Medical knowledge | Diseases » Pediatrics final topics

Datasheet

Year, pagecount:2008, 297 page(s)

Language:English

Downloads:12

Uploaded:July 05, 2013

Size:1 MB

Institution:
-

Comments:

Attachment:-

Download in PDF:Please log in!



Comments

No comments yet. You can be the first!


Content extract

1) Failure to thrive in infancy – DD and treatment (see topic 4) 2) Lyme disiease (Boreliosis) Causative agent: Borrelia burdogferi Transmission: ƒ Ticks serve as vector. sheep and deer are the hosts ƒ Transmission occurs only after the tick has been in place for at least 36-48 ƒ hours. ƒ The agents enter the skin in the site of the bite. It may spread in the lymph, producing regional adenopathy, or disseminate in the blood to organs or other skin sites. Clinical findings: ƒ The classic description of the disease divides the illness into 3 stages: 1. Primary- erythema migrans 2. Secondary- develops weeks to months after (Skin, CNS, joints, heart) 3. Tertiary- develops months to years later (chronic neurologic/ joint disorder. ƒ In the real disease course there may be overlapping of the symptoms and each one of them can appear in the early or late state. ƒ More accurate classification divides the disease into early and late manifestations and specifies whether the disease is

localized or disseminated. Signs and symptoms: 1.Early localized infection: ƒ Erythema migrans (EM)- is the hallmark and best clinical indicator of the disease.it develops in 75% of pts. Beginning as a red papule or macule, 3-32 days after the bite the area expends, often with central clearing to a diameter of up to 50cm. ƒ DD: o cellulitis o urticaria o erythema multiforme o pityriasis rosea ƒ Flu like syndrome- malaise, fatigue, fever, headache, stiff neck, myalgias, and arthralgias. ƒ Resolve in 3-4 weeks. 2. Early disseminated infection: ƒ The spirochete may spread in the pts blood or lymph to cause a wide variety of symptoms and signs. ƒ Usually occurs within days to weeks after inoculation of the organism. ƒ Most common manifestations are: 1. Skin- multiple, usually smaller them EM, lesions (50%) 2. Neurologic- lymphocytic meningitis, Bells palsy (10-20%) 3. Cardiac- fluctuating degrees of AV block, rarely myopericarditis The AV block may be total and may require a pace

maker until it resolves. (410%) 4. Musculoskeletal 3. Late persistent infections: Occurs months to years after initial infection. 1 Most common manifestations are: 1. Musculoskeletal- up to 60% of pts the pathogenesis of those symptoms is rather immunologic in origin than persistent infection. include: - joint and periacticular pain. - frank arthritis, maily in large joints- knee, that is recurrent or chronic over years. Increase frequency in HLA-DR4 - chronic synovitis-may result in permanent disability. 2. Neurologic- both the CNS and the PNS may be involved - subacute encephalopathy, memory loss, mood changes,and sleep disorder. - axonal polyneuropathy. - Leukoencephalitis- a rare manifestation, more common in Europe. 3. Skin- acrodermatits chronicum atrophicans: bluish- red discoloration of distal extremity, usually with swelling. can occur up to 10 years after infection, more common in Europe. Diagnosis and lab: -Diagnosis is based on both clinical picture and lab findings.

-Criteria for diagnosis: 1. EM 2. At least one late manifestations 3. Lab conformation (detecting specific antibodies either by IFA or ELISA. -All specimens positive or equivocal by ELISA of IFA should be further tested by western blot.(detect both IgM and IgG) 1. IgM appears 2-4 wks after EM, peaks at 6-8 wks, decline after 4-6 months. 2. IgG appears 6-8 weeks, peaks 4-6 months and remains high indefinitely. -The diagnosis of early disease is only clinical. -Late disease requires objective evidence of clinical manifestation and lab results. -Pts without specific symptoms should not have serologic testing, which are in this case have very high incidence of false positive. Prevention: ƒ ƒ ƒ ƒ Simple preventive measures such as avoiding tick-infested areas, covering exposed body parts, using repellents will greatly reduce the incidence. The rle of antibiotics prophylaxis after a tick bite is controversial. (a single dose of 200mg doxycycline given within 72hrs after tick bite can

prevent the development of the disease) ƒ Vaccination with a recombinant outer surface protein is still under investigation to be well tolerated to humans. Treatment: oral ATB (doxycycline, amoxicillin) for early, localized infection, Bells palsy & arthritis. IV ATB (ceftriaxone, penicillin G) is recommended for pt. with meningitis & late neurologic disease 2 3) COAGULOPATHIES Normal hemostasis requires the integrity of 3 elements: blood vessels, platelets & clotting factors. Hemorrhage may result from deficiency or disorders of any of them. • Clinical features petechiae, purpura, severe recurrent epistaxis, prolonged bleeding after dental extractions, surgical procedures, or trauma, and recurrent hemarthrosis. • Screening the pt. with a suspected coagulopathy requires PLT count, bleeding time, partial thromboplastin time (PTT, to measure intrinsic & common pathways), and prothrombin time (PT, to measure extrinsic & common pathways). • General

approach drugs compromising platelet function (e.g aspirin) must be avoided, as should deep IV & IM injections. According to severity the pt should be protected against trauma (esp. the head) If the bleeding is life-threatening, fresh frozen plasma can be used temporarily until a specific factor deficiency is identified. Disorders of blood vessels • Vasculitis – mechanical damage to RBC & PLT. • Excessive capillary fragility – hereditary disorders of collagen (Ehlers-Danlose sy). • Vit-C deficiency (scurvy) – results in impaired collagen synthesis (fragile vessels). • Henoch-Schönlein purpura – associated with purpuric rash (esp. of legs & ass), arthritis, GN, urticaria & GI pain. Disorders of platelets – PLT defects can be quantitative or qualitative. Quantitative disorders are detected by peripheral blood smear or PLT count. Qualitative disorders are detected by bleeding time or PLT aggregation studies. 1. Quantitative disorders –

characterized by thrombocytopenia (TP) below the normal range of 150,000-300,000/μl. It is the most common cause for abnormal bleeding The low count may result from failure of production or from shortened survival. Production is evaluated by number of megakaryocytes in the bone marrow. ƒ TP due to decreased PLT production: o Bone marrow failure – associated with pancytopenia (see topic 4C). o Wiskott-Aldrich syn – X-linked recessive inheritance, characterized by eczema, recurrent infections (T & B-cell deficiency) & TP. ƒ TP due to shortened survival: o Immune-mediated – may be associated with viral infection or drugs (e.g heparin HIT), but mostly idiopathic (see topic 2C). o Non-immunological – related to vasculitis, DIC, HUS, TTP or prosthetic valves. o Hypersplenism – may affect all cell lines. Can result from: 3 ƒ • Myeloproliferative or lymphoproliferative disorders. • Malignancy infiltrating the spleen. • Lipid storage diseases e.g

Gauchers disease • Liver diseases. TP in the newborn – due to many causes congenital infections (e.g toxoplasmosis, rubella, CMV, HSV); bacterial sepsis; immune-mediated causes (e.g ITP); and DIC 2. Qualitative platelet disorders – congenital or acquired They are associated with platelets that although sufficient in number, are dysfunctional. Congenital cases are rare (eg gray platelet syn, with no granules in the PLT). Acquired cases are mostly drug-induced (esp aspirin, by inhibiting degranulation from platelets). Disorders of soluble procoagulant factors – coagulopathies due to deficiency of a single factor are usually hereditary, whereas multiple factor deficiencies are usually acquired. Disorders of single coagulation factors: Hemophilia A & vW disease (factor-VIII deficiency) the most common hereditary coagulation disorders, that can be distinguished by clinical & lab. means: Hemophilia A X-linked, with spontaneous or traumatic hemorrhages (SC, IM, within

joints). As a rule only males are affected In infants, excessive bleeding can occur after circumcision. Usually hemophilia-A is usually not a problem within the 1st year of life The PTT is prolonged & bleeding time is normal. VIII:C (antihemophilic factor) activity is low; VIII:R (vWF) is normal. Therapy: Supportive care includes immobilization, local pressure on bleeding sites & ice packs. In mild cases, desmopressin (releases factor VIII from endothelial storage), can be used. In severe cases, infusion of factor VIIIcontaining products are needed Von Willebrand’s disease clinically similar to hemophilia-A. Severity varies with the degree of deficiency of factor VIII (4 variants). There is prolonged PTT & bleeding time. Both VIII:C and VIII:R levels are low Therapy: see hemophilia A Hemophilia-B (factor IX deficiency; Christmas disease) X-linked recessive disorder. Clinically: see hemophilia A. PTT is prolonged; factor IX activity is decreased Recombinant factor IX is

now available for treatment. Hemophilia-C (factor-XI deficiency) autosomal recessive, mainly affecting Ashkenazi Jews. Usually mild Treated with factor-XI replacement via FFP when needed 4 Disorders of multiple coagulation factors: Vit-K deficiency – Affecting dependant factors II, VII, IX & X (synthesised in the liver), as well as the antithrombotic factors protein C & S. Deficiency can occur in malabsorption states, other GI diseases, and drugs (e.g coumarin, antagonists to vit-K) Hemorrhagic disease can occur in neonates if vit-K is not given. Therapy: nutritional disorders and malabsorption respond to parenteral vit K. FFP (fresh frozen plasma) or prothrombin complex concentrate (factor II, VII, IX, X) in severe cases. Liver disease – all the coagulation factors, except factor VIII, may become deficient. In addition, hepatic clearance of activated clotting factors may be impaired. Thus, both PTT and PT are prolonged. FFP is given as therapy DIC –

thrombohemorrhagic condition, caused by endothelial damage (renal disease, sepsis, giant hemangioma); or impairment of clearance of activated clotting factors (liver disease). The bleeding is diffuse (GI, pulmonary bleeding, hematuria & from traumatised sites). Thrombotic lesions affect the extremities, skin, kidney & brain. PTT and PT are prolonged There is thrombocytopenia and hypofibrinogenemia. The levels of FDP are elevated Therapy: treat the primary disease. For supportive measures see hemophilia A If bleeding persists or thromboses present, heparinization with PLT & clot. factors replacement (FFP) should be given Thrombophilic disorders – predisposition to venous or arterial thrombosis may result from defects in proteins that inhibit coagulation or fibrinolysis. Congenital thrombophilic disorders include: • AT-III deficiency. • Dysfibrinogenemia. • Protein S or C deficiency. • Hyperhomocystinemia. • Factor V Leiden mutation. • Defective

plasminogen activator. The 2 principle endogenous anticoagulant pathways are the antithrombin-heparan sulfate pathway and the protein C/protein S pathway (for more details see pathophysiology). Defects in either of these factors may predispose to thrombosis. Therapy & management: - Primary prophylaxis heparin &/or warfarin in case of prolonged surgery or immobilization. - Acute thrombotic events beginning with heparin, followed by warfarin after 2 days of heparinization. Heparin is continued until the desired warfarin effect (INR) is achieved - Secondary postthrombotic prophylaxis lifelong warfarin, maintaining INR 2-3. 5 4) Failure to thrive (FTT) in childhood – DD and treatment Weight consistently below the 3rd percentile for age; progressive decrease in weight to below the 3rd percentile; weight < 80% of ideal weight for height-age; or a decrease in expected rate of growth based on the child’s previously defined growth curve, irrespective of whether

below the 3rd %. Etiology & pathophysiology – the physiologic basis of any FTT is inadequate nutrition. Organic FTT – the growth failure is due to an acute or chronic disorder that can interfere with normal nutrient intake: Cleft lip or palate. GERD. Laryngomalacia, tracheomalacia. Malabsorption (celiac, CF, disaccharidase deficiency). Metabolism (fructose intolerance, galactosemia). Excretion (DM, proteinuria). ↑↑ energy requirements (bronchopulmonary dysplasia, cystic fibrosis, hyperthyroidism). Non-organic FTT – the growth failure is due to: Environmental neglect – e.g lack of food that can be associated with poverty, poor feeding techniques, improperly prepared formula (overdiluting to save money), or an inadequate supply of breast milk (e.g maternal stress, exhaustion, malnourishment) Stimulus deprivation – the unstimulated child becomes depressed, apathetic & anorectic. The stimulus may be lacking because of caregiver (usually mother) depression, anxiety,

poor parenting skills, or hostility towards the child. Also related to external stress (eg demands of other children, marital dysfunction). Mixed FTT – the organic & nonorganic overlap. The Dr must recognize the relative contributions of each factor. Diagnosed in: Children who were born prematurely & have growth failure later in infancy. Those with a defect that doesnt explain the current FTT (e.g repaired cleft palate) Those who are frustrating (e.g because of neurologically impaired sucking) Diagnosis – children with organic FTT may present at any age depending on the underlying disorder; most children with nonorganic FTT manifest growth failure before age 1. Weight is the most sensitive indicator of nutritional status. The brain is preferentially spared in protein-energy malnutrition, so reduced growth in head circumference occurs late and indicates severe malnutrition. When growth failure is noted, a history (including diet) is obtained, diet counseling is given, and

the child’s weight is monitored frequently. If the child does not gain weight, he is admitted to the 6 hospital so that all necessary observations can be made and diagnostic tests performed quickly (CBC, ESR, BUN, Cr, urinalysis, urine culture, stool examination, and other more specific tests). Without evidence of a specific underlying etiology, no single clinical feature or test can reliably distinguish organic from nonorganic FTT. Search for an underlying medical problem and for personal and family characteristics that support a psychosocial etiology for FTT. Get the parents to co-investigate with you. It avoids blaming parents, who may already feel frustrated or guilty because of inability to nurture their child. Suspected neglect or abuse must be reported Prognosis – with nonorganic FTT, 50-75% of children >1 year achieve a stable weight above the 3rd percentile. ~50% may have later cognitive functions below normal range, esp verbal skills & general behavioral

problems. Treatment: a nutritious diet containing adequate calories for catch-up growth (150 % of normal kcal requirements) and individualized medical and social supports are required. For children with organic or mixed FTT, the underlying disorder should be treated as quickly as possible. For children with nonorganic FTT, management consists of providing education and emotional support to correct problems interfering with the parent-child relationship. 7 5) Infectious mononucleosis An acute infection by EBV (heterophil positive) or CMV (heterophil negative), Transmitted primarily via saliva exchange. Mono most often affects adolescents & young adults Very common Clinical features – variable. Symptoms are usually less severe in younger children than adolescents: Prodrome of fever, malaise, sore throat & headache for 3-7 days before onset of more profound symptoms. Fever is always present and may reach 40°C Pharyngitis occurs in 80% of pt. and may be severe

Lymphadenopathy usually generalized, mostly involves the cervical nodes. Splenomegaly noted in most pt. Rash occurs in 5% of pt. and is variable Administration of ampicillin to mono pt will cause a pruritic maculopapular rash. It may appear after cessation of the ATB treatment!!! Other clinical findings include fatigue, eyelid edema & abdominal pain. In CMV mono theres no pharyngitis or respiratory symptoms. Heterophil Ab are absent Complications: Splenic rupture is a relatively rare complication. Airway obstruction is also rare. Treat with steroids Neurologic complications aseptic meningitis, encephalitis & peripheral neuropathies. Subclinical hepatitis occurs in 30%. Diagnosis: Lab: Absolute lymphocytosis, with atypical T-lymphocytes in blood smears. Hemolytic anemia secondary to anti-i (fetal RBC) Ab & thrombocytopenia. Positive heterophil (sheep RBC) agglutination test (Paul-Bunnell test, monospot test). During acute attack rise & drop of IgM and rise in IgG

to Viral Capsid Antigen (VCA). Ab to Nuclear Antigens (EBNA) appear 4wk after onset. The lifetime immunity to IM is through Ab to Viral Membrane Ag (VMA). Hepatic aminotransferases & bilirubin are usually elevated. Therapy: recovery may take weeks to months. Treatment: Specific treatment is not very effective, except maybe penciclovir. In most cases, rest is the only treatment. Paracetamol for symptomatic relief (not aspirin) & throat irrigations. Airway obstruction, anemia, thrombocytopenia 5 days of corticosteroids. Avoid palpation of spleen to prevent rupture, which requires splenectomy. 8 6) Idiopathic / immune thrombocytopenic purpura (ITP) PLT defects can be qualitative or quantitative. Qualitative disorders are detected by bleeding time or platelet aggregation studies. Quantitative disorders are detected by ↓PLT count ITP belongs to the quantitative group of disorders, to the subgroup of thrombocytopenias due to shortened platelet survival. ITP is an autoimmune

disorder, with immune-mediated thrombocytopenia. Opsonized platelets are trapped and destroyed in the RES. 2 subtypes: Acute ITP – transient, common in children, after viral & upper respiratory tract infections. Viral proteins are modified and the Ab formed against them cross react with PLT. Chronic ITP – more severe, affects mainly middle aged females. AutoAb are formed to platelet adhesion molecules platelet destruction thrombocytopenic purpura. Clinical features: ITP may be seen after a mild viral illness or an immunization. In children the onset is usually abrupt, with bleeding of the skin and mucous membranes. Bleeding is severe after trauma. Severe internal hemorrhage is rare In 80% of pt, ITP resolves spontaneously within 1 to 6 months. However, some cases become relapsing or chronic. Therapy: mild cases (PLT >20,000/μl) may be managed with observation alone. More severe acute cases are treated by IVIG, anti-RhD Ab, or corticosteroids. These agents interfere with

splenic sequestration of antibody-coated platelets. Chronic cases are treated with splenectomy or immunosuppressive agents. 9 7) Congenital adrenal hyperplasia (CAH) Salt loosing syndrome is the most common clinical presentation of congenital adrenal hyperplasia. - Caused by a family of autosomal recessive disorders of adrenal steroidogenesis leading to deficiency in cortisol. - The obligatory decreased production of cortisol(due to enzyme deficiency) causes hypersecretion of ACTH, which in turn stimulates the synthesis of steroids immediately proximal to the block and causes shunting of precursors to the androgen pathway leading to the overproduction of testosterone. - There are a few enzymes that have been shown to be deficient, from them the most common is 21-hydroxylase. Deficiency of 21-hydroxylase - Usually presented in the classic form of salt-wasting and virilizing (75%), others (25%) have the simple virilizing form, which is not diagnosed until puberty usually it is

asymptomatic or can cause hirsutism ,cane in girls or spermatogenesis disturbance in boys. - Newborn screening programs are available. Patient without salt loosing males: -Premature isosexual develops and hirsutism. -Enlargement of the penis, scrotum and prostate. -Tall in early childhood but due to early closure of the epiphysis their adult stature is short. females: -Pseudohermaphroditism. -Musculinization at birth, which progress. enlargement of the clitoris The verilization of the external genitalia of the female occurs in utero the development of the ovaries, fallopian tubes and uterus is unaffected. -Body build of a boy. -No breast development and menstruation. Salt loosing patients - Symptoms begin shortly after birth. - in male there is FTT, in females FTT and virilized genitalia. - vomiting and cyanosis. - The FTT, dehydration and poor general condition can lead to collapse and death within few weeks. DD in boys ƒ pyloric stenosis. ƒ bowel obstruction. ƒ heart

insufficiency. ƒ cow milk intolerance. LAB: ƒ Decreased serum Na+ and Clƒ Increased serum K+ conc. ,rennin conc, highly increased 17-αhydroxypregesterone (17-α-OHP) conc ƒ Increased urea 17-ketosteroids conc. and pregnantreiol 10 Diagnosis: -Suspicious about CAH should always rise when there is: ƒ Previous family history. ƒ FTT. ƒ Ambiguous genitalia. non decendent testicles ƒ Virilization. ƒ Severe dehydration and vomiting. ƒ Hypoglycemia -CT and US should be done to roll out adrenal or pituitary tumor. -Prenatal diagnosis is possible with villi sampling or with 17-OHP measurement in amniotic fluid. Treatment: -Glucocorticoids- cortisol 15-20mg/m2/day. -In a severe salt loosing babies intensive treatment with IV hydrocortisone and glucose-saline infusion. after the water and electrolytes balance correction ,salt should be added to the diet. mineralocorticoids and cortisol -Reconstruction surgery for the female genitalia should be done at 6-12 months of age.

11-β-hydroxylase deficiency No production of cortisol from 11-deoxycortisolÆ 11-deoxycortisol, androgens and deoxycorticosterone accumulationÆ veriliztion, salt storing, hypokalemia and alkalosis. clinical picture: Girls -pseudohermaphroditism (ambiguous genitalia). -hypertenson. -early epiphysial fusion. Boys -early age hirsutism and penile enlargement. -hypertension. -gyenicomastia. -early epiphysial fusion. 3β-hydroxysteroid dehydrogenase deficiency Deficiency in this enzyme causes no synthesis of glococorticoids and minaralocorticoidsÆ DHA (dehydro-piandrosterone) accumulation. low synthesis of the sex hormones Girls -normal to mildly virilized genitalia. -severe salt loosing shortly after birth. Boys -pseudohermaphrodism (hypospadias, ambiguous genitalia). -severe slat losing shortly after birth. 17α-hydroxylase deficiency Disturbance in glococorticoids , mineralocorticoids and sex hormones synthesis. -salt storing may occur. Girls -normal female genotype. -late secondary

sex change and late menstruation appearance. -hypertension. Boys -peudohermaphrodism with ambiguous genitalia. -hypertension. 11 8) Management of common poisons in childhood. Epidemiology: Poisoning is the third most common injury treated in the ER for children age <16. Poisonings peaks at age 1-5 & at adolescence. The younger cases are involuntary, unexpectable poisonings. The older group are related to suicide attempts Initial treatment is the same either way Environmental factors: 95% are related to home exposure. Circumstances that ↑ risk of ingestion include storing dangerous chemicals (cleaning agents, motor oil, pesticides) in drinking glasses or bottles; visiting friends (with their medications); lacking childproof caps for medicine containers, and storing different medications in the same container. Most common poisoning agents: Cosmetic and personal care products, cleaning substances, analgesics, plants, cough and cold remedies, antidepressants, sedatives,

hypnotics, antipsychotics. Examination: Careful history and screening blood and urine for toxic substances. Assess & continuously monitor the LOC. Pupil size & reactivity, breath odor, cardiorespiratory status, and a careful neurologic examination are key points. Principles of therapy: Adsorption of poison – by activated charcoal, which binds the poison & inhibit GI absorption. It works within few min and may be inhibited by food. It doesnt affect iron salts, boric acid, cyanide, caustics, or lithium. The charcoal should be 5-12 times more than the ingested stuff The recommended pediatric dose is 15-30 g for children <12 yr and 50-100 g for children >12 yr. its given in an aqueous slurry, which children often refuse to ingest use an NG tube Removal of gastric contents – may be the 1st step in case of less serious ingestions. Removal is done by induction of emesis or gastric lavage. Emesis is done by ingestion of ipecac syrup, which works within 20 min. Never use

in depressed LOC or volatile hydrocarbons may result in lung aspiration. Dont use with caustics risk of ↑↑ esophageal damage Gastric lavage is used for children who cannot take or dont respond to ipecac, or have a depressed LOC. For severe mental status depression intubation with a cuffed ET tube is required to prevent aspiration. Lavage with normal saline should continue till the returned fluid is clear for 2 or 3 passes. Intestinal cleansing – cathartics (magnesium sulphate, sodium sulphate, magnesium citrate) decrease the transit time through the GI, thus making them less available for absorption. Specific poisonings & antidotes: Paracetamol (along with ibuprofen) has completely replaced aspirin as an antipyretic-analgesic for use in children. Children <5 yr seem more resistant to severe toxic sequelae than adults Clinical features: Phase I begins 30-60 min after ingestion and lasts for 24-48 hrs. Most mild cases are asymptomatic. In severe cases, GI signs A/N/V,

pallor & sweating 12 In the later Phase II that may last up to 4 days, the pt. is clinically asymptomatic Liver function tests rise as hepatic necrosis ↑↑. Phase III occurs few days later, w/ symptoms of hepatotoxicity A/N/V, abd. pain, jaundice, hypoglycemia, confusion, stupor & encephalopathy Therapy: If dose is unknown or >100 mg/kg intervention with ipecac syrup & N-acetylcysteine (antidote). Aspirin – widely used by adults. It is absorbed rapidly from the stomach, metabolised by the liver and excreted through the kidney. When urine pH is >74, the drug is ionised & not reabsorbed, thus shortening the t1/2. Aspirin is a weak acid producing metabolic acidosis & stimulates the respiratory center to ↑↑ rate & depth of respiration resulting in respiratory alkalosis. It also ↑↑ metabolic rate tachycardia, tachypnea, fever, and hypoglycemia. Clinical features: N/V, tinnitus, hyperpnea, fever, confusion, convulsions, coma, and

respiratory or cardiac failure. The serum level after 6 hrs is predictive of the course Risk of Reye syn Therapy: vomiting, followed by activated charcoal & cathartics. Alkalinization with IV NaHCO3 Dialysis or hemofiltration in severe cases (respiratory, renal, or cardiac failure). Iron-containing products – the leading pharmaceutical cause of death in children <5 yr. It is widely available without prescription e.g in multivitamin capsules Clinical features: GI symptoms: Within 30-60 min, vomiting, colicky abdominal pain, GI hemorrhage & diarrhea occur. Iron acutely & directly damages the GI Circulatory shock occurs after 48 hrs, due to GI fluid & blood loss, ↑↑ capillary permeability, and loss of vascular tone. Late manifestations: coagulopathy, gastric scarring (within few weeks). Therapy: emesis and cathartics (charcoal has no effect). Iron levels should be measured after 5h If the concentration is toxic, IV deferoxamine is recommended for chelation.

Hydrocarbons (petroleum distillates) – include gasoline, lighter fluid, paint thinner, etc. Aspiration into the respiratory tract is the major risk. The major effect is chemical irritation, which damages the alveolar lining and the capillaries, causing pneumonitis, atelectasis, or pulmonary edema. Results are severe hypoxemia and respiratory acidosis Because of the risk, emesis should not be induced!!! CNS depression results from absorbed hydrocarbons. Clinical features: Early, theres burning in the mouth & throat, choking and gagging, coughing, N/V, and hemoptysis. Tachycardia and tachypnea reflect the degree of pulmonary insult A CXR shows changes before the onset of significant clinical findings. 1 hour after ingestion, CNS depression occurs (weakness, dizziness, confusion, lethargy, agitation). Therapy: For large ingestions, spont. vomiting usually occurs Comatose pt should be intubated and lavaged. 13 Poison control centers – cecause the number of potential poisons is

so high and gathering information about them is so time consuming, there are state-wide and regional poison control. 9) Single-gene defects (AD & AR inheritance, X-linked dominant and recessive). Autosomal dominant AD disorders – when 1 gene of a gene pair is altered or mutated. General characteristics – often caused by a mutation in a gene coding for a structural protein or in a protein involved in signaling or developmental pathway. Any pt with an AD disorder has a 50% chance of passing it to each offspring. Such gene is usually inherited from one parent who is affected with that condition. Sometimes, a person will be the first person with the defect This is due to a new mutation. Clinical features – AD genes produce conditions that manifest differently and vary in degree of severity this is the variable expressivity. For example, in polycystic kidney disease, some people have early CRF, whereas others have only HTN & normal renal function at the same age. Carriers of

some AD mutations may have no identifiable abnormality, whereas others in the family have a recognizable disorder. This is called decreased penetrance (eg polydactyly, as some carriers are normal). Mutant AD gene often affects more than 1 tissue/organ called pleiotropy Marfan’s syndrome – AD; affecting 1:10.000 newborns It has variable expressivity, pleiotropy, and a high rate of new mutation (only 70% of affected pt. have an affected parent) Caused by mutation in the fibrillin gene, leading to defective CT production. Prenatal diagnosis available Affected pt. should be followed up with physical exams, eye evaluations, and echo Symptoms: Skeletal long thin face, limbs & digits; high arched palate, tall stature, sternum deformity, hypermobile joints, scoliosis. Cardiac aortic root dilation, MV prolapse, risk of aortic aneurysm rupture. Ophthalmologic lens subluxation, flat corneas, severe myopia. Pulmonary spontaneous PTX, emphysema. Predisposition to cancer – can be

inherited, usually in an AD manner. Many genetic syndromes have an increased risk of both cancer and developmental abnormalities: Beckwith-Wiedemann syndrome – characterized by general overgrowth, macroglossia, neonatal hypoglycemia, umbilical abnormalities, and predisposition to cancers (hepatoblastoma, nephroblastoma). Usually caused by a new mutation Neurofibromatosis type I – characterized by fibromas & nerve sheath tumors, cafe au lait spots & freckles. Pt have a higher risk of pheochromocytoma, fibrosarcoma, neurofibrosarcoma, and some benign tumors such as optic gliomas. 14 In other individuals, the predisposition to cancer is the major finding. Most cancers occur in adulthood. Oncogenes cause cancer when they are overexpressed due to duplication of the mutant gene, or because a mutation causes activation or loss of suppression of the gene. Tumor suppressor genes cause cancer when the normal function of the gene is lost. Cancer may also develop d/t mutations in

genes whose proteins are involved in DNA repair. Autosomal recessive AR disorders – occur when both genes of a gene pair have mutations. Many AR disorders are caused by mutations in genes coding for enzymes e.g most inborn errors of metabolism. Since half of normal enzyme activity is usually enough, only one mutant gene will not affect the pt. Each parent of a child who has an AR disorder is presumed to be heterozygous for that mutation; each child of such a couple has a 25% risk of having the disorder. Cystic fibrosis – one of the most common AR disorders, affecting 1:2000 newborns. A defect in membrane transport of Cl- results in an inability to clear mucous secretions in the lungs & causes ↓↓pancreatic exocrine function. >800 different mutations were identified, however, 1 mutation named F508 is found in 70% of US CF pt. The features vary in nature and severity Diagnosis: sweat chloride testing, direct and indirect DNA analysis. Prenatal diagnosis can be made if

mutations have been identified in the parents. Symptoms: - Respiratory insufficiency, chronic sinusitis, malabsorption. - Meconium ileus, biliary cirrhosis, abnormal glucose intolerance. - Reproductive defects. X-linked disorders – when a male inherits a mutant gene (X-linked), which is always maternal in origin. Common X-linked disorders include hemophilia A, color blindness, Duchenne muscular dystrophy, and fragile X syndrome. - General characteristics – The mother is usually heterozygous for the gene. It is estimated that 1/3 of lethal X-linked disorders are due to a new mutation. - Recurrence risk – if the mother has the mutation on 1 X-chromosome (she is a carrier), there is 50% that the gene will pass to each offspring. If the offspring inheriting the mutant gene is a girl, she too will be a carrier. If its a son, he will be affected since theres no other X to compensate. Thus, each daughter has a 50% of being a carrier, and each son has a 50% of being affected.

If the father has the mutation (hes affected), he passes only that abnormal gene to his daughters all his daughters will be carriers. Because the Y is normal, all sons will be normal 15 Fragile X syndrome – a common cause of mental retardation. The incidence is ~1:4000 males and females. Diagnosis by molecular genetic testing, based on the number of repeats of the nucleotide sequence CGG within a specific gene. Affected pt have more than 200 repeats, which inactivate the gene (normal is 6-52). This results in mental retardation The number of repeats in part determines whether the baby will be affected. This complicates genetic counseling and risk prediction Clinical features are variable large birth size, macrocephaly, long face, large ears, long chin, macroorchidism, join laxity, MV prolapse, mental retardation, and hyperactivity. 10) Short stature – DD and treatment. Definition of short stature - A height < 0.4 percentile - Predicted height< target MPH (Mid Parental

Height = dad+mom/2). - Height changing by more than one percentile band over 1-2 years (Abnormal growth velocity). Causes of short stature 1. Familial (short family) 2. Constitutional delay of growth spurt- very common, normal GH the child is below the lower 10% but growth rate is normal and the child can reach its expected genetic high. 3. Endocrine disorders: ƒ GH deficiency: a. Can be associated with hypoglycemia, cyanosis and micropenis in male. b. The child has a special external appearance: round face, short broad face, bulging frontal bone, depressed nose bridge, small nose, bulging eyes. c. High pitched voice d. Short neck but normal body proportion e. Weigh and height at birth are normal but there is growth abnormality, usually in the 1st year of their life. f. Under developed genitalia- may cause late puberty of even lack of puberty. Lab exams- GH level and GHRH, GH reaction to injection of GHRH etc. X-ray- bone age. ƒ Hypopituitarism: a. Congenital abnormalities-

anacephalus, ectopic gland, cleft lip (may cause GH deficiency in 4% of patients). b. Idiopathic hypopituitarism- may be associated with birth trauma or forceps delivery, increase incidence in breech birth. c. Acquired disturbances-kariopharyngioma, hypophysial tumors,TB, sarcoidosis, toxoplasmosis, trauma, anoxia and radiation. ƒ Hypothyroidism: a. Congenital – apalsia and hypoplasia of the gland, glandular iodine conc disturbance, thyroid ectopy ETC. 16 4. 5. 6. 7. Clinical – LGA babies with jaundice, drowsiness, sleepiness, apneas, low appetite, don’t cry much, growth retardation- short limbs, long duration of undiagnosed case will lead to mental retardation. Labs –TSH and free T4 levels. Treatment- L-thyroxine begin with 10-15 microgram/kg and gradually dcrease it to a dose that will keep the free T4 in the normal range. b. Acquired- appearance of hypothyroidism in a previously healthy child, diagnosis and treatment are according to the congenital type. ƒ Cushing

syndrome- growth retardation to complete stop of growth with late bone age. ƒ Steroid therapy. Chromosomal disorders: ƒ Turner syndrome. ƒ Silver-Russell syndrome- short stature, frontal bossing, triangular shaped face, LGA(low for gestational age) babies. Skeletal dysplasia ƒ Achondroplasia. Emotional and psychological deprivation- sleeping disturbance, hungry agitated child, nucturia. Chronic illness: ƒ CHD ƒ CF ƒ Cerebropalsy ƒ CRF ƒ FTT Approach to short-stature - When a patient with a short stature is presented, few things should be always kept in mind: ƒThe majority of children who are presumed to have short stature are normal and have either familial short stature or constitutional delay of the growth spurt. ƒShort stature can be due to many systemic conditions, such as IBD, cardiac disorders, renal disease and other. ƒTurner syndrome is commonly presented with short stature. ƒEmotional deprivation is common and treatable etiology for short stature. - Height should

be monitored over 6-12 months in response to parental concern regardless of current percentile. History - Pregnancy and birth. - Parental height. - Family history. Examination - Height. - Growth velocity- Two measurements at least 6 months apart on growth chart. - Dysmorphic features ƒ Turner syndrome: webbing of neck, wide spaced nipple, low set ears. ƒ Prader-willi syndrome: obesity, hypotonia, small genitals( males) ƒ Skeletal dysplasias: disproportions short limbs- achondroplasia. short trunk- Mucipolysaccharides (MPS). - Weight, endocrine disorders are associated with increased weight. - Visual fields, may be decreased because of a pituitary tumor pressing the optic chiasm. - Stages of puberty (tanner). 17 - Lab exams- if there is endocrine deficiency. Treatment -Treat the underlying cause. -If there is GH deficiency – 7 injection per week the dose according to the childs weight, in puberty the dosage should be higher.younger children react better to this treatment

rather the older ones. -Give thyroxin and hydrocortisone as necessary. -In case of micropenis treatment with testosterone for 1-3 months will give good results. -In case of GH deficienct accompanied with sex hormones deficiency we should add sex hormones to the treatment with GH. 11) Changes in the circulation at birth. Diagnostic evaluation of the CV system in the newborn period. Prenatal circulation Blood oxygenation occurs in the placenta, an organ of low vascular resistance, and the oxygenated blood reaches the fetus via the umbilical vein, which enters the through the liver. Blood passes via the ductus venosus to the IVC. Shunting allows oxygenated blood to be shunted through the foramen ovale to the LA and LV systemic, whereas the unsaturated blood of the IVC & SVC goes to the pulmonary artery. Due to the high vascular resistance in the lung, 90% of the pulmonary artery blood bypasses the lung and shunted through the ductus arteriosus into the aorta, thus giving the most

oxygenated blood to the heart and brain. Blood in the descending aorta, which is intermediately oxygenated, is returned to the placenta by way of the umbilical arteries. Postnatal circulation At birth, there is a rise in systemic vascular resistance as a result of the stopped blood flow through the placenta. With the first few breaths, pulmonary vascular resistance falls, the foramen ovale closes, and the ductus arteriosus begins to constrict. These processes allow all deoxygenated blood returning to the right ventricle to go on to the lung and become oxygenated. The oxygenated blood returns to the LV and then is pumped throughout the body. Diagnostic evaluation of the CV system in the newborn period Physical examination Peripheral cyanosis (fingers, toes, around the mouth) – it is common in normal newborns. However, cyanosis of the mucous membranes is diagnostic of a RL shunt. Cyanosis of pulmonary origin can usually be improved with oxygen administration, whereas cyanosis of

cardiac origin does not respond to oxygen. Shortness of breath, dyspnea or syncope with exertion, feeding difficult, and disturbances in growth may be due to abnormal cardiac function. 18 Pulse – note the presence & quality of peripheral pulses. It is important to palpate radial and femoral arteries simultaneously to rule out coarctation of aorta. The pulse may appear delayed if the arteries are filled via collateral vessels. BP – pressure in the lower extremities should normally be the same as in the upper. A wide pulse pressure (low diastolic) may indicate lesions such as aortic insufficiency or PDA. Precordial palpation – a thrill or palpable murmur defines an area of maximal turbulence. A diffuse impulse in the parasternal region may indicate RV enlargement. Cardiac auscultation: 1. Heart sounds – S1 may be single or split S2 is split during inspiration Abnormal wide splitting occurs with RV overload and prolonged RV emptying. The pulmonary component of S2 is

accentuated in pulmonary HTN. The aortic component of S2 is accentuated in systemic HTN or if the aortic valve is near the chest wall, e.g transposition of the great arteries. S3 is usually a normal in children S4 is always abnormal 2. Clicks – an ejection click occurs shortly after S1 d/t the opening of a stenotic semilunar valve or the sudden distention of an hypertensive pulmonary artery. A mid or late systolic click originates from prolapse of the mitral or tricuspid valve. 3. Murmurs – Functional murmurs (sounds of turbulence) are usually present at some time during childhood and are often age-related. Systolic murmurs that begin with S1 are called rugurgitant murmurs, caused by regurgitation through the AV valve or d/t a LR VSD. Systolic murmurs that begin immediately after S1 are called ejection murmurs (opening of the semilunar valves). Ejection murmurs are caused by ventricular outflow obstruction. Murmurs that begin late in systole are characteristic of MV prolapse.

Diastolic murmurs begin with S2 are caused by semilunar valve regurgitation; mid– diastolic murmurs are caused by increased flow across one of the AV valves or AV valve stenosis. Continuous murmurs start in systole and continue into diastole 19 Laboratory evaluation 1. CXR – to see the location of the heart and its size, and whether the pulmonary vasculature is normal, diminished, or increased. 2. ECG – for rhythm; it may reflect anatomic changes (eg hypertrophy), and ischemia 3. Echo – for cardiac structure & function Direction and velocity with Doppler, permitting visualization of shunt direction, valve regurgitation & pressure gradients across valves. 4. Cardiac catheterization – provides info abt systolic P in the chambers and great vessels (evaluating gradients), ventricular end-diastolic pressure (evaluating myocardial efficiency), pulmonary and systemic artery pressure (evaluation the resistance). 5. Arterial blood gasses – for oxygen content & SAT,

permitting detection of LR & RL shunts, measurement of CO, and calculation of systemic and pulmonary vascular resistance. 6. Selective angiography – visualizes cardiac and vascular anatomy 7. Therapeutic catheter interventions – include balloon atrial septostomy, balloon angioplasty of stenotic valves and vessels, occlusion of communications such as shunts and collateral arteries, and insertion of stents to maintain dilation of stenotic vessels. 12) Aplastic and hypoplastic anemias. Anemias of reduced or absent RBC production. Congenital hypoplastic anemias 1. Diamond-Blackfan syndrome- mostly is sporadic but 15% are familial, it is a disturbance of RBC production in the bone marrow. Clinical ƒSevere anemia appears in young age(2-6 months). ƒWithout blood transfusion heart insufficiency will develop and may cause death. ƒNo spleno- or hepatomegaly. ƒIn 1/3 of cases there are accompanied congenital defects ƒ(turner, triphalangeal thumb). Lab ƒ Macrocytic anemia. ƒ

Normochromic anemia ƒ Reticulocyte level is decreased and may reach 0. ƒ Normal erythrocyte life cycle. ƒ Increased fetal i antigen. ƒ High serum: Fe, ferritin, erythropoietin, folic acid and B12. Prognosis ƒ Not good, a lot of blood transfusions are needed unless steroids are causing remission and in that case hemosiderosis may develop with splenomegaly and hepatomegly hypersplenism, heart insufficiency and even diabetes. ƒ Death in the 2nd or 3rd decays of life. ƒ In less then 5% of patients leukemia and osteosarcoma may appear. 20 Treatment ƒ Steroids when given early- Prednisone 2-4mg/kg/day. ƒ 2-4% of patient have spontaneous remissions. ƒ 10-15% are steroid resistant,they may get testosterone or immunosuppressive treatment + multi blood transfusions. ƒ In hypersplenism conditions splenectomy is indicated. ƒ Unresponsive patients which have a right donor need BM transplantation. 2. Pearson syndrome- BM failure syndrome may resemble Diamond Blackfan anemia Etiology

This multisystem disease is cause due to mitochondrial DNA deletion. Clinical ƒ Macrocytic anemia ƒ Nuetoropenia ƒ Thrombocytopenia ƒ FTT ƒ Pancreatic fibrosis with IDDM, exocrinic pancreatic insufficiency. ƒ Neurological disturbances. ƒ Muscle disease. ƒ Early death. Lab -Fetal hemoglobin levels are elevated. -BM- erythroblasts and myeloblasts with vacules (ringed sideroblasts) are seen. Treatment -blood transfusions when necessary. -G-CSF. Acquired pure cell anemias ƒ Acquired depression of RBC synthesis in the BM. ƒ May be associated with thymoma. ƒ Some cases have erythropoietin or erythroblast antibodies. ƒ Some patients react to steroids, if not immunosuppressive treatment may be indicated (cyclophosphamide). ƒ Viral infercion may cause temporrly depretion of the erythropoesis. may pass spontaneously. ƒ Patients with hemolytic anemia Parvo Virus can cause aplastic anemia. Transient Erythroblastopenia of childhood ƒ Severe RBC production disturbance in the BM, in a

child who is healthy. ƒ Usually appears at 6 month to 5 years. ƒ Spontaneous remission through 1-2 months. ƒ Unclear mechanism.usually there is a previous viral infection Lab -Normal MCV, HbF, antigen i and enzyme levels. -serum Fe is high. -Normal adenosine deaminase activity in RBC. Treatment - No steroids are necessary. - Sometimes blood transfusion is indicated. 21 Congenital Dyserythropoetic Anemias -This is a rare group of disturbance of RBC production in the BM. -The anemia can be normocytic or macrocytic. -AD or AR inheritance. Clinical -In all forms of the disease there is chronic hemolysis findings: jaundice, gall stones and splenomegaly. Treatment -Blood infusions -If blood infusions are given frequently splenectomy should be considered. Anemia of chronic disorder and renal failure This anemia is mostly due to BM failure, but can be caused from other factors in infection or chronic disease. Clinical: ƒ Erythropoietin levels are low compare to anemia. ƒ Erythrocyte

destruction is increased and there is a problem with the iron metabolism- problem with its release from the tissues to the plasma. ƒ In CRF there are toxins in the blood that depress erythropoiesis. Lab ƒ Usually there is normocytic normochromatic anemia but some times hypochromatic ,microcytic anemia is found. ƒ Normal reticulocytes but in low amount. ƒ Serom Fe is low. but there is no increase in the iron binding capacity ƒ Normal ferritin level or it may be elevated, in the BM hemosiderin may be increased. Treatment ƒ No response to iron treatment. ƒ Deal with the underlying disease. ƒ In severe kidney failure the hemoglobin level and the clinical picture may be improved by erythropoietin treatment. Physiological Anemia of Infancy In a newborn the hemoglobin level is higher that in a child or in an adult. After birth there is a decrement in the hemoglobin level for few weeks, which peaks in 2-3 weeks with a low level of hemoglobin 9-11g/dl. The reasons for this anemia: 1.

Low production of hemoglobin in this period due to decreased erythropoietin level after birth with the increase blood oxygen saturation. 2. Shorter life spam of the embryos erythrocyes 3. Fast fat gaining and so blood volume in the first few months of life ƒ In prematures the hemoglobin level drop is faster (lowest in 3-6 weeks) and more severe, with hemoglobin values of 7-9 g/dl. mostly because the growing rate is faster in a premature, and the erythropoietin synthesis is not enough because it is synthesized in the liver in premature and the liver doesn’t sense hypoxia very well. ƒ Low folic acid level can cause physiological anemia. ƒ Vitamin E deficiency can cause hemolytic anemia. 22 13) Obesity – DD and treatment. A body weight >20% of the appropriate for height is considered obesity. Underwater weights are the most accurate measure of lean body mass. Triceps skinfold thickness is the most practical way to measure obesity in teenagers. The BMI (weight [kg] / height

squared [m2]) is used to assess obesity in adults, but correlates less with the percentage of body fat in children and adolescents. Background obesity is the most common nutritional disorder of children in western countries, and the prevalence increases between the elementary & high school yrs. The reduction of fitness is parallel to a rise in hrs of TV watching. Etiology: - Overeating (most common). - Genetic factors many obese children also have obese parents: - Leptin gene mutation defective control over appetite. - β3-receptor mutation in adipose tissue (involved in lipolysis & thermogenesis). - Syndromes e.g Pradder-Willi, Down, Turner, Frolich, PCOS - Environmental hypothalamic irradiation, encephalitis, trauma, - Secondary causes (rare) e.g hypothyroidism, Cushing’s syndrome Obesity risk factors include having obese parents & being an only child. Associated risks: 1. HTN, DM, CVA 2. Hypertriglyceridemia, gallstones, reflux esophagitis 3. Slipped

capital femoral epiphyses, degenerative arthritis 4. Higher surgical & obstetric risks 5. Pulmonary functional impairment, endocrine abnormalities 6. Psychosocial alienation, poor self-esteem, guilt, depression, altered body image Diagnosis a triceps skinfold measurement of >1 standard deviation above the mean (85th percentile) defines obesity. An adolescent with a BMI above the 95th percentile is considered overweight. The history of obese pt or those at risk for obesity should include onset of obesity, eating and exercise habits, amount of time spent in sedentary activities such as watching TV, previous successful and unsuccessful attempts at weight loss, and family history of obesity. It is also important to assess if the pt. is aware of or denies a weight problem When obesity is diagnosed, physical & lab. exams follow, which can exclude endocrine causes, such as hypothyroidism or Cushing disease. If an adolescent is healthy and has no delay of growth or sexual

maturation, an underlying endocrinologic, neurologic, or genetic cause is unlikely. Treatment the adolescent should be taught appropriate eating and exercise habits to maintain weight reduction yet meet nutritional needs for growth and development. Appetite-suppressing drugs, fasting, and bypass surgery have no role in the management of obese adolescents!!! 23 14) birth injury Although the forceps of delivery can cause physical injury to the infant, even severe molding of the head usually does not cause problems. The incidence of neonatal injury from difficult or traumatic delivery is decreasing. A traumatic delivery may occur when the mother has small pelvic measurements; the infant is large for gestational age (DM mothers), or when there is a breech or other abnormal malposition, especially in primigravidas. If there is fetal distress, the mother should be positioned on her L side and given O2. If fetal distress persists, labor induction, instrumental delivery of C/S is

indicated Head trauma: Caput succedaneum (scalp edema) – mild, as this area is forced against the uterine cervix. Subgaleal hemorrhage – greater trauma, characterized by a boggy feeling over the scalp. Cephalhematoma (subperiosteal hemorrhage) – can be diff. from more superficial bleeding because its sharply limited to the area overlying a single bone (the periosteum is adherent at the sutures). Cephalhematomas are commonly unilateral & parietal No treatment required Depressed skull fractures – uncommon, most result from forceps pressure. These fractures or other head trauma may be associated with subdural bleeding, SAH, contusion or laceration of the brain. X-ray confirms diagnosis; neurosurgical elevation is needed Cranial nerve trauma – the facial nerve is injured most often. Although usually thought to be from the forceps pressure, most trauma probably result from pressure on the nerve in utero. It occurs usually distal to its exit from the stylomastoid foramen and

results in facial asymmetry, especially during crying (mm on the injured side cannot move). Injuries can also occur in individual branches of VII, most often the mandibular. Another cause of facial asymmetry is mandibular asymmetry resulting from intrauterine pressure. These conditions usually resolve spontaneously by age 2 to 3 mo Brachial plexus injuries – may follow stretching caused by shoulder dystocia, breech extraction, or hyperabduction of the neck. Injuries may be due to simple stretching, hemorrhage within a nerve, tearing of nerve or root, or avulsion of roots with associated cervical cord injury. Associated traumatic injuries (fractures of the clavicle or humerus or subluxation of the shoulder or cervical spine) may occur. The site and type of nerve root injury determine the prognosis Injuries of the upper brachial plexus (C5–6) affect mm around the shoulder & elbow. Eg Erb’s palsy, causing adduction & internal rotation of the shoulder with pronation of the

forearm, ipsilateral paralysis of the diaphragm is common. Treatment includes protecting the shoulder from excessive motion by immobilizing the arm and preventing contractures by passive motion. Lesions of the lower plexus (C7-8 & T1) affect mm of the forearm & hand. Eg Klumpke’s palsy, causing paralysis of the hand & wrist, often with ipsilateral Horner syn 24 (ptosis, miosis, anhidrosis). Passive motion exercise is the only treatment needed Both conditions are not associated with sensory losses (which suggests a tear or avulsion). These conditions usually improve rapidly, but deficits can persist. Surgical exploration and repair may be helpful. When the entire brachial plexus is inured, the involved upper extremity cannot move & sensory loss is present. Ipsilateral pyramidal signs indicate associated spinal cord trauma do an MRI The prognosis is poor. Passive motion exercises can prevent contractures Spinal cord trauma – rare & variable, often with

hemorrhage; complete transection is very rare. Trauma usually occurs in breech deliveries after excess traction to the spine. Intracranial hemorrhage (see topic 6B) Fractures – midclavicular fracture, most common during birth, usually occurs with shoulder dystocia. The infant is irritable & does not move the arm Most heal rapidly (within 1mo) The fractures may cause brachial plexus injury or PTX. The humerus & femur may be fractured during difficult deliveries. Excellent prognosis Soft tissue injuries – edema and ecchymosis often follow injury, particularly the periorbital and facial tissues in face presentations and of the scrotum and labia during breech deliveries. 25 15) Otitis. Hearing deficits A. Otitis externa – there is often a history of recent water exposure or mechanical trauma It is usually caused by gram (–) rods (Proteus, Pseudomonas) or fungi (Aspergillus), which grow near excessive moisture. The most common underlying cause is loss of the protective

function of cerumen, leading to maceration of the underlying skin and bacterial invasion. Remove any purulent debris before starting treatment. Lack of, or wrong treatment may result in chronic otitis externa. In such cases treat the specific bacteria Symptoms: Treatment: • It presents with otalgia, often with pruritus & purulent discharge. • Erythema & edema of the ear canal, which may block vision of the eardrum. • Protect the ear from more moisture & avoid further mechanical injury. • Otic drops a mixture of neomycin, polymyxin B & hydrocortisone. B. Acute otitis media – very common, in newborns, E coli & S aureus cause suppurative otitis media. In older infants & children <14yr pneumococi, strep, staph & H influenzaeand S The bacteria reach via the auditory tube & causing accumulation of purulent material in the middle ear. It is usually a complication of a viral upper respiratory tract infection that causes auditory tube

edema, resulting in fluid & mucus accumulation secondary infection. Symptoms: • It starts with severe headache and accompanied with fever, A/N/V & diarrhea. • Theres otalgia, aural pressure, hearing loss & often fever. • If middle ear empyema is severe, the membrane bulges outward and may rupture. • Rupture is followed by sudden decrease in pain & onset of otorrhea. • With good therapy, the membrane may spontaneously heal, but if the perforation persists, chronic otitis media frequently occurs (usually caused by P aeruginosa, Proteus, S aureus & mixed anaerobic infections). • Complications mastoiditis, osteomyelitis, Bells palsy, sigmoid sinus thrombosis, brain abscess, epidural empyema & meningitis. Diagnosis: Otoscopy tympanic membrane erythema & opacity, decreased mobility, and later pin-sized hole in the PS quadrant with possible purulent discharge. Treatment: 1. Specific ATB (amoxicillin or erythromycin plus sulfonamide),

often with nasal decongestants. In resistant cases give cefaclor or amoxicillin-clavulanate 2. Tympanocentesis for bacterial & fungal cultures, esp in immunocompromised pt. & recurrences despite multiple courses of ATB 3. Surgical drainage (myringotomy) for severe otalgia or if complications occur 26 C. Acute mastoiditis – an extension of infection from the middle ear to the pneumatic system of the temporal bone. It appears as a change in the resolution of acute otitis media, with hearing loss, tenderness over the mastoid process & retroauricular swelling. It may complicate into a subperiosteal abscess or extend via the zygomatic process into the cheek & eyelid. Treatment ATB, unless the bone is involved. In such cases perform a mastoidectomy D. Secretory otitis media – an effusion resulting from incomplete resolution of acute otitis media or obstruction of the eustachian tube. Very common It may be sterile but usually contains bacteria Eustachian

obstruction may be due to nasopharyngeal inflammation, allergy, hypertrophic adenoids, or benign or malignant neoplasms. Normally, the middle ear is ventilated 3–4x/min as the eustachian tube opens during swallowing. If the patency of the tube is impaired, a relative negative pressure develops within the middle ear. At first, the tympanic membrane retracts mildly, with light reflex displacement. Then a transudate forms in the middle ear giving the tympanic membrane grey appearance and immobility. Conductive hearing loss occurs Treatment: ATB are given first. Systemic sympathomimetic agents open the eustachian tube by their vasoconstrictive effect. Myringotomy may be necessary for aspiration of the fluid and for allowing ventilation of the middle ear. Correction of the underlying condition in the nasopharynx is required (adenoidectomy, infection treatment, allergen avoidance). E. herpes zoster otitis – may affect cranial nn VII, VIII, V, etc as part of Ramsay-Hunt synd Symptoms:

Treatment: - Constitutional symptoms (fever, subfebrility, malaise, etc). - Erythema & vesicles on the auricle, meatus & tympanic membrane. - Severe neuralgic pain & regional lymphadenopathy. - Severe retrocochlear deafness (demonstrated by audiography). - Labirynthine symptoms vertigo, dizziness & nystagmus. Acyclovir & analgesics. F. Labirynthitis – may be caused by many other viruses It is usually self limited (influenza, adenovirus, coxsacki, mumps & measles). Theres risk of hair cell & spiral ggl destruction & degeneration of the organ of Corti. 27 Hearing defects ƒ Definition: hearing impairment applies to anything from the mild-to-profound range. Mild loss 20–40dB loss. Moderate 40–60dB loss and severe 60–80dB loss have impact on speech learning. Profound loss (deafness) – >80dB loss A deaf child is unable to process language through hearing, even with a hearing aid. ƒ Functional subtypes of hearing loss: -

Conductive hearing loss – theres interference in the transmission of sound from the ext. auditory canal to the inner ear, causes: • Otitis media (most common). • Otosclerosis. • Trauma. • Foreign bodies. • Ext. canal or ossicles abnormalities • Cerumen Most conductive loss can be corrected by medical treatment or surgery. - Sensorineural hearing loss – is secondary to damage to the inner ear or auditory nerve. Causes include: • Congenital infection. • Meningitis or other postnatal infection. • Rubella or mumps. • Prematurity. • Perinatal asphyxia or trauma. • Kernicterus. • Ototoxic drugs. • Genetic causes (e.g Alport’s syndrome) • Environmental (e.g noise–induced) This type is nearly always irreversible, and treatment is usually by amplification. ƒ A mixed-type – involves both sensorineural & conductive loss simultaneously. Assessment – check of abnormalities around the pinna, ear canal or tympanic

membrane. Abnormalities that may suggest genetic explanation should be identified (e.g renal disease) Audiologic evaluations are useful, including: o Behavioral & observational hearing assessments. o Pure tone audiometry. o Auditory brainstorm response. o Otoacustic emissions. These children require special education to allow maximal development. 28 16) Precocious puberty – DD and treatment. If there are any secondary sex characteristics present in girls before age 7 and in boys before age 9, it is considered precocious puberty. True central (gonadotropin-dependent) precocious puberty is more common in girls. In girls, there is rarely underlying CNS disease ( idiopathic) In boys with PP, there is a higher incidence of CNS pathology (tumors). Premature thelarche – refers to frequent isolated breast development in very young girls (12-24 months). Caused by estrogen either from the prepubertal ovary or increased sensitivity to low levels estrogen. Gonadotropins and serum

estrogen are normal, growth acceleration and advanced skeletal maturation is not present. Breast development does not progress, other signs of puberty are not present. The condition is nonprogressive and benign Premature adrenarche – describes the early appearance of sexual hair (girls <7, boys <9). This benign condition is caused by early maturation of adrenal androgen secretion. Levels of adrenal androgens are as in puberty. Bone age is slightly advanced These children must be evaluated for other causes of increased androgen production, such as CAH. Precocious isosexual puberty – divided to 2 types: gonadotropin-dependent & independent. 1. Gonadotropin-dependent PP – may be considered as normal puberty beginning at an abnormally early stage. Etiology: either idiopathic (most girls) or d/t CNS disturbance (tumors: glioma, pinealoma, hydrocephalus, head injury, congenital malformation or infection). Clinical features: theres progressive development of secondary sex

characteristics, together with a growth spurt. Since sex steroids stimulate growth while promoting epiphyseal fusion, PP causes tall stature in childhood, premature epiphyseal closure & adult short stature. Diagnosis: based on growth acceleration, advanced bone age, and pubertal levels of gonadotropins and estrogen or testosterone. Elevated gonadotropins after an IV infusion of GnRH is indicative Treatment: long-acting GnRH analogues that inhibit gonadotropin release. 2. Gonadotropin-independent PP – rare (eg Leydig cell tumors) Levels of gonadotropins are low and there is no increase in gonadotropins after GnRH. 3. Ectopic hCG production – by neoplasms may stimulate Leydig cells growth & testosterone hypersecretion in boys. Tumors include germ cell tumors of the CNS, hepatoblastoma, pinealoma, and retroperitoneal carcinoma. LH & FSH levels are suppressed, testosterone is high. Elevated hCG confirms diagnosis 4. Hypothyroidism – may be associated with PP Levels of

both gonadotropins & PRL are elevated. Galactorrhea may be present Growth is arrested and bone age is delayed Treatment of hypothyroidism usually stops progression of sexual development. 29 17) BRAIN AND NEUROLOGIC DISORDERS OF THE NEWBORN INFANT These disorders generally result in abnormalities of tone, strength, and state of consciousness. Asphyxial brain injury – the most common neurologic abnormality in the neonatal period. Pathophysiology: during severe perinatal asphyxia, cerebral hypoxia & ischemia occur, initially in the cortex and eventually in the cerebellum and brain stem. Risk factors: ƒ Extremes maternal age (<20 & >35). ƒ Meconium-stained amniotic fluid. ƒ Placental abruption & previa. ƒ Fetal bradycardia. ƒ Preeclampsia, multiple gestation ƒ Prolonged fetal ROM. ƒ Preterm & postterm gestation. ƒ Maternal diabetes. Clinical features: the respiratory effort stops suddenly, causing infant apnea & gasps, and if

resuscitation is not initiated, there is progression to terminal apnea. ↓↓ blood oxygenation results in respiratory acidosis. Hypoxia causes a rapid decrease in HR BP increases initially but later falls to hypotension. The fall in BP decreases the flow of blood to the organs and causes tissue damage. Postnatal symptoms of asphyxia vary with the degree of asphyxia Some infants have multiorgan involvement, others have only one or two organs involved. - Brain infants with mild asphyxia are depressed initially & followed by hyperalertness, which resolves in 2 days (excellent prognosis). Moderate asphyxia may cause generalized seizures, resolving in a few days (variable prognosis). Severe asphyxia is associated with coma, seizures, cerebral edema, and intracranial hemorrhage (poor long-term survival). - Heart: severe asphyxia may result in hypoxic cardiomyopathy. There is hypotension, poor contractility, cardiomegaly, and CHF. - Lung: respiratory distress can occur due to a

delayed drop of pulmonary vascular resistance. - GI: it is associated with poor GI motility or ileus. The hypoxia also predisposes to necrotizing enterocolitis. - Blood: hypoxia depresses bone marrow function and initiates an intravascular coagulopathy, which results in thrombocytopenia, prolonged PT, PTT & bleeding time. Therapy resuscitation. Ventilation with oxygen, restoration of adequate cardiac output Prognosis variable and difficult to predict. Mild cases are associated with good outcome, severe cases with significant morbidity and mortality. Pericranial & intracranial hemorrhage – can be classified according to the location: 1. Subaponeurotic (subgaleal) hemorrhage – a blood collection beneath the thin tendineous sheet covering the skull & above the periosteum. This is a large potential space crossing cranial suture lines. It follows head trauma at birth The scalp and head are firm, there may be scalp discoloration and a large amount of blood loss. 30

2. Cephalhematoma – a subperiostal blood collection, thus does not cross sutures It comes after head trauma and is self–limited. It is sometimes confused with caput succedaneum, which is localized scalp edema secondary to compression. This condition has boggy consistency and crosses cranial sutures. 3. Subarachnoid hemorrhage (SAH) – may occur after a normal or traumatic delivery Bleeding is self–limited, and symptoms (irritability, seizures) resolve in a few days. 4. Subdural hemorrhage – occurs with birth trauma Blood can accumulate and cause focal neurologic deficits (pressure on brain). Drainage is necessary if symptoms are severe 5. Intraventricular hemorrhage – seen in preterm infants frequently after asphyxia Large hemorrhage is associated with permanent functional impairment and hydrocephalus. Diagnosis: cranial US and CT can detect blood and are useful. If in doubt, the CSF can be examined for RBC; it usually contains gross blood. An EEG helps in establishing the

prognosis if the infants survives. The dg of SAH should be suspected in any infant with apnea, seizures, lethargy, or an abnormal neurologic finding and is confirmed by CT or CSF examination. In subdural hemorrhage, CT or MRI are used. Prognosis: generally good except for large intraventricular hemorrhage, for which it is poor. Treatment: mostly supportive unless a hematologic abnormality contributed to the bleeding. vit-K, platelets or clotting factors may be given if indicated. Seizures (See topic 115) Periventricular leucomalacia (PVL) – referring to cystic lesions in the cerebral cortex, lateral to the lat. ventricles It is a problem of preterm infants The cause is probably cerebral ischemia in the perinatal period. PVL correlates with long-term abnormalities in neurologic function The degree of dysfunction depends on the size, number, and location of the cysts. The development of PVL from edema to cysts is easily identified with US. Hydrocephalus – a CSF collection due to

imbalanced production or absorption: High–pressure (obstructive) hydrocephalus – d/t defective drainage in the ventricular system. This may be part of congenital anomalies, or after intraventricular hemorrhage or meningitis. Low– pressure (communicating) hydrocephalus – defective resorption, while flow is normal. Seen after intracranial hemorrhage and in some malformations. No therapy needed Hypotonia – characterized by diminished skeletal mm tone. Its the most common neurologic motor disorder of the neonatal period. Common causes include: ƒ Muscular or myotonic dystrophy (mm defects) ƒ Hypothyroidism (metabolic defect). ƒ Werdnig–Hoffmann disease (s. cord defect) ƒ Asphyxia (brain defect). ƒ Myasthenia gravis (NM junction defect). 31 Myelomeningocele – the most common congenital anomaly of the nervous system. It results from failure of the neural tube closure. The risk can be diminished by folate admin to the mother before conception & during the

1st trimester. Clinical features depend on the location Prenatal screening of for α-fetoprotein & US allow early diagnosis. Surgical closure prevents infection, but it may cause hydrocephalus. Prognosis depends on the location, presence or absence of other associated anomalies, infection, and the effectiveness of physical therapy and other supportive care. 32 18) BRONCHITIS An inflammation of the tracheobronchial tree, generally divided into acute and chronic. Acute bronchitis – generally self-limited with eventual complete healing and return of function. Although commonly mild, it may be serious in debilitated pt. and pt with chronic lung or heart disease. Airflow obstruction is a common consequence, and pneumonia is a critical complication Etiology: it is most prevalent in winter, generally as part of an acute URI. It may develop after a common cold or other viral infection of the nasopharynx, throat & tracheobronchial tree, often with secondary bacterial infection.

Causative viruses include adeno, corona, rhino, coxsackie-A21, influenza A & B, parainfluenza, RSV, rubella and measles. Mycoplasma pneumoniae, Bordetella pertussis, and Chlamydia pneumoniae also cause infectious bronchitis, commonly in young adults. Malnutrition and exposure to air pollutants are predisposing factors. Bronchitis often recurs in pt with chronic pulmonary diseases that impair bronchial clearance and may recur in those with chronic sinusitis, bronchiectasis, bronchopulmonary allergy, or COPD and in children with hypertrophic tonsils and adenoids. Acute irritative bronchitis may be caused by dusts, strong acid fumes, ammonia, certain volatile organic solvents, chlorine, bromine, ozone, nitrogen dioxide, or tobacco and other smoke. Symptoms: ƒ Often preceded by URI symptoms: coryza (rhinitis), slight fever, malaise, chilliness, back pain, muscle pain & sore throat. ƒ Onset of distressing cough signals onset of bronchitis. It is initially dry and nonproductive

and later becomes mucopurulent. ƒ Frank purulent sputum suggests superimposed bacterial infection. ƒ Some pt. have burning substernal chest pain, which is aggravated by coughing ƒ Fever for a few days. Persistent fever suggests complicating pneumonia ƒ Dyspnea may occur secondary to airway obstruction. ƒ Auscultation scattered crackles or moist rales at the bases, and wheezing. ƒ Serious complications (acute respiratory failure) occur only in pt. with an underlying chronic respiratory disorder. Diagnosis: based on symptoms and signs. X-ray is indicated in severe cases Treatment: bed rest, oral fluids, antipyretics, analgesics. ATB (amoxicillin) is indicated if there is concomitant COPD, when the sputum is purulent, or when high fever persists. When the symptoms persist, smear & sputum cultures should be done specific ATB. For M pneumoniae & Chlamydia give erythromycin. Cough-suppressing agents control exhausting cough and mucolytics help in sputum

evacuation. 33 Chronic bronchitis – characterized by chronic productive cough for at least 3 mo in each of 2 successive yrs for which other causes, such as infection with TB, lung cc or CHF, were excluded. Most children with chronic cough are hyperreactive to some inhalant, such as cigarette smoke, pollens, or pollutants. The possibility of cough-variant asthma should be considered Diagnosis depends on exclusion of other causes, bronchoscopy, X–ray, or CT, but all these diagnostic methods are rarely performed. Chronic cough with airway hyperreactivity responds to theophylline. If infection is present, ATB are given. If there is no response, investigation for immunodeficiencies and ciliary function are performed. 34 19) Amenorrhea. sexual infantilism (primary and secondary testicular failure) Amenorrhea May be primary or secondary. Always consider the possibility of pregnancy first (urine hCG test) Primary amenorrhea – not reaching menarche by age 16. May be caused by

many factors including: 1. Severe malnutrition, vigorous exertion, or chronic disease 2. Error in embryonic sexual differentiation 50% are due to syndrome of gonadal dysgenesis (Turner, topic 9C), 1/3 to müllerian dysgenesis & 1/6 to errors in genital development (topic 8A). 3. Anatomic abnormality eg absent uterus, imperforate hymen 4. Most factors causing delayed puberty also cause primary amenorrhea 5. Primary ovarian failure due to autoimmunity, postoophorectomy, resistant ovary syn) 6. Lack of gonadotropin stimulation of normal ovaries d/t: a. Hypothalamic &/or pituitary trauma b. Congenital anomalies or idiopathic hypopituitarism, c. Tumors, infection, inflammation d. Prader-Willi sy, functional gonadotropin deficiency e. Anorexia & bulimia nervosa, exercise, psychologic, constitutional delayed puberty 7. Excessive androgen production may be 2o to a virilizing tumor or adrenal hyperplasia 8. Testicular feminization or true hermaphroditism Anatomic abnormalities

– if the hymen is imperforate, lower abdominal swelling may be felt. Rectal examination may reveal distended vagina. The hymen should be perforated to allow menstrual bleeding to escape. If the vagina is patent, examination for a uterine cervix should be performed. Absence of a cervix (& uterus) suggests possible chromosomal abnormality The pt. with a Y chromosome and no masculinization usually has androgen insensitivity or complete testicular feminization. Müllerian dysgenesis may be accompanied by bony abnormalities and renal anomalies. US helps diagnosis Tissues inappropriate to the phenotypic sex of the individual (e.g testes in testicular feminization, appearing as a girl) should be removed surgically Testicular feminization syndrome X–linked, in which affected genotypic males are phenotypic females. Etiology: In the complete forms, either androgen receptors are absent in target cells or postreceptor activity is defective. Ineffective androgen action during embryogenesis

results in lack of male differentiation of the wolffian ducts and the external genitalia. Since testes are present, müllerian inhibiting factor is secreted normally and the müllerian ducts regress. Thus, these pt lack fallopian tubes, uterus, and the upper portion of vagina. The external genitalia and the urogenital sinus are female in development. 35 Clinical features: Since these individuals appear as normal females, the diagnosis is not suspected unless other family member is affected. No problem is suspected until pubertal dvelopmebnt occurs, but no menses follow. On physical examination, the vagina is short and no cervix is present These girls have little sexual hair. A chromosomal analysis will show 46, XY karyotype Incomplete forms result in pt. with various degrees of masculinization Plasma levels of LH are high, FSH are normal. Testosterone plasma levels are high Treatment: There is an increased risk of gonadal neoplasia, therefore, gonadectomy should be performed after

pubertal feminization. Vaginal dilation by the pt or her sexual partner is usually sufficient and vaginal surgery is rarely necessary. Estrogen therapy is required 2. Secondary amenorrhea It is defined as the absence of menses for at least 6 months after menarche has occurred. Etiology: The most common causes in young girls are those affecting hypothalamic-pituitary function. Some pt have high prolactine levels Since prolactine secretion is episodic, a single determination may not reveal hyperprolactinemia– measurement should be repeated. Many pt with hyperprolactinemia do not have Galactorrhea and vice versa. Amenorrhea may result from nutritional and/or psychogenic factors. Anorexia nervosa may be associated with primary or secondary amenorrhea. Weight reduction associated w3ith loss of 30% of body fat will result in amenorrhea. Women undertaking vigorous exercise (long distance running, ballet dancers) commonly have amenorrhea. In addition to hypothalamic–pituitary dysfunction,

ovarian failure may be cause of secondary amenorrhea. Destruction of the ovary may be due to autoimmune oophoritis, irradiation or chemotherapy, trauma, or systemic disease, such as galactosemia and cerebellar ataxia. Other causes include ovarian tumors and polycystic ovary sy that are associated with excessive androgen production. Diagnosis: see picture Galactorrhea – amenorrhea syndrome It means persistent discharge of milk from the breast in the absence of labor or beyond postpartum in a non–breast feeding mother. When accompanied by amenorrhea and hyperprolactinemia, there is chronic anovulation. Etiology: Galactorrhea occurs in many disorders, most commonly in pt. with normal menstrual periods and no associated endocrine disease. Often the galactorrhea represents a residuum of postpartum lactation. Prolactine levels in these cases are usually normal In these cases, women have breast tissue that is extremely sensitive to normal levels of prolactine. Galactorrhea– amenorrhea

is associated with high prolactin which is often secondary to prolactine–secreting 36 adenomata (prolactinomas) of the pituitary gland. When GnRH is administered to women with hyperprolactinemia from prolactinomas, there is prompt secretion of LH and FSH; therefore, the functional defect lies in the hypothalamus. Diagnosis: Persistent hyperprolactinemia, typical radiographic, CT, or MRI changes in the sella turcica, visual field abnormalities may be present. Treatment: Transsphenoidal surgery (microadenomas) or bromocriptine (a dopamine agonist – suppressing prolactine secretion) for surgically not treatable macroadenomas. b) Primary and secondary testicular failure It is gonadal failure either due to intrinsic defects (hypergonadotropic hypogonadism) or secondary to a lack of gonadotropins (hypogonadotropic hypogonadism). Both of the conditions may be primary or secondary. 1. Hypergonadotropic hypogonadism Any condition in which testosterone production by the testis is

impaired results in increased production of GnRH by the hypothalamus and greater production and secretion of gonadotropin. Primary causes include Klinefelter syndrome, gonadal dysgenesis (topic 9C), Noonan sy, anorchia, and abnormalities in testosterone synthesis. Secondary causes include: torsion of the testes, castration, Leydig cell failure, seminiferous tubule failure, and orchitis. Noonan syndrome This syndrome is characterized by hypogonadism, webbed neck, ptosis, congenital heart disease, and short stature. The incidence is 1:8000 It is familial The chromosomal constitution is normal and the gonadal differentiation is appropriate for the chromosomal sex. Clinical features: Males are usually cryptorchid with hypoplastic testes. Androgen deficiency is common. These pt have a characteristic triangular facies with narrow maxilla and micrognathia Other clinical features are similar to those found in Turner sy (low–set ears and posterior hairline, cubitus valgus, lymphedema,

clinodactyly, shield chest, widely–spaced hypoplastic nipples). Renal anomalies and impaired mental development are common. Diagnosis: There are no chromosomal abnormalities! Those male pt. manifesting features of Turner sy plus chromosomal abnormalities, such as 45,XO/46,XY mosaicism, might labeled as having male Turner syndrome. Treatment: Androgen hormone replacement. Enzymatic defects in testosterone and/or DHT biosynthesis These disorders include any defect in one or more of the enzymes involved in the conversion of cholesterol to testosterone and DHT. Defects in enzymes involved in the early steps of testosterone 37 synthesis are discussed in topic 3A. Those specific for androgen synthesis include 17, 20 desmolase deficiency and 17–hydroxysteroid dehydrogenase deficiency. In addition, a defect in 5alfa– reductase severely impairs conversion of testosterone to DHT, which is necessary for masculinization of the external genitalia. These conditions have an autosomal

recessive inheritance and are very rare. Clinical features: There is incomplete masculinization. The defect may be so severe that a female sex is apparent at birth. Some pt may have a gender change at puberty from female to male Hormonal analysis revealing accumulation of steroids proximal to the enzymatic defect is diagnostic. Treatment: Androgen replacement therapy for pt. brought up as males, gonadectomy and estrogen replacement for those brought up as females. 2. Hypogonadotropic hypogonadism A deficiency of gonadotropins may be due to pituitary failure (hypopituitarism, pituitary tumors, GH deficiency) or to hypothalamic dysfunction (Kallmann sy, suprasellar tumors, Prader–Willi sy). Constitutional delayed puberty may also be responsible. Hypopituitarism Deficiencies of FSH and LH can occur as part of idiopathic hypopituitarism or secondary to pituitary or hypothalamic pathology (see above). Prolactinomas are rare in males Kallmann syndrome It is a disorder of isolated

gonadotropin deficiency associated with anosmia. Micropenis and/or cryptorchidism are common. Treatment: androgens, GnRH in pulses to induce fertility 38 20) GI DISEASES OF THE NEWBORN INFANTS. INTESTINAL ATRESIA Congenital anomalies Newborns with congenital bowel obstruction present with distension and emesis at birth or within 1 or 2 days. Immediate management includes bowel decompression by continuous nasogastric suction to prevent emesis, which can lead to aspiration pneumonia. Also vital are maintenance of body temperature, prevention of hypoglycemia, and treatment of acidosis, and infections, so that the infant is in optimal condition for surgery. An infant with one congenital anomaly should be evaluated for malformations of other organ systems (CNS, heart, kidneys). High alimentary tract obstruction – high (esophageal, gastric, duodenal, jejunal) obstruction should be considered when hydramnios (excessive volume of amniotic fluid) is diagnosed. Hydramnios occurs when

the fetus cannot swallow and absorb fluid or when fetal emesis is present. A nasogastric tube should be passed into the newborn’s stomach immediately after delivery, the finding of fluid, especially if bile–stained, supports the diagnosis of upper GI obstruction, whereas inability to pass the tube into the stomach suggests esophageal atresia. 1. Esophageal atresia – usually associated with tracheoesophageal fistula Characteristic signs are excessive secretions, coughing and cyanosis after feeding attempts, & aspiration pneumonia. A type III lesion soon leads to abdominal distension because as the infant cries, air from the trachea is forced through the fistula into the lower esophagus and stomach. Diagnosis is suggested by inability to pass a nasogastric tube into the stomach; a radio-opaque catheter will determine the location of the atresia on X–ray. Treatment consists of no oral feeding and continuous suction to prevent aspiration of swallowed saliva. When the infant’s

condition is stable, surgical repair of the esophageal atresia and closure of the tracheoesophageal fistula can be performed. 2. Diaphragmatic hernia – a protrusion of abdominal contents into the thorax through a defect in the diaphragm. Diaphragmatic hernia usually occurs in the posterolateral portion of the diaphragm (Bochdalek’s hernia) and is on the left side in most of the cases. If the hernia is large, the lung on the affected side will be hypoplastic. Respiratory distress occurs after delivery Bowel sounds may be heard over the involved hemithorax. The hypoplastic lung causes pulmonary hypertension with decreased pulmonary blood flow; RL shunting occurs, and severe hypoxemia results. Persistent pulmonary hypertension is the major cause of death among infants with congenital diaphragmatic hernia. 39 Diagnosis: X–ray (air–filled loops of bowel filling the hemithorax). Treatment: The infant should be immediately intubated and ventilated. Continuous suction prevents

swallowed air from progressing through the GI tract and causing further lung compression. Surgery is required to place the bowel in the abdomen and to close the diaphragmatic defect. Pulmonary vessel constriction may be reduced by alkalinization with the IV sodium bicarbonate. Inhaled nitric oxide helps dilating the pulmonary arteries. 3. Hypertrophic pyloric stenosis – obstruction of the pyloric lumen due to mm hypertrophy It usually develops during the initial 4th to 6th week of life. Forceful projectile vomiting of feedings without bile is usually the first sign. Delayed diagnosis may lead to repeated vomiting, dehydration, failure to gain weight, and hypochloremic metabolic alkalosis. Diagnosis: palpation of firm, movable, pyloric mass or by ultrasonography. Treatment: Surgery. 4. Duodenal obstruction – may result from atresia, stenosis, or pressure from extrinsic mass After the ileum, duodenum is the most common site of primary bowel atresia. Duodenal atresia is more common in

infants with Down syndrome. Choledochal cyst or annular pancreas may also cause obstruction by extrinsic pressure. Because the obstruction is high, there is usually a history of maternal hydramnios and of forceful bilious vomiting after the first feedings. X-rays show one large air-filled bubble (stomach) and a second bubble (dilated duodenum), and no air is seen distal to the blockage. When duodenal obstruction is suspected, the infant should not be fed. Continuous suction decompresses the stomach and prevents vomiting and aspiration of vomitus. Surgery is performed to correct the obstruction. Distal small–bowel and large–bowel obstruction – usually there is no history of maternal hydramnios because much of the swallowed amniotic fluid has been absorbed from the fetal bowel proximal to the obstruction. The first feedings are tolerated, but on day 2 abdominal distention develops, often accompanied by bilious or fecal vomiting. There are no passed stools The preoperative

management includes giving nothing by mouth, placing nasogastric tube to prevent further bowel distention or aspiration of vomitus, correcting fluid and electrolyte disturbances, and taking a plain X–ray and contrast enema to show the anatomy. 1. Meconium plug syndrome – a bowel obstruction caused by thick meconium, which forms a cast of the colon. It is more common in infants of diabetic mothers Thick, inspissated, rubbery meconium forms a plug and can cause complete obstruction, with failure to pass stools, distention, and vomiting. Contrast enema shows the meconium plug, separates it from the bowel wall and expels it. 40 2. Meconium ileus – a bowel obstruction caused by meconium in the terminal ileum that is extremely tenacious and adheres to the ileal mucosa. It is almost always an early sign of CF Distal to the obstruction, the colon is narrow in Ø. This relatively empty colon of small caliber is called microcolon. Distended small bowel loops can sometimes be palpated

These loops, distended with thick, tenacious meconium, may twist to form a volvulus in utero. If the bowel then loses its vascular supply and infarcts, sterile meconium peritonitis will result. The infarcted bowel may resolve leaving an area of bowel atresia, or it may be walled off as a large fluidcontaining cyst. Diagnosis is supported by the presence of undigested protein in meconium. A heavy white precipitate indicates undigested albumin. A positive sweat test confirms the diagnosis of CF If the meconium ileus is diagnosed, the obstruction may be relieved in uncomplicated cases by an enema with a dilute contrast medium. If the enema does not help, laparotomy is done 3. Bowel atresia – occurs most frequently in the ileum Ileal atresia usually presents late during day 1 or on day 2. Abdominal distention progressively increases, the infant fails to pass stools, and feedings are regurgitated. Diagnostic approach and preoperative management are similar to those above. Prognosis is

good At surgery, the entire bowel is inspected to exclude other areas of atresia. The atretic lesions are resected and anastomosis is made 4. Anomalies of bowel position due to malrotation – during embryonic development, the primary bowel protrudes from the abdominal cavity. As it returns to the abdomen, the large bowel normally rotates counterclockwise, with the cecum coming to rest in the right lower quadrant. Incomplete rotation in which cecum ends up elsewhere may cause bowel obstruction from retroperitoneal bands that stretch across the duodenum or from a volvulus of the small bowel that, lacking its normal peritoneal attachment, twists on its narrow stalk-like mesentery. Clinical presentation initially is the same as with other forms of bowel obstruction. The infant vomits bile and may appear acutely ill. Abdominal X-ray showing bowel gas distal to the duodenum suggest a midgut volvulus. Volvulus is a surgical emergency because the involved bowel may become gangrenous if the

obstruction is not quickly relieved. a. Hirschsprung’s disease (Congenital Megacolon) – a congenital abnormality of innervation of the lower bowel, usually limited to the colon, resulting in partial or total functional obstruction. It is caused by congenital absence of Meissner’s and Auerbach’s autonomic plexus in the bowel wall. Peristalsis in the involved segment is absent or abnormal, resulting in continuous smooth muscle spasm and partial or complete obstruction with accumulation of bowel contents and massive dilation of the more proximal, normally innervated bowel. The infant presents with obstipation, distention, and finally vomiting, as in other forms of 41 distal bowel obstruction. It is important to make the correct diagnosis as early in infancy as possible. The longer the disease goes untreated, the greater the chance of developing toxic megacolon, which may be fulminant and fatal. Diagnosis: A barium enema shows a transition in diameter between the dilated colon

proximal to the obstruction and the narrow, distal segment, which lacks normal innervation. Rectal biopsy confirms the diagnosis. Treatment: Colostomy proximal to the aganglionic segment. Resection of the entire aganglionic portion of the colon and definitive repair can be done. After definitive repair, the prognosis is good; most infants will achieve control of bowel movements. b. Toxic enterocolitis (toxic megacolon) – may result from the chronic obstipation of Hirschsprung’s disease, with overgrowth of bacteria in the dilated bowel resulting in production of bacterial toxins. Consequent fulminant water losses into the bowel lumen result in massive fluid and electrolyte losses and may rapidly lead to dehydration and death. Fluid replacement and antibiotics are important but may not prevent death unless the obstruction is quickly relieved by performing a colostomy. c. Anal atresia – obvious on routine physical exam because the anus is not patent Should the diagnosis be missed

and the infant be fed, signs of distal bowel obstruction soon develop. A fistula often extends from the anal pouch to the perineum or the urethra in males and to the vagina in females. The urine should be filtered and examined for meconium, indicating the presence of a fistula to the urinary tract. Plain X–rays and fistulograms can define the level of the lesion. A cutaneous fistula generally indicates low atresia; in such cases, definitive repair via the perineum is possible. If no perineal fistula exists, a high lesion is likely, and definitive repair is usually deferred until the infant is older and the structures to be repaired are larger. Until then, a colostomy is done to relieve the obstruction Defects in abdominal wall closure: 1. Omphalocele – a protrusion of variable amounts of abdominal viscera from a midline defect at the base of the umbilicus. The herniated viscera are covered by a thin membrane and may be small (only a few loops of bowel) or may contain most of the

abdominal viscera. Immediate dangers are drying of the viscera, hypothermia due to heat loss and dehydration from water evaporation, and infection of the peritoneal surfaces. These infants have higher incidence of other anomalies (bowel atresia, cardiac & renal anomalies). At delivery, the exposed viscera should be immediately covered with sterile saline-soaked sponges and then with an occlusive dressing to maintain sterility and to prevent evaporation. Primary closure is performed then. 42 2. Gastroschisis – a protrusion of the abdominal viscera through an abdominal wall defect, usually to the right of the umbilicus. Here, there is no membranous covering over the bowel, which is markedly edematous and erythematous and is often enclosed in a fibrin mat. These findings indicate long-standing inflammation. It is not commonly associated with other anomalies It takes several weeks before GI function recovers after surgical repair. Acquired disorders Neonatal necrotizing

enterocolitis Incidence: Affected infants are most commonly premature, asphyxiated, and suffering from other medical problems. Etiology: Bowel ischemia secondary to perinatal asphyxia is regarded as the cause of bowel wall injury. Human milk or formula then provides the substrate for bacterial overgrowth Bacterial invasion, often with gas production, leads to bowel wall necrosis and perforation. Other predisposing factors include systemic hypotension, patent ductus arteriosus, umbilical catheter, exchange transfusion, and systemic ATB. Clinical features: Signs and symptoms are noted during the first 2 weeks, shortly after enteric feeding has begun. These include: ƒ Abdominal distention, bloody stool & bile stained gastric residue. ƒ Apnea, lethargy & hypotension. ƒ Abdominal wall discoloration, unstable temperature. ƒ Hyperglycemia, metabolic acidosis. ƒ Lab leukocytosis or neutropenia, thrombocytopenia. ƒ X–ray shows dilated bowel loops, gas in the bowel

wall, perforation (abdominal air). Course: Most infants have feeding intolerance, abdominal distention, occult blood in the stool, and dilated bowel loops on X–ray. These infants improve with therapy The other group has severe symptoms (abdominal tenderness, hypotension, DIC, and sepsis. Perforation is common Therapy: No enteric feeding, gastric drainage, and IV fluid. Systemic ATB (according to cultures), surgical resection of the necrotic segment in severe cases. Mortality (highest in the most premature infants) is 30%. 43 21) TUMORS OF THE SOFT TISSUES. BONE TUMORS Soft tissue tumors 1. Rhabdomyosarcoma – arises from the embryonal mesenchyme from which skeletal muscle originates. It can be divided into two categories => tumors of favorable histology occur in 70% and are embryonal; tumors of unfavorable histology occur in the remaining 30% and are of various subtypes. Alveolar type is the most common The most common sites are: - Head & neck (38%) – orbital tumors

have a rapid onset of symptoms due to their location in the bony orbit. They cause exophthalmos, ptosis, and eyelid swelling Nasopharyngeal and middle ear tumors cause discharge, polypoid mass, airway obstruction, chronic otitis media, and spread to the adjacent meninges, causing increased intracranial pressure and cranial nerve palsies. Neck tumors cause mass, pain, and cervical and brachia plexus palsy - Genitourinary tract (21%) – bladder and prostate tumors cause urinary obstruction and hematuria. Vaginal and uterine tumors cause vaginal bleeding and polypoid tumor (sarcoma botryoides, or “cluster of grapes”) extruding from the vaginal orifice. - Extremity tumors (18%) – involve solid masses on the upper or lower extremities. - Miscellaneous presentations (21%) – present as a mass or obstruction in the trunk, retroperitoneum, paratesticular region, perianal region, gastrointestinal and biliary tracts. Therapy: Surgery – Complete excision is indicated if possible.

If deformation would result, chemotherapy, radiotherapy, or both are used as alternatives to surgery or to shrink the tumor to permit easier removal. Radiotherapy – It is used for local tumors and metastases. Chemotherapy – It is used as an adjuvant to other therapy and to destroy local and metastatic tumors. The intensity of the therapy depends on the location, stage, and histology Prognosis – has dramatically improved during last years. Children with an orbital tumor have an excellent survival rate (90%). Genitourinary tract tumors have 75% survival rate Poor prognosis is still with extremity tumors, retroperitoneal tumors, and metastatic disease. 2. Rare soft–tissue sarcomas: - Fibrosarcomas – often arise in the distal portion of extremities. They are more common in young children for whom the prognosis is excellent with surgical treatment. - Liposarcomas – are also more common in young children and have an excellent prognosis with surgery - Synovial sarcomas –

most commonly develop around the knee joint. 44 - Primitive neuroectodermal tumors – of the PNS usually develop in the soft tissues but can sometimes arise in a bone. They are closely related to Ewing’s sarcoma The presentation often involves masses on the chest or extremities. Unlike other rare soft-tissue sarcomas, these make more metastases and require aggressive systemic chemo. Bone tumors – primary malignant account for 4% of childhood cancer. Two highly malignant: 1. Osteogenic sarcoma - Epidemiology: It is the most common primary malignant bone tumor seen in pediatrics. It occurs mainly in adolescence and is twice as common in males as in females. - Clinical features: Pain and swelling are common. Half of all cases occur near the knee (distal femur, proximal tibia). It also occurs in proximal humerus and proximal femur Metastases are mainly to the lungs. X-ray may show destructive lesions, periosteal reaction - Therapy: Surgery plays an important role (limb

salvage procedures are common). Surgery occurs after an initial period of chemotherapy that aims to kill as much tumor as possible. Amputation is performed when limb salvage is not possible. Chemotherapy is also important (improves disease–free survival). - Prognosis: With aggressive adjuvant chemotherapy, survival is 50%. 2. Ewing’s sarcoma – an undifferentiated sarcoma of uncertain histogenesis, which arises primarily in bone. Occasionally, it arises in the soft tissues of the extremities and paravertebral region (extraosseous Ewing’s sarcoma). - Epidemiology: It is seen in adolescents and is 1.5 times more common in males - Clinical features: Pain and localized swelling are the most common. Sometimes there is also fever, leukocytosis, elevated FWD. Common sites are mid– to proximal femur and pelvic bones, other long bones, the ribs, and the scapula. X–ray shows a destructive lesion associated with periostal elevation or a soft–tissue mass. Metastases should be

suspected (lungs, other bones, and bone marrow). - Therapy: Radiotherapy is common, surgery is done instead of radiotherapy, especially if it is limb salvaging. Chemotherapy reduces primary tumor mass, prevents metastases, and is given to pt. with metastases at diagnosis - Prognosis: It is good for pt. with distal extremity nonmetastatic tumors It is poor for pt with metastases at diagnosis, tumors of the pelvic bones, and proximal femur. 45 22) ANOMALIES OF EXTERNAL GENITAL ORGANS – DD AND MANAGEMENT a) Ambiguous genitalia Physical exam: Presence of at least 1 testis suggests the presence of a Y chromosome. Absence of palpable testes implies that the child is a girl with salt–losing congenital adrenal hyperplasia (CAH –see topic 3A) until proven otherwise!!! Associated abnormalities in the genitourinary system suggest a dysmorphic syndrome. Karyotyping should be done in all cases. The result of karyotyping separates cases into genetic males (XY) with incomplete

virilization and genetic females (XX) with excessive virilization, and chromosomal aberrations fitting neither genotype. Internal anatomy should be investigated by pelvic US. Evaluation of incomplete virilization in a genetic male should include measurements of serum LH, testosterone (T) and dihydrotestosterone (DHT). - Normal testicular tissue (seen during operation in the abdomen) with reduced Wolffian structures associated with high LH, normal circulating T and DHT indicates a receptor defect (androgen insensitivity – see topic 7A) - Combination of low LH, T, and DHT suggests a hypothalamo– pituitary defect. These defects do not cause ambiguity but there is micropenis. - High LH with low T and DHT suggest an inborn error of androgen biosynthesis. Because errors of androgen biosynthesis involves enzymes necessary for the production of cortisol and aldosterone (rare defects in 17–hydroxylase and 3beta–dehydrogenase), these infants should have an ACTH stimulation test (no

increase in these hormones after ACTH stimulation). A deficiency in 5alfa–reductase is characterized by high LH, normal T, and low DHT. - Maternal virilization may be responsible for ambiguous genitalia in a girl. - The absence of maternal virilization and elevated androgens suggests idiopathic ambiguity (dysmorphia) or true hermaphroditism. Gonadal biopsy is indicated in such cases – karyotyping the gonadal cells shows mixed XY and XX. Therapy: Hormonal replacement (see topics 3A and 7A for details) Surgery – reconstructions. 46 b) Anomalies of male genitalia 1. Micropenis – the growth of penis depends on normal pituitary and testicular production of gonadotropins, testosterone, and growth hormone. Micropenis may be the only sign of hypopituitarism or disorders of hypothalamic–pituitary function. Disorders associated with micropenis include: hypothalamic causes (Kallmann sy, Prader–Willi sy), pituitary causes (GH deficiency, gonadotropin deficiency), gonadal

causes (Klinefelter sy, dysgenetic testes), and end–organ causes (androgen insensitivity). Pt. with micropenis should be tested for responsiveness to androgens (testosterone should produce lengthening of the penis). If the response is good, the pt is followed only If there is no response, change to female sex should be considered. 2. Cryptorchidism – the descent into the scrotum occurs during the last trimester of gestation Cryptorchidism is defined as failure of the testes to descend normally through the inguinal canal to their normal position at the bottom of scrotum at birth. The term ectopic testis describes a testis that is lodged in the abdominal wall, upper thigh, or within the perineum after passing through the external inguinal ring. The migration of the testis depends on the normal secretion of LH by the fetal pituitary and of testosterone by the fetal testis. Thus, abnormalities of the either the fetal pituitary (hypopituitarism) or the fetal testis causes

cryptorchidism. Concomitant hypospadias suggest an abnormality of sexual differentiation. The incidence is 3% in normal infants and 30% in premature infants. In most boys, descent is completed in the first 3 months of life. Bilateral cryptorchidism is less common The undescended testis has a decreased number of spermatogonia after 2 yr of age and it is also more prone to tumor formation. Diagnosis: Cryptorchidism must be differentiated from ectopic and retractile testes. If the parents say they have ever seen a testis in the child’s scrotum, the testis is probably retractile. If the testis can be palpated in the inguinal canal but cannot be brought down, ectopic testis is probable. If the testis isn’t felt anywhere, it is probably atrophied or placed in the abdomen CT or US help in locating the testis. Evaluation of hormones is helpful High gonadotropin and low testosterone levels indicate primary gonadal failure. Low levels of both gonadotropin and testosterone suggest the use of

gonadotropin releasing hormone stimulation test (GnRH or hCG administration increases the testosterone level and causes the testes descent). Cryptorchidism associated with abnormalities of external genitalia requires evaluation. Cryptorchidism may be associated with various endocrine and chromosomal abnormalities, such as Noonan or Klinefelter syndromes and trisomy of 13, 18, 21. Bilateral cryptorchidism with no palpable glands requires investigation to rule out female pseudohermaphroditism. 47 Treatment: Hormone stimulation test (hCG, GnRH) for 4 weeks. If it fails, orchiopexy is indicated and has to be performed before age 2 yr to prevent irreversible damage. 3. Anorchia – defined as either unilateral or bilateral absence of testicular tissue It is found in 3– 5% of boys operated on for cryptorchidism. The internal ducts and external genitalia may be male or there may be ambiguity, depending whether testicular tissue was functional at 8–12 weeks’ gestation. This

condition is called “vanishing testis syndrome” The loss of testes after 12 weeks probably occurs as a result of torsion or vascular occlusion that happened at some time during the testicular descent. Occasionally, testes are absent, but Leydig cells are present scattered along the genital ducts. This is called “Leydig cell only syndrome” Diagnosis: The absence can only be seen during operative search. Serum gonadotropin levels are elevated in the first few months of life and again in the prepubertal period. Treatment: Testosterone replacement in puberty, prosthetic testes for cosmetic purposes. c) Anomalies of female genitalia Developmental abnormalities are rare in girls. Imperforate hymen presents either at birth, when mucus secreted by the vagina accumulates beneath a bulging imperforate hymen, or at puberty, with amenorrhea and hematocolpos, with or without cyclical attacks of abdominal pain. Removal of the membrane provides drainage and relief of symptoms. 48 23)

TOXOPLASMOSIS. Maternal infection with Toxoplasma gondii occurs in 0.1–05% of pregnancies and is usually asymptomatic. When primary infection occurs during pregnancy, 40% of the fetuses become infected, of whom 15% have severe damage. The sources of transmission include exposure to cat feces and ingestion of undercooked meat. Fetal damage is most likely when maternal infection occurs in the second to sixth months of gestation. Clinical findings: - Growth retardation. - Chorioretinitis, cataracts. - Hydrocephalus, microcephaly, cerebral calcifications, seizures. - Jaundice, hepatosplenomegaly, lymphadenopathy, and pneumonia. The majority of infants are asymptomatic at birth but develop problems at a later time. Serologic tests, first for IgG and then for the specific IgM Ab, make the diagnosis. Organisms isolation and PCR are also available both from the neonate or by amniocentesis. Treatment: Spiramycin to the acutely infected pregnant woman. It does not prevent transmission

but reduces the rate of sequelae. Neonatal treatment with pyrimethamine and sulfadiazine with folinic acid can improve long-term outcome. 49 24) TUMOURS OF THE CENTRAL NERVOUS SYSTEM. RETINOBLASTOMA Collectively, brain tumors are the 2nd most common form of childhood cancer (20% of the total). They are of diverse types, each with unique characteristics, location, and growth rates. Special problems concerning CNS tumors include: the BBB limits delivery of chemotherapy by systemic route, and the developing brain is vulnerable to the toxicity of chemo– and radiotherapy. Some tumors are next to important areas – surgical resection is not possible. There is a tendency for tumors to spread within the neuraxis. Classification: 2/3 are infra- and only 1/3 are supratentorial. Most fall into two groups: - Tumors of astrocytic origin: High-grade astrocytomas (very rarely GBM) arise primarily above the tentorium; they cause focal neurologic deficits, signs of increased ICP, and focal

seizures. Low–grade astrocytomas arise primarily in the cerebellum; they cause signs of ↑ICP and signs of cerebellar dysfunction (ataxia, nystagmus, dysdiadochokinesis, dysmetria). The presentation of brain–stem gliomas involves multiple cranial nerve palsies, ataxia, long tract signs. Ependymomas develop below the tentorium and have symptoms of ↑ICP when they obstruct the fourth ventricle. Also oligodendrogliomas, choroids plexus tumors, etc - Tumors of neuroepithelial origin: Medulloblastoma is the most common malignant brain tumor in children. It causes ↑ICP and often obstructs the 4th ventricle hydrocephalus Pt. have headache, lethargy, vomiting, cerebellar dysfunction The tumors may spread rapidly via the CSF & neuraxis poor prognosis. Signs of ↑ICP: papilledema, diplopia, N/V, headache & reduced concsiousness. Therapy: - Surgery plays an important role if the tumor location permits resection. Brainstem gliomas are usually not resectable. - Radiotherapy

is important for tumors in all locations (except completely resected low-grade astrocytomas). However, it has a big toxicity For tumors spreading throughout the whole neuraxis, the whole neuraxis must be irradiated. - Chemotherapy prolongs survival and helps postponing radiotherapy to a later age, when it may be less toxic to the developing nervous system. Prognosis: it is excellent for completely resected low-grade cerebellar astrocytomas. It is good for some medulloblastomas. It is poor for brainstem gliomas and medulloblastomas that are large, occur at age <4 years, and spread into the CSF and throughout the neuraxis. 50 Retinoblastoma This rare malignant tumor of childhood is fatal if untreated. It is usually diagnosed before the end of the third year. In 30% of cases, it is bilateral Retinoblastoma results from a loss of a dominant protective allele. This usually occurs by mutation, either in somatic retinal cells alone (nonheritable retinoblastoma – 95%) or in the

germ line cells as well (heritable retinoblastoma). In heritable cases, the genetic predisposition is inherited as an AD trait; children of survivors have 50% chance of having the disease; and the tumor is usually bilateral and multifocal. The Rb gene is located on chromosome 13, on the same locus as the osteosarcoma gene. In sporadic cases, the tumor is usually not discovered until it has advanced far enough to produce an opaque pupil. The eye becomes red, with infiltration of the vitreous body, ant chamber, and orbit A deviating eye may be also the first sign of the tumor. Calcifications frequently occur in this tumor easier confirmation by radiographs and CT. Treatment: Enucleation is the treatment of choice in extensive unilateral cases. If bilateral, conservative therapy (radiotherapy, photocoagulation) is used to preserve the less severely affected eye. If left untreated, it will metastasize to the brain death. 51 25) DISEASES WITH FEVER AND RASH – DD AND TREATMENT Rash

and fever are common presenting signs of infectious diseases. In some diseases, the rash is always present and is typical for that disease (exanthematous diseases). Rash can occur in many infectious and non-infectious diseases. In the diagnosis of exanthematous infections, check: - Epidemiologic history contacts, season, traveling. - Past history past diseases, vaccination, passive immunization, allergies. - History of the present illness insect bites, drugs, onset, prodromes, site and extent of rash, spreading, type of fever, and systemic presentations. - Physical examination investigate the oral mucosa, organomegaly & LN enlargements. - Specific lab findings CBC & specific microbiological investigations. The microbiology is becoming most important because easily recognisable diseases such as rubella or measles are rare today (associated with vaccination), and since more and more diseases presenting with nonspecific exanthemas, which cannot explain the diagnosis

alone. In the DD, we should never forget to consider allergic exanthema and in indicated cases, syphilis. 1. Measles (see topic 18B) 2. Scarlatina (see topic 48C) 3. Rubella (see topic 29B) 4. VZV (see topic 31B) & HSV (topic 33B) 5. Fifth disease (Erythema infectiosum) – associated with human parvovirus-B19 infection (the name derives from its place as the fifth classic exanthema of childhood: measles, scarlet fever, rubella, smallpox, erythema infectiosum and roseola infantum. The parvovirus B19 can also cause aplastic crisis in pt. with hemolytic disease and polyarthralgia in normal adults Clinical features: It occurs in periodic outbreaks (spring). Many attacks are asymptomatic The virus first infects the epithelium of the respiratory through bloodstream to the bone marrow where it replicates and temporarily arrests erythropoiesis. This phase normally goes unnoticed (except for mild fever) in a normal host but causes aplastic crises in those with chronic hemolytic anemias.

Later, rash and arthralgias occur, both probably immune-mediated Mostly, there is a typical facial rash characterized by “slapped cheek appearance”, and later a lace-like rash mostly over the extremities but not on palms and soles. Both rashes are exacerbated by heat. Diagnosis: Clinical + serologic. CBC shows relative lymphocytosis Treatment: Supportive only. In immunedeficient, IVIG induces a remission Prognosis: excellent in immunocompetent host. If during pregnancy, there is a chance of fetal infection, with fetal death secondary to anemia and hydrops. 52 6. Sixth disease (roseola infantum, exanthema subitum) – caused by human herpes virus 6 (HHV6). It is an acute febrile illness that occurs commonly during the summer and early fall It is rare in infants before age 3 mo, and also rare after 4 yr. One attack causes immunity It usually occurs sporadically, but when occurs in epidemics, the incubation period is 5-15 days. Clinical: There acute onset of high fever without

other severe systemic signs lasting for 3-5 days, followed by an erythematous maculopapular rash mostly on the neck and trunk, less in face and proximal extremities. It is not itchy and there is no desquamation The rash persists for 24-48 hours and then disappears. Diagnosis: difficult during the high fever period before the rash appears. There is suspicion of sepsis or meningitis and these children are sometimes treated for pharyngitis and otitis. The rash may be also attributed to drugs used during the febrile illness, and the diagnosis may be missed at all. An important distinguishing feature is that most rashes in association with drugs have fever concurrent with the rash. In roseola, the fever has subsided by the time the rash appears There is no specific treatment. When the temperature is > 40ºC, acetaminophen is given 7. Enteroviral exanthemas: - Coxsackieviruses – associated with many clinical syndromes, including aseptic meningitis, febrile illness with a maculopapular

erythematous rash, vesicular stomatitis and pharyngitis, hand-foot-and-mouth disease, myocarditis, pericarditis, upper respiratory illness, pneumonia, diarrhea, encephalitis, orchitis, or pancreatitis. Perinatally acquired Coxsackie or echovirus infection is a cause of hepatitis and encephalomyocarditis. Epidemic outbreaks of enteroviruses occur in warmer months in temperate climates. These outbreaks are characterized by illnesses ranging from mild to severe. Aseptic meningitis is usually part of the picture, as is fever and rash Diagnosis: Recovery of the virus in tissue culture. Serologic tests are of no value Treatment: Supportive only. - Echoviruses – frequently cause neurologic illness. Like other enteroviruses, the virus is acquired by the oral or respiratory routes and extends to regional lymph nodes. Many infections are asymptomatic. Echoviruses produce nonspecific febrile illnesses of about 3 days’ duration. Pharyngitis may be present Vomiting and diarrhea are usually

mild Abdominal pain is present and poorly localized. In newborns, the disease is much more serious and may be fatal. There are a variety of skin manifestations The rash is usually erythematous and macular but may be petechial. Conjunctivitis is frequent Aseptic meningitis is common. The simultaneous appearance of fever and rash should suggests the diagnosis. Virus isolation confirms the dg, especially when recovered from the CSF Treatment: Supportive only. Good hygiene to lessens fecal–oral spread is preventive 53 Hand–foot–and–mouth syndrome – caused by Coxsackie virus A16. It is most common in children < 5 yr, has a short incubation period of < 1 week, and presents as mild fever, malaise, and sore mouth. After a few days an enanthem appears, especially in the mouth The lesions are 4–8 mm and frequently ulcerate. The hands, more commonly than feet, are the site of vesicular lesions that clear in 1 week. Aseptic meningitis may be associated Treatment is

supportive only. 8. Skin presentations of meningococcemia - caused by G(–) diplococcus Neisseria meningitidis. Only some strains cause skin manifestations that are seen in 70% of menigococcal meningitis. In meningococcemia, there is variable degree of damage to the skin capillaries and arterioles. Clinical features: Skin manifestations are variable. It may involve only few purple patches or papules resembling urtica but usually it presents as petechiae. Sometimes we have to search for them (on legs, arms, buttocks, abdomen, back), and sometimes they are generalized. If pressure is applied ion them, they don’t disappear. Sometimes there are large ecchymoses and bleeding into abdominal organs occurs. The spread of petechiae is usually accompanied by fever, sore throat, and sometimes arthralgias, myalgia, and a herpes–like lesion on the lips. If meningitis develops, meningeal signs become apparent. Shock may develop as well Diagnosis: It is important to differentiate petechiae

accompanying sepsis of different etiology (in such case, petechiae appear only after few days of fever), infectious endocarditis, and gonococcemia. In viral infections, petechiae can be associated with enteroviral infection (appearing in two phases – with a break with no lesions between the two attacks + leukopenia and lymphocytosis in blood count), viral hepatitis, infectious mononucleosis, and rubella. Henoch–Schönlein purpura is also associated with petechiae (starting as urtica, being around joints and on the extensor part of the extremities). Trombopenic purpura also has petechiae (but there is no fever) and petechiae in face or chest also appear while coughing or crying heavily. Treatment: Meningococcemia requires immediate hospitalization in ICU (see meningitis). 54 9. Toxic shock syndrome Etiology: Staphylococcus aureus producing exotoxin (TSST–1 = toxic shock syndrome toxin) or one of the staphylococcal enterotoxins. S aureus producing toxin may proliferate in an

infection focus (folliculitis, furuncle, skin wound, surgical wound, or peritonsilar abscess) and the toxin gets into the blood causing rash, fever, mucosal edema, hypotension, and shock. TSS may be also associated with vaginal S. aureus infections in menstruating women who use tampons. Clinical features: It is defined by high fever, low blood pressure, and diffuse erythematous “sunburn–like” rash that begins on palms and soles. It resembles scarlet fever but may also be confused with Kawasaki disease or leptospirosis. Desquamation is common late in the course Proteinuria and elevation of liver enzymes is common, and in severe cases, renal and hepatic failure can lead to death. Severe vomiting and diarrhea are common Diagnosis: It depends on isolation of the organism from the site of infection. Sensitivity to ATBs should be always assessed because not only penicillin but lately also oxacillinresistance becomes common in hospital environment. Treatment: ATB to which the organism

is sensitive. 10. Rickettsial infections 11. Kawasaki disease (see topic 46B) 12. Autoimmune disorders (see topics 41B, 46B, 27B) 55 26) IDIOPATHIC RESPIRATORY DISTRESS SYNDROME (Hyaline membrane disease, wet lung) The most common cause of respiratory distress in the preterm infant. This condition is caused by a deficiency of surfactant (the earlier the infant is born, there’s less surfactant). Surfactant decreases surface tension in the alveolus during expiration, allowing the alveolus to remain partly expanded and in that way maintaining a functional residual capacity. The absence of surfactant results in poor lung compliance and atelectasis. The infant must work hard to expand the lungs with each breath, and this leads to respiratory failure. Clinical features: Tachypnea, cyanosis, and expiratory grunting; poor air movement despite increased work of breathing occurring in the first 3 hours of life. Diagnosis: CXR – shows hypoexpansion and air bronchograms (diffuse

atelectasis). Course: It is progressively worsening over the 1st 48 to 72 hrs of life. Later, there is increased production and release of surfactant, allowing improvement. Treatment: 1. Supplemental oxygen, early intubation and ventilation 2. Placement of umbilical artery and vein lines 3. Exogenous surfactant replacement therapy, used both in the delivery room as prophylaxis and with established hyaline membrane disease as rescue, decreases the mortality rate in preterm infants and decreases air leak complications of the disease. 4. Antenatal dexamethasone administration to the mother for accelerating lung maturation and surfactant production. Complications: related to organ immaturity and associated with asphyxia and mechanical ventilation. They include PTX, patent ductus arteriosus, intraventricular hemorrhage, necrotizing enterocolitis, bronchopulmonary dysplasia, and retinopathy of prematurity. 56 27) CHROMOSOME ABNORMALITIES An alteration in the amount or nature of

chromosome material is associated with birth defects or other abnormalities. Chromosome disorders are seen in 5 in 1000 liveborn infants General characteristics: most chromosome defects arise de novo (no family members have a defect). The defects are classified as abnormalities of number or structure and content. They involve either autosomes or the sex chromosomes; autosomal abnormalities are usually more severe. Numeric defects - are abnormalities of the euploid number of chromosomes (46). Examples: trisomy 21 (Down sy), trisomy 18, trisomy 13, Klinefelter sy (47,XXY), and Turner sy (45,X). Structural defects - result from chromosome breakage and rearrangement. It includes unbalanced translocation, deletion, duplication, inversion, isochromosome, and centric fragment. Examples: deletion of 22q11, cri du chat sy (5p deletion), and Wilms tumor with aniridia (11p deletion). Prader–Willi sy is associated with deletion of the maternally derived chromosome 15q. Mosaicism - the presence

of 2 or more cell lines with different chromosome compositions in one individual. The phenotype is affected by the number and distribution of chromosomally abnormal cells. Mosaicism occurs as a result of mitotic nondisjunction (division error) after fertilization Methods of chromosome analysis: chromosome studies can be performed on any tissue in which cells are actively undergoing mitosis. 1. Karyotype (arrangement of chromosomes) allows analysis of chromosome number & structure 2. Peripheral blood lymphocytes are the most commonly studied tissue It takes 3 days 3. Fluorescence in situ hybridization (FISH) uses labeled DNA probes specific to a given chromosome segment to rapidly identify the origin of extra or missing genetic material. 4. Bone marrow studies provide results in 6 hours 5. Organ tissue studies (eg fibroblasts from skin biopsy) – it takes at least 3 weeks to obtain results, but it can be cultured from aborted fetuses, whose peripheral blood is not available. It is

also used in mosaicisms. a) Numeric autosomal abnormalities: 1. Down sy – the most common autosomal trisomy compatible with life Trisomy 21 is characteristic of DS (95%), although some cases result from translocation (4%) or, more rarely, mosaicisms (1%). Trisomy 21 occurs in 1:700 liveborn infants The risk for DS increases with advancing maternal age. This risk rises dramatically after 35 yr of age Clinical features: children have a characteristic appearance and also many functional and structural abnormalities, including: a. Mental retardation, retarded growth b. Flat hypoplastic face with small low set ears and short nose 57 c. Prominent epicanthic skin folds, transverse palmar crease, thick fissured tongue d. Joint ligament laxity, pelvic dysplasia & heart diseases, e. High incidence of leukemia and Alzheimer’s disease at later life (abt 40) Children with mosaicisms have a milder clinical presentation. Prognosis: With improved medical management, life expectancy can be

into adulthood. 2. Trisomy 13, Patau’s syndrome – 1:4000-10000 newborns are affected There is also a relationship between advanced maternal age and trisomy 13. 20% result from translocation, and 5% from mosaicisms. Prognosis is extremely poor, 50% die before 1 month of age Clinically, - All have mental retardation and malformed ears. - Most have cleft lip or palate, small mandible or microphthalmia. - Intestinal malrotation, umbilical hernia. - Polydactily, renal & cardiac defects. 3. Trisomy 18, Edward’s syndrome – occurs in 1:8000 There is a relationship with advanced maternal age but it is lower than the previous 2. 90% result from meiotic nondisjunction, 10% from mosaicisms. Prognosis is extremely poor; 90% die within 1 year of birth Clinically, - There’s mental retardation. - Abnormal skull, lowest malformed ears, small mandible. - Diaphragmatic & inguinal hernias, Meckel’s diverticulum. - Cardiac defects. b) Sex chromosome disorders: 1. Turner syndrome -

with a 45,X0 karyotype is associated with a female phenotype but with streak ovaries (gonadal dysgenesis). The incidence is 1:10000 females Clinical features: - Newborns have webbed neck, triangular facies, edema of hands & feet, aortic coarctation. - Later symptoms include short stature, shield chest with wide-set nipples. - Streak ovaries, amenorrhea, absence of secondary sex characteristics, and infertility. Some affected girls have only short stature and amenorrhea. Mosaicisms (e.g 45,X/46,XX) may be present Mosaic forms of gonadal dysgenesis that contain a Y–bearing cell line have variable ambiguous internal and external phenotypes. The risk for gonadoblastoma is higher in individuals with Turner sy. Treatment: Estrogen replacement helps in development of secondary sex characteristics and normal menstruation and prevent osteoporosis. Growth hormone therapy is used to increase height. Females with Turner syndrome have low fertility rate and their offspring have an increased

frequency of chromosome abnormalities and congenital malformations. 58 2. Klinefelter syndrome - with a 47, XXY karyotype, is associated with a male phenotype but with poorly functioning testes. The incidence of Klinefelter syndrome is 1:1000 The maternal age at birth is often advanced. Klinefelter syndrome is rarely the cause of spontaneous abortions. Clinical features: - Prepubertal boys have a normal phenotype. - The characteristic findings after puberty include micro-orchidism associated with otherwise normal external genitalia, azoospermia, sterility, gynecomastia. - Normal to borderline IQ, diminished facial hair, lack of libido and potency, and tall, eunuchoid build. - In chromosome variants with three or four X chromosomes, mental retardation may be severe, and anomalies of the external genitalia as well as cryptorchidism may be present. In general, the physical and mental abnormalities associated with Klinefelter syndrome increase as the number of sex chromosomes

increases. A variety of mosaicism have been reported (some pt. with 46,XY/47,XXY mosaicism are fertile) Treatment: Testosterone replacement. 3. Other sex chromosome abnormalities: a. 47,XXX karyotype - is seen in 1:1000 liveborn females Most individuals have a normal phenotype, though tall stature is common. Average IQ is 90, but mental retardation is rare. b. 47,XYY karyotype - is seen in 1:1000 liveborn males The phenotype is usually normal, though most of them are taller. There may be behavioral disorders & severe acne c) Structural chromosome abnormalities 1. Partial deletions – Some syndromes can be caused by the loss of chromosome material from the ends of a chromosome (terminal deletion) or loss from the middle or inner portion (interstitial deletion). Most deletions arise de novo Contiguous gene deletion syndromes refer to phenotypes resulting from the absence of several neighbouring genes. These defects cause variable conditions with several manifestations (e.g Williams

sy) Clinical features: Children with terminal deletions usually have growth deficiency, mental retardation, dysmorphic features, and multiple malformations. Examples: a. Deletion 22q11 occurs in 1:4000 It causes a variety of syndromes (eg DiGeorge sy) with variable manifestations: short stature, cleft palate, small ears, cardial defects, thymic aplasia, hypocalcemia, low IQ, attention deficits, psychiatric disorders (bipolar, psychotic disorders). 59 b. Cri du chat sy occurs in 1:50.000 There is terminal end deletion of the short arm of chromosome 5 (5p). Affected children have a characteristic catlike cry, there is mental retardation and CNS abnormalities, congenital heart disease and ocular malformations. Many pt. can survive into adulthood 2. Partial trisomy – It results when extra chromosome material is found in the karyotype, but there is less than an entire extra chromosome present. The origin of the extra material determines the phenotype. When it is of autosomal

origin, dysmorphic features, growth failure, malformations, and developmental abnormalities are common. 60 28) ACUTE DISEASES WITH FEVER WITHOUT RASH – DD AND TREATMENT Dr. have to evaluate children with fever frequently Fever is defined as rectal temp >38 °C, an oral temp >37.5 °C, or axillary temp >372 °C Body temp normally fluctuates during the day and may be 0.5 °C below normal in the morning, progressing to 05 °C above normal in the evening Mild elevations of 1 °C can be caused by exercise, hot bath, or hot weather. Warm food or drink can elevate an oral temperature. Fever occurs when there is a rise in the hypothalamic set-point (d/t infection, malignancy, collagenvascular disease, some drugs, etc.), when the body’s heat production or environmental heat exceeds heat loss mechanisms (e.g malignant hyperthermia or excessive environmental heat), or when heat loss mechanisms are defective (as in ectodermal dysplasia). The vast majority of fevers are caused by

viral infections and last no longer than 3 days. Teething does not cause fever over 38.4 °C Fever causes no harm, such as brain damage, when it is less than 41.7 °C Fortunately, the brain’s thermostat keeps untreated fevers resulting from infection below 41 °C. Febrile convulsions develop in about 4 % of febrile children and are generally harmless. Children under age 3 who present with fever but no obvious source of infection present a challenge. Both minor illnesses and serious bacterial infections sepsis, meningitis, UTI, pneumonia, septic arthritis, osteomyelitis, bacterial enteritis – occur at this age. Bacterial pathogens most commonly seen in infants < 2 months are group B strep and E coli. Pneumococci are more common in older children. The incidence of H influenzae is low because of vaccination. Other common pathogens include Staph aureus, Salmonella, & other G (–) organisms In general, as the height of fever increases, the risk of serious bacterial infection

increases. Treatment of fever a. Antipyretics paracetamol, liquid ibuprofen Don’t use aspirin d/t risk of Reye syndrome b. Sponging with lukewarm water (indicated in febrile delirium, febrile seizures, and any fever >41; acetaminophen should always be given prior to sponging). c. Extra fluids & dressing the child in one layer of light clothing Management of infants with fever Any child who appears toxic should be hospitalized. Hospitalized infants are usually given parenteral ATB until culture results. Low-risk febrile infants can stay at home until culture results 61 29) FOLLOW UP CARE OF THE PRETERM INFANT. BRONCHOPULMONARY DYSPLASIA. ROP (RETINOPATHY OF PREMATURITY) Preterm infant – follow up care Preterm infant is any infant born before 37 W of gestation Etiology: the cause of premature labor, whether or not preceded by premature ROM, is usually unknown. However, often there’s maternal history showing low SES, inadequate prenatal care, poor nutrition, poor

education, and untreated illnesses or infections. Signs: - The premature infant is small, weighing < 2.5 kg - The skin is thin, shiny & pink, through which the underlying veins are easily seen. - Little subcutaneous fat, hair, or external ear cartilage exists. - Spontaneous activity and tone are reduced, and extremities are not held in flexed position. - Testes may be undescended in males. In females, labia majora don’t yet cover the labia minora Complications: most complications relate to the immature functioning organ systems: 1. Lungs: Surfactant production is inadequate to prevent alveolar collapse and atelectasis, which results in RDS. 2. CNS: Because of inadequate coordination of sucking and swallowing reflexes, the infant born before 34 week gestation may need to be fed IV. Immaturity of the respiratory center in the brainstem results in apneic spells. The periventricular germinal matrix is prone to hemorrhage, which may extend into the ventricles

intraventricular hemorrhage. Hypotension, and inadequate brain perfusion contribute to cerebral hemorrhage or infarction. 3. Infection: Sepsis and meningitis is 4x more common than in full-terms It results from indwelling intravascular catheters and endotracheal tubes and from areas of skin breakdown and the markedly reduced serum immunoglobulin levels in preterm infants. 4. Temperature regulation: They have an exceptionally large body surface area to body mass ratio; therefore, they rapidly loose heat and have difficulty maintaining their temperature. 5. GIT: Small stomach and immature reflexes prevent adequate oral or nasogastric tube feedings and create a risk of aspiration. Breast milk does not provide sufficient calcium, phosphorus, and protein for very low weight infants, in whom it should be mixed with milk fortifiers. 6. Kidney: Late metabolic acidosis and growth failure may result from the immature kidney’s inability to excrete fixed acids. As a result, sodium and

bicarbonate are lost in urine 7. Hyperbilirubinemia: (see topic ) 8. Hypoglycemia: Premature infants are especially prone to it It can lead to neurologic damage! Treatment and follow-up care is directed towards treatment and prevention of complications (see specific topics – e.g: 9B, 6B, 21B, 36B, 43B, 41A, 46C, 19A) 62 Bronchopulmonary dysplasia A chronic pulmonary disease of infants that is characterized by the need for oxygen therapy beyond 28 days of life and by characteristic changes on the lung on CXR. It follows neonatal respiratory failure and is especially likely to occur after oxygen and mechanical ventilation for hyaline membrane disease in a preterms. This condition also may occur in term infants after acute lung injury (e.g pneumonia, meconium aspiration, diaphragmatic hernia) Pathogenesis: - Oxygen toxicity from prior oxygen therapy (FiO2 > 0.8) - Barotrauma from mechanical ventilation with high airway pressure. - Other factors include prematurity, fluid

overload, PDA & damage d/t severe pulmonary disease. Pathology: Diffuse alveolar injury with endothelial cell damage, resulting in interstitial pulmonary edema and fibrosis. Necrotizing bronchiolitis with smooth muscle hypertrophy, resulting in areas of atelectasis and emphysema. Clinical features: - Retractions, tachypnea, wheezing, and cyanosis, esp with stress. - Some infants show poor lung compliance, pulmonary edema, and pulmonary HTN. Diagnosis: - Phys exam: high sleeping respiratory rate, breathing effort, auscultation (crackles & wheezes). - CXR: near-total lung opacification (acute stage), thickened fibrotic markings and cystic changes (chronic stage). - Blood gas analysis. - ECG & echo: may demonstrate cor pulmonale. Therapy: - Oxygen: delivered by nasal cannula to maintain SAT >90% and minimize the work of breathing. Some infants require mechanical ventilation - Diuretics: for infants with pulmonary edema, high pCO2, and persistent hypoxemia.

- Bronchodilators: inhaled β2-agonists improve bronchospasm and decrease work of breathing. - Dietary supplements: more calories are needed (higher BMR + increased work of breathing). Prognosis: Short–term prognosis: Most infants will get better with time, and by age 5 yr their pulmonary function may be normal. However, they may require oxygen for 6 to 12 month after hospital discharge. Many infants have hyperreactive airways and may require hospitalization for acute viral infections. They have greater risk of asthma Long–term prognosis is not known yet 63 Retinopathy of prematurity (ROP) It was first seen after oxygen therapy became commonly used to sustain life in very small premature infants. This condition is estimated to result in 550 new cases of infant blindness each year in the USA!!! The cause of retinopathy of prematurity seems to be supplemental oxygen. The major risk factors are decreasing gestational age and low birth weight Retinal vascularization comes

within the optic nerve. Retinal vessels normally reach the nasal ora serrata at 8 month and the temporal ora serrata at 9 month. Retinopathy of prematurity develops if this process is disturbed (hyperoxia induces vasospasm and endothelial damage in the retinal vessels with resulting tissue edema and injury). It is usually bilateral but often asymmetric It begins at the junction of vascularized and avascular retina, initially as an obvious demarcation line (stage 1), followed by formation of a ridge (stage 2), then extraretinal fibrovascular proliferation (stage 3). Even among pt with stage 3, there is a high incidence of spontaneous regression The cicatrical phase (stages 4 and 5) is manifested by increasingly severe retinal detachment. Blindness follows retinal detachment. Treatment: cryotherapy or photocoagulation. 64 30) HODGKIN’S DISEASE. NON HODGKIN’S LYMPHOMA Hodgkin’s disease Neoplasm of large activated lymphoid cells with distinctive cytomorphology & histology.

It’s principally a disease of LN, containing bizarre atypical giant cells with a mixed reactive background. These atypical cells include RS-cells (Reed-Sternberg, multinucleated), mirror-image cells (binucleated), Hodgkin’s cells (mononuclear), L&H (‘popcorn’) cells, and lacunar cells. Hodgkin’s disease accounts for ~1/3 of all malignant lymphomas. It accounts for 4% of all childhood cancers/ mainly in older children & teenagers, with a slight female predominance. It is primarily a nodal disease, spreading slowly & orderly to the following anatomical contiguous LN. Hematogenous dissemination & extranodal involvement occurs in advanced stages Etiology: 1. Latent EBV infection (40-60%) 2. Other herpes viruses 3. T-cell dysfunction 4. Immunodeficiency (rare) Symptoms: ƒ Diagnosis: Painless lymphadenopathy, usually in the neck. ƒ Constitutional symptoms fever, weight loss, night sweats, pruritus & skin rash. ƒ Rarely there’s pain in an involved

LN following alcohol ingestion. ƒ Hepatosplenomegaly (in advanced stages). ƒ Bone marrow involvement (in advanced stages). LN biopsy there are several histological subtypes: lymphocyte predominance (5%), lymphocyte depletion (5%), mixed cellularity (30%), and nodular sclerosis (60%). DD: NHL, infectious mononucleosis, cat-scratch disease & drug reactions (e.g phenytoin) Pt. should undergo staging to determine the extent of the disease, which will determine whether localized treatment (radiotherapy) or systemic chemotherapy should be given. Ann Arbor scheme: Stage I: Single lymph node region or extralymphatic site involvement. Stage II: Involvement of more than one region/site on the same side of diaphragm. Stage III: LN involvement of both sides of the diaphragm. Stage IV: Disseminated (multifocal) disease with bone marrow or liver involvement. All stages are further divided to A (absence of systemic symptoms) & B (presence of such symptoms) best prognosis

for stage IA and worst for IVB. Treatment: 1. Localized disease (stages IA, IIA) radiotherapy 2. Disseminated disease (IIIB, IV) aggressive combination chemotherapy E.g ABVD : adriamycin (doxorubicin), bleomycin, vincristine, dacarbazine 3. Stages IIB or IIIA controversial between the 2 65 Prognosis: Best for stages IA & IIA, with a 10-year survival rates >80%. For disseminated disease (IIIB, IV), the 5-year survival rate is 50–60%. If the disease recurs after initial radiotherapy it may still be curable with chemo. If it relapses after initial chemo, it may be cured with intensive therapy involving autologous bone marrow transplantation. Most serious late effects of therapy are: secondary malignancies, thyroid gland dysfunction after neck irradiation, growth disturbances after irradiation, and sterility. Non-Hodgkin’s lymphomas, NHL: A heterogenous group of diseases characterized by neoplastic proliferations of immature lymphoid cells, which unlike malignant

lymphoid cells of ALL, accumulate mainly outside the bone marrow. Epidemiology: NHL represent 6% of all childhood cancers. They occur in older children and teenagers and have a strong predilection for males. Childhood NHL differ from adult cases (predominant extranodal presentation, T–cell lymphomas as likely as B–cell lymphomas, highly aggressive). Burkitt’s lymphoma occurs in an endemic form in Africa, presenting as a mass in the jaw or abdomen. It is linked to EBV and also develops in children with HIV Classification: - Morphology: Almost all cases are diffuse lymphomas, highly aggressive. There are two major types: lymphoblastic and nonlymphoblastic. - Immunology: T–cell origin in ½ of the cases. The cells have a lymphoblastic morphology and contain TdT. B–cell origin in most other cases The cells are nonlymphoblastic and lack TdT activity. Non-T, non-B-cell origin is uncommon Clinical features: All childhood NHL are rapidly growing, and thus, symptom duration is

short: - Anterior mediastinal masses sometimes associated with pleural effusion, are the most common presentation of T-cell NHL. They can produce respiratory distress from airway compromise and SVC syndrome. - Abdominal masses most common presentation of B-cell cases. They arise from either abdominal LN or from intestinal Peyer’s patches. They can cause abdominal enlargement, which may produce pain, ascites, and intestinal & urinary tract obstructions (intussusception). - Jaw masses common presentation for endemic Burkitt’s lymphoma. Peripheral LN enlargement can be seen along any time of NHL. - Less common presentations obstructing nasopharyngeal tumor, bone tumor, skin tumor. 66 Staging: Local disease – limited bulk, confined to one side of the diaphragm and having good prognosis. Extensive disease – within the mediastinum or abdomen. Hematogenous dissemination – especially bone marrow and meninges. Therapy: 1. Extensive surgical removal (at least 90%)

improves survival in case of abdominal lymphomas 2. Systemic chemotherapy is needed in all cases to shrink the local tumor and prevent dissemination. CNS prophylaxis is required in most pt 3. Radiotherapy is indicated in case of life–threatening obstruction that does not respond to chemotherapy and in bulk tumor for which chemotherapy is inadequate. Prognosis: With appropriate management a good outcome is achieved. Without therapy, rapid and widespread dissemination occurs (mainly bone marrow and meninges). 67 31) DISEASES WITH CHRONIC FEVER – DD AND TREATMENT Some pt. come to the pediatrist with chronic fever whose source is unknown and should be investigated. The fever may resolve spontaneously, and the cause often remains unclear Etiology: 1. Infectious origin (50%) These include in decreasing frequency viral syndromes, URI, LRI, UTI, gastroenteritis, osteomyelitis, CNS infections, TB, bacteremia, subacute bacterial endocarditis, mononucleosis, brucellosis, abscess, and

malaria. 2. Collagen vascular disease (10% RA, 3% SLE, 2% vasculitis) 3. Neoplasms (7%, mostly leukemia, lymphoma, neuroblastoma) 4. IBD (4%) 5. Other causes (12%) History: note the following: - The characteristics of fever onset, duration, pattern. - Nonspecific symptoms such as anorexia, fatigue, chills, headache, and mild abdominal pain are rarely helpful in diagnosis. - Animal exposure may be important: ticks (Lyme disease, relapsing fever), rats (plaque, rat-bite fever, leptospirosis), rabbits (tularemia), cattle, goats, dogs (brucellosis), birds (psittacosis), and cats (cat–scratch fever). - Any history of travel (especially travel to tropical areas and developing countries. Pt are at higher risk for malaria, dengue fever, typhoid fever, endemic typhus, borreliosis, brucellosis, and leptospirosis), - foreign contact, or drug exposure should be noted. - Specific symptoms related to some organ dysfunction may help. - Risk factors for HIV (blood transfusions, drug

abuse, perinatal transmission), sickle cell disease, malignancies, immunodeficiencies, DM, and autoimmunity should be investigated. Physical examination: The presence of arthralgias, arthritis, myalgia, or localized limb pain suggests collagen vascular disease, neoplasms, or infections (osteomyelitis, septic arthritis). Heart murmur suggests bacterial endocarditis. IBD is suggested by abdominal pain, bloody stools, diarrhea, or weight loss. Jaundice is present in hepatitis A rash may indicate collagen vascular disease, neoplasm, or infection. Pharyngitis, tonsillitis, or peritonsilar abscess can be caused by the usual bacteria or infectious mononucleosis, CMV, tularemia, pr leptospirosis. Respiratory distress may relate to an underlying neoplasm, collagen vascular disease, or infection. Meningeal or focal neurologic signs suggest encephalitis, meningitis, or neoplasm. Lymphadenopathy and hepatosplenomegaly may be related to several infectious and non-infectious causes, but it may

indicate a malignancy!!! 68 Lab: differential CBC may help in finding the cause: Pancytopenia, unexplained neutropenia with thrombocytopenia, or lymphoblasts on the peripheral smear requires a hematology/oncology consultation and bone marrow test. Reactive lymphocytes on differential suggest mononucleosis or other viral infection. Severe neutropenia in a moderately ill pt may be associated with many infections. Leukocytosis and elevated sedimentation rate suggest bacterial infection or collagen vascular disease. Hemolytic anemia suggests endocarditis or collagen vascular disease Nonhemolytic anemia suggests chronic illness or malignancy. Pyuria and bacteriuria suggest UTI, hematuria suggests endocarditis. Inappropriate ATBs may alter typical signs of occult infections (especially abdominal abscess or osteomyelitis) and delayed diagnosis. Therapeutic trials of ATBs and other drugs should be avoided before the diagnosis is made. Treatment: Depends on the diagnosis (see related

topics). 69 32) DISORDERS OF THE SKIN AND THE UMBILICUS IN THE NEWBORN. Color of the skin may suggest cyanosis, pallor, or jaundice. The color of the skin is a useful indicator of cardiac output. Because there is normally a high blood flow to the skin, any stress that triggers a catecholamine response produces changes in skin color secondary to changes in the distribution of the cardiac output and perfusion of the skin. Cyanosis and pallor are 2 signs reflecting inadequate oxygenation and skin blood flow. However, perioral cyanosis and cyanosis of the hands and feet, with lips, mucous membranes, and nailbeds remaining pink, suggests normal peripheral vascular instability shortly after birth. For jaundice see topic 19A Cracking and desquamation of the skin is normal in the term infant. In the term infant, fine downy hair known as lanugo covers the skin, particularly the shoulders and upper back. Common birth marks visible at birth include flat vascular nevi and mongolian spots.

Raised vascular nevi (hemangiomas) usually become apparent several weeks after birth. Benign rashes are common: Erythema toxicum neonatorum has a “flea–bite” appearance with scattered maculae that may contain papulopustular centers. This rash changes distribution from day to day Milia are fine, pinpoint, yellow–white papules caused by retained sebum that cover the bridge of the nose, chin, and cheeks. These are transient Neonatal pustular melanosis consists of small vesiculopustules that are present at birth and rupture within a few days. The umbilical cord is clamped and cut after the first breath; one vein and two arteries should be visible on the fresh cut surface. The clamp should be removed in 24h to avoid tension on the drying stump. Daily application of 70% alcohol helps drying and reduces infections The cord should be observed daily for redness and drainage, since it is a portal of entry for infection. The umbilical site heals by 1 month of age. Once the cord has

separated and healed, fluid discharge suggests an abnormal connection between the surface of the abdomen and underlying structures, such as a patent urachus. Umbilical hernias are commonly noted in infants, often with diastasis of the rectus muscles. Most umbilical hernias resolve by school age 70 33) LEUKEMIAS a) Acute lymphoblastic leukemia (ALL) It is a malignant disease characterized by progressive infiltration of bone marrow and lymphatic organs by immature lymphoid cells known as lymphoblasts. It is the most common childhood malignancy accounting for 25–30% of all childhood cancers. Etiology: The exact etiology is unknown. However, oncogene activation and/or tumor–suppressor gene inactivation contribute. The mutation is probably sporadic Certain chromosomal instability syndromes, such as ataxia teleangiectasia and Fanconi’s sy have a higher incidence of ALL. Regardless of etiology, childhood ALL is a heterogenous disorder that is classified according to: - Morphology

it is classified into 3 groups (L1–L3). L1 has the best prognosis, L3 the worst - Immunology classified according to whether the lymphoblasts are of B or T cell origin. Specific monoclonal antibodies are used to recognize them. They are further subdivided according to the stage of development. This is important because some groups have worse prognosis and require more aggressive treatment. - Cytology correlates with immunologic subtypes. This classification is also used diagnostically and prognostically to divide pt. into appropriate treatment regimens - Enzyme activities are used as markers. For example, TdT is an enzyme not found in mature lymphocytes but is present in lymphoblasts of ALL. Therefore, finding TdT differentiates ALL from AML and helps to diagnose CNS relapse. Epidemiology: In the USA, 15000 cases are diagnosed each year. The peak age incidence is 2–6 yr It is more common in males. ALL occurs in 1% of pt with trisomy 21 Clinical presentation: It may be

focal or systemic, minor or life-threatening depending on the degree of bone marrow infiltration and extent of extramedullary spread. - Usual findings are related to bone marrow infiltration: anemia, thrombocytopenia & neutropenia. - Fever, lethargy, pallor, and fatigue. - Mucosal bleeding with petechiae or purpura. - Extremity or joint pains. - Abdominal fullness due to HSM may be present. Anorexia is common - Meningeal infiltration is more common than in AML. Diagnosis: CBC shows anemia, neutropenia, and thrombocytopenia. However, bone marrow biopsy and aspirate is necessary. More than 5% of lymphoblasts in the bone marrow biopsy, and more than 25% on aspirate is diagnostic. Special tests (see above) are done by the pathologist to distinguish the type and decide about the treatment and prognosis. 71 There may be some other metabolic abnormalities in the child, such as hyperuricemia from increased proliferation and destruction of the cells that may lead to uric

acid nephropathy and renal failure if untreated; high potassium, phosphate, and LDH levels in the serum,. Treatment: Before antileukemic therapy is begun, complications should be treated (ATBs to fight infections due to neutropenia, platelet transfusions, RBCs transfusions, correction of electrolytes; and hydration, alkalinization, and allopurin to prevent uric acid nephropathy). The treatment depends on the type of ALL but is generally subdivided into 3 phases: remission/ induction, CNS prophylaxis, and maintenance. Induction refers to the 1st month of treatment when the tumor reduction (99% of tumor cells disappear) is the goal. After this, such pt is in remission CNS prophylaxis consists of radiation and intrathecal methotrexate. Maintenance therapy is given usually for 3 yr. Prognosis: 2/3 of ALL pt. remain disease–free 5 yr from diagnosis and are likely to be cured Poor prognosis is associated with children < 2 yr or > 10 yr, initial WBC > 50.000, male sex, black race,

massive organomegaly, lower platelet count, and certain immunophenotype. Pt whose leukemic cells have more than 50 chromosomes have a good prognosis. Pt with translocations have a poor prognosis. Pt with good prognosis may be treated with less toxic agents Sites of relapse are most likely bone marrow and testes (after treatment, hormone replacement is necessary and the pt. becomes sterile) With each relapse the ability to have complete remission decreases. Pt with early relapse should undergone bone marrow transplantation, if a compatible donor is found. Pt. that survive and had CNS radiotherapy and intrathecal methotrexate may have impaired psychomotor and intellectual functions. Other complications are short stature, testicular dysfunction, and secondary malignancies. b) Acute myelogenous leukemia (AML) It is a primary malignant disease characterized by predominance of immature myeloid precursors. Etiology: It remains unclear, but is it evident that the blasts (leukemic cells) are

all from a single cell. In addition, in 90% there is a clonal chromosomal abnormality in leukemic blast cells that disappear during remission and appear again in relapse, suggesting clonal origin of these cells. Like in ALL, many of the translocations seen in AML result in the activation of oncogenes. Since the malignant transformation can occur at any point from the pluripotent stem cell to precursors, there are heterogenous subtypes of AML (M1 – M7) depending on the cell of origin (e.g myeloblasts, promyelocytes, monoblasts, megakaryoblasts) Immunologic phenotypes are not recognized in AML (as opposed to ALL). Biochemical markers are useful 72 Epidemiology: It includes 25% of leukemias in childhood (200-300 cases in USA each yr). There is no age peak incidence. Conditions that predispose for AML: treatment with alkylating agents for other tumor, hereditable diseases or chromosomal disorders, such as Down sy or Fanconi’s sy. Clinical presentation: The signs and symptoms may

be insidious or life-threatening and are usually secondary to pancytopenia following bone marrow infiltration, or from infiltration of other organs. - Anemia and thrombocytopenia are common. - Because the absolute granulocyte count is low, bacterial infection, esp of the respiratory, dental, sinus, perirectal, urinary, and skin. Most pt have WBC (blast) count around 50,000 If the count is >200,000, leukostasis may occur, with intravascular clumping of blasts & subsequent infarction. Leukostasis usually occurs in the brain and lung, and these pt may present with somnolence, stroke, or tachypnea (bc of hypoxia). - There is usually HSM and LN enlargement. - Occasionally, myeloblasts form tumor masses (chloromas) in the skin, spinal cord, or airways. Sometimes, the CNS may be involved. Diagnosis: Bone marrow aspirate and biopsy followed by morphologic and histochemical analysis. In an aspirate, more than 30% of myeloblasts make the diagnosis. Chromosome abnormalities,

surface and biochemical markers influence prognosis and treatment. Treatment: First, complications have to be managed (see ALL). Remission induction is achieved by cytosine arabinoside and doxorubicine, but these agents cause periods of bone marrow hypoplasia – during this time, pt. are extremely prone to infections and require ATB as well as blood components. CNS prophylaxis is used An alternative to chemotherapy after remission is bone marrow transplant if HLA compatible donors are available. 50% survive 5 yr, but there is a problem of graft vs host reaction. Regardless of therapy used, overall survival 5 yr after diagnosis is about 30-40%. 73 34) ACUTE ABDOMINAL PAIN – DD AND TREATMENT. The most urgent consideration in evaluating a child with AAP is to determine whether it is requiring surgery. Most causes of AAP in children do not require surgery Important causes possibly requiring surgery include: intestinal obstruction due to malrotation and volvulus, intussusception,

strangulated hernia, or adhesions; appendicitis, Meckel’s diverticulum, or an abdominal abscess; toxic megacolon; perforated duodenal ulcer or perforation of intestine; cholecystitis; rupture of the spleen or other organ due to trauma. Clinical features suggesting a cause requiring surgery include: vomiting (especially bilious or feculent); sudden onset of abdominal distention; absent bowel sounds or high–pitched sounds (intestinal obstruction); abdominal signs of peritonitis (rigidity, rebound tenderness). Important causes of AAP not requiring surgery include: enteritis and colitis of any cause, Henoch–Schönlein purpura, hemolytic–uremic syndrome, and other types of vasculitis, fecal impaction, hepatitis, pancreatitis, vaso–occlusive crisis of sickle cell anemia, primary peritonitis, mesenteric adenitis, UTI or urinary calculi, extra–abdominal causes (pneumonia, osteomyelitis, acute neurologic processes). Appendicitis It is the most common indication for acute abdominal

surgery in childhood. It occurs frequently in children 10–15 years old. Less than 10% are < 5 years Pathogenesis: Bacterial invasion of the appendix occurs, especially if the lumen is obstructed by a fecalith, parasite, or lymph node. Clinical features: Fever, vomiting, anorexia, and diffuse periumbilical pain develop. Later, pain and abdominal tenderness localize to the right lower quadrant as the parietal peritoneum becomes involved. The incidence of perforation and diffuse peritonitis is high Atypical presentations are common in childhood. Certain bacterial infections (eg C jejuni, Yersinia) may be associated with right lower quadrant pain and tenderness and may mimic appendicitis. Diagnosis: Clinically and by physical examination, Laboratory tests show WBC count moderately elevated, X–ray may show a fecalith, occasionally, barium enema may be useful. Therapy: Laparotomy + appendectomy. The mortality is low unless perforation has occurred 74 Intussusception It is the

invagination of one part of the intestine into another. It is one of the most common causes of intestinal obstruction in infancy. Pathogenesis: Most intussusceptions are ileocolic. In pt until the age of 2 years, no cause of the intussusception is usually found. A previous viral infection may cause hypertrophy of the Peyer’s patches or mesenteric nodes, which may play a role. Specific anatomic abnormalities leading to intussusception (Meckel’s diverticulum, intestinal polyp, lymphoma, or a foreign body) are more common in children > 5 yr. However, Meckel’s diverticulum usually manifests as melena unassociated with abdominal pain or intussusception. As a result of impaired venous return, the intussuscepted bowel may swell, become ischemic and necrotic, and perforate. Clinical features: Irritability and colicky pain start suddenly. Vomiting is common Rectal bleeding may occur. Lethargy and altered consciousness may be striking A tubular mass is palpable in about ½ of the pt.

Diagnosis: X–ray may show gas in the right lower quadrant or evidence of obstruction. A barium enema shows a coiled–spring appearance to the bowel. Therapy: Hydrostatic or air reduction by careful barium enema; if it does not help – surgery. Prognosis: Immediate recurrence rate=15%, in case of an anatomic abnormality it is higher. 75 35) PERINATAL BACTERIAL INFECTIONS. Infection is a major cause of neonatal morbidity and mortality. The newborn is susceptible to infection due to immaturity of the immune system. Perinatal infections include infections acquired after rupture of the fetal membranes, and infections acquired via the birth canal. Common causative organisms include: Group B beta–hemolytic streptococcus, E. coli, Klebsiella, Streptococcus pneumoniae, Chlamydia trachomatis, N. gonorrhoeae, and N meningitidis The infection is usually bacteremic (often affecting meninges by the blood resulting in meningitis) and associated with systemic symptoms => neonatal sepsis.

Neonatal sepsis is common in premature infants. Sepsis in term infants is rare Risk factors for early neonatal sepsis include premature labor, prolonged rupture of the fetal membranes, low birth weight, chorioamnionitis, and maternal fever. Clinical features of bacterial infection: Respiratory distress, feeding intolerance, temperature instability, hypo- or hyperglycemia, apnea, lethargy, and irritability. Laboratory findings include: neutropenia or neutrophilia, thrombocytopenia, prolonged PT and PTT, tracheal aspirate containing bacteria and neutrophils. Therapy: Empiric antibiotic therapy (broad–spectrum penicillin + aminoglycoside). Once culture data (blood, lumbar puncture, urine, tracheal aspirate, gastric aspirate) are available, therapy should be given according to the specific organism. The duration therapy is 7–10 days (meningitis and osteomyelitis longer). Complications (eg DIC) must be treated 76 36) INFECTIVE ENDOCARDITIS. MYOCARDITIS, PERICARDITIS a) Infective

endocarditis Bacterial infection of the endocardial surface of the heart or the intimal surface of certain arterial vessels (coarcted segment of aorta or ductus arteriosus) is a rare condition that usually occurs when a preexisting abnormality of the heart or great arteries is present. It may develop in a normal heart during the course of septicemia. The incidence of infective endocarditis is increasing owing to greater use of central venous catheters and prosthetic material and valves. Pediatric pt without preexisting heart disease also are at increased risk because of immune deficiencies, use of lines in critically ill newborns, and increased incidence of intravenous drug abuse. Pt at greatest risk include those with aorticopulmonary shunts, left–side outflow obstruction, and VSDs. The actual cause (identified in 30% of cases) of the episode of infective endocarditis is usually dental procedure, nonsterile surgical procedures, and cardiovascular surgery. Causative organisms include

viridans streptococci, Staphylococcus aureus, and fungal agents. Symptoms, signs, and diagnosis - Changing murmurs, fever, positive blood culture. - Weight loss, cardiomegaly, elevated ESR. - Splenomegaly, petechiae, embolism, and leukocytosis. - Other findings are hematuria, signs of CHF, clubbing, joint pains, and hepatomegaly. - Echocardiography identifies vegetations. Prevention It is recommended that pt. at risk for infective endocarditis be given appropriate antibiotics before any type of dental work, and before operations within the oropharynx, GI, & GU tracts. Treatment Even if blood cultures are negative after 48 hours, antibiotic therapy should begin. If congestive heart failure occurs, surgical excision of the infected area and prosthetic valve replacement must be considered. Embolization may occur from the vegetations! b) Myocarditis It is most commonly of infectious origin. It is not frequent in childhood Most cases are caused by enteroviruses, predominantly

coxsackie B virus and echovirus. Clinical features include fever, congestive heart failure, and arrythmias. The ECG is abnormal, with ST segment depression and T wave inversion. Therapy for pt with viral myocarditis is supportive Non–infectious inflammatory lesions are associated with collagen vascular diseases. Some pt may be asymptomatic, and the diagnosis is made only by observing changes in the ST segment and T 77 wave. Other pt may have signs of congestive heart failure, low cardiac output, or rhythm disturbances. Myocardial inflammation may be acutely fatal or progress into chronic congestive heart failure. Anticongestive therapy is necessary; myocardial biopsy can help find an etiology Cardiac transplantation may be necessary for pt. with myocardial failure c) Pericarditis It is an inflammation of the pericardium. Etiology The most common causes are viruses – esp coxsackie B, echovirus, influenza virus, and adenovirus. Bacteria include S aureus, S pneumoniae, and N

meningitides. Other causes include fungi and M tuberculosis. Clinical features Left shoulder pain and back pain, which decrease when the pt. is sitting Fever, tachypnea, tachycardia, cough, and decreased heart sounds due to pericardial fluid and pericardial friction rub are also common. It is important to recognize pericardial fluid accumulation, which can lead to cardiac tamponade. Signs of tamponade include neck vein distention on inspiration and paradoxical pulse (greater–than–normal inspiratory decrease in arterial blood pressure and an absence of the normal inspiratory fall in venous pressure). Diagnosis ECG findings include a low–voltage QRS complex, ST segment changes, and T wave inversion. Chest X–ray shows increasing cardiothoracic ratio without increasing pulmonary vascular markings. Echo estimates the amount of pericardial fluid Therapy In severe cases – pericardial drainage and administration of oxygen and isoproterenol. Prolonged ATB therapy is given for cases of

pericarditis due to bacteria. 78 37) RECURRENT ABDOMINAL PAIN – DD AND TREATMENT Recurrent abdominal pain It is defined as three or more episodes of abdominal pain over > 3 months. The three types of recurrent abdominal pain (RAP)–psychogenic, organic, and functional–are differentiated by the underlying cause. The incidence is slightly > 10% RAP is rare before ages 4–5 years and most common between ages 8 to 10 years. RAP is psychogenic in 85% of pt Etiology and Pathophysiology Psychogenic RAP arises from stress, anxiety, or depression. Its pathophysiology is unknown Pt susceptible to RAP appear to be stressed easily. RAP itself may cause stress by creating new problems. Organic RAP is due to an organic disorder, most commonly inflammatory bowel disease, chronic appendicitis, peptic ulcer disease, H. pylori infection, parasitism, urinary tract disease, and sickle cell disease. In adolescent girls, pelvic inflammatory disease and ovarian cyst are possible causes

Functional RAP arises from abnormal functioning of a nondiseased organ as a result of interaction between constitutional and environmental factors. Why some pt develop abdominal pain and others do not is unknown. Perhaps anxiety alters autonomic & GI function, causing pain in susceptible pt Symptoms and signs Psychogenic RAP may occur daily or several times per week or month. The pain is generally vague and ill–defined but sometimes is crampy or colicky. Some pt wake early because of discomfort Pain is most often periumbilical. The symptoms progress little or not at all Any change in the location or pattern of pain suggests an organic condition! Organic RAP is described as constant or cyclic (related to activities or diet); is well localized, especially to areas other than periumbilical region; and may penetrate to the back. It frequently wakes the pt. Associated findings (depending on the underlying disease) include fever, changes in stool consistency, color, or elimination

pattern, blood in the stools, vomiting, hematemesis, abdominal distention, joint symptoms, change in appetite, and weight loss. Functional RAP depends on the underlying cause: Cramping and bloating are common in lactase deficiency; cramping pain in a lower quadrant is common with menstruation. Diagnosis The persistence, recurrence, and chronicity of RAP differentiate it from an acute abdomen pain. However, determining whether RAP is psychogenic, organic, or functional may be difficult. The history should begin with initial onset of pain and its frequency, nature, and location; the relationship to meals, defecation and voiding; and the results of any treatment. Psychogenic RAP is commonly associated with headache, dizziness, facial pallor. Psychosocial characteristics include immaturity, dependence on parents, anxiety, depression, and perfectionism. 79 Parents are often anxious, overprotective, authoritarian, and preoccupied with the child. A family history of chronic somatic

complaints (headache, “nerves”, depression) is common. Except for periumbilical discomfort on palpation, findings are typically negative. Organic RAP requires appropriate testing (e.g US, barium swallow, endoscopy, gynecologic exam, urine cultures, blood tests, biochemistry, stool examination, liver tests etc.) Functional RAP is best diagnosed by a through history defining associated symptoms or precipitating factors. Causes of functional RAP include inappropriate diet, ineffective toilet training leading to constipation, dysmenorrhea, or lactose intolerance secondary to physiologic decline in lactase activity occurring between ages 10 to 20 years in some pt. Prognosis and treatment Psychogenic RAP has a long–term prognosis, and no treatment regimen is universally successful. Some children later develop other somatic complaints or emotional difficulties. In organic or functional RAP, the prognosis and treatment depend on the underlying condition. In case of psychogenic RAP, the

family should be reassured that the child is not in physical danger. The physician should explain the negative laboratory findings and the nature of the problem. The first step in treatment is to avoid the negative psychosocial consequences of chronic pain (e.g absences from school). The next step is to work with the family to eliminate unnecessary stress and to help the child to cope effectively with unavoidable stress. Psychiatric referral may be required when symptoms persist. 80 38) ACQUIRED IMMUNODEFICIENCY (see also topic 45B) Causes: 1. Premature and newborn infants 2. Infections (bacterial, viral (AIDS, EBV, CMV, hepatitis, measles, rubella), fungal, TBC 3. Neoplastic disease (leukemia, Hodgkin disease, nonlymphoid cancer) 4. Surgery and trauma (burns – loss of immunoglobulins through the skin, splenectomy – results in loss of phagocytes, stress) 5. Autoimmune diseases (SLE, rheumatoid arthritis, graft–versus–host disease) 6. Protein–losing states

(protein–losing enteropathy, nephrotic syndrome – loss of immunoglobulins, intestinal lymphangiectasia – loss of immunoglobulins and cells) 7. Immunosuppressive treatment (cancer chemotherapy, radiation therapy, immunosuppressants, corticosteroids) 8. Hereditary and metabolic diseases (malnutrition – zinc deficiency causes impaired cell–mediated immunity; copper and iron deficiency causes impaired neutrophil function; uremia, DM, sickle cell anemia, chromosomal abnormalities) Secondary immunodeficiency is a common cause of pediatric illness. Symptoms result from a combination of the primary disorder and the complicating immunodeficiency. Whenever possible, treatment should be directed at the primary disorder. Occasionally, immunologic methods may help; for example, varicella–zoster immune globulin may prevent varicella in pt. with leukemia and iv IgG replacement provides protection where antibody production is decreased. IgG dos not help when loss (as in nephrotic

syndrome) is responsible for hypogammaglobulinemia. 81 39) INJURY OF THE CNS. INTRACRANIAL HYPERTENSION Head injury Clinical features: - Concussion produces a transient loss of consciousness, with amnesia for the event but with no obvious pathologic cerebral changes. Diagnosis: Neurologic examination is not remarkably changed except for the change in mental status. Nystagmus and a positive Babinski’s reflex may be present for several hours. Therapy is symptomatic Headache, dizziness, and poor attention may persist for up to 1 year after the injury. - Contusion and laceration of the brain can be the result of a depressed skull fracture, a penetrating injury, or a closed injury. Control of intracranial pressure is essential because cerebral edema commonly occurs. The level of coma is usually greatest by the 4th day, when cerebral swelling is at its maximum level. Depressed skull fractures and penetrating injuries require surgery as soon as the pt. is stable to minimize the

possibility of meningitis Complications: - Epidural hematoma occurs within hours of the injury in an adult but may not develop 1 to 2 days in a child. Tearing of the dural veins of the middle meningeal artery is responsible for this complication, which should be suspected when a pt.’s condition deteriorates The diagnosis is made by CT scan. Surgical evacuation is necessary - Subdural hematoma can develop even more slowly than epidural hematoma. Especially in infants < 8 months, this problem might not develop for several weeks after injury. A common presenting sign is an enlarging head circumference and change in feeding habits or personality. A form of childhood abuse, shaken–impact syndrome, may involve subdural hematoma and retinal hemorrhages. Treatment requires surgical drainage and treatment of increased IP - Parenchymal hematoma (blood clot within the brain) rarely requires surgical intervention. However, if medications don’t prevent swelling, evacuation may be

necessary. - Transtentorial herniation may occur as a result of generalized cerebral edema or a space– occupying lesion. This complication is suspected if pupils are dilated and do not respond to light and 6th nerve palsy develops. Rapid treatment with a dehydrating agent (eg mannitol) and hyperventilation are often necessary. - Recurrent meningitis is a risk when the injury provides an entrance to bacteria. The cause is not always obvious. CSF rhinorrhea and otorrhea should be looked for - Arachnoid (leptomeningeal) cyst occurs in linear fracture. It is more common in children < 3 years. Onset is several months after injury The pt presents with enlarging head circumference Excision of the cyst may be necessary. 82 Prognosis: The duration of coma after head injury correlates with the future disability. Normal cognitive and motor function is unlikely if the coma lasts > 1 week in an adolescent and 2–4 weeks in children. A seizure at the time of injury does not

correlate with future epilepsy On the other hand, coma lasting > 1 day correlates with future epilepsy. A depressed skull fracture or penetrating trauma leads to a seizure disorder in 80%. Posttraumatic personality changes and learning problems are common after significant injury. Spinal cord injuries Clinical features: The level of injury is determined by the lack of sensation below the level of the lesion. Diagnosis: MRI scan determines if the lesion needs surgery Therapy: After stabilization of the pt. and surgery, the most immediate concern is urinary retention Cathetrization of the bladder is necessary. Prognosis: The pt may be areflectic distal to the injury for several weeks, after which spasticity commonly appears. Complete paralysis 5 to 10 days after the injury suggests permanence Intracranial hypertension The intracranial contents usually consist of brain tissue (80%), cerebrospinal fluid (12%), and blood (8%). Cerebral blood flow remains stable over a wide range of

systemic blood pressures (autoregulation). Normal ICP is < 10 mm Hg ICP remains constant for some time after the volume of intracranial content begins to increase via compensatory mechanisms: - ↓CBF: Shunting of blood out of the venes to the central veins of the chest. - ↓CSF: Shunting of CSF towards the lumbar cistern. - Expansion of cranial sutures and fontanelles in infants. - ↓ECF: a later compensation. When these compensation mechanisms fail to work, IP raises rapidly and cerebral blood flow is decreased. If the situation is not reversed, herniation and brain death will occur due to ischemia Causes of increased ICP: - Cerebral mass (tumor, traumatic hemorrhage), - increased CSF volume (hydrocephalus), - increased intravascular blood volume (AV malformation), - increased interstitial fluid (cerebral edema), - extravascular fluid collection (epidural, subdural, subarachnoid, or intracerebral hemorrhages). Approach: - Children with increased ICP are at

risk for loss of airway control and hypoventilation. Intubation should be undertaken. 83 - Monitoring: Pulse oximetry and cardiac rate and rhythm continuously. Arterial pressure Central venous pressure monitoring helps to guide i.v fluid administration ICP is measured directly. Obtained values may be used to calculate cerebral perfusion pressure (CPP) - Reduction of intracranial contents volume: Intracranial volume can be normalized by removal of space–occupying lesions (e.g tumor resection) CSF can be removed from the ventricles via a ventriculostomy. Interstitial fluid/edema reduction involves: Steroids help in cerebral edema of the vasogenic type (associated with trauma or tumors). Ischemia causes cell death and cytotoxic edema, which rarely responds to therapy. Osmotic agents such as mannitol reduce ICP within minutes. Within 1 hour, osmotic diuresis occurs but intravascular volume depletion must be prevented because systemic dehydration causes secondary ischemia to

brain cells! Reduction of cerebral blood volume involves: Hyperventilation reduces ICP because cerebral blood flow is directly proportional to pCO2. Short–acting barbiturates constrict cerebral blood vessels and also decrease cerebral metabolic demand. Both of these effects reduce cerebral blood volume, thus reducing ICP Modest elevation of the head may promote drainage of the venous blood. 84 40) DISEASES WITH HEPATOMEGALY AND/OR SPLENOMEGALY Hepatomegaly In the history ask about trauma, abdominal distention, pain, fever, appetite, weight loss, jaundice. Note exposure to hepatotoxins, infections (hepatitis, mononucleosis, tuberculosis, amebiasis, other protozoan parasites). Ask about underlying conditions such as sickle cell disease and other hematologic disorders, cardiac disease, collagen vascular disorders, and storage diseases. All these condition may cause hepatomegaly. Physical examination: Determine liver size by using percussion to identify the upper and lower borders

of the liver. The mean span changes from 5 cm at 1 week of age to 7 cm in early adolescence. The lower edge of the liver should not extend below the costal margin more than 2 cm in infancy and 1 cm in childhood. Conditions that may displace a normal liver downward include pulmonary hyperinflation, pneumothorax, retroperitoneal mass, and subdiaphragmatic abscess. Riedel lobe, a normal anatomic elongation of the right lobe may be mistaken for hepatomegaly. The quality of liver on palpation is as important as its size. Normally, its edge is rubbery soft, sharp, and smooth. This consistency is often maintained when the liver is enlarged because of acute hepatitis, fatty infiltration, passive congestion, or early biliary obstruction. The cirrhotic liver edge is firm, blunt, and irregular. Hard lumps suggest malignant infiltration Audible friction rubs, although rare, also indicate tumor. Associated ascites, splenomegaly (suggesting increased portal vein pressure) should be noted. Skin

lesions, subcutaneous nodules, lymphadenopathy, spider angiomata, and signs of storage diseases should be noted as well. Laboratory: Elevated α–fetoprotein and carcinoembryonic antigen suggest malignancy. Prolongation of clotting time, elevation of serum ammonia, and decreased serum albumin indicate liver failure. Elevation of bilirubin, bile acids, lipids, and ALP suggest a problem with bile secretion. Elevations in AST and ALT reflect the amount of acute damage to hepatocytes Hepatomegaly may be caused by extramedullary hematopoiesis, as erythropoietic tissue is present within the liver sinusoids of term infants. Causes of extramedullary hematopoiesis include erythroblastosis fetalis and other severe neonatal anemias. Congenital infections that can produce hepatomegaly include toxoplasmosis, rubella, herpes simplex, CMV, and syphilis. Pyogenic and mycotic abscesses are commonly multiple, and amebic abscess is usually solitary. X– ray may show calcification in TBC. Helminthic

infections include ascaris in the common bile duct Mycotic infections are actinomycosis and cryptococcosis. Fatty infiltration of the liver can result from malnutrition and vitamin deficiencies. Malnutrition can be related to inadequate caloric and protein intake or malabsorption. 85 Hepatoblastoma and hepatocellular carcinoma account for most primary hepatic malignancies. Hepatoblastoma are most frequent in children < 5 yr, and HCC has peaks < 4 & between 12-15 yr. Disorders that predispose to HCC include: - Hepatitis B, C & D. - Biliary atresia. - Liver cirrhosis. - Storage diseases & tyrosinemia. The only effective treatment is resection (33% survival). Storage disorders related to inborn errors of metabolism include glycogen storage disease, galactosemia, lipid storage disease, Gaucher disease, mucopolysaccharidosis, amyloidosis, and alfa–1 antitrypsin deficiency. Hemosiderosis and hemochromatosis are caused by deposit6ion of iron in the liver.

Splenomegaly The tip of the spleen is normally palpable in premature infants and in about 1/3 of full term infants. It may be felt up to 3 to 5 yr. Thereafter, a palpable spleen is enlarged In the history, it is important to ask about acute illness, associated personal and family hematologic disorders, and possible food or drug exposure precipitating the episode. Presence or recurrence of systemic symptoms such as fever, jaundice, pallor, bleeding, tea–colored urine, bone and joint pains, weight loss, abdominal sweating, anorexia, and abdominal enlargement is important to investigate. Physical examination should be performed then. If the spleen is huge, its edge extends down into the left lower quadrant. Size and consistency should be noted Auscultation may reveal a bruit or friction rub. Other masses on the left side of the abdomen that should be differentiated from the spleen are a large kidney, pancreatic or ovarian cyst, omental mass, mesenteric cyst, retroperitoneal tumor, and

adrenal neoplasm. Lymph nodes and liver enlargements should be checked as well Any signs of storage diseases and ascites should be noted. Signs of a hematologic disorder are pallor and jaundice associated with anemia, an abnormal smear, elevated indirect bilirubin, and reticulocytosis. Hemolytic anemias that can cause splenomegaly (due to increased trapping and destruction of abnormal RBCs) include hemoglobinopathies, thalassemia, enzyme defects, hereditary spherocytosis, and other autoimmune hemolytic anemias. All syndromes in which circulating cytopenia is associated with splenomegaly are called hypersplenism. Splenectomy is sometimes necessary in such cases 86 Acute or chronic infection causes splenomegaly. Bacterial causes include sepsis, bacterial endocarditis, typhoid fever, tuberculosis, cat–scratch disease, brucellosis, typhus, syphilis, and Lyme disease. Viral agents include EBV, CMV, HIV, and hepatitis viruses Protozoans include malaria, toxoplasmosis, kala–azar and

visceral larva migrans. Mycotic agents are histoplasmosis and coccidiodomycosis. Congestive splenomegaly: Chronically increased splenic venous pressure may result from hepatic cirrhosis, portal or splenic vein thrombosis, or certain malformations of the portal venous vasculature. Associated bleeding from esophageal varices may be worsened by the superimposed thrombocytopenia induced by splenomegaly (increased trapping). Splenic venography and MRI, which may demonstrate or exclude extrahepatic portal obstruction, aid in diagnosis. Treatment is determined by the underlying disorder Myeloproliferative disorders: These disorders include polycythemia vera, myelofibrosis, chronic myeloid leukemia, and essential thrombocythemia. The spleen become enlarged, particularly in myelofibrosis, in which bone marrow is obliterated and the spleen assumes an increasing hematopoietic function. Splenomegaly may be massive and splenectomy may help Lymphoproliferative disorders: The spleen is enlarged in

chronic lymphocytic leukemia and in the lymphomas. Splenomegaly is usually associated with lymphadenopathy, immunoglobulin abnormalities, and lymphocytic dysfunction. Lipid storage diseases: Glucocerebroside (in Gaucher’s disease) or sphingomyelin (in NiemannPick disease) may accumulate in spleen. Collagen vascular disorders: Splenomegaly and leukopenia may coexist in SLE and JRA. The pathogenesis of splenomegaly is unknown, and splenectomy is beneficial in 50%. 87 41) MUMPS (EPIDEMIC PAROTITIS). A highly contagious, acute viral disease. The most characteristic feature is painful enlargement of the salivary glands, primarily the parotid glands. The disease is almost always benign and resolves spontaneously; 20–40% of infections are subclinical. Epidemiology: Mumps if found throughout the world and occurs year round, although epidemics are more frequent during the winter and spring. Etiology and pathogenesis: Mumps is caused by a paramyxovirus, the mumps virus. The virus is

spread by direct contact and airborne droplet nuclei. Viral transmission usually occurs during the period 48 hours before to 7 days after the appearance of the swollen salivary glands. Clinical features: - The incubation period varies from 2 to 4 weeks (usually 16-18 days). - Within 1 day, the illness manifests as pain and swelling in one or both parotid glands. - Pain and erythema often occur at the opening of the parotid duct. - The swelling peaks in 1 to 3 days and then resolves over a 3–7 day period. - Fever is usually moderate; it is absent in 20% of pt. Diagnosis: Known exposure, clinical symptoms, physical examination. An elevated serum amylase level due to parotid involvement or pancreatitis is an lab indicator. Definitive diagnosis requires culture of the virus or significant rise in circulating mumps antibody. Therapy: It is symptomatic and supportive. Prevention: Active immunization with live attenuated mumps virus is effective. Complications are uncommon and

usually not severe, including: - meningoencephalitis, unilateral deafness. - orchitis, oophoritis, - pancreatitis, nephritis, myocarditis, arthritis, and thyroiditis. 88 42) TERATOGEN FACTORS IN DISEASES OF CHILDREN. General principles – proving the relationship between a substance to which a fetus is exposed and a birth defect involves several important factors: - Timing of exposure is vital, because morphogenesis occurs only for the first 8 to 12 weeks’ gestation. Growth and CNS systems are affected thereafter Exposure before implantation (days 7 to 10 postconception) results in loss of embryo or produces no effect. - Dosage is important – many teratogens have a threshold level below which there is no effect. - Genetic constitution of the mother and the fetus determines whether the fetus will be affected. Medication, drugs, and chemicals can interfere with embryonic and fetal development. - Anticonvulsant drug – Hydantoin causes a specific pattern (growth

disturbance, skeletal and CNS abnormalities. Valproic acid causes neural tube defects - Thalidomide has been associated with limb malformation and cleft palate. - Retinoic acid results in brain, ear, and heart malformations. - Tetracycline causes dark staining of teeth. - Other teratogenic drugs and chemicals include anticoagulants, antithyroid, cancer chemotherapeutics, ACE inhibitors, lead, lithium, and mercury. Infectious agents: Especially the TORCH (Toxoplasma gondii, Rubella, CMV, and HSV organisms cause poor growth, abnormal CNS, hearing loss, cataract, congenital heart defects). Treponema pallidum causes poor growth, abnormal CNS and skeletal development. Radiation (usually therapeutic) causes fetal malformations. Usually the dose received by the fetus during maternal diagnostic X-ray is below the threshold for teratogenic effect. Maternal metabolic disorders: - Diabetes: 15% of fetuses have risk for birth defects involving heart, skeleton, brain, and spinal cord.

Hypoglycemia seem to be responsible Careful control of diabetes decreases the risk - Phenylketonuria: Infants are exposed to metabolites of phenylalanine. Brain and heart defects occur in nearly every fetus. Dietary control of the mother decreases the risk significantly Mechanical forces: - Intrauterine forces result in deformations: Tumors, fibroids, or abnormal uterine anatomy result in constrained fetus => breech position, facial distortion, dysplasia of the hip, club feet. - External forces result in disruption of fetal blood supply. It can also cause bands of tissue from the amniotic sac resulting in limbs hypoplasia or transverse amputations. Maternal alcohol use: Alcohol is the most common teratogen. A genetic predisposition may play a role in the severity of the alcohol effect. The timing and amount of use is also important Increased risk of miscarriage is proportional to the amount of alcohol consumed. 89 Fetal alcohol syndrome occurs 2:1000 newborns (an

additional 7:1000 may have milder defects!). It affects 45% of fetuses of females who consume more than 5 alcoholic drinks per day while pregnant. Clinical features include CNS dysfunction (mental retardation, childhood hyperactivity), growth deficiencies, facial dysmorphism, joint contractures, and kidney malformations. h) Environmental pollutants are suggested but it is difficult to study such agents. 90 43) ACUTE DIARRHEA – DD AND TREATMENT. Acute infectious diarrhea (Gastroenteritis) Viral Viruses are the most common cause of acute gastroenteritis. Rotavirus, with 8 serotypic variants, is the most common. As with most viral pathogens, rotavirus affects the small intestine, causing voluminous watery diarrhea without leukocytes or blood. Rotavirus affects mainly infants aged 3– 15 months. Peak incidence is in the winter months The virus is transmitted via the fecal–oral route and survives for hours on hands and for days on environmental fomites. Diagnosis and treatment The

incubation period for rotavirus is 24–48 hours. Vomiting is the first symptom followed by low grade fever and profuse watery diarrhea. Diarrhea lasts for 4–8 days Pt. become dehydrated from unreplaced fecal water loss and they may become hypernatremic Metabolic acidosis results from bicarbonate loss in the stool, ketosis from poor intake, and lactic acidemia from hypotension and hypoperfusion. Replacement of fluid and electrolyte deficits and ongoing losses is critical, esp in small infants. Intestinal lactase levels are reduced during rotavirus infection. Lactose-free diet shorten the diarrhea Reduced fat intake reduces nausea and vomiting. Antidiarrheal medications are ineffective Rotavirus antigens can be identified in stool. Serum antibodies are present, but their role in prevention is not clear. Prevention: good hygiene and prevention of fecal–oral contamination Other viral pathogens in stool are identified by electron microscopy, special viral cultures, or enzyme–linked

immunoassays in infants with diarrhea. Second most common pathogens are adenoviruses and caliciviruses. The symptoms of enteric adenovirus infection is similar to rotaviral, but infection is not seasonal and duration may be longer. Caliciviruses cause mainly vomiting, but also some diarrhea, usually in common source outbreaks (water–born?). The duration of symptoms is 24–48 hours Other pathogenic viruses include astroviruses, corona-like viruses, & CMV (immunocompromised). Bacterial Most bacterial agents cause acute disease. The most common agents include enterotoxigenic bacteria (Bacillus cereus, C difficile, C perfringens, E coli, S aureus, Vibrio cholerae); or invasive bacteria (E coli, Salmonella, Shigella, Yersinia enterocolitica). Epidemiology, duration, character, and frequency of vomiting and diarrhea in relation to the child’s age may indicate the cause. Usually, 1 family member has recently had gastroenteritis Infants < 6 months may develop dehydration, metabolic

acidosis, and electrolyte imbalance as early as 24 hours after onset. Lethargy, oliguria, and weight loss are signs of dehydration 91 Stool cultures may be useful for differentiating bacterial from viral gastroenteritis and sensitivity studies guide antibiotic therapy. There are abundant PMN in stool when bact infection is present Treatment consists of fluids and electrolyte replacement. Antibiotics should only be used for specific indications (e.g Vibrio cholerae, Clostridium difficile, Shigella, or Giardia lamblia) Parasitic Giardia is a small bowel pathogen causing malabsorptive diarrhea, nausea, and vomiting. It may be chronic in IgA deficient hosts. Entameba histiolytica may be a chronic colon pathogen especially in caecum and rectum. It causes a chronic mucoid or bloody diarrhea Cryptosporidium causes severe watery diarrhea in immunocompromised pt. Microsporidial diarrhea is common in HIV and rare in immunocompetent children. 92 44) PREVENTION OF RICKETS (VIT-D

DEFICIENCY). HYPERVITAMINOSIS D Metabolic bone disease resulting from vitamin D deficiency is called rickets in children and osteomalacia in adults. Etiology Inadequate exposure to sunlight and low dietary intake are usually necessary for development of clinical vitamin D deficiency. Nutritional rickets is rare in the USA now Rarely, very low intake of calcium or phosphorus may be the cause. Vitamin D deficiency may also be caused by defects in the production of 25(OH)D3 or the action of 1,25(OH)2D3. The deficiency may occur in hypoparathyroidism; in hereditary diseases, such as familial hypophosphatemic (vitamin D–resistant) rickets; and in various other diseases. Some diseases interfere with the absorption of vitamin D or with the formation of its active metabolites (chronic liver disease, severe renal damage or renal tubular acidosis). Deficiency of vitamin D metabolites results in vitamin D–resistant states. Generally, rickets may occur when the supply of vitamin D is

inadequate, its metabolism is abnormal, or tissues are resistant to its action. Categorising rickets by etiology is important in relation to its effective treatment! Pathology In children, changes include defective calcification of growing bone and hypertrophy of the epiphyseal cartilages. Epiphyseal cartilage cells cease to degenerate normally, but new cartilage continues to form, so that the epiphyseal cartilage becomes irregularly increased in width. Calcification then stops, and osteoid material accumulates. Treatment with vitamin D permits calcium and phosphate deposition, osteoid material ceases to form, and normal enchondral production of new bone is resumed. Symptoms and signs - Young infants are restless and sleep poorly. - They have reduced mineralization of the skull. - In older infants, sitting and crawling are delayed as is fontanelle closure. - In children age 1-4, epiphyseal cartilages enlarge at the lower ends of radius, ulna, tibia & fibula. -

Kyphoscoliosis develops, and walking is delayed. - In older children walking is painful, and in extreme cases, such deformities as bowlegs and knockknees develop. - Rachitic tetany is caused by hypocalcemia and may accompany infantile vitamin D deficiency. - Bone changes, visible on X-rays, precede clinical signs. The diaphyseal ends loose their sharp, clear outline; are cup-shaped, and show rarefaction. Later, the distance between bones appears 93 increased because the true ends are noncalcified and invisible. Characteristic deformities result from the bones bending at the cartilage-shaft junction because the shaft is weak. Diagnosis: 25(OH)D3 and other vitamin D metabolites may be measured in plasma. In nutritional rickets, 25(OH)D3 levels are very low, and 1,25(OH)2D3 is undetectable. A low serum phosphorus and a high serum alkaline phosphatase are characteristic. Serum PTH is elevated, serum calcium is normal, and urinary calcium is low. Treatment: With adequate calcium

and phosphorus intake, uncomplicated rickets may be cured by giving vitamin D. Rickets that results from defective production of vitamin D metabolites do not respond to the usual doses effective in nutritional rickets. Hypervitaminosis D Vitamin D is toxic in large amounts. Because it is not water-soluble, it cannot be excreted by urine if toxic amounts are present in the body. Manifestation of vitamin D toxicity includes: - Hypercalcemia, hypercalciuria, and hyperphosphatemia. - The result is anorexia, renal failure & metastatic calcifications. Treatment include stopping therapy, low calcium diet, high fluid intake. Furosemide may increase calcium secretion. Calcitonin may decrease hypercalcemia 94 45) MENTAL RETARDATION Definition Significantly subaverage intellectual quotient (IQ < 70) with related limitations in two or more of the following: communication, self–care, home living, social skills, community use, health and safety, functional academics, leisure, and

work. Classification based on IQ alone (mild: 52–68; moderate 36–51; severe: 20–35; profound: < 20) has been replaced by that based on level of support needed (intermittent – does not require constant support; limited – ongoing support of varying intensity; extensive – daily and ongoing consistent levels of support; pervasive – high level of support for all activities of daily living). About 3 % of the total population are with an IQ < 70. Etiology Intelligence is polygenetically and environmentally determined. When both parents are retarded, 40% of their offspring are retarded. In 60–80 % of cases, the cause of MR is unknown 1. Prenatal factors: a. Chromosomal abnormalities – i. Trisomies: eg Down syndrome d/t trisomy of 21 ii. Partial deletion eg of chromosome 5 in the cri du chat syndrome iii. Sex chromosome abnormalities eg Klinefelter’s syndrome [XXY], Turner’s syndrome [X0], or various mosaicisms. iv. Person with mild familial mental retardation may

have the fragile X syndrome (1:1000 births, affecting more males than females; physical features include increased head size, macro-orchidism, prominent jaw, & protruding ears). b. Genetic metabolic and neurologic disorders – Retarded infants with idiopathic hypotonia should be screened for neuromuscular disorders. c. Congenital infections – A major cause of mental retardation It may be due to rubella virus, CMV, Toxoplasma gondii, and Treponema pallidum. d. Prenatal drug exposure – eg fetal alcohol syndrome and fetal hydantoin syndrome e. Malnutrition – It may affect fetal brain development, resulting in mental retardation f. Radiation & chemicals 2. Perinatal factors: Prematurity, CNS bleeding, breech or high forceps delivery, multiple births, placenta previa, preclampsia, and asphyxia may increase the risk of mental retardation. 3. Postnatal factors: Viral and bacterial encephalitis or meningitis, severe malnutrition, severe head injuries or asphyxia. 95 Symptoms

and signs - Low IQ combined with limitations in social, language, and self–help skills. - Seizures, psychiatric disorders, and behavioral disorders (explosive outbursts, aggressive behavior to normal stress) may be present. - Factors that predispose to unacceptable behavior include lack of training, inconsistent discipline, brain damage, and impaired ability to communicate. - A mentally retarded adolescent may develop depression when he is socially rejected. Diagnosis A history (perinatal, developmental, neurological, and family history) identifies infants at risk of mental retardation. Visual and auditory assessments, CBC, and urinalysis should be performed For infants at high risk, tests to assess development and intelligence are performed. Genetic metabolic disorders may be suggested by their clinical manifestations. Isolated delays in sitting or walking (gross motor skills) and in pincer grasp or writing (fine motor skills) may indicate a neuromuscular disorder. Specific

lab tests are performed depending on the suspected cause Prevention Genetic counseling, amniocentesis, chorionic villus sampling, US, maternal serum α-fetoprotein (Down syndrome, myelodysplasia screening). Prenatal diagnosis enables a couple to consider abortion and family planning. Amniocentesis is indicated for all pregnant women > 35 years (increased risk of Down syndrome) and for women with family history of mucopolysaccharidosis, galactosemia, or Tay–Sachs disease. Other methods of prevention include rubella vaccination, and use of exchange transfusion to prevent hemolytic disease. Prognosis and treatment The more severe the retardation and the greater the immobility, the higher the mortality. Developmental retardation due to a perinatal insult may be overcome in a good learning environment. Family support and counselling are crucial An individualised program is developed with the help of appropriate specialists (neurologist, orthopedist, physical therapist, occupational

therapist, speech pathologist, audiologist, nutritionists, social workers, educators, psychiatrists). Every effort should be made to have the child live at home or in a community–based residence. The family with a retarded child living at home must have psychological support. Whenever possible, a retarded child should attend a normal day care centre or a regular public school. 96 46) CHRONIC DIARRHEA – DD AND TREATMENT. It is difficult to define chronic diarrhea because there are wide variations in normal bowel habits. Some infants pass one form stool every second to third day, whereas others may have 5-8 softy small stools daily. A gradual or sudden increase in the number and volume of stools combined with increased fluidity suggests that an organic cause of chronic diarrhea is present. Diarrhea may result from any of the following mechanisms: 1) interruption of normal cell transport processes for water, electrolytes, or nutrients. 2) decrease absorption surface area

(shortening of the bowel or mucosal disease). 3) increase in intestinal motility. 4) increase in unabsorbable osmotically active molecules in the intestinal lumen. 5) increase in intestinal permeability, leading to increased loss of water and electrolytes. Causes: 1. Antibiotic therapy – Diarrhea accompanies ATBs therapy 60% of cases Eradication of normal gut flora and overgrowth of other organisms may cause diarrhea. This diarrhea is watery, not associated with systemic symptoms, and decreases when ATBs therapy is stopped. Pseudomembranous colitis caused by the toxins produced by Clostridium difficile occurs in some pt. taking ATBs Pt develop fever, tenesms, and abdominal pain with diarrhea, which contains leukocytes, and sometimes blood. Treatment with oral metronidazole or vancomycin is recommended. 2. Extraintestinal infections – Infections of the urinary tract and upper respiratory tract (especially otitis media) are sometimes associated with diarrhea. 3. Malnutrition –

Malnutrition is associated with an increased frequency of enteral infections Decreased bile acid synthesis, pancreatic enzyme output, and disaccharidase activity, altered motility, and changes of the intestinal flora – all of which may cause diarrhea. 4. Diet – Overfeeding may cause diarrhea, especially in young infants Fruit juices, especially those high in fructose or sorbitol, produce osmotic diarrhea. Intestinal irritants (spices) and histamine-releasing foods (tomatoes, citruses, cheeses, wines, and fish) also cause diarrhea. 5. Allergic diarrhea – Protein allergy (often milk protein) is more common in infants under age 12 months, who may experience colitis. A history of atopy is common Older children may develop celiac-like syndrome. In infants, the condition usually disappears after 12 months Allergies to fish, peanuts, and eggs are more likely to be life-long. 6. Chronic nonspecific diarrhea – It is the most common cause of loose stools in thriving children The typical

pt. is a healthy child aged 6-20 months with 3-6 loose mucoid stools per day The diarrhea worsens with low-residue, low-fat, or high carbohydrate diet and during stress and infections. It clears spontaneously at about age 3 1/2 years No organic cause is discoverable A 97 high familial incidence of functional bowel disease is observed. Stool tests are negative Institution of high-fat, low-carbohydrate, high-fibre diet; avoidance of chilled fluids, and between-meal snacks may be helpful. Antidiarrheal medication may be helpful 7. Inflammatory bowel disease – ulcerative colitis, Crohn’s disease, eosinophilic gastroenteritis, allergic colitis 8. Secretory diarrhea – Toxin-associated secretory diarrhea is caused by enterotoxins secreted by enteric organisms including toxigenic E.coli, cholera, and other enteric organisms It causes acute watery diarrhea but may be chronic in pt. with dysmotility syndromes and bacterial overgrowth. Tumor-associated secretory diarrhea may be

associated with neuroblastoma, ganglioneuroma, carcinoid tumors, VIP-oma, or Zollinger-Ellison tumor. 9. Radiation enteritis – Causes diffuse mucosal damage and may cause strictures and obstruction 10. Chemotherapy – Most agents cause diffuse mucosal damage 98 47) IMMUNIZATION (ACTIVE AND PASSIVE VACCINES). Immunization is the process of creating immunity to a specific disease in an individual. Active immunization is a stimulation with a specific antigen to promote antibody formation in the body. The antigenic substance may be in one of four forms: 1. dead bacteria, as in typhoid fever immunization, pertussis, cholera, HAV 2. dead viruses, as in the Salk poliomyelitis injection (IPV), rabies, HBV, influenza 3. live attenuated virus, as in smallpox vaccine and Sabin polio vaccine (taken orally-OPV), BCG, measles, rubella, mumps (MMR), chickenpox, yellow fever 4. toxoids, altered forms of toxins produced by bacteria, as in immunization against tetanus and diphtheria Active

immunization induces the body to produce its own antibodies and to go on producing them, protection against disease will last several years, in some cases for life. Active immunization is not without risks and has recently become an issue among activist parents whose children have suffered injury from immunization, mainly whooping cough. Circumstances that require postponement of immunization include acute febrile illness, immunologic deficiency, hypersensitivity, some neurological disorders, pregnancy, immunosuppressive therapy, and administration of gammaglobulin, plasma, or whole blood transfusion 6 to 8 weeks prior to the scheduled immunization. Because of their potential for triggering anaphylaxis in hypersensitive persons, all immunizing agents should be given with caution and only after complete medical history of the pt. Emergency equipment and drugs should be readily at hand. Minor reactions to immunization occur in most vaccinated subjects, major reactions are consequences of

the special immunological state of the subject, immunization accidents are consequences of bad quality vaccine (vaccine must be stored and utilized according to manufacture advise) or malpractice. Some vaccines are given in combinations, which offers advantages, particularly if the child may be inaccessible for further immunization. However, separate vaccines must never be given in combinations! Passive immunization is a transient immunization produced by administration of pre-formed antibody or specifically sensitized lymphoid cells. The person is protected only as long as these antibodies remain in his blood and are active - usually 4-6 weeks. 99 Human immune globulin (IG) is an antibody-rich fraction of plasma obtained from normal healthy donors. It contains IgG Since maximal serum antibody level may not be achieved until 48 hours after IM injection, IG must be given as soon after exposure as possible. IG may be used for prophylaxis in hepatitis A, immunoglobulin deficiency,

varicella, and rubella exposure in the first trimester of pregnancy. Administration is painful, anaphylaxis may occur Hyperimmune globulin is prepared from persons with high titers of antibody against a specific organism or antigen. It is derived from artificially hyperimmunized donors or from persons after natural infections. It is available for hepatitis B, rabies, tetanus, and varicella-zoster Administration is painful and anaphylaxis may occur. IV immune globulin (IVIG) was developed to provide larger and repeated doses of immune globulin. IVIG is the product of choice in the treatment and prophylaxis of severe bacterial and viral infections. Administration is painless and adverse effects uncommon In case of unavailable human serum, animal serum is applicated, but hypersensitivity test must be done before. The most common vaccines include: 1. DTP vaccine - diphtheria, tetanus, and pertussis Adverse effects include encephalopathy, convulsions, collapse, and anaphylaxis. The initial

series of three primary doses (2, 4, and 6 months) is followed by a booster at age 15 to 20 months and another at 4 to 6 years. Subsequent routine tetanus boosters every 10 years maintain protection. 2. Poliomyelitis - OPV (oral polio vaccine) is given in series of three doses with at least 4 weeks in-between them. The only known adverse effect of OPV is paralysis, which occurs in 006/ 1.000000 doses IPV (inactivated polio vaccine) is incapable of causing paralysis and is therefore recommended in immunocompromised pt. However, it is less effective, administered parenterally, and more expensive than OPV. 3. Measles, Mumps, and Rubella vaccines are usually given in combination (MMR vaccine) Recommendation is for two doses of vaccine, one at 12 to 15 months and another at age 4 to 6 years to boost immunity. A mild, noncommunicable infection (fever, malaise, rash) is produced in 15 % of vaccine recipients. Subacute sclerosing panencephalitis (SSPE) has been reported in association with

measles vaccination. Immunity after MMR vaccination is probably life-long 4. Haemophilus influenzae type b (Hib) - Vaccines produce from the purified capsule conjugated to a protein carrier are affective in preventing Hib disease in children. Three doses are given at the age of 2, 4, and 6 months, and booster vaccination over age of 12 months. 100 5. Hepatitis B - Vaccines are produced by recombinant DNA technology Three IM doses are required and should be initiated during the newborn period, a second dose is given 2 months later, and a third dose given by 6 to 18 months of age. Adult vaccination with hepatitis B is possible as well. 6. Influenza - Influenza occurs each winter and early spring, often associated with significant morbidity and mortality rates in certain high-risk persons (adults and children with chronic disorders of pulmonary and cardiovascular systems, residents of nursing homes, persons above 65 years, adults and children with chronic metabolic diseases (e.g DM),

renal dysfunction, hemoglobinopathies, and immunosuppression). The vaccine may be administered to children over age 6 months. The vaccine is given each year 7. Immunization for travelers - required for travel to regions where infectious diseases not seen in our countries are endemic (e.g tick-borne encephalitis, cholera, yellow fever, hepatitis A) 8. Tetanus - active vaccine (toxoid) is used as prevention or immediately in case of deep dirty wounds, bites from animals, operations, circumcision, piercing, chronic ulcers, miscarriage, abortion, or infected umbilical cord in the newborns. In some cases, immunization is not successful. Factors influencing the effects of immunization: - Way of immunization (e.g hepatitis B admin intradermally causes decreased antigenicity) - Age & immunodeficiency. - Antibodies in the circulation. - Dose of antigen and its antigenicity, persistence of antigens. - Timing of immunization. Recommended vaccination schedule for children: AGE

VACCINATION Birth-hospital discharge HBV 2 months DTP 1, OPV 1, Hib, HBV 4 months DTP 2, OPV 2, Hib 6 months DTP 3, HBV 12 months Hib 15 months MMR 1, DTP 4, OPV 3 4-6 years MMR 2, DTP 5, OPV 4 14 - 16 years (repeat every 10 years) Td 101 Compulsory vaccination in Hungary AGE VACCINATION Birth - hospital discharge HBV 0-6 weeks BCG 2 months Hib 3 months DTP Ia + IPV 4 months DTP Ib + OPV + Hib 5 months DTP Ic + OPV + Hib 15 months MMR + OPV + Hib 3 years DTP II + OPV 6 years DTP III + OPV 11 - 12 years DT 11 years MMR (reactivation) 14 years HBV Compulsory vaccination for persons at risk: Typhoid fever, diphtheria, pertussis, tetanus, measles, rubella, HAV, HBV, rabies, travelers (e.g yellow fever) Recommended vaccination for persons at risk: Tetanus, influenza, HBV, travelers (diphtheria, pertussis, tetanus, poliomyelitis, typhoid, HAV, HBV), vaccinations connected to occupational risk 102 48) SHOCK TREATMENT. Shock is s complex

metabolic state characterized by impaired delivery and utilization of oxygen and other substrates by the tissues. Causes: 1. Noncardiovascular causes: hypoxemia, hypoglycemia, and toxins (eg cyanide) that impair delivery and/utilization of oxygen. 2. Cardiovascular causes: derangements of the 3 components of CO: preload (eg hemorrhage); contractility (e.g myocarditis); and afterload (eg sepsis, anaphylaxis) Phases: All forms of shock mat progress through 3 phases: 1. Compensated shock - signs of shock with maintained BP 2. Decompensated shock - signs of shock associated with hypotension 3. Irreversible shock - prolonged ischemia causing permanent organ failures Types of shock: a) Hypovolemic shock is caused by decreased circulating volume (preload) b) Cardiogenic shock is caused by decreased contractility. c) Distributive shock is caused by disordered perfusion of tissues, often caused by sepsis or toxins or allergens (anaphylactic shock). If you need, for more details about clinical

picture and pathophysiology see intensive care books. Treatment: a) Hypovolemic shock: - Primary therapy is restoration of circulating volume with crystalloid or colloid solutions, or blood products if shock is due to hemorrhage. - Secondary therapy is inotropic support, often used during initial resuscitation with IV fluids. - Fluid balance and physical examination must be frequently reassessed. If there is a poor response to fluid therapy, central venous or pulmonary artery pressure monitoring may help. b) Cardiogenic shock - Inotropic support with DA, dobutamine, epinephrine, or milrinone. When pharmacologic vasodilation to reduce afterload is desired as well, dobutamine or milrinone are preferred. - Diuresis - Continuous monitoring of cardiac rhythm, arterial BP, and urine output is indicated. Central venous pressure monitoring is helpful. c) Distributive shock - Monitor cardiac rhythm, arterial and central venous pressure, and urinary output; pulmonary arterial

cathetrization may be helpful. 103 - Attempt to correct triggering insult (ATBs-sepsis, antihistamines, corticosteroids anaphylaxis, antidotum - intoxication). - Maintain circulating blood volume (fluid i.v) - Inotropic support - In refractory cases with persistent hypotension, promote vasoconstriction by alfaadrenergics. 104 49) ALTERED STATES OF CONSCIOUSNESS – DD AND TREATMENT. When assessing a child’s altered state of behaviour or decreased responsiveness, the developmental age and typical responses to various stimuli must be considered. Definitions: - Delirium is an altered state of behavior in which the pt. appears alert, but is confused, irritable, and has inappropriate reactions to stimuli. - Coma is a state of unarousable unresponsiveness. This is a consequence of bilateral cerebral dysfunction or involvement of the periventricular grey matter in the brain stem. Assessment of a pt. in coma should include a description of the stimulus used and the

respond observed The Glasgow coma scale is reliable. Etiology: - Infection – meningitis, encephalitis (fever, nuchal rigidity, previous illness, exanthema). - Cerebrovascular disease – AV malformation, stroke (focal neurologic signs, nuchal rigidity). - Metabolic disorders – abnormalities of electrolytes, glucose, oxygen content, cyanosis (signs of dehydration, acetone on breath, laboratory evaluation is necessary to establish the etiology) - Trauma – scalp or facial bruises, hemotympanum (blood behind the eardrum) - Toxic (poisoning) – one of the most common causes of altered consciousness in toddlers and adolescents (breath may smell of the ingested agent, such as cleaning fluids, alcohol; medications present at home should be suspected) - Postictal state – depressed level of consciousness secondary to an unwitnessed seizure (history of seizures; abnormal EEG may be helpful) Diagnosis: a) History: It must be obtained rapidly. Attention should be paid to

events leading to altered consciousness, past medical conditions, and available toxins. b) Physical examination: Eyes: Pupil size and reaction to light suggests a toxic substance or brain stem injury. Abnormal ocular movements, proved by caloric testing suggests brain stem damage. Papilledema suggest elevated intracranial pressure Motor status: Spontaneous movements are to be recorded. Decorticate or decerebrate posturing or seizures should be noted Respiratory pattern: The rate of respiration as well as abnormalities (e.g Cheyne-Stokes respiration, central neurologic hyperventilation, Biot’s breathing) should be noted. c) Laboratory studies: Blood – metabolic abnormalities detection. Urine – presence of toxic substances, heavy metals, sugar, and acetone. CT scan – detects hemorrhage, mass lesions, blunt trauma. MRI provides more detailed information EEG can diagnose seizures Lumbar puncture should be performed if an infection is suspected. 105 Therapy: - Supportive

treatment includes establishing an airway, maintaining hydration, and decreasing intracranial pressure. Hyperventilating the pt is the quickest way to lower intracranial pressure Medications (mannitol, steroids) are also helpful. - Specific treatment depends on the cause of the delirium or coma. 106 50) PRIMARY IMMUNODEFICIENCY a) Complement disorders Activation of the complement (C) system, by either the classic or alternative pathways, leads to generation of potent complement components, which have many activities. Faulty complement activation or regulation can lead to immune-mediated damage of host tissues and various disorders. Most genetic complement deficiencies are inherited as autosomal recessive disorders. C1 inhibitor deficiency is inherited as autosomal dominant trait. Clinical features (see picture) b) Phagocyte disorders Types of defects: Phagocyte disorders affect one or more of the cell’s functions, including: - Adherence to vascular endothelium - Recognition

of and migration toward a chemical stimulus (chemotaxis) - Phagocytosis - Intracellular killing Clinical features: - Affected children are prone to infections with low virulence-bacteria, such as S. aureus and Gnegative enteric bacteria They range from mild skin lesions to severe systemic infections Typical infections include furunculosis, organ abscess, lymphadenitis, and perirectal abscess. - Pt. with disorders of leukocyte movement are unable to accumulate neutrophils at the sites of infection (e.g Chediak-Higasi syndrome) - Pt. with leukocyte adhesion deficiency have defective expression of cell membrane adhesion glycoproteins. They may have history of delayed separation of the umbilical cord (> 6 weeks after birth) and poor wound healing. - Impaired intracellular killing (especially of catalase positive organisms) arises from a defect of one cytochrome. Chronic granulomatous disease is the result c) B-cell deficiency disorders These defects lead to antibody

deficiencies (hypogammaglobulinemias). Pt suffer from recurrent infections. Causative agents are most commonly extracellular organisms, namely pyogenic and enteric bacteria, because pt. are deficient in serum opsonins (antibodies) necessary for phagocytosis. Pt with X-linked (Bruton’s agammaglobulinemia may also have problems with certain enteric viruses (polio, echo, coxsackieviruses). Pt with IgA deficiency or common variable hypogammaglobulinemia are affected by Giardia lamblia, a GI parasite. Sites of infection include the skin, middle ear, conjunctiva, sinuses, meninges, and the respiratory, urinary, and GI tracts. 107 d) T-cell deficiency disorders They result from abnormalities in T-cell functions. Antibody production may be also affected in pt. with severe T-cell abnormalities because T-cells are important immunoregulators of B-cell differentiation and function, resulting in combined T-cell and B-cell deficiencies. Recurrent infections are also common. Causative agents

are intracellular pathogens (eg HSV, mycobacteria, fungi (Candida, Pneumocystis carinii), or toxoplasmosis. There are various sites of infection, both local and systemic. Congenital T-cell deficiencies: At one extreme are defects in lymphoid stem-cell differentiation, which results in severe combined immunodeficiency disorders. At the other extreme are isolated defects that affect only cell-mediated immunity to one specific pathogen (e.g chronic mucocutaneous candidiasis) Evaluation for immunodeficiency disorders - History: Severe, recurrent, or persistent infections strongly suggest an immune deficit. In general, the earlier the onset, the more serious the underlying immunodeficiency. Serious sites of infection (meningitis, organ abscess, pneumonia, sepsis, or generalized dermatitis) suggest an underlying immunodeficiency. - Causative agents: Usually, infectious agents in phagocyte dysfunction are bacteria of low virulence (S. aureus, Klebsiella, Serratia) Infectious agents in

B-cell deficiency disorders include highly virulent bacteria (Strept. Pneumoniae and other pyogenes streptococci, H influenzae, meningococci), and Giardia lamblia. Infectious agents in T-cell deficiency disorders include viruses (HSV, CMV), fungi (Candida sp.), and opportunistic organisms (Pneumocystis carinii, mycobacteria). - Family history: Many immunodeficiencies are autosomal recessive or X-linked. - Adverse reaction to drugs and vaccines (e.g polio and blood product transfusion reactions occur more commonly in these pt.) - Physical findings relating to immune deficiency (see picture) - Laboratory testing (see picture) 108 51) URINARY TRACT INFECTIONS. (see topic 73) 52) ANEMIAS. Anemia is an abnormal decrease in the number of circulating RBCs, the hemoglobin concentration, and the hematocrit. It is not a disease itself but is a symptom of another disorder Anemia is a relatively common finding, and finding the cause is always important. The cause is suggested by

careful history. If nutritional cases are suspected, we have to ask about dietary intake, growth and development; any symptoms of chronic disease, malabsorption, or blood loss. Hemolytic disease may be suggested by a history of jaundice (including neonatal jaundice) or by a family history of anemia, jaundice, gallbladder disease, splenomegaly, or splenectomy. The child’s ethnic background may suggest the possibility of certain hemoglobinopathies or of deficiencies of red cell enzymes such as glucose-6-phosphate dehydrogenase (G6PD). The pt age is important because some causes of anemia are age related. For example, pt with iron deficiency anemia and beta-globin disorders present more commonly at ages 6-36 months. The physical examination may also reveal clues to the cause of anemia. Poor growth may suggest chronic disease or hypothyroidism. Congenital anomalies may be associated with Fanconi’s anemia or with Diamond-Blackfan anemia (see topic 4C). Other disorders may be suggested

by the findings of petechiae or purpura (leukemia, aplastic anemia, hemolytic uremic syndrome), jaundice (hemolysis or liver disease), generalized lymphadenopathy (leukemia, HIV infection), splenomegaly (leukemia, sickle cell syndromes, hereditary spherocytosis, liver disease, hypersplenism), or evidence of chronic recurrent infections. The initial laboratory evaluation of anemic child consists of a complete blood count (CBC) with differential and platelet count, review of the peripheral blood smear, and reticulocyte count. Hemoglobin level and hematocrit are relatively high in the newborn; these values subsequently decline until 2 - 3 month of age. (This condition is termed physiologic anemia of infancy) Total hemoglobin concentration and hematocrit rise gradually during childhood, reaching adult values after puberty. Hb F is the major hemoglobin of prenatal and early postnatal life Hb F values decline postnatally. Anemias are classified according to the morphologic appearance (i.e,

color and size) of the RBCs on the peripheral smear as well as the MCV (mean corpuscular volume). MCV determines whether the anemia is microcytic, normocytic, or macrocytic (the suffix “cytic” refers to size, the suffix “chromic” refers to color). 109 The classifications are: a) Hypochromic, microcytic (small, pale RBCs, a low MCV) b) Macrocytic (large RBCs, a high MCV) c) Normochromic, normocytic (cells of normal size and shape; a normal MCV) Microcytic anemia due to iron deficiency has decreased with improvements in infant nutrition. Still, it is an important cause of microcytic anemia, especially at ages 6-24 months. Another key element is the use of both reticulocyte count and the peripheral blood smear to determine whether normocytic anemia is due to hemolysis. Typically, hemolytic disease is associated with an elevated reticulocyte count, but not in some children with chronic hemolysis. Thus review of the peripheral blood smear for evidence of hemolysis (spherocytes,

red cell fragmentation, sickle forms) is important. Autoimmune hemolysis can be excluded by Coombs testing. Children with normocytic and macrocytic anemias with no evidence of hemolysis, usually have anemias caused by inadequate erythropoiesis in the bone marrow. The presence of neutropenia or thrombocytopenia in such children suggests bone marrow failure and the possibility of aplastic anemia or malignancy. Pure red cell aplasia may be congenital or acquired (see topic 4C) 110 53) MORBILLI (Measles, rubeolla) It is an acute, highly contagious viral disease that occurs chiefly in young children living in densely populated areas. Epidemiology: It is uncommon in countries where vaccine is used, worldwide it is a problem. Etiology: Measles is caused by a paramyxovirus, of which only one serotype has been found. Clinical features: The course has three stages: 1. An incubation period - from 8 to 12 days after exposure, signs and symptoms are absent during this stage. 2. A prodrome

follows, consisting of malaise, fever, cough, coryza, conjunctivitis, and photophobia. Within 2 or 3 days after the onset of symptoms, Koplik’s spots (small, irregular red spots with grey or bluish-white centre) appear on the buccal mucosa. 3. An erythematous maculopapular rash erupts 5 days after the onset of symptoms The rash begins on the head and spreads downward, last 4-5 days, and then resolves from the head downward. Pt with measles appear quite ill compared to pt with other exanthematous disease (e.g rubella) Diagnosis: Can be made by clinical findings. > 4x rise in hemagglutination inhibition antibodies over 2 to 3 weeks confirms the diagnosis. Therapy: It is supportive. Vitamin A supplementation should be given to very young infants Prevention: A live attenuated vaccine given alone or as part of the measles, mumps, and rubella (MMR) vaccine is highly effective. The vaccine may be protective if given within 72h of measles exposure. Immunoglobulin can be given as

prevention if given within 6 days of exposure Complications - rare, but may occur in malnourished, vitamin A deficient, or immunocompromised children. It includes - pneumonia, pericarditis, and hepatitis. - encephalitis, subacute sclerosing panencephalitis (SSPE). The mortality is low in healthy children. 111 54) DISORDERS OF AMINO ACID METABOLISM. a) Phenylketonuria (PKU) The best-known disorder of amino acid metabolism is the classic form of PKU caused by decreased activity of phenylalanine hydroxylase (converts Phe to Tyr). PKU is AR, with an incidence of 1:10.000 On normal Phe intake, affected pt develop hyperphenylalaninemia and produce and excrete its metabolites. Untreated PKU exhibits severe brain damage with mental retardation, hyperactivity, seizures, skin hypopigmentation, and eczema. The pt. urine has a “mouse-like” odor Outcome is best with early treatment, so infants are screened during the first few days. Because the screening test measures a metabolite that

becomes abnormal only with time and diet, the disease cannot be detected reliably until intake of the enzyme substrate has become established! Prenatal diagnosis is possible. In classic PKU, there is little or no phenylalanine hydroxylase activity, but in the less severe hyperphenylalaninemias there is significant residual activity. Treatment: limit dietary Phe intake to amounts that permit normal growth & development (lifelong). Serum Phe concentrations must be monitored frequently. Phenylketonuric females must pay special attention during childbearing years (the woman’s diet should be monitored closely prior to conception and throughout pregnancy). Children with classic PKU who receive treatment promptly after birth will develop well physically & intellectually. b) Maple syrup urine disease Caused by deficiency of the enzyme that catalyses oxidative decarboxylation of the branched-chain keto acid derivatives of Leu, isoleucine, and Val. The accumulated keto acids of Leu and

isoleucine cause the characteristic odor, whereas only the keto acid leucine causes CNS dysfunction. Many variants of this disorder have been described, and all are AR. Pt. with classic maple syrup urine disease are normal at birth but soon develop the characteristic odor, lethargy, feeding difficulties, coma, and seizures. Unless dietary restrictions of branchedchain amino acids is begun, most will die in the first months of life, but nearly normal growth and development may be achieved if treatment is begun before age 10 days. Prenatal diagnosis is possible. 112 Treatment: Products deficient in branched-chain amino acids but otherwise similar to milk are commercially available but must be supplemented with normal milk and other foods to supply enough branched-chain amino acids to permit normal growth and development. Serum levels of branched-chain amino acids must be monitored frequently (every day at first). c) Homocystinuria It occurs in several biochemical forms. The

incidence is 1:100,000 liveborn infants It is an AR inherited deficiency of the enzyme cysthationine - beta synthase, leading to accumulation of homocystine, whose excretions can be measured in urine. Homocystinuria can be caused by abnormal metabolism of homocystein as well as due to abnormalities of tetrahydrofolate reductase and several defects of vitamin B12 metabolism. Clinical features: There are no symptoms in infancy. - The major manifestations during childhood are skeletal, ocular, and intellectual. - There is a typical body habitus similar to that seen in Marfan’s sy, with long, thin limbs and fingers. Scoliosis, sternal deformity and osteoporosis are common - Dislocated lenses are frequent. - Mental retardation (mild-to-moderate) is observed. - Vascular thrombosis is common, and childhood strokes and infarctions are reported. This is thought to be due damage to the vascular endothelium by homocystein. Management: Some pt. respond to treatment with pyridoxine

(vitamin B6), a cofactor for cysthationine beta-synthase. In other pt, dietary management is difficult, because restriction of sulfhydryl group leads to a very low-protein and unpleasant-tasting diet. d) Examples of other less common diseases of amino-acid metabolism: - Tyrosinemia type I (hepatorenal), type II (oculocutaneous) - Albinism - results from enzyme tyrosinase defect; presents as absent pigment in skin, hair, eyes. Management consists of protection of skin and eyes from actinic radiation. - Alkaptonuria - results from enzyme homogentisate oxidase deficiency. Clinical features include arthritis, and dark urine. There is no treatment available - Histidinemia - presents as neurologic manifestations, frequently benign. Treatment consists of low protein diet, and controlled histidine intake. 113 55) NEONATAL JAUNDICE – DD AND TREATMENT. There are two types of neonatal hyperbilirubinemia: unconjugated, which can be physiologic or pathologic, and conjugated, which is

always pathologic. Both types may lead to jaundice Neurotoxic concentrations of unconjugated bilirubin can cause kernicterus. Hyperbilirubinemia = serum bilirubin concentration > 10 mg/dL in preterm newborns or >15 mg/dL in full-term newborns. When the bilirubin blood level is 5 mg/dL, jaundice becomes apparent. With increasing bilirubin levels, visible jaundice advances in a head-to-foot direction Hyperbilirubinemia may be physiologic or due to a specific underlying disorder (neonatal hyperbilirubinemia can result from: 1. increased production of bilirubin [eg from elevated Hb from hypertransfusion, hemolytic diseases, hematomas]. 2. decreased excretion of bilirubin [eg from decreased glucuronyl transferase activity in the preterm newborn, hepatitis, biliary atresia]. 3. both [eg sepsis]) A specific cause for neonatal hyperbilirubinemia must be always sought, because it will require specific treatment. The exact cause of physiologic hyperbilirubinemia is unknown; it may be due

to limiting rates in the binding of bilirubin in the hepatocytes, in the conjugation of bilirubin with glucuronic acid, and in bile secretion. Delay in initiating feedings is often complicated by unconjugated hyperbilirubinemia, because enterohepatic circulation of bilirubin is increased when GI transit time is prolonged. Neonatal hyperbilirubinemia is most often unconjugated, and excess accumulation of unconjugated bilirubin can lead to kernicterus, especially in the preterm or sick newborns. Treatment Physiologic jaundice is usually not clinically significant and resolves within 1 week. In breast milk jaundice (see breast feeding), a temporary change from breast milk to formula feedings may be appropriate. Phototherapy may also be indicated: it is a safe and effective in treating hyperbilirubinemia. A maximal effect is obtained by exposing the newborn to visible light in the blue range. Phototherapy produces configurational photoisomers of bilirubin in the skin and subcutaneous

tissues; these are more water-soluble and can be excreted rapidly by the liver without glucuronidation. Phototherapy is not indicated with biliary or intestinal obstruction, since the photoisomers cannot be excreted. Exchange transfusion: Dangerous levels (about 20-25 mg/dL) of bilirubin are treated by exchange blood transfusion via an umbilical vein catheter. 114 Kernicterus = brain damage due to deposition of bilirubin in the basal ganglia and brain stem nuclei. Bilirubin is firmly bound to serum albumin and is not free to cross the blood-brain barrier and cause kernicterus. The risk of kernicterus is therefore greater in newborns who have a markedly elevated serum bilirubin concentration, a low serum albumin concentration, or substances in their serum that compete for bilirubin binding sites on albumin, including free fatty acids, hydrogen ions, and certain drugs, such as sulfisoxazole, ceftriaxone, and aspirin. Serum albumin concentrations are lower in preterm newborns,

putting them at greater risk. Symptoms, signs, and diagnosis - Early symptoms in term newborns are lethargy, poor feeding, and vomiting. - Opisthotonus, seizures, and death may follow. - In preterm newborns, kernicterus may not produce recognizable clinical signs. - It may cause mental retardation, choreoathetoid cerebral palsy, and sensorineural hearing loss. Prophylaxis and treatment Prevention involves avoiding high bilirubin levels (phototherapy or exchange transfusion). There is no curative treatment for kernicterus; treatment is symptomatic. Conjugated hyperbilirubinemia - most common causes: 1. Biliary atresia occurs when there is obstruction of the biliary tree due to progressive sclerosis of the common bile duct. In most cases, biliary atresia develops several weeks after birth, probably after inflammation and scarring of the extrahepatic bile duct. It is rarely found in newborns at birth. The cause of inflammatory response is unknown 2. Neonatal hepatitis (giant

cell hepatitis) is usually idiopathic or due to infections (hepatitis B virus, CMV, enteroviruses, toxoplasmosis, herpes virus, rubella). In both conditions, cholestatic jaundice with mixed hyperbilirubinemia, dark urine (conjugated bilirubin), acholic stools, and hepatomegaly are usually first noted about 2 weeks after birth. By age 2 to 3 months, retarded growth, irritability from pruritus, and signs of portal hypertension may be present. Biliary atresia and neonatal hepatitis may be difficult to differentiate In both, liver function tests reflect cholestasis and hepatocellular inflammation. The absence of recognizable gallbladder and of extrahepatic bile ducts on ultrasonography suggests biliary atresia. Presence of bile in the duodenum (identified by nasogastric suction) is an evidence against biliary atresia. Liver scanning identifies extrahepatic bile flow. In questionable cases, liver biopsy is performed Treatment 115 Laparotomy should be done before age 2 months because

irreversible biliary cirrhosis will develop if surgical repair is postponed. Atretic bile ducts can be successfully reanastomosed in only 10% of infants; in the remaining pt., hepatoportoenterostomy often re-establishes bile flow However, many infants have significant problems later. Biliary atresia is the most common pediatric indication for liver transplantation. Cholestasis due to neonatal hepatitis resolves slowly, but permanent liver damage may result, and some infants will not survive. 116 56) EPIDEMIC INFLUENZA It is the most serious viral airway infection in terms of morbidity and mortality. It is an important cause of worldwide respiratory diseases. Epidemiology: Worldwide pandemics occur every 10-20 years and have been responsible for 21 million deaths. Virus is transmitted by droplets (sneeze, cough, talk) Etiology: The influenza virus belongs into myxoviruses group and it occurs in 3 types (A, B, C) of which 2 types (A, B) cause the clinical syndrome of influenza. The

virus has 2 surface glycoproteins (antigens); neuraminidase and hemagglutinin. Both viral antigens play an important role in infecting host cells. Influenza A is associated with human and animal infection Influenza B occurs in humans only. Strain variations are noted each year with antigenic shifts and drifts; a shift indicates a major antigenic change and drift implies a minor change. Clinical presentation: after a short incubation period (24-72h) there is: - Sudden onset of fever, frequently with chills or rigors. - Headache, malaise, diffuse myalgia, and non-productive cough. - Subsequently, signs of respiratory tract involvement (sore throat, nasal congestion). - Conjunctival injection, N/V, abdominal pain may occur. - As infection spreads through the respiratory tract there may be nasal, pharyngeal, and tracheal symptoms, dry cough, and burning substernal discomfort. Complications: - Pulmonary: primary viral pneumonia, bacterial pneumonia (superinfection), exacerbation

of chronic lung disease or asthma, otitis and sinusitis in children. - Non-pulmonary: myositis, myocarditis or pericarditis, toxic shock syndrome, neurologic complications, Reye’s syndrome. Diagnosis: based on clinical symptoms during an outbreak of influenza. Definitive diagnosis can be confirmed by laboratory tests: virus isolation or antigen detection from nasal washing/swab or sputum; serology: hemagglutinin inhibition test, complement fixation test and enzyme immunoassays; or reverse transcriptase PCR. Therapy: Symptomatic in immunocompetent hosts; in groups at higher risk: virostatic agents such as amantadin, rimantadin, or zanamivir (neuraminidase inhibitor). ATBs are indicated for secondary bacterial infections (pneumonia, otitis media, sinusitis). Prevention: Vaccination (inactivated virus) is available. The vaccine is changed annually according to expected influenza strain in the upcoming winter. Protection is transient and must be repeated every year. It is indicated

for pt with chronic cardiopulmonary disease who are at risk of developing complications. 117 57) NEPHROLITHIASIS Etiology: It may be divided into several classes based on stone composition, metabolic causes, site of origin, symptomatology, and the presence or absence of obstruction, infection, or foreign body. One simple way to consider stones is as primary (no complicating structural factors) or secondary (due to obstructed and/or infected urinary tracts). Classification: 1. Renal tubular syndromes (renal tubular acidosis, cystinuria) 2. Enzyme disorders (primary hyperoxaluria) 3. Hypercalciuric states (primary hyperparathyroidism, sarcoidosis, hypervitaminosis D, milkalkali syndrome, neoplasma, immobilization) 4. Uric acid lithiasis (myeloproliferative disorders, oxaluria from increased absorption), 5. Idiopathic renal lithiasis 6. Infected and obstructed urolithiasis, endemic calculi Pathophysiology: The urinary tract stones are composed of a mucoprotein matrix and crystals of

varying composition. The major components are: calcium oxalate, magnesium ammonium phosphate (struvite), calcium phosphate, uric acid, and cystine. Several factors, including urinary pH, solute load, and activity of urinary inhibitors, influence crystal formation. Secondary factors, including obstruction with stasis and infections with ureasplitting organisms, can also be important. Infected stones most commonly associated with obstruction usually consist of struvite. Metabolic factors can be important. Uric acid lithiasis is rare in children, but can present as a complication of treatment of myeloproliferative disorders. Hyperoxaluria occurs in inflammatory bowel disease from increased absorption oxalate and may result ion calcium oxalate stones. The reason for increased absorption is from binding of intestinal calcium to fat, leaving oxalate in its soluble (hence, absorbable) form. Idiopathic renal lithiasis is a term used for poorly understood metabolic disorders. Minor defects,

such as renal leak hypercalciuria, can cause recurrent stone formation Inherited disorders of metabolism, such as hyperoxaluria or cystinuria, must also be excluded. Environmental factors may play a role in the development of stone disease (e.g predisposition to calculi is endemic in Southeast Asia). Children immobilized after trauma or surgery are at risk for calcium urolithiasis. Clinical features and diagnosis: - Gross hematuria, urosepsis, abdominal or flank pain, and renal colic are most common. - Painless hematuria in children may precede later symptomatic calcium urolithiasis. - Special attention must be paid to past medical history, such as immobilization, excess vitamin D or calcium intake. 118 - Lab investigation includes urine sediment – detection of crystals and infection, collection of urine for calcium, phosphate, uric acid, oxalate levels, and cystine, X-ray for stone localization, and screening for electrolyte abnormalities, hypercalcemia, or hyperuricemia

in serum. Analysis of the stone may be helpful. Prevention and treatment: depends on the underlying cause and the degree of obstruction. 1. Renal tubular acidosis is treated by oral alkalinization, cystinuria requires hydration, alkalinization, and sometimes D-penicillamine (binding cystine in the gut). 2. Enzymatic defects cannot be treated successfully 3. For hypercalciuria, increasing fluid intake, reducing excess calcium intake, and thiazides diuretics are helpful. Symptomatic obstructing and infected stones are treated surgically 4. New techniques (percutaneous extraction or dissolution, extracorporeal shock wave lithotripsy) are used lately. 119 58) JAUNDICE IN INFANCY AND CHILDHOOD (See also topic 19A for definitions) Investigations: 1. History: Ask about the feeding pattern, color of stools, and any fever, malaise, vomiting, abdominal pain, jaundice, pruritus, and dark urine. Document exposure to hepatitis or to blood products, medications, or drugs of abuse. Identify

hematologic abnormalities such as hemolytic anemia, sickle cell disease, or thalassemia. Identify preexisting liver disease that causes elevations in indirect bilirubin (hepatitis, Gilbert disease, Crigler-Najjar sy) and direct bilirubin (Rotor sy, Dubin-Johnson sy, hepatitis). Note extrahepatic symptoms such as arthritis, polyarthralgia, amenorrhea, colitis, thyroiditis, glomerulonephritis, pleurisy, or rash. 2. Physical examination: Note upper quadrant abdominal mass or tender hepatomegaly Exsudative tonsillitis, adenopathy, or splenomegaly suggests mononucleosis. Signs of chronic liver disease (cirrhosis): acne, ascites, cushingoid face, digital clubbing, gynecomastia. 3. Laboratory findings: The degree of cholestasis and obstructive jaundice is reflected by the level of direct bilirubin and elevation of alkaline phosphatase and GGT. Hepatocyte damage correlates with the level of serum transaminases (AST, ALT). With extensive hepatocytes damage, serum albumin is low and the

prothrombin time is prolonged. Anemia or thrombocytopenia suggests severe liver disease. A Coombs-positive hemolytic anemia, aplastic anemia, or pancytopenia may complicate hepatitis. Causes: 1. Hepatitis (see topic 37B, 25C) 2. Acute cholecystitis can be caused by a bacterial infection (Salmonella, Shigella, E coli), virus, or parasites (Giardia and Ascaris). 3. Sclerosing cholangitis usually associated with inflammatory bowel disease is a rare progressive disease that causes stenosis of the extrahepatic bile ducts; when it is suspected, an endoscopic retrograde cholangiogram (ERCP) should be performed. There is no treatment of this condition 4. Acute hydrops of the bladder should be suspected in children with Kawasaki disease when US demonstrates distention of the gallbladder without calculi and with normal extrahepatic bile ducts. Treatment includes supportive care (IV fluids) followed by a low-fat diet If it is not successful, surgery follows (cystostomy). 5. Choledochal cyst a

congenital dilation of the common bile duct The degree of jaundice, abdominal pain, vomiting, and alteration of stool color is variable. 120 6. Cholelithiasis, or gall stones, may be related to prolonged total parenteral nutrition, disease of the terminal ileum, dehydration, prematurity, hemolytic disease, immaturity of liver glucuronosyltransferase, or ceftriaxon therapy. However, no associated cause is found in half of the cases. The stones consist of calcium bilirubinate and/or cholesterol crystals In some children, biliary sludge may be responsible for the gallbladder dysfunction. US usually shows dilation of the intrahepatic and extrahepatic bile ducts even when no stones are detected. ERCP can identify the cause and remove the sludge or stone. Cholecystectomy is indicated for symptomatic pt., especially if strictures or anatomic anomalies are present 7. A Kayser-Fleischer ring revealed by slit-lamp examination, reduced serum copper and ceruloplasmin, and elevated urine and

liver tissue copper occurs in Wilson disease. 8. Galactosemia presents with vomiting, diarrhea, failure to thrive, hypoglycemia, cataracts, developmental delay, and seizures. 9. Alfa-1-antitrypsin deficiency presents with neonatal cholestatic jaundice and hepatomegaly, and elevated aminotransferases. 10. Hereditary fructose intolerance the pt is deficient in fructose-metabolizing enzymes 11. Urine should be tested for identification of hereditary tyrosinemia 12. The chronic form is characterized by cirrhosis, vitamin D-resistant rickets, failure to thrive, and Fanconi sy. 13. Cystic fibrosis may be identified by sweat test 14. Abnormalities in iron, TIBC, and ferritin levels suggest hemochromatosis 15. Autoantibodies suggest autoimmune hepatitis Liver biopsy may be necessary to identify the cause. 16. Liver mass (tumor, abscess) is diagnosed by US or CT 121 59) ENCEPHALITIS. Etiology: A specific pathogen is not identified in 75% of cases. Of the known causes, a viral cause is most

common. - HSV causes encephalitis year round and is the most common cause of sporadic acute encephalitis. HSV 2 is the usual cause in neonates, whereas type 1 in older children - Arboviruses cause outbreaks during the summer because they are transmitted by insects. - Enteroviruses (i.e coxsackievirus, echovirus) cause outbreaks in the summer - Viruses associated with childhood illnesses (e.g mumps, measles, varicella, rubella) may cause acute or postinfectious encephalitis (mediated by immune mechanisms). - EBV is a rare cause; occasionally encephalitis may develop during infectious mononucleosis. - Nonviral causes include Mycoplasma pneumoniae, Toxoplasma gondii, Bartonella henselae, Treponema pallidum, and B. burgdorferi) Clinical features: They are variable in severity, but most commonly include: a) Early (nonspecific and typical for acute systemic illness - vomiting, fever, headache). b) Neurologic (develop abruptly; most commonly, there is a decreased level of

consciousness ranging from confusion to coma, seizures, paralysis, and abnormal reflexes. Increased intracranial pressure can result in papilledema Diagnosis: History (exposure to infected people, insects, animals). Lumbar puncture and CSF examination is essential. Typical CSF findings include increased intracranial pressure, pleocytosis (mainly mononuclear cells), increased protein level, normal glucose level). The CSF should also be examined directly for bacteria and fungi and cultured for bacteria, mycobacteria, fungi, and viruses. Brain biopsy may be performed. Serologic tests may be used to detect viral antibodies (mainly arboviral encephalitis). EEG, CT, MRI, and brain scans may reveal focal or generalized abnormalities. Therapy: Acyclovir is the drug of choice for herpes simplex encephalitis. There is no specific therapy for other types of viral encephalitis. Supportive care: pt. should be placed in an intensive care unit, with close cardiac and intracranial pressure monitoring.

Phenobarbital is given to prevent convulsions. Severe cerebral edema can be decreased by dexamethasone parenterally, mannitol IV, and furosemide given IV. 122 60) CONGENITAL DISORDERS OF BONES AND JOINTS 1. Congenital dislocation of the hip This is a spontaneous dislocation of the hip occurring either before or during birth or shortly afterwards. In Western countries it is one of the most common skeletal deformities Causes: A combination of factors is involved – some genetic and some environmental: 1) Genetically determined joint laxity – It leads to lack of stability at the hip, so that dislocation may easily occur in certain positions of the joint. 2) Hormonal joint laxity – In females a ligament-relaxing hormone may be secreted by the fetal uterus. This may cause instability in the same way as does genetically determined joint laxity It explains the greater incidence of dislocation in girls (the relaxing agent is not produced in boys). This factor, however, requires

further investigation 3) Genetically determined dysplasia of the hip – Defective development of the acetabulum and femoral head is bilateral, and is as common in boys as in girls. It predisposes to dislocation If dislocation does not occur, the defect may show itself in adult life 4) Breech malposition – The incidence of hip dislocation is higher when the infant is delivered by the breech than with normal delivery. It may precipitate dislocation when there is already a predisposition to it from joint laxity or acetabular dysplasia. Generally, we distinguish 2 main types. The first type is caused predominantly by ligamentous laxity (genetically determined or hormonal) in which dislocation occurs with some precipitating movement (extension of the hips during delivery). The dislocation in this type is often unilateral and easily correctable. The second type is due to genetically determined dysplasia of the acetabulum, which is always bilateral and more difficult to treat. Clinical

symptoms, radiological symptoms: Girls are affected six times as often as boys. In one third of all cases both hips are affected Abnormality may not be noticed until the child begins to walk, therefore it has to be specially looked for!!! Walking is often delayed and there is limp. On examination the main features in unilateral cases are asymmetry, shortening of the affected limb, and restricted abduction in flexion. In bilateral cases there is widening of the perineum and marked lumbar lordosis. Abduction in flexion is slightly restricted Imaging: X-ray shows the ossific centre of the head of the femur that appears late and its development is retarded; the bony acetabular roof that has an upward slope, and the femoral head that is displaced upwards and laterally from its normal position. US shows the position of the femoral head in the new-borne. 123 Diagnosis: Dislocation or instability of the hip may be detected in the first few days of life by a diagnostic manoeuvre. In this

test the surgeon faces the child’s perineum and grasps the upper part of each thigh between fingers behind and thumb in front, the child’s knees being fully flexed and the hips flexed to a right angle. While each thigh in turn is steadily abducted towards the couch the middle finger applies forward pressure behind the great trochanter and alternately the thumb placed anteriorly applies backward pressure. One of two abnormal states may be detected: 1) a dislocated femoral head snaps back into the acetabulum with a palpable and audible jerk 2) backward pressure with the thumb dislocates a joint that is unstable Course and prognosis: The earlier the dislocation is reduced the better the prognosis. Only about half of the pt. treated after the first year of life can be free of trouble later! Treatment: 1. If diagnosed at birth: may be corrected by wearing a double nappy, or by using a harness, holding the hip abducted and flexed. 2. At 2 months: may be diagnosed by routine examination

Can be managed conservatively with a brace traction or plaster immobilization. 3. At 12-18 months: since the child start walking with a limp or rolling gait The limb cannot be completely stable anymore, but series of operations may achieve a stable joint. Missed diagnosis is a disaster to the pt, though if it’s bilateral at least the problem will be symmetrical. 2. Torticollis In infantile torticollis (wry neck) the head is tilted and rotated by contracture of the sternocleidomastoid muscle of one side. It arises after birth Cause: This is uncertain. Probably there is interference with the blood supply (causing infarction) of the sternocleidomastoid muscle, caused by injury during birth. Pathology: Part of the affected muscle is replaced by contracted fibrous tissue. Clinical features: The child, often between 6 months and 3 years old, is holding his head on one side. On examination, the contracted muscle is felt as a tight cord The ear on the affected side is closer to the shoulder.

In long-standing cases there is retarded development of the face on the affected side, with asymmetry. Treatment: If the condition is seen at the stage of muscle infarction, physiotherapy is effective. In established cases the contracted muscle is divided. After operation exercise should be encouraged 124 3. Congenital clubfoot (talipes equinovarus) This term describes the commonest and most important congenital deformity of the foot. Cause: A defect of fetal development is responsible, with imbalance between the inventor-plantarflexor muscles and the evertor-dorsiflexor muscles. Pathology: There is subluxation of the talo-navicular joint, so that the navicular bone lies partly on the medial aspect of the head of the talus instead of on its distal aspect. The soft tissues at the medial side of the foot are under-developed and shorter than normal. The foot is adducted and inverted and is held in equinus (plantar flexion). In most cases there is also under-development of the calf.

If there is no early treatment, the developing tarsal bones assume an abnormal shape as growth occurs, worsening the deformity. Clinical features: It is commoner in boys. One or both feet may be affected The foot cannot be pushed passively through the normal range of eversion and dorsiflexion. Prognosis: Even with early treatment recurrence is common! Treatment in a fresh case: a) primary conservative treatment - It should be begun not more than one week after birth! It consists of correction of the deformity by repeated firm manual pressure and maintenance of correction (plaster, metal splints, or adhesive strapping) b) operative treatment - If at the end of three or four months the feet are not normal, operation is done. After operation a plaster is worn for 2 months Treatment in neglected cases: In late cases no treatment is capable to restore the foot to normal! The most that can be done is restoration of the foot so that the sole is on the ground. 4. Congenital valgus deformity of

the foot (talipes calcaneo-valgus) This is the opposite deformity to talipes equinovarus. The foot is everted and dorsiflexed, so that its dorsum lies almost in contact with the shin. It is a less serious deformity than talipes equinovarus and it responds well to treatment. Cause: unknown Clinical features: One foot or both feet may be affected. The foot cannot be brought easily into inversion and equinus. Treatment: Repeated manual stretching by the parents may be sufficient. If it is not enough, the foot is manually corrected and plaster is applied. The plaster is changed weekly until a full range of inversion and equinus is gained. 125 5. Osteogenesis imperfecta It is a congenital and inherited disorder in which the bones are abnormally soft and brittle, on account of defective collagen formation. In addition to the bones, other collagen-containing tissues such as teeth, skin, tendons and ligaments may be abnormal. It is usually transmitted as an autosomal dominant, but in

severe cases of the disease the parents are normal and a fresh gene mutation is responsible. Clinical features: In the worst cases, the child is borne with multiple fractures and does not survive. In the less severe examples fractures occur after birth, often from trivial violence. As many as fifty fractures can occur in the first few years of life! The fractures heal readily but in some cases deformities develop and such pt. may be badly crippled In the mild cases there is a tendency for fractures to occur less frequently in later life. Other congenital disorders of the extremities: 1) Congenital amputation - Part or whole of one or more limbs is absent. 2) Phocomelia - Aplasia of proximal part of limb, the distal part being present (“seal limb”) 3) Absence of radius - Hand deviated laterally from lack of normal support by radius. 4) Absence of thumb - Thumb alone may be absent. 5) Syndactyly - Webbing of two or more digits 6) Polydactyly - More than five digits 126 61)

ABDOMINAL TUMORS – DD AND TREATMENT a) Neuroblastoma It is a malignancy of neural crest cells, which later, in normal development, form the paraspinal sympathetic ganglia and the adrenal medulla. Epidemiology: the second most common solid childhood tumor. Only brain tumors are more common. Neuroblastoma occur predominantly in infants and preschool children There is a slight male predominance. Clinical features: They are variable because of the distribution of neural crest tissue. 1/2 of newly diagnosed pt. have distant metastases, and they appear ill Primary sites: - Abdominal tumors are the most common presentation (70%), 1/2 arise from extra-adrenal tissue and 1/2 from adrenal medulla. Presenting features are abdominal mass displacing the kidneys, abdominal pain, and systemic hypertension, if there is compression of the renal vasculature. - Thoracic tumors are the next most common. They are located in the posterior mediastinum There is respiratory distress, and X-ray finding. -

Head and neck tumors are palpable masses that sometimes produce Horner’s syndrome (miosis, ptosis, enophthalmos, and anhidrosis). - Epidural tumors grow through the neural foramina into the epidural space, where they compress the spinal cord, producing back pain and cord compression symptoms. Rapid evaluation and treatment is necessary to prevent neurologic damage. Metastases: They are common at diagnosis and often cause symptoms that lead to the diagnosis. - Nonspecific symptoms include weight loss, fever. - Specific symptoms include bone marrow failure, cortical bone pain (may cause limp), ptosis, periorbital ecchymoses from retrobulbar and orbital infiltration, liver infiltration causing hepatomegaly, distant lymph node enlargement, skin infiltration (subcutaneous nodules) Tumor markers: They are extremely useful in evaluation (diagnosis, response to therapy, recurrence). They include: • serum markers (ferritin, lactate dehydrogenase – these are associated with poor

prognosis). • urinary markers (catecholamines and their metabolites), • oncogene markers (amplification of the n-myc oncogene – poor prognosis). 127 Therapy: - Surgery alone is sufficient for stage I and II pt. (tumor does not cross the midline) - Spontaneous regression without any therapy is common in stage IVS (special stage – metastases limited to the skin, liver, and bone marrow). - Aggressive multiagent chemotherapy produce dramatic reduction is size, but are not curative in stage III and IV, even with surgery and radiotherapy. - Autologous bone marrow transplantation: It is used for stage III and IV with poor prognosis. Up to 50% are cured. Prognosis: It depends on the age (infants < 1 yr have the best prognosis), stage (stage I, II, and IVS have a good prognosis, most stages III, and IV have a poor prognosis), tumor markers, histology, and degree of differentiation. b) Nephroblastoma (see topic 43C) c) Hodgkin’s disease and non-hodgkin’s lymphomas

(see topic 10C) d) Rhabdomyosarcomas (see topic 7C) e) Liver tumors They account for only 1% of childhood cancer. However, they are the most common epithelial malignancy of childhood. Clinical features: A mass occurring primarily in the right upper quadrant is present. These tumors often produce alfa-fetoprotein (AFP), which is a tumor marker useful for diagnosis, following up response to therapy, and detection of recurrence. There are 2 types of tumors: - Hepatoblastoma, the most common type, occurs in infants and young children. It is often associated with thrombocytosis and metastasizes to the lung. Outcome depends on whether it can be completely resected. Hepatoblastoma that is not resectable may be at least reduced in size by chemotherapeutics (e.g doxorubicin and cisplatin) - HCC, which is less common, is seen in older children and teenagers. It is often multifocal and is less curable than hepatoblastoma. It may be related to preceding hepatitis B infection, just as it is in

adults. 128 f) Germ cell tumors They arise from precursors of egg and sperm cells. Common sites of origin include the gonads (testes, ovaries), and ectopic sites (sacrococcyx, retroperitoneum, mediastinum). Histology: The tumors differ in the level of differentiation (from pluripotent to differentiated). Multipotential cells form embryonal carcinoma. Cells with embryonic or somatic differentiation form teratomas, which may be benign or malignant. Cells with extra-embryonic differentiation form yolk sac tumors or trophoblastic tumors or choriocarcinomas. Cell designed for germ cell differentiation form seminomas (rare in children) and dysgerminomas in females). Diagnostic markers: AFP is produced by embryonal carcinomas and yolk sac tumors. Human chorionic gonadotropin is produced by embryonal carcinomas and choriocarcinomas. Clinical features: - Testicular tumors present as painless testicular swelling; these tumors always require radical orchiectomy. Additional therapy and

prognosis depend on age (most infants < 2 yr have an excellent prognosis with surgery alone), stage, histology, and metastatic potential for spread to the retroperitoneal nodes and lungs via the lymphatics and blood. - Ovarian tumors present as abdominal or pelvic masses. They may produce severe pain of acute onset that results from torsion of the tumor. Uncommon presentations include precocious puberty or virilism. Therapy includes complete surgical resection and chemotherapy - Sacrococcygeal germ cell tumors are the most common germ cell tumors and the most common solid tumors in the newborn. They are usually benign and diagnosed in the first 2 months of life. Such tumors in older children are often malignant - Mediastinal teratomas are benign in 80%. In older adolescents, they may be malignant 129 62) DISEASES WITH SPECIAL DIET REQUIREMENT IN INFANCY 1. Preterm infancy - Rout of feeding The preterm infant who is less than 34 weeks’ gestational age does not have a

well-coordinated suck-and-swallow reflex, and therefore should be fed via a feeding tube. The feedings may be every 2 to 3 hours, except in infants weighing less than 1000 gm. Continuous gastric or transpyloric feeding is given to infants < 1000 gm, because such infants have a limited gastric volume and do not tolerate bolus feedings. Continuous transpyloric feedings are considered for the infant who requires endotracheal tube and mechanical ventilation to prevent gastric reflux and aspiration. - Feeding solutions Preterm infants have decreased gastric motility and intestinal lactase activity as well as increased calcium and phosphorus requirements, among other nutritional problems. The initial feeding solution should be a dilute, whey-based formula or human milk. As positive nitrogen balance is achieved, the infant may get formula high in calcium, phosphorus, and protein, or to fortified human milk. - Vitamins and minerals Due to small body stores and inadequate feeding

volumes, preterm infants should receive a multiple-vitamin supplement containing the fat-soluble vitamins (A+D) and the water-soluble vitamins (B+C). Infants < 36 weeks’ should receive vitamin E to prevent hemolytic anemia Calcium, phosphorus, and iron supplementation is necessary. Total parenteral nutrition Preterm and other sick infants may require total parenteral nutrition because of GI disorders (neonatal necrotizing enterocolitis) as well as non-GI disorders (respiratory disease, sepsis). An iv solution of dextrose, amino acids, fat, vitamins, and minerals can be administered by either peripheral or central venous access. 2. Infants with special metabolic needs Special formula solutions are available for infants who have selected intestinal enzyme deficiencies (e.g sucrase-isomaltase deficiency), metabolic disease (eg PKU), or allergies (eg protein allergy) 130 3. Nutrition in renal failure The nutrition must be low in protein (because of impaired excretion of urea

nitrogen), sodium (bc of impaired water excretion), and phosphate (bc of chronic impaired ability to synthesize vitamin D3 leading to hypocalcemia, hyperphosphatemia, and secondary hyperparathyroidism). The nutrition must provide adequate calories to cover high metabolic needs, calcium, and vitamin D3. Dietary potassium limitation is not needed until the GFR falls to very low levels. 4. Nutrition in hepatic failure Proteins must be limited (to minimize hyperammonemia). Especially aromatic amino acids must be limited in favour of branched-chain amino acids (bc of hepatic inability to metabolise aromatic amino acids). High-density carbohydrate sources should be used as continuous infusions to prevent hypoglycemia (bc of hepatic inability to store glucose and impaired gluconeogenesis). 131 63) RHINITIS, ADENOIDITIS, GRIPPE a) Rhinitis (rhinitis acuta, common cold) It is an acute illness characterized by rhinorrhea, sneezing, sore throat, cough, and in children, lowgrade fever. The

transmission occurs from person to person, requires close contact, and involves transfer of the virus from the hands of an infected person to an intermediate surface or directly to the hands of an susceptible individual. Etiology: Rhinoviruses, coronaviruses, and adenoviruses account for the majority of infectious rhinitis in children. Less common are parainfluenza viruses and RSV Other causes of rhinitis can be non-infectious, such as vasomotor rhinitis, allergic rhinitis (see topic 48A), nasal septal defect, foreign bodies, nasal polyps, atrophic rhinitis or nasal tumors. Clinical features of infectious rhinitis: The pt. has a sudden onset of clear or mucoid rhinorrhea, nasal congestion, and fever. Mild sore throat and cough may develop The nose, throat, and tympanic membrane can appear red and inflamed. The usual duration is 3 to 7 days with peak symptoms on the second and third day. Mucopurulent rhinitis (thick, opaque nasal discharge) follows the clear-phase rhinitis frequently

and is not an indication for ATB treatment. Rhinitis is usually accompanied by light sinusitis which resolves without ATBs as well. Therapy: Symptomatic – antihistamines, NSAIDs, intranasal or oral decongestants. Complications: They are rare (1-5%) and involve acute otitis media or bacterial sinusitis. b) Adenoiditis = refers to inflammation of the tonsils of the pharynx and the Waldeyer’s ring (see tonsillopharyngitis – topic 23B) c) Grippe = Influenza (see topic 19B) 132 64) PLEURAL EFFUSION – DD AND TREATMENT Normally, there is a small amount of fluid spread over the visceral and parietal pleurae. The fluid is similar to plasma except that it is lower in protein. Pleural fluid enters from the pleural capillaries and exists via parietal pleura and the lymphatics. 1. Pleural effusion (excess fluid in the pleural space) are classified as transudates or exudates Transudates are due to elevations in microvascular pressure or to decrease in oncotic pressure, exudates are due

to pleural inflammation (pleurisy), with an increased permeability of the pleural surface to proteins. Lymphatic obstruction can also contribute to effusion 2. Hemothorax (blood in the pleural space) occurs most often from trauma and rarely after rupture of a vessel in a parietopleural adhesion associated with spontaneous pneumothorax. Spontaneous hemothorax occurs rarely as a complication of a coagulation defect. Plural blood does not clot and may be easily withdrawn via a needle or a water-sealed tube thoracostomy. 3. Chylothorax (a milky or chylous effusion) is caused by traumatic or neoplastic (most often lymphomatous) injury to the thoracic duct. The lipid content is high, fat droplets are seen in microscope. Cholesterol content is low Conditions that can cause transudate: 1. Heart failure, the most common cause, usually bilateral If unilateral, it’s generally right-sided 2. Hypoalbuminemia is usually bilateral and associated with edema elsewhere in the body 3. Ascites may be

associated with effusion (fluid can move from the peritoneum into the pleural space through diaphragmatic vessels or lymphatic channels). In 70%, they are right-sided 4. Meigs’ syndrome (pleural ascites + ovarian tumor) has a similar mechanism 5. Peritoneal dialysis or acute pancreatitis 6. Myxedema, effusions are usually transudates but may be exudates 7. Iatrogenic pleural effusions results when fluid infused through a catheter meant for a subclavian vein enters the pleural space. Conditions causing exudates: 1. Mycotic (blastomycosis, coccidiodomycosis) pleurisy produces an exudate, and pleural biopsy shows granulomas. Organisms can be cultured from pleural fluid and tissue There may be an underlying parenchymal disease. 2. Parapneumonic effusions, the visceral pleura overlying a pneumonia becomes inflamed The fluid contains many neutrophils and may contain bacteria. Parapneumonic effusions are usually caused by bacteria, small effusions may occur with viral or mycoplasmal

pneumonia. 3. PE - produces pleural effusions in 30-50% 80% are exudates, often blood-colored 133 4. Metastatic neoplasms are the most common cause in old people, but not children The fluid is bloody or hemorrhagic. Most neoplasmatic effusions can be diagnosed by cytologic examination. Pleural biopsy is less sensitive 5. Hodgkin disease and NHL - common; it may be the presenting sign! Lymphatic obstruction is predominantly the cause of effusion in Hodgkin disease, pleural infiltration in NHL. Cytology or biopsy is usually rarely needed for diagnosis. 6. SLE or drug-induced lupus-like syndromes (eg phenytoin) produces effusions in 40% of pt The symptoms usually disappear with the drug discontinuation. In SLE, fever, pleuritic pain, and some systemic manifestations are usual, rarely, isolated pleural effusion occurs. Neutrophils predominate early, monocytes later. Pleural fluid antinuclear antibody (ANA) titer is high 7. Rheumatoid disease causes pleural effusion (especially

systemic onset JRA) 8. Subdiaphragmatic abscess produces a sympathetic effusion, a sterile exudate with neutrophils predominating. The dg is made by US or abdominal CT 9. Acute pancreatitis associated effusion has a high amylase levels and is rich in neutrophils It may be exudate or transudate. Pleural exudate may also occur after cardiac surgery It responds to NSAIDs and corticosteroids. Uremia is often complicated by generalized serositis, the fluid is grossly bloody and contains few cells, mainly mononuclears. Creatinine level is high. Symptoms, signs, and diagnosis: - Pleuritic pain and dyspnea are the most common, but many are asymptomatic and discovered incidentally (dullness, decreased hemithorax motion, decreased breath sounds) or on CXR. - Despite extensive effort, the etiology of an effusion is not found in 20% of cases. - CT is valuable in evaluating the underlying lung parenchyma (abscess, pneumonia, carcinoma). - US can also identify loculated pleural effusion

(between adhesions). - Thoracentesis is performed to confirm the presence and to analyze it (see above). - Specimens should be taken for chemical, bacteriologic, and cytologic examination. The fluid sediment is also examined microscopically. - Exudates have at least one of the following features: - pleural/serum protein ratio > 0.5, with pleural fluid protein usually > 30g/dL - pleural/serum LDH ratio > 0.6 - pleural LDH > 2/3 of the upper normal limit for serum. 134 - Transudates have no such findings. If neutrophils and bacterias increases in the fluid, it becomes pus. The result is empyema (= purulent exudate in the pleural space) Most empyemas are caused by anaerobic bacteria. It may be a sequelae of a penetrating wound, a thoracotomy, ruptured abscess, or infection from a hepatic or subdiaphragmatic abscess. Lower amounts of neutrophils and bacteria suggest pneumonia and parapneumonic effusion. The presence of small mature lymphocytes is suggestive of

TB. - Other important findings include glucose levels (low levels in TBC, malignancy, parapneumonic effusion, or rheumatoid disease), - amylase levels (high in pancreatitis). Whenever the diagnosis is not clear, a needle biopsy of the parietal pleura should be performed. Treatment: Thoracentesis relieves dyspnea due to a large plural effusion. It must not be removed too quickly bc of cardiovascular collapse danger. Pneumothorax may complicate thoracentesis Infection in the plural space must be treated by a long course ATB treatment. Plural fluid is usually reabsorbed spontaneously then. Empyema must be drained (aspirations or thoracostomy). Surgery may be necessary when the empyema cavity is lined by a thick, organizing, fibrous exudate. Treatment of pleural effusion due to malignancy is often difficult. Pleural fluid often reaccumulates Pleurodesis is done in such cases (instillation of a sclerosing agent, e.g free talk) The result is an intense pleuritis that results in

obliteration of the space. 135 65) HELMINTHIC DISEASES. 1) Enterobiasis (Pinworm infection) Identification: intestinal helminthic infection, perianal itching, disturbed sleep, irritability, infection of the scratched skin. Diagnosis is made by applying scotch tape to the perianal region and examining the tape microscopically for eggs Infectious agent: Enterobius vermicularis (intestinal nematode) Occurrence: Worldwide Reservoir: Human (particularly school children) Mode of transmission: Direct transfer of infective eggs by hand from the anus to the mouth of the same or other person, or indirectly by cloths, bed linen & food contaminated with eggs. Incubation period: 2-6 weeks Period of communicability: As long as the females are discharging eggs on the perianal skin Pathogenesis: Worm and egg cause perianal pruritus Treatment: Mebendazole. Prevention: Hygiene 2) Ascariasis (roundworm infection) Caused by Ascaris lumbricoides. A common infection, esp in preschool and younger

children Life cycle: Eggs are found in the soil, and humans are infected by contact with the soil. The eggs are ingested and hatch in the intestine, where they become larvae. The larvae penetrate the intestine, enter the venules or lymphatics, and subsequently reach the lungs, where they mature. Mature roundworms then migrate up the bronchioles into the pharynx, where they are swallowed and finally pass into the small intestine, and the cycle is repeated. The adult worm is 25 cm long Clinical features: Most pt. are asymptomatic Abdominal pain may occur with heavy infection Pulmonary symptoms are rare and result from large numbers of larvae passing through the lungs, causing cough, blood-stained sputum, eosinophilia, and pulmonary infiltrates. Diagnosis: Worms that appear like long, pink, earthworms are seen in the stool. Eggs are found in the stool and can be identified microscopically. Therapy: Pyrantel in a single dose is curative in 90% of pt. The drug is well tolerated Mebendazole

and piperazine are also effective. 136 3) Taenia solium (cysticercosis) It is the pork tapeworm. Larval infection causes cerebral cysticercosis Man serves as a host for adult worms and also for the larval stage (cysticerci). Frequent infections are found in Asia, eastern Europe, and Latin America. Infection with the adult worm is acquired by ingesting pork containing cysticerci; it may survive for years in the human intestine and produce 50.000 eggs per day Cysticercosis is acquired by ingestion of eggs in water contaminated with raw sewage or vegetables fertilized with soil containing worm eggs. The eggs hatch in the small intestine, and the embryos penetrate the intestinal wall and may be distributed to subcutaneous tissue, muscle, viscera, and CNS. Diagnosis: Segments of the adult worm may be found in stools. In the brain, a cyst containing lowdensity fluid with a calcified dead larva is characteristic Treatment: Praziquantel, niclosamid. Brain cysts may be resected Cooking

pork prevents the infection with adult worms. Mortality is low if treatment is adequate 4) Toxocariasis: visceral larva migrans It is a syndrome caused by the migrating larvae of the helminth Toxocara canis or catti. It is widespread wherever there is dog or cat population. Human infection takes place when eggs from dog or cat feces are ingested. Toddlers and young children who tend to eat soil and dirt are most likely to be infected. The eggs hatch in the small intestine and the larvae migrate to various organs, mainly the liver, lungs, eye, and brain. In time, granuloma forms around the larvae. During the migration stage, a high eosinophil; count is typical Man is an abnormal host, and the larvae do not become adult worms. Diagnosis: Many infections are subclinical with the only evidence being an elevated eosinophil count and positive fluorescent antibody test. ELISA is also available Clinical features may include muscular pain, urticarial rashes, and hepatomegaly. Ocular involvement

may lead to blindness. CNS involvement may cause epilepsy Treatment: The lesions usually subside in 6-18 mo, and the prognosis is good. Thiabendazole is recommended, corticosteroids for ocular forms and severe symptoms. 137 66) CONGENITAL DISORDERS OF THE CARBOHYDRATE METABOLISM. a) Galactosemia AR with 1:40,000 prevalence. Classic galactosemia is caused by deficiency of galactose-1phosphate uridyltransferase, causing accumulation of galactose-1-phosphate in the liver, brain, and renal tubules causes hepatic parenchymal disease, mental retardation, and renal Fanconi syndrome. Accumulation of galactitol in the lens produces cataract. With prompt galactose-free diet, the prognosis for survival without severe mental retardation and liver disease is excellent. Onset of the disease is accompanied by vomiting, jaundice, and hepatomegaly in the newborn period after milk feeding. Without treatment, death occurs in the first month of life In less severe cases, survival is possible but

mental retardation is irreversible. Newborn screening is becoming common. Prenatal diagnosis is possible Treatment: A galactose-free diet should be started as soon as possible. Avoidance of galactose should be lifelong in severe cases, with appropriate calcium replacement. b) Glycogen storage diseases Glycogen is a highly branched polymer of glucose that is stored in liver and muscle. Different enzyme defects affect its biosynthesis and degradation. The most common glycogenoses cause growth failure, hepatomegaly, and fasting hypoglycemia. Muscular effects are mainly easy fatigue, mm weakness & stiffness. Other manifestations include macroglossia & cardiomegaly. 1. Type-1: von-Gierke disease: glu-6-phosphatase def with accumulation of excessive normal glycogen in liver & kidney. 2. Type-2: Pompe’s disease: 1,4glucosidase def accumulation of normal glycogen in heart, mm, liver & CNS. 3. Type-3: Cori’s disease: 1,6 glucosidase def accum of abnormal glycogen in liver

& mm 4. Type-4: Andersen’s disease: glucan branching enzyme def accum of abnormal glycogen in liver & mm, kidney cirrhosis. 5. Type-5: McArdle’s: mm glycogen phosphorylase def accum of normal glycogen in mm 6. Type-6: Her’s disease: hepatic glycogen phosphorylase def accum of normal glycogen in liver & WBC. 7. Type-7: phosphofructokinase def in mm 8. Type-9: hepatic phosphorylase kinase def, which regulates glycogen phosphorylase in the liver Diagnosis: Biochemical tests (responsiveness of blood glucose to fasting and glucagon), and enzyme assays on leukocytes, liver, or muscle. Treatment: Prevention of hypoglycemia while avoiding storage of even more glycogen in the liver. This requires monitoring of specific diets, using restriction of free sugars & measured cornstarch. 138 67) ASCITES. EDEMA – DD AND TREATMENT a) Ascites It is accumulation of fluid in the peritoneal cavity. The ascites frequently is a component or a complication of hepatic cirrhosis,

CHF, nephrosis, or carcinoma. In such disorders the fluid has the characteristic of a transudate (< 25g/L of protein, specific gravity < 1.016, and white blood cell count < 250/m3). Diagnosis: - Shifting dullness to percussion is the most sensitive clinical sign of ascites. - Ascites can be confirmed by US. - Diagnostic paracentesis is important in a pt. who has no obvious cause of ascites - The fluid should be examined for its gross appearance, specific gravity, protein content, total lipids, serum and fluid lactate, cell count, and differential. - Gram stain, acid-fast stain, and culture are requested if infection is suspected. - Cytology should be done. Cloudy or turbid fluid with a predominance of polymorphonuclears, a positive Gram stain, and a lactate level or pH less than that of the serum are characteristic of bacterial peritonitis. Antibiotics are indicated. If most cells are lymphocytes, TBC should be suspected Peritoneal biopsy, stain and culture for

acid-fast bacilli should be performed. If it is positive, start antituberculotics Blood-stained or hemorrhagic peritoneal fluid is seen in pt. with malignancy with peritoneal seeding. Cytology is requested in such cases The DD of hemorrhagic peritoneal fluid includes pancreatic ascites secondary to pancreatitis and pancreatic pseudocyst. An increase of amylase levels supports it (+ cytology is normal). TB peritonitis may sometimes manifest as bloody peritoneal fluid. Chylous ascites refers to a turbid, milky, or creamy peritoneal fluid. Such fluid contains fat This disorder results from a congenital or acquired intestinal lymph obstruction. Among the causes of acquired intestinal lymph obstruction are trauma, TBC, tumor, nephrosis, and filariasis, and previous surgery with trauma to the main lymphatic ducts. A fluid that is turbid because of leukocytes or tumor cells (no fat) may be confused with chylous fluid. Cytology is performed in such cases. If it is normal, ascites probably

results from nephrosis, cirrhosis, or cardiac disease If it is abnormal, malignancy is the cause. Straw-colored fluid is always a sign of cirrhosis, renal, cardiac or nutritional disorder. In liver disease or intra-abdominal disorders, ascites is usually isolated and disproportionate to peripheral edema; in systemic disease (heart failure), the reverse is true. 139 Symptoms: Massive ascites may cause nonspecific abdominal discomfort and dyspnea, but lesser amounts are usually asymptomatic. Treatment: Bed rest and dietary Na restriction. Diuretics are used if Na restriction fails Changes in body weight measure response to treatment. Therapeutic paracentesis is an alternative Edema It is the excess accumulation of interstitial fluid. It may occur as a result of increased capillary pressure (congestive heart failure – CHF) or decreased plasma protein concentration. Decreased plasma protein may be due to increased losses (nephrotic syndrome, burns), decreased intake (malnutrition),

or impaired lymphatic flow. The history and physical examination are very important. Signs and symptoms specific to heart failure, liver failure, and renal disease should be obtained. 1. Localized edema - Angioedema is a form of urticaria affecting deeper tissues, including the skin and subcutaneous tissues. It often involves the lips, dorsum of the hands and feet, scalp, scrotum, and periorbital tissues. There are many causes, including drugs, food, contactants, inhalants (pollen, dander), and infections. Episodic angioedema is associated with fever and eosinophilia Hereditary angioedema, which results from decreased synthesis of C1 esterase inhibitor, occurs as sudden attacks of edema often precipitated by minor trauma, strong exercise, or any stressors. - Lymphedema is caused by the obstruction of lymphatic flow. Congenital lymphedema may occur in Turner’ sy, Noonan sy, and Milroy disease. Acquired obstruction may be due to tumors, lymphoma, filariasis, postirradiation

fibrosis, and postinflammatory or postsurgical scarring. Injury to major lymphatic vessels may result in chylous ascites - Venous thrombosis – edema occurs distal to the site of thrombosis. 140 2. Generalized edema - Heart failure occurs when the heart cannot deliver adequate output to meet the metabolic needs of the body. Signs and symptoms include tachycardia, tachypnea, systemic venous congestion (hepatomegaly), and cardiomegaly. Other features may include feeding difficulties, excessive sweating, and failure to thrive. Respiratory symptoms (wheezing, rales, cough) are due to pulmonary congestion. Jugular venous distention may be present An older child may have orthopnea or syncopal symptoms. Poor peripheral perfusion results in cool extremities, prolonged capillary refill time, and weaker peripheral pulses. Causes of heart failure in childhood: Congenital heart defects are the most common cause of heart failure in infants and children. Other causes include

hyperthyroidism, cardiomyopathy, myocarditis, arrhythmias, coronary artery lesions (Kawasaki sy), rheumatic heart disease, hypertension, cor pulmonale, or an arteriovenous malformation. - Renal etiology – edema accompanies the nephrotic sy, tubulointerstitial disease, glomerulonephritis, and renal failure. For details see topics 24A, 26A, 28C, 32C - Liver failure Features include progressive jaundice, fetor hepaticus, fever, anorexia, vomiting, and abdominal pain. There may be a rapid decrease in liver size, hemorrhagic diathesis, and ascites Infants may present with irritability, lethargy, poor feeding, and sleep disturbances. Mental status changes occur with progression of symptoms. Older children may have asterixis In liver failure, hypoalbuminemia results in edema and bilirubin levels (both direct and indirect) are elevated. Serum aminotransferase levels are elevated early but may decrease as the pt condition deteriorates. Prothrombin time is prolonged The serum ammonia level

is elevated, and there may be hypoglycemia, hypokalemia, hyponatremia, metabolic acidosis, or respiratory alkalosis. Causes of liver failure include viral hepatitis, EB virus, HSV, adenovirus, enterovirus, parvovirus B19, and varicella-zoster infections. Known hepatotoxins include acetaminophen overdose, carbon tetrachloride, and Amanita phalloides mushroom. Idiosyncrastic damage may occur after halotane, phenytoin, or carbamazepine. Metabolic disorders associated with liver failure include Wilson disease, galactosemia, hereditary tyrosinemia, hereditary fructose intolerance, and urea cycle defects. - Burns 141 - Kwashiorkor It is a clinical syndrome resulting from severe protein deficie3ncy and inadequate calorie intake (protein calorie malnutrition). Early in the dise3asae, symptoms include anorexia, lethargy, apathy, and irritability. Later, there is decreased growth, muscle loss, increased susceptibility to infections, and edema. The edema may mask poor gain weight Skin

changes may be present, and the hair becomes coarse and grey. Laboratory findings include decreased serum albumin, hypoglycemia, hypophosphatemia, hypokalemia, and hypomagnesemia There is anemia. Signs of vitamin and mineral deficiencies may be present (vit A, zinc) - Protein-losing enteropathy It may result in edema secondary to hypoalbuminemia. Alfa-1 antitrypsin, unlike albumin, is resistant to digestion. Measurements of levels in the stool is helpful in dg Causes include intolerance to protein in cow’s milk or soy formula and postinfectious enteropathy. Eosinophilic gastroenteritis is another cause, it may be associated with dietary protein hypersensitivity as well as food allergies. There is elevated IgE and eosinophilia - Other causes of edema include premenstrual edema, edema associated with oral contraceptives intake, associated with hypercortisolism (both from adrenal or pituitary hyperfunction or due to therapeutical administration). Syndrome of inappropriate secretion

of ADH is also accompanied by edema, and excessive administration of infusions, too. Some drugs may cause edema Treatment: It is variable according to the underlying condition. For more details see topic 34C 142 68) ANGINA, TONSILLITIS, DIPHTHERIA 1) Angina, Tonsillitis (Acute pharyngotonsillitis) The majority of pharyngitis are not exsudative, and usually are due to viruses (70–80%): adenoviruses, enteroviruses, EBV, CMV. Group A beta-hemolytic strep are the major cause of bacterial pharyngitis (causing mainly exsudative tonsillitis: angina), which is the only commonly occurring form of acute pharyngitis for which antibiotic therapy is indicated. Rare infections are due to C diphteriae and N gonorrhoae. Clinical manifestation: Strep tonsillitis: onset in winter or spring, school-aged children, sore throat, abdominal pains. Having 3 of the following is enough for empirical treatment: fever, tonsillar exsudate, tender cervical LN, and no cough. Pt with one or two symptoms should

have further diagnosis: culture - to detect group A streptococci (ASLO). Viral infection is suggested by absence of fever, rhinorrhea, obstruction of the nose, cough, conjunctivitis, hoarseness and diarrhea. Viral infections are particularly common in children < 3 yr Infectious mono (EBV) occurs most commonly at ages 15-25 yr. The pt with IM presents with fever, sore throat, often with petechiae on the soft palate, general lymphadenopathy & HSM. A rash is seen in only 5% but the rash is seen very often in pt. who have by mistake received an aminopenicillin for their symptoms. Lab testing: take a throat swab from both tonsils. Serology is used for IM detection (monospot test) Therapy: ATB therapy is advised for group A strep infection to prevent both suppurative (abscess, otitis media, sinusitis) and nonsuppurative complications (rheumatic fever, glomerulonephritis). The drug of choice is penicillin (or erythromycin as an alternative) for 10 d. Tosillectomy should be considered

if acute tonsillitis recurs repeatedly after treatment. 2) Diphtheria An acute bacterial disease involving tonsils, pharynx, larynx, nose, and mucous membranes (it involves formation of a fibrinous pseudomembrane) producing sore throat, & enlarged cervical LN. The exotoxin may also cause myocardial and neural tissue damage. Infection can occur in immunized pt. (mainly people > 50 yr old) Caused by Corynebacterium diphtheriae, gram (+) rod Occurrence: Cold areas; Reservoir: Humans throat; Incubation period: 2-5 days. Mode of transmission: contact with ill pt. or carrier by respiratory droplets Period of communicability: As long as bacilli are discharged and lesions present Pathogenesis: The organism secrets an exotoxin that inhibits protein synthesis. The toxin has two components: (1) subunit A which has enzymatic activity; and (2) subunit B which binds the toxin to the surface receptors 143 Clinical features: - Initially, the pt. has only mild sore throat, dysphagia,

low-grade fever, and rising NEUT count - N/V, chills, headache, and fever are more common in children. - The membrane is dirty gray, tough, and adheres firmly, so that removal causes bleeding. - The disease may remain mild but when it progresses, dysphagia, toxemia, and prostration are prominent. Pharyngeal and laryngeal membrane may obstruct breathing completely - The cervical LN are enlarged. - Nasopharyngeal involvement may produce a serosanguineous nasal discharge. Diagnosis: Clinical appearance, Gram stain, culture, Treatment: Antitoxin must be given early, penicillin G (or erythromycin), bed rest, fluid intake, intubation if necessary. Cardiac and neurologic monitoring Prevention: toxoid vaccine (with tetanus and pertussis - DTP). Pt should be isolated 144 69) DISORDERS OF CALCIUM HOMEOSTASIS (see also rickets - topic 15B) Ca and phosphorus homeostasis is maintained by adequate nutritional intake of Ca, phosphorus, and vitamin D and a normally mineralized

skeleton, which is the major reservoir of these minerals. Calcium and phosphorus levels are regulated by PTH, which acts on bone and kidneys to raise the serum levels of calcium and lower the levels of phosphorus. Disorders of the parathyroid glands 1. Primary hyperparathyroidism - rare in childhood, d/t an adenoma or hyperplasia It may be isolated or part of MEN I or II. Increased PTH levels cause mobilization of calcium and phosphorus from bone. In kidneys, it decreases tubular reabsorption of phosphorus. Thus, the serum levels of calcium are elevated (hypercalcemia) and the phosphorus levels are low. Symptoms are related to hypercalcemia and include N/V, constipation, lethargy, confusion, and weakness. HTN and renal colic secondary to kidney stones are common Lab findings include elevated serum Ca, low phosphorus, increased ALP and low tubular reabsorption of phosphorus. PTH levels are elevated PTH secretion is autonomous X-rays shows subperiostal bone resorption. Therapy involves

excision of the adenoma and subtotal parathyroidectomy for hyperplasia. 2. Secondary hyperparathyroidism: diseases that cause hypocalcemia stimulate PTH release The elevated PTH then restored the serum calcium to normal, but at the expense of having low phosphorus. Conditions associated with secondary hyperparathyroidism are lack of intake, absorption, or metabolism D (e.g CRF, liver disease) Diagnosis & therapy: see rickets (15B) 3. Tertiary hyperparathyroidism: when 2o cases are not treated, the gland may start to secrete autonomously. Treat the secondary cause 4. Hypoparathyroidism a) Idiopathic hypoparathyroidism may occur in the neonatal period or at any time during childhood. The cause may be autoimmune Symptoms are caused by low se Ca and include seizures, tetany, numbness of the face and extremities, and carpopedal spasm. Diagnosis: calcium levels are low, phosphorus levels high, the PTH levels low (lack of response to the hypocalcemic signal). b) Pseudohypoparathyroidism

also manifests as symptomatic hypocalcemia, but PTH levels are very high, indicating PTH unresponsiveness due to a receptor defect. Pt. have skeletal and facial characteristics, including short stature; round face; a short, thick neck; and short metacarpals. Cognitive impairment is common 145 Treatment: - Vitamin D is the treatment for idiopathic hypoparathyroidism and pseudohypoparathyroidism: because low levels of PTH inhibit 1,25-hydroxylation of vitamin D in the kidneys, PTH deficiency is associated with a deficit of 1,25 dihydroxyvitamin D3 (calcitriol), the most active vitamin D metabolite. Treatment with 1,25 dihydroxyvitamin D3 overcomes the deficit and also stimulates increased calcium absorption in the intestine. - Oral calcium supplementation. Calcium and phosphorus levels should be monitored frequently to avoid hypercalcemia and potential nephrocalcinosis and renal damage. 5. Neonatal hypocalcemia Early hypocalcemia in the first 24-48 hours is usually associated

with prematurity It may be treated with oral or IV calcium supplementation. Late hypocalcemic tetany may occur in the first few weeks of life in infants receiving highphosphate diets (e.g cow milk) Idiopathic hypoparathyroidism may manifest in this period with hypocalcemic seizures. 146 70) HEMATURIA – DD AND TREATMENT Hematuria may be gross or microscopic. Causes: Virtually any congenital anomaly, injury, or inflammatory disease of the kidney or urinary tract may cause hematuria. Isolated microscopic hematuria is relatively common and usually does not mean any serious renal disease. Most cases are idiopathic However, when microscopic hematuria occurs in association with proteinuria, it may indicate a disease. A common cause of microscopic hematuria in childhood is idiopathic hypercalciuria. Children with painful hematuria should be investigated for direct injury to the urinary tract; whether the pain is dysuria, associated with cystitis or urethritis; back pain secondary to

pyelonephritis; or colicky flank pain accompanying the passage of a stone. Bright red blood or clots in the urine are associated with bleeding disorders, trauma, and arteriovenous malformations. Abdominal masses suggest the presence of urinary tract obstruction, cystic disease, or tumors involving the renal or perirenal structures. Evaluation: Gross inspection of the urine and the urine dipstick test for blood may be useful, but microscopic examination of fresh urine is most important. Microscopic hematuria is significant if it is persistent and there are > 5-10 RBC per high-powered field. Urine that is brown or tea-colored suggests glomerular bleeding; dysmorphic RBS or RBC casts are even more specific. Persistent, unexplained microscopic hematuria for > 6 to 12 months or a single episode of gross hematuria should be evaluated by US to exclude obstruction, renal cysts, or Wilms’ tumor. The most common causes of hematuria in childhood include: 1. Glomerulonephritis (GN) Acute

poststreptococcal GN is the most common form of postinfectious GN. The epidemiologic relationship between certain strains of streptococci and GN is well recognized. Ag-Ab complexes are formed in the bloodstream and deposited in the glomeruli. These deposited complexes may start glomerular inflammation and activate the complex system. The diagnosis may be supported by a recent history (7-14 days previously) of group A betahemolytic strep infection. If a positive culture is not available, recent infection may be supported by an elevated antistreptolysin (ASLO) titer. The clinical presentation is usually with gross hematuria (coffee or tea-colored urine), with or without edema (e.g periorbital) Symptoms are usually nonspecific; in cases of severe HTN, headache may be present. Fever is not expected Severe glomerular injury may be accompanied by massive proteinuria (nephrotic syndrome), anasarca or ascites, and severe compromise of renal function. 147 There is no specific treatment.

ATB therapy is indicated for infection if still present Resulting HTN may require dietary management, diuretics, or antihypertensive drugs. In severe cases, dialysis may be necessary. The acute abnormalities generally resolve in 2-3 weeks; although microscopic hematuria may persist for as long as a year, most children recover completely. Persistent deterioration in renal function is an indication for renal biopsy. The various types of GN (MPGN, IgA nephropathy, Henoch-Schönlein purpura, SLE nephropathy, hereditary GN (e.g Alport syndrome) have similar manifestations Minimal change nephropathy is one of the most common forms in childhood. Diagnosed only by EM since histology is normal. Treatment is supportive 2. Acute tubulointerstitial nephritis Characterized by diffuse or focal inflammation and edema of the renal interstitium and secondary involvement of the tubules. Types: a. drug induced (eg methicillin, penicillin, cephalosporins, thiazides diuretics, and NSAIDs) b.

Infection-related tubulointerstitial nephritis It may be the direct result of pyelonephritis or an immune-mediated response. Bacterial, viral, fungal, and parasitic infections may be involved Tubulointerstitial nephritis may manifest in several ways. Manifestations vary from abnormalities in urinalysis only to anuric renal failure. Definitive diagnosis may require renal biopsy S&S: - Fever, rigor, abdominal or flank pain, and rashes may occur in drug-associated cases. - Urinalysis can show leukocyturia and hematuria. - The inflammation can be severe enough to cause significant deterioration of renal function. The underlying cause should be treated if possible. Immediate identification and removal of the causative agent is important in case of drug related TIN. A course with progressive renal insufficiency may require dialysis and treatment with corticosteroids. Most have a full recovery 148 71) PERTUSSIS (whooping cough) An acute bacterial infection involving the

respiratory tract, producing violent barking cough (highpitched inspiratory whoop). It is rarely accompanied by fever Pt at risk are those who are not vaccinated. All age groups may be affected Causative: Bordetella pertussis, gram (-) rod, which produces the pertusis toxin. Occurrence: Endemic disease worldwide, periodical outbreaks Reservoir: Human respiratory tract Mode of transmission: Respiratory droplets. It is highly contagious Incubation period: 6- 20 days Period of communicability: highly communicable in the early catarrhal stage before the paroxysmal cough stage, thereafter communicability gradually decreases despite persisting cough Clinical presentation: A catarrhal cold with simple cough is followed by paroxysmal cough after several days. These may be followed by a rapid inspiration causing the characteristic “whoop” Frequency of paroxysms is high (30-40x daily), more frequent at night, often terminated by vomiting. The cough may persist more than 3 weeks

Complications: Apnea, cyanosis, convulsions, aspiration, epistaxis, facial petechiae. Lab: Leukocytosis with relative lymphocytosis. Cultivation (laryngeal swabs) Therapy: Careful observation, antitussic drugs, intubation if necessary. Erythromycin helps if given during the first 5 days of illness. The pt should be isolated Prevention: 2 vaccines: (1) killed organisms, and (2) purified protein (acellular vaccine) given to children in combination with diphtheria and tetanus toxoids (DTP) 149 72) LYSOSOMAL STORAGE DISEASES. Lysosomes are cellular organelles in which complex macromolecules are degraded by specific hydrolases. Deficiency of a lysosomal enzyme causes its substrate to accumulate in lysosomes of tissues that degrade it, creating a characteristic clinical picture. These disorders are classified as: 1. Mucopolysaccharidoses (eg Hurler syndrome, Scheie syndrome, Hunter syndrome, Sanfilippo syndrome – type A, B, C, D, Morquio syndrome). 2. Lipidoses (Niemann-Pick disease,

Metachromatic leukodystrophy, Krabbe disease, Fabry disease, Farber disease, Gaucher disease, GM1 gangliosidosis, GM2 gangliosidosis [TaySachs disease, Sandhoff disease]). 3. Mucolipidoses (eg Mannosidosis, Fucosidosis) depending on the nature of the stored material. Most are autosomal recessive traits, and all can be diagnosed in utero The diagnosis of mucopolysaccharidosis is suggested by certain clinical and radiological findings and urine screening tests. Diagnosis should be confirmed by enzyme assays of cultured leukocytes or skin fibroblasts; this is esp important when parents want to have more children. When a lipidosis or mucolipidosis is suspected, diagnosis is made by appropriate enzyme assays of cultured leukocytes or skin fibroblasts. Most of these conditions cannot be treated effectively. 150 73) INFECTION OF THE URINARY TRACT. PYURIA – DD AND TREATMENT UTI occur in 1% of premature infants and newborns. Many are of hematogenously spread UTI raise the possibility of

urinary tract abnormalities. During the first year of life, males are more likely to have an anatomic basis for developing UTI (see topic 33C). Older boys with 1st infection should be examined for UT abnormalities; conservative approach is taken with older girls, especially if sexual activity or poor hygiene is suspected. The most common causes of UTI are E coli, Klebsiella, Staphylococcus, and enteric streptococci – all present in normal rectal and perineal flora. Unless there is a suspicion that bacteremia is responsible, bacteria gain access to the UT via the urethra. The problem is aggravated by poor hygiene, perineal infection, sexual activity, and instrumentation. Voiding provides safeguard against bacterial contamination of the urethra developing into infection, & abnormalities of the collecting system lead to upper tract disease and renal parenchymal damage. Symptoms and signs: - Newborns & infants present with nonspecific symptoms (fever, hypothermia, poor feeding,

jaundice, FTT, or sepsis). - They may have strong-smelling urine and be irritable. - Older children may have abdominal pain, vomiting, strong-smelling urine, fever, enuresis, increased frequency, dysuria, urgency, and flank pain. - Symptoms of pyelonephritis may include those of cystitis (dysuria, frequency, hematuria, retention, suprapubic pain,) plus high fever, chills, and costovertebral pain & tenderness. - Some infections may be relatively asymptomatic. - The presence of UTI should be documented by urinalysis and urine culture. Lab tests: Clean-catch, midstream method is useful in children who can void upon request. Bladder cathetrization or suprapubic bladder tap may be performed if necessary. Because of risk of contamination, more invasive procedures should be used only with appropriate indications. The presence of bacteria in the specimen is highly suggestive of UTI. Pyuria (> 5 WBCs/highpower field) usually indicates UTI but is absent in 60% of cases

Non-culture methods (nitrite sticks) for early detection of UTI are used in follow-up of children with recurrent infections. Treatment: Uncomplicated cases of urethritis or cystitis can be treated by a single oral antibiotic for 10 days. The choice of antibiotic must be reviewed when culture and sensitivity results are available. A pt. with suspected pyelonephritis should be covered by broad antibiotic until sensitivity results Follow-up urinalysis or use of nitrite sticks within 2-3 days can confirm therapeutic success. Persistent bacteriuria indicates superinfection with a different organism or with the same one due to obstruction or presence of a foreign body 151 Urologic intervention: Voiding cystouretherography should be obtained with proved infection. Renal US evaluates the anatomy of the urinary tract. Vesicouretheral reflux (VUR) is common in younger children. Mild reflux doesn’t cause renal damage and disappears if infections are prevented. Severe reflux requires

ureteral reimplantation In pt. without structural or functional urinary tract abnormalities, possible causes of recurrent infection include infrequent or incomplete voiding, poor perianal hygiene, pinworms, constipation, and the use of bubble bath. Prognosis: Lower urinary tract (bladder and below) infection has an excellent prognosis. Once it enters the kidney, the prognosis gets worse. 152 74) NUTRITION OF THE HEALTHY INFANT. BREAST FEEDING WEANING Infant nutrition: For infants, drinking and eating are intense experiences, comprise most of their socializing, and are integral parts of their developmental progress. Thus, the act of feeding provides emotional and psychological benefits as well as nutritional needs. The normal newborn has active rooting and sucking reflexes and can receive oral feedings immediately after birth. Normally, the feedings should not be delayed > 4 hours Spitting and regurgitating of mucus are common the 1st day but should clear spontaneously.

Nutritional requirements The nutrient requirements of the child are influenced substantially by the rate of growth. It varies with the child’s age and it is especially important during early postnatal life. Growth rates are higher in early infancy but then the rate of growth decreases rapidly during the first year of life and then decreases more gradually until the beginning of the adolescent growth spurt. Nutrient requirements also depend on body composition. In the adult, the brain, which accounts for only 2 % of body weight, contributes 19 % to the total basal energy expenditure. In contrast, in a term neonate, the brain accounts for 10 % of body weight and for 44 % of total energy needs. Thus, the brain is especially vulnerable to the effects of malnutrition in early postnatal life. Breast feeding: Breast feeding is one of the most important influences on children’s health worldwide. It provides optimal nutrition for the normal infant during the early months of life.

Immunologic factors in breast milk (including secretory IgA, lysozyme, lactoferrin, bifidus factor, and macrophages) help to provide protection against GI and upper respiratory infections. Allergic diseases are less common in infants who have been breast fed. Although formulas have improved progressively and are made to resemble breast milk as closely as possible, it is impossible to mimic the nutritional or immune composition of human milk. Absolute contraindications to breast feeding are rare. They include maternal TB and severe mastitis, and infant galactosemia. The premature infant weighing <1500g may benefit from the addition of a milk fortifier. Infants with CF can be breast fed if exogenous pancreatic enzymes are provided. Breast feeding is promoted by prenatal and postpartum education and follow-up. It is important for the mother to know that very few women are unable to nurse their babies. The newborn is generally fed on demand every 2-3 hours, with longer intervals ( 4-5

hours) at night. Thus a newborn infant nurses at least eight to ten times a day Mothers also need to be reassured about stooling pattern. In early stages, a loose stool is often passed with each feeding; 153 later (age 3 - 4 months), there may be an interval of several days between stools. Failure to pass several stools a day in the early weeks of breast feeding suggests inadequate milk intake. Technique of breast feeding Breast feeding can be started after delivery as soon as both mother and baby are stable. Correct positioning and breast-feeding technique are necessary to ensure effective nipple stimulation and optimal breast emptying with minimal nipple discomfort. Some breast-fed infants fail to thrive. The most common cause is poorly managed mammary engorgement which leads to diminished milk production. It can result from long intervals between feeding, improper infant sucking, a nondemanding infant, sore nipples, maternal or infant illness, and nursing from only one breast.

The mother’s ignorance of technique, inappropriate feeding routines, and inadequate amounts of fluid and rest for the mother all can be factors. Some infants are too sleepy and may need waking to feed at night. Primary lactation failure is rare but does occur Common problems - Mild nipple tenderness, severe nipple pain & cracking (may be managed by proper positioning, - Breast feeding jaundice (indicating inadequate intake of breast milk). - Maternal mastitis (flu-like illness with local breast tenderness). Nutrient composition 1. the relatively low protein content, which is, however, quite adequate for the normal infant 2. not excessive quantity of essential fatty acids 3. the presence of long-chain unsaturated fatty acids 4. a relatively low sodium and solute load 5. lower concentration of calcium, iron, and zinc, which with favorable absorption provides adequate quantities. A source of iron, either from iron-fortified cereal, meat, or supplement, however, should be

given by age 6 months. Weaning It can take place according to the needs of both infant and mother. Gradual weaning, starting typically after 4-6 months, is preferred. Bottle feedings are increased progressively over a period of several weeks as breast feeding is decreased. 154 Supplementation: Bottle feeding Bottle fed infants are fed on demand and tend to wake for feedings every 3 to 4 hours. The volume consumed in the first feeding is up to 15 mL. The volume offered in the next 48 hours is gradually increased to about 60 - 75 mL per feeding. Prepackaged formulas are available in the hospital in sterile bottles providing 20 kcal/30 mL with adequate vitamins. Feeding should increase gradually during the first week of life up to 100 - 120 mL approximately 6 times/day. The newborn should be offered water between feedings, especially in hot, dry environments. If the infant is exceeding his calculated intake of formula, water should be offered to avoid overfeeding. Those who do not

retain enough fluid are given 5 % glucose in 025 % sodium chloride by IV to make up the deficit. Commercially prepared formulas are available as powder, concentrated liquid, and prediluted liquid. All are preferable to whole cow’s milk It is recommended not to use whole cow’s milk in the first year of life. Instructions for special formulas should be written by the physician (eg soy protein formulas in lactose intolerance). It is recommended that all bottle fed infants receive a formula containing iron or an iron supplement. Special hypoallergenic or carbohydrate-free formulas are available, as are predigested formulas with TAG, amino acids, and monosaccharides. Starting solid foods Infants do not need solids before age 6 months, but they can handle solids at 4 months (full-term infants). To prevent food sensitivities, wheat, eggs, and chocolate should be avoided until age 1 155 75) CHRONIC LIVER DISEASES IN CHILDHOOD. a. Chronic hepatitis can be defined as an inflammatory

process of the liver lasting longer than 6 months. On the basis of pathology it is differentiated into chronic persistent hepatitis and chronic active hepatitis. Chronic persistent hepatitis is defined by an inflammatory reaction limited to the portal zone, with little or no fibrosis. Chronic active hepatitis is defined by an inflammatory reaction that is not limited to the portal area and periportal fibrosis. Pt. with chronic persistent hepatitis have better prognosis Etiology: - Infectious: Chronic HBV is the most common causative, though HCV gradually becoming more common. Coinfection with other viruses such as HCV/HBV and HBV/HIV may also lead to chronic liver disease. Other viruses include EBV and CMV - Autoimmune: It is the result of primary or secondary defect in T-cells. These pt initially have hypergammaglobulinemia (autoAb). Classification is based on antibodies - Metabolic/genetic: Wilson’s disease, alfa1-antitrypsin deficiency, CF. - Toxins/drugs Clinical

features: - Pt. with chronic persistent hepatitis usually have symptoms similar to acute hepatitis, such as fever, malaise, anorexia, and abdominal pain. - The presentation in chronic active hepatitis is more severe, with jaundice, HSM, and RUQ pain. - Ascites, digital clubbing, spider angioma, caput medusae, arthritis or GN may also occur. - Lab findings include elevated serum bilirubin, elevated aminotransferases (at least 10-fold), serum albumin is low, gamma globulin is elevated, FW is also elevated. 25% of pt have detectable HBsAg. Other pt may have positive autoAb It is important to test for anti-HCV or HCV RNA. Hypersplenism may result in anemia, leukopenia, and thrombocytopenia Therapy and prognosis: Hepatitis B: Recombinant INFα. Some pt may relapse Side effects include fever, fatigue, headaches, muscle aches, abdominal discomfort. Hepatitis C: The relapse rate is very high. Pt with the highest risk for progressing to cirrhosis benefit from INFβ. Sometimes it is used

in combination with ribavirin Autoimmune hepatitis: It is treated with immunosuppression. Prednisone is the first-line medication Azathioprine and cyclosporine are alternatives in severe cases. Liver transplantation may be necessary for pt. with cirrhosis and its complications (portal HTN, esophageal varices, ascites, liver failure). 156 b. Metabolic liver disease: They are numerous and vary in presentation For example α1-AT deficiency can occur in neonates as cholestasis and in older children as cirrhosis. The most important metabolic liver diseases for the pediatrician include: 1. α1-antitrypsin deficiency – α1AT is a serum protease inhibitor synthesized in the liver Deficiency results from homozygous inheritance of a specific gene. This results in low serum levels and in a slow moving protein on electrophoresis. Clinical features: It is the most common metabolic disease to cause neonatal hepatitis and for which children require liver transplantation. Some pt have neonatal

cholestasis Jaundice resolves in most cases. Sometimes, severe disease causes death in the 1st year of life Older infants may be seen with failure to thrive, hepatomegaly, or cirrhosis. In the adolescent, it may cause early pulmonary emphysema, cirrhosis, and HCC. Diagnosis: Low serum α1-AT, characteristic liver biopsy. Treatment: Cigarette smoking accelerates the onset of lung disease and should be avoided. Transplantation may be necessary in severe cases. 2. Wilson’s disease - a treatable AR disorder, which is the result of a mutation of a gene producing a copper-binding protein (ceruloplasmin). Pathogenesis: Organ damage occurs as the result of toxicity from copper deposition. Although levels of ceruloplasmin are low in 95% of pt., the exact mechanism is not known Clinical features: Liver disease is the most common presentation in pediatric pt., but it is rarely clinically evident before age 5. It may include acute hepatitis, fulminant hepatitis, chronic hepatitis, or cirrhosis

Neurologic symptoms (tremor, dysarthria, loss of fine motor control, chorea, ataxia, seizures) usually occur in young adulthood. Psychiatric symptoms may lead to diagnosis of OCD, schizophrenia, or mania-depressia. Other hemolytic anemia, Fanconi-like syndrome, & corneal deposition of copper, which causes formation of Kayser-Fleischer rings. Diagnosis: Kayser-Fleischer rings, low ceruloplasmin, elevated urinary copper, liver biopsy. Treatment: Dietary restrictions (chocolate, nuts, liver, shellfish, mushrooms and other foods rich in copper). Life-long treatment with chelating agents (penicillamine, trientine, and oral zinc for reduction of intestinal copper absorption. Prognosis: excellent with early treatment. Fulminant hepatitis has poor prognosis 157 76) PROTEINURIA – DD AND TREATMENT Urine is not completely protein free, but the average excretion is below 150 mg/24 h. Small increases in urinary protein can accompany febrile illnesses or exertion and in some cases are

produced while in the upright posture. Nephrotic syndrome – characteristics: - Proteinuria >5g/d with hypoalbuminemia < 28 g/L. - Massive edema. - Hyperlipidemia, hypercholesterolemia & lipiduria. Primary nephrotic syndromes can be classified according to their pathological lesions, which also correlate with various clinical courses. It may occur as a result of any form of glomerular disease and may be associated with a variety of extrarenal conditions. In children under age 5 years, the disease usually takes the form of idiopathic nephrotic syndrome of childhood. 1. Idiopathic nephrotic syndrome of childhood (lipoid nephrosis, minimal change disease) Affected pt. are under age 5 years Often following an influenza-like syndrome, periorbital edema, and oliguria are noticed. Within few days, increasing edema - even anasarca become evident With significant “third spacing” of plasma volume, however, some children may present with hypotension. With marked edema,

dyspnea due to pleural effusions may also occur There is heavy proteinuria, sometimes microscopic hematuria, plasma albumin levels are low, and lipid levels increased. Glomerular changes are minimal Treatment: As soon as diagnosed, corticosteroids should be started. Careful restoration of compromised circulating volume with intravenous albumin and administration of diuretics is helpful in mobilizing edema. 2. Focal glomerular sclerosis Can appear at any age, but is mostly up to age 5. It may be familial It is one cause of corticosteroid-resistant or frequent relapsing nephrotic syndrome. The etiology is unknown Pt. are prone to intercurrent infections, including septicemia, pneumonia, peritonitis, or UTI Diagnosis: renal biopsy (normal, as well as sclerotic glomeruli). The lesion has serious prognosis: 20% of cases can progress to end-stage renal failure. 3. Mesangial nephropathy (mesangial proliferative, membranoproliferative glomerulonephritis) It occurs in children of all ages.

Nephrotic syndrome is present in association with macro- or microscopic hematuria. HTN occurs in 1/3 of pt It is another cause of corticosteroid-resistant nephrotic syndrome. The renal biopsy shows an increase in the mesangial matrix of the glomeruli The origin is unknown. Treatment: high doses of prednisone Cyclosporine may help Relapses are associated with infection. 50% of pt will develop renal failure by 10 years The other half will have permanent remission even without therapy. 158 4. Membranous nephropathy (membranous GN) Although usually idiopathic, it can be found in association with diseases such as hepatitis B antigenemia, SLE, syphilis, and renal vein thrombosis; with immunologic disorders; and with administration of some drugs (e.g penicillamine) The pathogenesis is unknown, but the glomerular lesion is probably due to prolonged deposition of circulating antigen-antibody complexes. The onset may be insidious or may resemble that of idiopathic nephrotic syndrome. It

occurs more often in older children. It responds little to corticosteroids Diagnosis: renal biopsy 5. Congenital nephrotic syndrome It is hereditary and is transmitted by an autosomal recessive trait. It is characterized by proteinuria shortly after birth. The etiology is unknown It is most common in Finland The course is progressive and most children die of sepsis or thrombotic complications in the 1st year. 159 77) PERINATAL VIRAL INFECTIONS. Viral infection is uncommon in the newborn but can be devastating. Common agents include HSV, VZV, CMV, RSV, hepatitis A and B, echo, and coxsackie. Clinical features: - HSV symptoms do not appear until 3 to 7 days and up to 4 weeks after birth. These infections manifest as vesicular skin eruptions, DIC, shock, pneumonia, and encephalitis. - RSV manifested as temperature instability, respiratory distress, apnea, clear nasal discharge, and poor feeding. Diagnosis: Cultures, cord IgM level, changing Ab titer, rapid antigen or antibody

tests, maternal medical history, time of the year. Therapy: for herpes virus (systemic vidarabin or acyclovir, ophthalmologic vidarabin ointment) and RSV (aerosolized ribavirin) infections. Congenital CMV may be treated with gancyclovir 160 78) MALABSORPTION SYNDROMES Malabsorption of ingested food has many causes. Shortening of the small bowel (surgical resection) or mucosal damage (celiac disease) both reduce surface area. Impaired motility of the small intestine may interfere with normal movements and mixing of food with pancreatic & biliary secretions. Anaerobic bacteria proliferate under these conditions and impair fat absorption by deconjugation of bile acids (postoperative blind loop syndrome). Impaired intestinal lymphatic (congenital lymphangiectasis) or venous drainage also causes malabsorption. Diseases reducing pancreatic exocrine function (CF) or the production or flow of biliary secretions cause nutrient malabsorption. Malabsorption may be genetically determined

(disaccharidase deficiency, glucose-galactose malabsorption) Clinical findings - GI symptoms such as A/N/V, abdominal pain, diarrhea, steatorrhea and bloating are common. - Observe the stools for abnormal color, consistency, bulkiness, odor, mucus, and blood. - Microscopic exam of stools for neutral fat and fatty acids is helpful because most malabsorption syndromes involve some fat malabsorption. Pancreatic insufficiency is associated with neutral fat in stool. FA are the major fatty material found in the stool of mucosal and liver disease - Quantitative assessment of fat absorption requires measurement of fat excreted in the feces as a proportion of fat intake for a defined period. Serum carotene, vitamin E and vitamin D levels may be depressed by fat malabsorption. - Prothrombin time is increased (vitamin K deficiency). - Assessment of protein absorption requires isotopic labelling of amino acids. - Disaccharide or monosaccharide malabsorption is estimated by reduction

in stool pH, increased breath hydrogen, or decreased intestinal mucosal disaccharidase activity. - Other tests suggesting cause of malabsorption include sweat chloride test (CF), intestinal mucosal biopsy, liver function tests, and pancreatic secretion of enzymes after stimulation with secretin and cholecystokinin. 1. Protein-losing enteropathies – excessive loss of plasma proteins into the GI tract occurs in association with a number of disorders such as: - Vascular obstruction (e.g CHF, ASD, constrictive pericarditis) - Stomach (e.g Menetrier disease, polyps, gastritis due to H pylori) - Small intestine (celiac, intestinal lymphangiectasia, blind loop syndrome, IBD, malrotation). - Colon (e.g UC, Hirschsprungs disease, pseudomembranous colitis, villous adenoma) Clinical findings edema, chylous ascites, poor weight gain, and anemia. Serum albumin is low Fecal α1-antitrypsin, a marker of serum protein loss, is elevated. 161 Treatment: Albumin infusions, diuretics,

and high protein and low fat diet. However, treatment must be directed toward identifying and treating the underlying cause. 2. Celiac disease (Gluten Enteropathy) - results from intestinal sensitivity to gliadin fraction of gluten from wheat, rye, oats & barley. Most cases present during the second year of life It is thought that intestinal damage and atrophy results from a cell-mediated immune response to gliadin. The inheritance is probably polygenic, but may be associated with an intestinal adenovirus Symptoms and signs • Diarrhea – intermittent, later continuous, with bulky, foul-smelling stools. During celiac crises, dehydration, shock, and acidosis occur. • Constipation, vomiting, and abdominal pain – the triad may dominate the clinical picture. • FTT. • Anemia and vitamin deficiencies. Lab findings: Fat content in the stools, impaired carbohydrate absorption, hypoproteinemia (hypoalbuminemia can lead to edema, there is increased protein loss in the gut

lumen, and poor hepatic synthesis secondary to malnutrition). Imaging: A small bowel X-ray shows a malabsorption pattern, which is nonspecific and can be found in other malabsorption states. Biopsy: the most reliable test for celiac disease. Serologic tests: Antigliadin Ab are often positive, but this test is not specific. Antiendomysial and tissue trans-glutaminase Ab are the most sensitive and specific screening tests. Treatment • Diet – Dietary gluten restriction for life. All sources of wheat, rye, barley, and oat gluten must be eliminated. Supplemental calories, vitamins, and minerals are indicated in the acute stage • Corticosteroids – Corticosteroids are indicated only in very ill pt. with celiac crises (malnutrition, diarrhea, edema, abdominal distention, and hypokalemia). 3. Disaccharidase deficiency – starches and the disaccharides sucrose and lactose are the most important dietary carbohydrates. The dietary disaccharides and oligosaccharide products of

pancreatic amylase action on starch require hydrolysis by intestinal disaccharidases before absorption can take place. In primary disaccharidase deficiency, a single enzyme is affected, disaccharides are not tolerated, but histologic findings are normal. Family history is common 162 Because disaccharidases are confined to the intestinal epithelium, they are susceptible to mucosal damage. Many conditions cause disaccharidase deficiency, with lactase usually most severely depressed. Histologic examination reveals changes of the underlying disorder Clinical findings a) Primary (congenital) 1. Lactase deficiency – A rare condition leading to diarrhea after lactose ingestion The stools are frothy and acid. Vomiting is common Blood glucose fails to rise after lactose ingestion There is a rise in breath hydrogen after oral administration of lactose (from hydrogen produced by normal colon flora during fermentation of unabsorbed carbohydrate). Severe malnutrition may occur. Pt. respond

to reduction of dietary lactose 2. Sucrase and isomaltase deficiency – A combined defect inherited as an AR trait Rare b) Secondary (acquired) 1. Secondary lactase deficiency – There is a high prevalence of genetically determined lactase deficiency in certain races (e.g 100% of Asiats, 40% of whites) after age 3-5 years Certain antibiotics (e.g neomycin), celiac disease, malnutrition, gastroenteritis, or intestinal mucosal injury secondary to radiation and cancer chemotherapy can decrease lactase activity. 2. Secondary sucrase deficiency – Intestinal mucosal damage tends to lower the levels of all disaccharidases. Infectious diarrhea is the most common cause 4. Glucose-galactose malabsorption – rare The Na/glucose co-transporter protein is defective Severe diarrhea begins with the 1st feedings. The diarrhea disappears promptly on glucose and galactose withdrawal. The acquired form occurs in infants < 6 mo following acute infections The prognosis of the congenital form is

good if the disease is diagnosed early. In the secondary form, prolonged TPN may be required until intestinal transport mechanisms return. 5. Intestinal lymphangiectasia – this form of protein-losing enteropathy results from a congenital ectasia of the bowel lymphatic system, often associated with abnormalities of the lymphatics in the extremities. Obstruction of the lymphatic drainage of the intestine leads to rupture of the lymphatic vessels with leakage of lymph into the lumen of the bowel. Fat loss may be high and lead to steatorrhea. Chronic loss of lymphocytes and immunoglobulins increases the susceptibility to infection. Clinical findings: - Peripheral edema, diarrhea, abd distention, lymphedema, chylous effusions & repeated infections. Lab: low Ig and albumin levels, lymphocytopenia, and anemia 163 - In the stools, fat, lymphocytes, and α1-antitrypsin is elevated. Treatment: Vitamin and calcium supplements. ATB are used for specific infections Surgery is helpful

when the lesion is localized to a small area. Generally, the prognosis is not good 6. Cow’s milk protein intolerance - milk protein intolerance is more common in males and in young infants with a family history of atopy. Colic, vomiting, and diarrhea are the major symptoms Stools often contain blood and mucus. Sigmoidoscopic examination reveals a superficial colitis Less commonly, milk protein may induce eosinophilic gastroenteritis with protein losing enteropathy, hypoalbuminemia, and hypogammaglobulinemia. Some pt. with milk protein allergy have also sensitivity to soy protein It is therefore best to use a partially or completely hydrolyzed formula as an elimination diet. 164 79) HYPOGLYCEMIA IN INFANTS AND CHILDREN - DD AND TREATMENT. 1. Neonatal hypoglycemia - the normal infant born at term shows an immediate fall in glucose level after birth. This triggers an immediate release of glucose from hepatic glycogen stores to normalize the glucose level. Transient hypoglycemia of

the neonates is therefore increased in preterm infants, infants with intrauterine growth retardation (IUGR), and in infants in stress. This is partly due to reduced hepatic glycogen stores, and partly due to increased use of glucose by the relatively large brain. In infants of diabetic mothers and infants with erythroblastosis or Beckwith-Wiedemann sy, a state of hyperinsulinism exists and hypoglycemia occurs after birth commonly. Symptoms and signs pallor, sweating, tremor, tachycardia. Newborn signs may be nonspecific: apnea, poor feeding, lethargy, seizures, and low body temperature. Treatment: Acute hypoglycemia is treated by glucose. A dose of glucagon may be given in some cases. Hydrocortisone may be helpful in prolonged difficulty in maintaining normal glucose level 2. Hypoglycemia in infants and children – rare in the neonatal period Causes: - Hyperinsulinism, GH deficiency, hypocortisolism secondary to ACTH deficiency, and primary adrenal insufficiency may manifest as

symptomatic hypoglycemia. - Some hepatic enzyme deficiencies, (glycogen storage diseases, hereditary fructose intolerance, galactosemia) which result in impaired gluconeogenesis or glycogenolysis, may present as metabolic acidosis, ketonuria, and hypoglycemia. Marked hepatomegaly with FTT and hypotonia suggests one of these defects. The diagnosis requires liver biopsy for assay of the specific enzyme. a) Ketotic hypoglycemia: In children 1 to 4 yr, it is the most common cause of hypoglycemia. The toddler has few episodes of early morning lethargy, pallor, sweating, with or without seizure. The blood glucose level during an episode is low Ketones are present in the urine The child is otherwise well. Episodes are associated with low food intake due to other illness Etiology: It appears to be the result of a low tolerance for fasting. Diagnosis: Glucose level monitoring. Glucose response to injected glucagon is minimal Therapy: Avoidance of fasting. Symptoms disappear usually after the

age 8 yr b) Hyperinsulinism: In the older child and adolescent, it may manifest as episodes of unpredictable or bizarre behavior, loss of consciousness, and seizures. Etiology: As opposed to the neonate who has persistent hyperinsulinemic hypoglycemia due to an inherited defect of beta cells, in older child, an insulin-secreting tumor may be found. Diagnosis: Monitoring of blood glucose level. Ketones are not present in the urine and the glucose response to glucagon is brisk. Insulin levels are inappropriately elevated Therapy: tumor may be localized by CT scan, ultrasound, or arteriography. Surgery is curative 165 80) INFLAMMATORY DISEASES OF THE BONES AND JOINTS a) Infections 1. Osteomyelitis (see topic 32B) 2. Septic arthritis - may be primary isolated infection or it may be secondary to adjacent osteomyelitis. There may be associated localized trauma When 2o to osteomyelitis, organisms enter the metaphysis via the blood stream & proceed to the capsule. In the joint,

bacteria cause release of synovial collagenase, which, together with lysosomal enzymes, destroy the cartilage. Clinical features: 80% involve the joints of the lower extremity, and 2/3 occur before age 3 yr. Streptococci and E. coli are the most common organisms in the newborns H influenzae is common in children < 2, and staphylococci are most common in older children. Findings include swollen, painful, erythematous joint, elevated ESR, and fever. Diagnosis: Aspiration from the joint and identification of the organisms by smear and culture. Bacteremia is common. X-ray shows widening of the joint space; joint destruction is a late finding. Long-term complications (joint destruction, ankylosis) occur when the diagnosis is late and adequate treatment is delayed. Prompt and appropriate treatment results in complete resolution Treatment: In hip infections, surgical drainage is an emergency. Systemic ATB are required Open drainage with irrigation of the purulent joint with saline is used

in other joints. b) Inflammations 1. Juvenile rheumatoid arthritis (JRA) - is not a single disease, but rather a group of conditions that are characterized by the presence of chronic synovitis in children. The arthritis results from chronic inflammation of unknown cause. Affected joints are swollen and may be painful, limited in motion, warm, and sometimes erythematous. Synovitis may persist for months without causing joint destruction; however, in some pt. damage to the joint may occur JRA is also associated with many extra-articular conditions. The severity of joint disease and the types of extraarticular manifestations differ in the various disease subgroups. These subgroups also differ in onset age, sex of affected pt, genetic predisposition, & prognosis. A. Systemic onset JRA - 20%; boys are slightly more affected, and can begin at any age The disease has dramatic extra-articular manifestations, including: High intermittent fevers, occurring once or twice a day & may be

associated with shaking chills. Erythematous rash which is maximal during periods of fever. HSM, lymphadenopathy, pericarditis, pleuritis, abdominal pain & peritonitis. Anemia and leukocytosis. The systemic complaints are usually self-limited to 6 months, 166 regardless of therapy. Pt may have arthralgia, myalgia, or transient arthritis affecting multiple joints, both small and large. B. Rheumatoid factor-negative polyarthritis - 25%; affects girls more, and can begin at any age Affected children have arthritis in multiple joints, both large and small, and no systemic symptoms. The arthritis is frequently mild C. Rheumatoid factor-positive polyarthritis - This condition is the childhood equivalent of classic adult rheumatoid arthritis, includes 5% of children with JRA, and rarely begins < 8 yr. Rheumatoid factor is present from the beginning. Multiple joints, both large and small are affected. The arthritis is often severe, with joint destruction occurring in the 1st yr of

the disease Subcutaneous nodules are present, and sometimes rheumatoid vasculitis, splenomegaly and leukopenia, and Sjorgen syndrome are associated. D. Pauciarticular JRA type I - this condition appears only in childhood, begins before age 5 yr, and includes 40%. Girls are affected more Arthritis may affect 1 or more joints, generally large, often asymmetrically. Knees, ankles, and elbows are most commonly affected 1/3 of children develop chronic iridocyclitis, which may progress to eye damage. Iridocyclitis may begin as long as 10 yr after the onset of disease. E. Pauciarticular JRA type II - this type is closely related to spondyloarthropathies, and accounts for 15% of JRA. Boys are predominantly affected and the diseases begins in children > 8 yr A family history of ankylosing spondylitis or other spondyloarthropathies is common. Affected pt. have arthritis in a few joints, most often of the lower extremities, hips are often affected. Arthritis may be acute and episodic or rather

chronic Pt may have sacroiliitis Extra-articular features include acute iridocyclitis and occasional fever. Diagnosis: Clinical picture, blood count, immunologic findings, X-ray. Treatment: Salicylates, NSAIDs, methotrexate in severe arthritis. Steroids should be used only in severe systemic disease which is life-threatening and in iridocyclitis. Physical and occupational therapy are essential components of therapy. Surgery: synovectomy & total joint replacement, generally for severe cases & older pt. 2. Acute rheumatic fever - potentially fatal and is still a leading cause of death in underdeveloped countries. It’s rare in developed world There’s a prior infection with group A beta-strep, almost always in the form of strep pharyngitis. It affects the endocardium The myocardium & pericardium may also be affected. Diagnosis depends on echo & Jones criteria arthritis (transient, migratory), rash (erythema marginatum), subcutaneous nodules, Sydenham’s chorea, and

nonspecific findings such as fever and malaise. Attacks are generally self-limited (wks to mo) and may recur Treatment anti-inflammatory drugs during active phase & prophylactic penicillin for prevention. 167 3. Spondyloarthropathies - a group of disorders characterized by chronic or recurrent arthritis affecting the spine and axial joints with or without the peripheral arthritis. These conditions occur in males, have seronegativity for both ANA and rheumatoid factors, familial occurrence, and association with HLA B27. Onset is rarely before age 8 yr A. Ankylosing spondylitis - begins with pain and stiffness in the hips, or back, or peripheral arthritis affecting few large joints. In children, the disease usually begins with peripheral arthritis, hence, these pt. are regarded as pauciarticular JRA type II Later, lower back involvement occurs, followed by spine in an ascending fashion. It manifests as pain, stiffness, and loss of motion in the back. Stiffness of the neck comes

late in the disease. Heel pain from Achilles tendonitis is common Acute iridocyclitis may occur B. Reiter syndrome - quite rare in childhood There is peripheral arthritis, conjunctivitis or acute iritis, and urethritis manifested by abnormal urinalysis or genitourinary symptoms. C. IBD - may be associated with a peripheral arthritis which is usually benign and resembles pauciarticular JRA type II. Bouts of arthritis are usually parallel to the underlying IBD Some children with IBD may have spondyloarthropathy which is associated with HLA B27. D. Psoriatic arthritis – not common in children E. Reactive arthritis - a pauciarticular arthritis, generally transient, occurs after GI infections with Shigella, Salmonella, Y. enterocolitica, or Campylobacter in individuals who are genetically predisposed; such individuals have HLA B27. F. Enthesopathy - inflammation of sites where ligaments and tendons attach to bone May be associated with peripheral arthritis. It is usually transient and

recurrent Course: Ankylosing spondylitis and the spondyloarthropathies may be progressive or may have long remissions without further progression. Ankylosing spondylitis is commonly related to immobility and deformity and severe hip disease. Reiter syndrome can also be associated with disabling peripheral arthritis. Psoriatic arthritis is sometimes disabling Other forms are generally mild and not progressive. Diagnosis: Clinical picture, there are no specific laboratory abnormalities, HLA B27 positivity, sacroiliitis and spine changes by X-ray (bamboo spine). Treatment: NSAIDs, physical therapy. Methotrexate in some cases Steroids are not used 168 81) DISORDERS OF ESOPHAGUS. a) Gastroesophageal reflux (GER) A common disorder in children. Mild GER is common in all babies It may be severe, persisting beyond 18 mo, or is associated with complications (FTT, blood loss, chronic lung disease, or pain). Pathophysiology: there’s low tone of the lower esophageal sphincter [LES]

associated with: 1. Neurological problems and impaired esophageal motility 2. Large hiatal hernia or previous repair of esophageal atresia 3. Delayed gastric emptying (causing gastric distention) 4. Increase in abdominal pressure (crying, valsalva, positional changes) Clinical features: Vomiting occurs after feeding. It is painless and consists of small amounts In older children, heartburn, dysphagia, and loss of dental enamel may occur. The vomiting does not contain bile or blood. Persistent GER can lead to complications Diagnosis: based on clinical picture. If in doubt tests: - Barium swallow is the best for exclusion of other abnormalities. - Esophageal pH monitoring assesses the quantity of GER. - Radionuclide scans with Tc99m added to formula for measure gastric emptying. - Subsequent scanning of the lung demonstrates reflux with aspiration. - Esophageal mannometry measures the LES and esophageal motility. - Endoscopy shows the esophageal mucosa and allows biopsy.

Therapy: 1. Conservative consists of positioning (sitting after meals, no diaper changing at the back, sleeping with elevated head) and dietary changes (smaller, more frequent meals, no food 1h before bedtime; fatty foods, alcohol, chocolate, caffeine, and NSAIDs should be avoided). 2. Medications: reserved for symptomatic pt omeprazole, ranitidine, famotidine 3. Surgery for pt who fail medical therapy, have complications eg strictures or Barrett b) Achalasia It is the most common disorder of esophageal motility. The cause is not known It is characterized by a hypertonic LES that fails to relax adequately with swallowing. Clinical features: Most children are diagnosed when older than 5 yr. - Dysphagia for both solid and liquid foods is the main symptom. - Slow eating, regurgitation of undigested food, weight loss. - Substernal pain and respiratory symptoms such as cough at night due to aspiration. 169 Diagnosis: Barium swallow widened, tortuous esophagus with a narrowed

distal end (birdbeak). Esophageal mannometry documents lack of peristalsis, an abnormally high LES pressure, and incomplete LES relaxation with swallowing. Endoscopy exclude other esophageal disease Therapy: Medical therapy (e.g Ca channel blockers) may help Disruption of the LES by pneumatic dilatation or surgery is usually required. c) Structural abnormalities 1. Esophageal atresia, tracheoesophageal fistula (see topic 7B) 2. Congenital strictures usually occur at the junction of the middle and distal thirds and result in dysphagia. Congenital webs have similar presentations Diagnosis is made by barium swallow and endoscopy. Treatment involves dilatation and, if necessary, surgery d) Esophageal damage by exogenous agents 1. Caustic agents – common by accidentally ingesting materials Strong alkali solutions (eg lye) used in drain cleaners are most dangerous. Less damaging are ammonia cleaners, bleaches, and dishwasher detergents. Acids cause more damage to stomach than to esophagus

Clinical features: - Acute: burns of hands, face, and oral cavity; local pain, drooling, dysphagia, stridor, or dyspnea, abdominal and chest pain, and shock in case of mediastinal perforation. - Chronic: Stricture develops 2 to 4 weeks after ingestion and causes persistent dysphagia. Diagnosis: - X-ray shows perforation and mediastinitis if present. - Endoscopy should be performed under general anesthesia to see esophagitis or gastritis. - A barium swallow can be performed 1 to 2 weeks after ingestion to detect stricture. Therapy: do not induce vomiting or neutralize the caustic agent. IV fluids are necessary (shock may develop). Cardiorespiratory status should be monitored frequently ATB may be given in suspected perforation. Repetitive esophageal dilation or reconstructive surgery may be needed 2. Foreign bodies These should be removed so that respiratory complications and esophageal ulceration and perforation do not occur. Coins are the most common esophageal foreign

bodies in children Children with ingested esophageal body will not always have symptoms of dysphagia, drooling, or chest discomfort! Infants may have stridor due to compression of the trachea. CXR will detect radioopaque foreign body, barium swallow outlines a radiolucent foreign body. 170 82) CARDIAC ARRHYTHMIAS - DD AND TREATMENT. a) Unstable arrhythmias requiring emergent treatment 1. Bradycardias: Most episodes in childhood are causes by hypoxemia, therefore oxygenation and ventilation must be established first. If bradycardia persists, therapy with epinephrine is started If it still does not respond, atropine is administered. If these agents are not helpful, and bradycardia persists, ventricular pacing is considered. 2. Tachycardias: If the rhythm is a narrow complex tachycardia, sinus tachycardia must be distinguished from supraventricular tachycardia (SVT): Sinus tachycardia: caused by fever, anxiety, dehydration, pain. Treat the source of tachycardia SVT: is treated with

IV adenosine in stable pt. Unstable pt with SVT is treated with synchronized cardioversion. Unstable, wide complex tachycardia is usually ventricular. If such a pt has palpable pulses, he is treated with synchronized cardioversion. If becomes pulseless defibrillation 3. Asystole: It is the most common finding in pulseless pediatric pt However, pulses may also be absent in ventricular tachycardia, ventricular fibrillation, or pulseless electrical activity (PEA). Management: Ventilation with 100% oxygen and initiation of basic life support measures (cardiac monitoring, intubation, vascular access). Asystole is treated as cardiopulmonary arrest (CPR, followed by IV epinephrine or endotracheal tube). If ventricular fibrillation or pulseless ventricular tachycardia is present, defibrillation is performed. If three consecutive shocks are ineffective, epinephrine, lidocaine, bretylium, or procainamide may be used. !!!Ingestion of sympathomimetic or anticholinergic drugs is the most common

cause of ventricular tachycardia in childhood!!! Causes for PEA may be: - mechanical tension PTX or cardiac tamponade - metabolic hypoxemia, hypovolemia, hypoglycemia, hyperkalemia, or hypocalcemia. These conditions and presenting symptoms of PEA must be treated. b) Stable arrythmias 1. Premature beats may originate from the atrium or ventricle - Premature atrial beats (abnormally shaped P wave occurring prematurely, a normal QRS complex, and no compensatory pause. They are common and benign - Premature ventricular beats (wide QRS complex, no relationship between the P and QRS waves, an inverted T wave, and a compensatory pause. They are usually benign, unless they are multiform, increase with exercise, or are associated with cardiomyopathy. 171 2. SVT is the most common symptomatic arrhythmia in children It is usually caused by reentry, is often paroxysmal, and may occur in any age. The ECG manifestation of the most common bypass tract is the Wolf-Parkinson-White syndrome,

which consists of a short PR interval and a wide QRS complex with a delta wave. Hemodynamic consequences depend on the HR, age, and presence of underlying heart disease. Symptoms of low CO develop in severe cases. Therapy includes vagal maneuvers, and IV adenosine, digoxin, or beta-blocking agents. In risk of cardiovascular collapse synchronized cardioversion. If the paroxysms are difficult to control, radiofrequency ablation of the bypass eliminates the reentry circuit. 3. Complete heart block - there is loss of conduction from the atria to the ventricles The idioventricular rhythm is slower than normal. Congenital heart block may be caused by maternal antibodies formed in mothers with collagen vascular disease that cross the placenta and produce fibrosis of the conduction system. The lower the level of the block, the slower the HR and the greater the symptoms of inadequate CO. Pacemaker implantation is the therapy 172 83) PROTOZOAN INFECTIONS a) Amebiasis - caused by Entamoeba

histiolytica, one of the intestinal protozoa. Prevalence is 50% in densely crowded areas with poverty. It is most common in warm climates Clinical features: The infection is spread by infective cysts in stools that can contaminate food and drinking water. Cockroaches and house flies can spread the cysts Trophozoites penetrate the mucous membrane of the bowel. A small abscess forms, and may ulcerate Amebae can enter the portal vein and lodge in the liver, or travel to lungs, pleura, and CNS causing abscess formation. Diagnosis: Most individuals are asymptomatic. Symptoms occurs when tissue is invaded and a regional ulcerous colitis results. The onset is insidious and systemic symptoms are unusual Stools have clear mucus or blood. Repeated fresh stool exams are required to identify the protozoa Liver abscess is considered if RUQ tenderness or mass occurs. There’s fever and leukocytosis Treatment: In severe cases, blood, fluid, and electrolyte correction comes first. For those with mild

GI symptoms and extraintestinal manifestation, metronidazole is used. Other alternatives include iodoquinol, diloxanide furoate, or paromomycin. b) Giardiasis - a noniflammatory infection of the small bowel that produces a protracted diarrhea. Widely distributed in drinking water, it is responsible for major outbreaks of GI symptoms, especially diarrhea in children. Transmission can be from infected animal to human and from human to human. Infection is usually asymptomatic in children When present, the symptoms range from mild foul-smelling diarrhea, flatulence, and anorexia, to crampy abdominal pain, epigastric tenderness, and celiac-like syndrome. Diagnosis: Cysts may be found in stools, but more often in duodenal aspirates. Diagnosis is now simplified by detection of Giardia antigen in the feces. Treatment: Metronidazole, quinacrine hydrochloride, furazolidone. c) Cryptosporidiosis - an intestinal parasite that is an important cause of self-limited diarrhea in infants and older

individuals. The life cycle is completed within one host who ingest oocysts The oocysts develop into adults in the small and large bowel and produce oocysts again. The illness is more common & serious in immunocompromised pt, such as AIDS, in whom it can produce voluminous, watery diarrheas. When it occurs in immunocompetent individuals, infection can lead to watery diarrhea for several weeks but is self-limited. Low-grade fever may be present and abdominal pain is prominent, symptoms persists usually for 1-10 days. Diagnosis: Seroconversion, stool smear, biopsy. Treatment: Supportive only. Spiramycin may help The prognosis is excellent, unless the host is immunocompromised. Hand washing is the best prevention 173 d) Toxoplasmosis (see topic 23) Other protozoan infections that are less common in developed countries include malaria, leishmaniosis, trypanosomiasis, microsporidium, and pneumocystis carinii. 174 84) GLOMERULAR DISEASES (see topics 24A, 26A) Hemolytic-uremic

syndrome, HUS The most common glomerular vascular cause of renal failure in childhood. The glomerular damage is exacerbated by severe fluid imbalances during the gastrointestinal prodrome. Epidemiologic studies suggest genetic and infectious or immunologic etiologic components. The primary lesion is in the arteriolar endothelium, especially in the kidney, with formation of platelet thrombi and resulting microangiopathic hemolysis. A disorder of immunoregulation (antiendothelial cell antibodies) may play a role in pathogenesis of the vascular injury. However, the epidemic form of the disease in most cases is precipitated by E. coli O157 HUS is most common in children under age 2 but is more severe in older children. The epidemic form begins with a prodrome of abdominal pain, diarrhea, and vomiting. Oliguria, pallor, and bleeding manifestations, esp gastrointestinal, occur next. HTN and seizures develop in some children - especially those who develop severe renal failure and fluid

overload. The triad of anemia, thrombocytopenia, and renal failure characterize the syndrome. A high reticulocyte count confirms the hemolytic nature of the anemia. Hematuria is often present Treatment: Attention to fluid and electrolyte status is crucial. Early dialysis improves the prognosis Plasma infusion or plasmapheresis may be necessary in severe cases. RBC and PLT transfusions may be necessary. Erythropoetin treatment may reduce red cell transfusion needs Prognosis: Most commonly, children recover from the acute episode within 2-3 weeks completely. However, some pt. have severe and occasionally progressive renal dysfunction or hypertension Mortality is of greatest concern in the early phase, resulting from complications of CNS disease. TTP 175 85) HYPERTENSION - DD AND TREATMENT. BP should be determined at every pediatric visit beginning at age 3 years. BP in children must be obtained when the child is relaxed, and an appropriate size cuff must always be used. According to

a recent study, the values for BP are similar for both sexes and all ethnic groups. If the BP taken in a quiet atmosphere and sitting position exceeds the 95th percentile, it should be repeated several times over a 2 to 4 week interval. If it is elevated persistently, an evaluation for the etiology should be undertaken. Although most hypertension in children is essential, there is a higher incidence of treatable conditions within pediatrics than in adult medicine, such as coarctation of the aorta, renal artery stenosis, chronic renal disease, and pheochromocytoma. Etiology: a) Primary HTN (idiopathic or essential) is the most common form of hypertension in children. As in adults, primary hypertension is a heterogenous group of disorders. The BP elevation is usually mild to moderate and asymptomatic. If lasting for longer period, it becomes an important risk factor in for cardiovascular disease. Often, family history is positive b) Secondary HTN - Renal disease is the most common cause

of secondary HTN. Any renal disease, glomerular or interstitial, may be the cause. The HTN may be transient or sustained Renal HTN is caused by salt and water retention with volume expansion and/or a renin-mediated ↑ in vascular resistance. - Vascular causes (coarctation of the aorta, renal artery stenosis, renal artery occlusion), although uncommon, are important to identify because they may cause severe, symptomatic hypertension and they may be curable. - Endocrine causes are very uncommon and are associated with excess levels of catecholamines or aldosterone. These include pheochromocytoma, primary or secondary aldosteronism, and congenital adrenal hyperplasia with 11- or 17-hydroxylase deficiency. Diagnosis is based on serum and urine concentrations of catecholamines and their metabolites, aldosterone, and 17-hydroxysteroids and 17-ketosteroids. - Neurologic disease as a cause of hypertension is hard to document. Increased intracranial pressure and Guillain-Barre sy are

well recognized causes. Cerebral palsy and seizure disorders are less associated with hypertension. - Miscellaneous causes drugs (ACTH, corticosteroids, amphetamines, OCT, sympathomimetic agents, phencyclidine). Acute significant rises in serum calcium (sudden immobilization) may increase BP. Obesity is a risk factor Renin secreting tumors are rare 176 Diagnosis: Includes BP readings and monitoring (see above), auscultation for murmurs and bruits, and funduscopic examination of retinal vessels. Laboratory studies should include urinalysis, serum levels of electrolytes, BUN, and creatinine. A CXR, ECG, and echo are indicated to exclude ventricular hypertrophy. Total cholesterol and HDL levels are important factors of additional risk for cardiovascular disease. Therapy: 1. Initial therapy should be nonpharmacologic in mild cases (salt restriction, weight reduction, physical activity). 2. Drug therapy follows if the previous is insufficient 3. Therapy for secondary HTN involves

eliminating the cause when possible as well as giving medication to stabilize the BP and the pt. 4. Surgery or angioplasty is indicated for coarctation of aorta, renal artery stenosis and endocrine causes. Hypertensive emergencies: When the BP is severely elevated (>180/110 mm Hg), producing symptoms, or increasing rapidly, treatment must be prompt, and monitoring is needed. 177 86) RUBELLA (German measles) It is a viral disease that is usually not serious when acquired postnatally; however, it can have devastating effects (premature deliveries, congenital defects, fetal death) when a fetus is infected transplacentally during maternal infection. Etiology: Rubella is caused by rubella virus, an RNA virus that is classified as togavirus. Clinical features: a) Postnatal rubella - Clinical manifestations are absent in many cases. An incubation period of 12 to 23 days if followed by a prodrome of malaise, fever, and anorexia in adults. There is no prodrome in children. Several days

after onset of symptoms, posterior auricular, cervical, and suboccipital lymphadenopathy develops, followed by maculopapular rash. The rash begins on the face and then becomes generalized; it rarely lasts more than 5 days. Fever resolves then b) Congenital rubella - mostly causes deafness, cataracts, glaucoma, CHD and mental retardation. Some complications, such as progressive encephalopathy, do not become apparent until the child is older. The risk of congenital defects increases with early uterine infection Disease at 1 to 3 months of gestation is associated with a 30-60% risk of multiple congenital defects and spontaneous abortion. Disease at 4 months of gestation is associated with 10% risk of a single defect. Disease at 5 to 9 month gestation is associated with a single defect Diagnosis: 1. Virus isolation can be performed in specialized laboratories, but it is difficult 2. Serologic confirmation - 4x or greater rise in titre of hemagglutination inhibition or complement-fixing

antibodies. Congenital rubella can be diagnosed in the neonatal period by IgM antibody to rubella virus in the newborn’s serum; the presence of IgM antibody indicates recent infection because maternally derived IgM does not cross the placenta. DD: symptoms are often mild and may be confused with those of enteroviral infections, roseola, toxoplasmosis, infectious mononucleosis, mild cases of measles, and scarlet fever. Therapy and prevention: Postnatal rubella usually requires no treatment. Treatment of congenital rubella is supportive. Prevention: A live attenuated vaccine, which is given at age 12 to 15 months and again at school entry as part of the MMR vaccine. 178 87) DIABETES MELLITUS. A heterogenous group of disorders characterized by hyperglycemia and abnormal energy metabolism, caused by an absent or low insulin secretion or action at the cellular level. Type 1 DM, is the most common endocrine disease in childhood (1:500). Type 2 DM, also occurs in childhood, but is less

frequently diagnosed because of its milder symptoms. DM may be associated with other diseases, such as CF or Prader-Willi sy A transient form of DM may be seen in the newborn. 1. Type 1 DM Etiology: The precise cause is unknown. The process eventually resulting in the loss of insulin secretion by β cells. It has been related to genetic (certain histocompatibility antigens among individuals with DM, increased incidence among 1st-degree relatives), autoimmune (Ab for the islet cells, insulin, GAD or tyrosine phosphatase, present in 90%. Usually there’s an increased incidence of other autoimmune diseases), and environmental factors (probably viruses). Pathophysiology: When 90% of the functioning β cells have been destroyed, loss of insulin becomes clinically significant. With the loss of insulin, the major anabolic hormone, a catabolic state develops, which is characterized by decreased glucose utilization and increased glucose production, which leads to hyperglycemia. Levels of

counter-regulatory hormones (glucagon, epinephrine, GH and cortisol) are elevated. These hormones stimulate lipolysis, fatty acid release, and ketoacid production. When the glucose concentration is above the renal threshold for glucose reabsorption, the resultant glucosuria causes an osmotic diuresis with increased urine output (polyuria) and increased fluid intake (polydipsia). Ketones are produces in big amounts when insulin deficiency is severe. If insulin treatment is not initiated, diabetic ketoacidosis occurs. This metabolic state is characterized by hyperglycemia, metabolic ketoacidosis, dehydration, and lethargy, which may progress to coma and death. Clinical features: The most common time of occurrence is early adolescence. Symptoms of polyuria, polydipsia, and nocturia or enuresis become constant. Significant unexplained weight loss may result, and acetone odor of breath may be noted. N/V and abdominal pain are symptoms of diabetic ketoacidosis that result in severe

dehydration. Diagnosis: A random blood glucose level greater than 7 mmol/L, which is verified on a repeat test, is sufficient for diagnosis. Early in the course of the disease, hyperglycemia may be transient In this situation, a fasting and 2-hour postmeal blood glucose measurement may be helpful. The presence of autoimmune markers in the serum, increased levels of glycosylated hemoglobin, a family history of autoimmune disorder, or all 3 features may increase suspicion. 179 Therapy: The immediate goal is to restore fluid and electrolyte losses and to reverse the catabolic state by replacing insulin. Children and adolescents require 075 to 10 U/kg/day The daily dose is divided between short-acting insulin and intermediate-acting. More intensive and flexible insulin regimens using a portable pump are used lately and achieve a better glycemic control. The insulin is recombinant human as opposed to insulin derived from animal sources (beef, pork). Human insulin is less immunogenic.

Diet: Because food must match insulin absorption, meals must be roughly equivalent in carbohydrate content from day to day and should be eaten on time. During and after periods of exercise, increased food intake may be required to avoid hypoglycemia. Medical follow-up: Regular visit every 4 months is indicated. The glycosylated hemoglobin level should be measured (objective control of blood glucose levels over the past 2 months) as well as lipid levels. Children with type 1 have an increased risk of autoimmune thyroid disease – assessment of thyroid function is necessary. After 5 years with type-1 DM, period examination for retinopathy, neuropathy, and nephropathy are to be carried out. Complications: Can be divided into immediate complications and late complications. a) Immediate complications: 1. Hypoglycemia (insulin reaction) Symptoms: May be related to Sym discharge (sweat, hunger, tremor). More severe symptoms (lethargy, bizarre behavior, slurred speech, unconsciousness,

seizures) are caused by glucose deprivation to the CNS. These may occur with sympathetic symptoms or alone Therapy: The child should consume calorie-containing drink or food if he is able to swallow. When neuroglycopenic symptoms occur, 1mg of glucagon should be injected intramuscularly. 2. Diabetic ketoacidosis - caused by relative or absolute insulin deficiency, which results in hyperglycemia and metabolic acidosis with respiratory compensation. The most common cause is omission of insulin doses. The condition may be triggered by intercurrent illness Symptoms: Polyuria, polydipsia, fatigue, headache, dry mouth, nausea, abdominal pain, and vomiting. Lethargy may progress to coma Physical findings: Tachycardia and hyperpnea (Kussmaul’s respirations), which is the respiratory compensation for metabolic acidosis. Hypotension indicates dehydration and intravascular volume depletion. The child appears acutely ill, anxious, and dehydrated The abdomen is mildly tender. Bowel sounds may be

diminished Lab: CBC, electrolytes, glucose, BUN, and Cr levels should be taken. For more severe illness, arterial blood gas determination is needed. The Hb and HTC are usually increased because of hemoconcentration. There may be leukocytosis Sodium is low (dilution from the osmotic effect of hyperglycemia). Potassium level is usually normal A low level indicates depletion and is 180 potentially life-threatening. The bicarbonate level is low The BUN level may be elevated (dehydration). Arterial blood gases indicate metabolic acidosis with respiratory compensation Therapy: For moderate and severe acidotic pt. (pH <72) should be carried out in an intensive care unit, with monitoring of the vital signs, neurologic status, and fluid input + output. A nasogastric tube should be placed to empty stomach and prevent aspiration. The first aim of therapy is replacement of fluids and electrolytes and to expand the vascular volume. Insulin replacement (continuous infusion of IV insulin)

comes next. 5% or 10% dextrose should be added to maintain the blood glucose level. Bicarbonate (correction of acidosis) is controversial Complications: Hypoglycemia, hypokalemia, and cerebral edema, which is frequently fatal. A worsening mental status, confusion, unequal pupils, or seizures indicate cerebral edema. b) Late complications: Long-term complications are related to chronic hyperglycemia; good glycemic control results in less complications. Complications associated with a long duration (> 10 yrs) of type-1 DM include microvascular disease of the eye (retinopathy), the kidney (nephropathy), and the nerves (neuropathy). The other major category is large vessel atherosclerotic complications, which lead to premature myocardial infarction and stroke. 2. Type-2 DM The most common type in older individuals but it can occur in childhood already. This trend may be related to diet and lifestyle changes leading to obesity in genetically susceptible children. Etiology: It is caused

by decreased insulin action due to impaired synthesis or resistance to insulin at the cellular level. In some families, it appears to be inherited as an AD trait Therapy: Consists of nutrition counseling to promote weight loss in the obese pt. Safety and efficacy of oral medications in childhood has not been fully proven. 181 88) HEADACHES It is a common symptom of many pathologies both within and outside the CNS. Most headaches are caused by benign diseases, but some may suggests serious conditions. There are only some structures on the head that are sensitive to pain. They include the 5th, 9th, and 10th cranial nerves and the roots of the 1st 3 cervical spine nerves. The basal dura and pia maters, and the blood vessels within them; the skin, scalp, and muscles surrounding the skull, and the periosteum of the skull. And finally the eye, ear, and nasal passages Stimulation of these structures provides pain. The brain parenchyma and the choroid plexus have no nociception Diagnosis:

detailed history and physical exam is crucial. Ask both the parents and child separately Quality of pain is important (e.g pulsatile pain suggests vascular etiology) The location is also important. If pain is in the face, sinus, ocular, and neck pathology should be considered as well as affections of the 5th cranial nerve (trigeminal neuralgia, zoster, or tumor), and cluster headache. Acute ocular pathology may present as periorbital and occipital pain Diseases of the cervical spine may present with neck pain + occipital region pain. More important is the course of headache. The acute onset of a severe headache suggests SAH Meningeal infection usually has more graduate onset, over hours, or days. Aggravating factors should be discussed. For instance, a headache that accompanies a 3rd ventricle cyst is precipitated by lying down or standing up. The headache that follows lumbar puncture is much worse on sitting or standing. Migraine headaches may be triggered by certain foods such as

cheese, red wine, or chocolate. - Migraine – either with or without aura. It’s a pulsatile throbbing headache, usually unilateral that is accompanied by photophobia, phonophobia. Common in late teens, esp females Treatment include prophylaxis with propranolol, verapamil. For acute termination of attacks use sumatryptan, aletriptan or ergotamine. - Tension headache It is characterized by a diffuse, dull, nonpulsatile ache that is bilateral and is associated by a sense of fullness or stiffness in the head or neck. It is common in adolescents The headache has a gradual onset and may persist for many hours, days, or weeks. Treatment: acetaminophen, aspirin - Headache as a manifestation of neuropathology Subarachnoid hemorrhage presents with hyperacute onset of severe headache with neck stiffness and pain. The pt appears ill and may become confused and develop other neurologic features In contrast to migraine, the neurologic features develop during or after the headache. CT scan

confirms diagnosis. If it is negative, LP should be done to exclude meningitis Bacterial or viral meningitis also presents with headache and neck stiffness. However, the onset is more gradual, photophobia, and features of infection (fever, elevated WBC & metabolic acidosis). 182 Intracranial neoplasms may present as headaches accompanied by focal neurological signs, increased ICP (papilledema), or personality changes. Headache associated with increased intracranial pressure is frequently accompanied by recurrent morning vomiting. Head injuries and cerebral hypoxia also present with headaches. - Other causes of headache include drugs, dehydration, lack of sleep, stress, emotions, acute systemic infection, or severe HTN. Treatment of headaches depends on the underlying cause. 183 89) SCARLET FEVER Scarlatina It is an acute illness characterized by fever, pharyngitis, and an erythematous rash. Scarlet fever is rare in infancy. It can occur more than once in a single pt

Etiology: Scarlet fever results from infection with group A streptococci that produce erythrogenic toxin. The disease is usually associated with pharyngeal infection Clinical features: - Characteristic rash is erythematous, finely punctate, and blanches with pressure. It appears initially on the trunk and becomes generalized within a few hours to several days. The face is flushed with circumoral pallor, and there is increased erythema in the skin folds. The skin may feel rough, similar to sandpaper. The skin rash fades over 1 week and is followed by desquamation, which may last for several weeks. - Strawberry tongue (rough, erythematous, swollen tongue) and pharyngeal erythema with exudate may be present. Diagnosis is made from clinical signs and isolation of group A streptococci on throat culture. Therapy: 10 days of oral penicillin. Complications: Both suppurative (e.g cellulitis) and nonsuppurative (eg GN, rheumatic fever) complications can occur, just as with streptococcal

pharyngitis. 184 90) GASTROINTESTINAL BLEEDING. CROHNS DISEASE UC GI bleeding – a common, and potentially life-threatening condition. Usually, a careful history and physical exam, as well as consideration of the pt. age, will suggest the diagnosis A. Acute bleeding of upper GI • Hematemesis – vomiting of fresh blood or coffee ground. • Melena – black stool. • Hematochezia – fresh bloody stool. Only in massive upper GI bleeding Etiology: 1. Peptic ulcer disease 4. Portal HT esophageal varices 2. Gastric neoplasms 5. Erosive gastritis or esophagitis 3. Mallory-Weiss lacerations 6 Vascular anomalies (AVM, ectasias) Evaluation: - Treatment: Stabilization assessment of BP & HR. Check PT, CBC & HTC - Assess volume status & give volume replacement (most important in acute cases). - Take history & physical examination liver disease, NSAID use, alcohol, etc. - Blood replacement. - NG tube or endoscopy to see the bleeding. 1.

Discontinue aspirin or other NSAIDs 2. Endoscopy to see the source of bleeding & determine risk of re-bleeding, or to perform therapy by cauterization, injection of sclerotant or placing a rubber band. 3. H2 blockers (famotidine, ranitidine) ↓symp, accelerate healing, preventative 4. IV octreotide to reduce splanchnic blood flow & portal pressure 5. Intraarterial vasopressin 6. Intraarterial embolization 7. TIPS: transvenous intrahepatic portosystemic shunts placing a stent from the hepatic vein through the liver to the portal vein provides effective decompression for control of acute variceal bleeding. 8. CV resuscitation – for severe cases, when orthostatic hypotension is present In massive bleeding, whole blood should be given. Once bleeding has stopped, packed cells alone are given. Vit K, PLT, and plasma should be given as needed to correct coagulopathy. 185 B. Acute lower GI bleeding When the bleeding arises from below the ligament of Treitz, resulting in

hematochezia. Melena will be from the upper GI. Most cases are from the colon & anorectal region Etiology: 1. Neoplasms 5. Diverticulosis. 2. Ischemic colitis 6. Portal HT hemorrhoids. 3. Radiation enterocolitis 7. IBD, esp. UC 4. Bacterial induced colitis 8. Vascular anomalies (AVM, ectasias). Evaluation: - Treatment: Assess volume status & give volume replacement. Blood replacement - Take history & physical examination, exclude upper GI bleeding. - Anoscopy, sigmoidoscopy or colonoscopy. - Angiography or nuclear bleeding scan, using Tc99m labeled RBC. 1. Therapeutic colonoscopy to see the source of bleeding and perform cauterization, injection of sclerotant or placing a rubber band. 2. Intraarterial vasopressin or embolization 3. Surgery hemicolectomy or total colectomy if the source cannot be identified C. Occult GI bleeding There’s no overt GI bleeding (no hematochezia or melena), but constant loss of small amounts of blood into the lumen.

Detected by fecal occult blood testing or by iron deficiency anemia Etiology: 1. Neoplasms colorectal cc, polyps, gastric cc, lymphomas 2. Infections amebiasis, TB, helminthes 3. Peptic ulcers, erosions & reflux esophagitis 4. Radiation enterocolitis 5. Drug induced NSAIDs & anticoagulants 6. IBD 7. Vascular anomalies (AV malformations, ectasias) Evaluation: a. Upper endoscopy, colonoscopy b. Barium enema or small bowel evaluation c. History dyspepsia, weight loss, nausea, vomiting, heartburn & bowel habits 186 Other tests that can assist: - Intraoperative endoscopy (laparotomy is performed and the bowel is telescoped over the endoscope) is reserved for pt. with bleeding and negative endoscopy and X-ray - Bleeding scans, involving injection of labelled red blood cells, detect very slow bleeding. Tc 99m pertechnetate IV may detect ectopic gastric mucosa in case of Meckel’s diverticulum. - Angiography - is used in more difficult cases for fast bleeding

areas. Inflammatory bowel disease, IBD Referring to 2 chronic inflammatory disorders - ulcerative colitic (UC) and Crohn’s disease (CD). Sometimes, they cannot be differentiated. Epidemiology: 30% of newly diagnosed cases occur in individuals younger than 20 yr. Pathogenesis: - Immunologic - The antigens to which the intestinal mucosa is chronically exposed stimulate the lymphoid tissue resulting in a state of “physiologic” inflammation. Down-regulation of this mechanism is compromised in these pt. which leads to pathologic inflammation - Infectious and psychological: It is thought it may play a role. But it is likely that the cause is multifactorial. Genetic predisposition may be a crucial factor Pathologic and clinical features, diagnosis: see picture Differential diagnosis: Appendicitis, enteric infection, pseudomembranous colitis, HUS, HenochSchönlein purpura, radiation enterocolitis, eosinophilic gastroenteritis. Therapy: 1. Medication: 5-aminosalicylate agents

(olsalazine, sulfasalazine) are used for mild-to-moderate cases, corticosteroids are most effective, mainly in severe cases, however, daily therapy inhibits growth, alternate day therapy is preferred, metronidazole treats perirectal fistulae in Crohn’s disease, 6-mercaptopurine is for severe cases, cyclosporine - good as short-term treatment, infusions of an antibody to tumor necrosis factor alpha (infliximab) are used for severe cases of Crohn, diphenoxylate and loperamide relieve mild symptoms only. 2. Nutrition - anorexia and increased nutrient losses through stool are common, so adequate calories and protein are essential (oral liquid supplements, nasogastric tube feedings, central venous hyperalimentation). Vitamin and mineral (esp iron) supplementation may be needed 187 3. Surgery - Ulcerative colitis: Indications: Fulminant colitis with severe blood loss, perforation, or toxic megacolon; high dose steroid requirement, growth failure, or invalidity. Procedure: colectomy

followed anastomosis. - Crohn’s disease: Indications: Hemorrhage, obstruction, perforation, severe fistula, ureteral obstruction, growth retardation. Procedure: After segmental resection, recurrence rates are 50%!!!. In general, conservative approach is preferred. 188 91) VOMITING - DD AND TREATMENT. Assessment of a child with recurrent episodes of vomiting should start with a complete history, physical examination, and description of vomitus. Emesis of gastric contents is characteristic of gastric outlet obstruction, CNS masses or infection, peptic disease, urinary tract infection, otitis or sinusitis, metabolic diseases (esp those causing acidosis), and psychogenic vomiting. GERD should be suspected in a healthy child with effortless postprandial spitting. An upper GI series is essential to rule out anatomic causes in young infants. Further evaluation may include serum electrolytes, calcium, magnesium, urea nitrogen, urinalysis, and urine culture. The child who vomits

bile-stained material may have small intestinal obstruction and should be examined immediately. Bile staining may be gold or green in color The history should include duration of vomiting, the presence of blood in the vomitus, the presence of abdominal pain or distention, the character of stools, and the presence of fever. Pain localized to the RLQ suggests appendicitis. Midline or diffuse abdominal pain suggests pancreatitis or generalized peritonitis. Abdominal distention suggests intestinal obstruction. Viral and bacterial gastroenteritis are associated with diarrhea and may produce generalized ileus with bilious vomiting. Gallbladder disease is uncommon in childhood but should be suspected in children with a positive family history. The presence of mucus or blood in the stool should arouse suspicion of intestinal intussusception or bacterial or toxic colitis. The evaluation of bloody vomitus should start with confirmation that the material vomited is indeed blood. Numerous causes

must be considered, including oropharyngeal lesions, nosebleed, peptic disease, bleeding disorders, foreign bodies, and esophageal varices. Mallory-Weiss tear of the gastroesophageal junction is common after prolonged vomiting. Passage of a nasogastric tube will help determine whether bleeding is ongoing. Hematocrit should be measured Further tests include upper intestinal endoscopy. 189 Common causes of vomiting and regurgitation 1. GI tract disorders • Esophagus: achalasia, GERD, hiatal hernia, esophagitis, atresia, stenosis, diverticulum, foreign body • Stomach: hypertrophic pyloric stenosis, pylorospasm, diaphragmatic hernia, peptic disease, gastritis • Duodenum: annular pancreas, duodenitis and ulcer, malrotation, mesenteric bands • Intestine and colon: atresia, stenosis, meconium ileus, malrotation, volvulus, duplication, intussusception, foreign body, polyposis, soy or cow’s milk intolerance, Celiac, food allergy, Hirschsprungs disease, appendicitis, IBD,

gastroenteritis, infection • Other abdominal organs: hepatitis, gallstones, pancreatitis, peritonitis 2. Extra-gastrointestinal disorders - Sepsis, pneumonia, otitis media, UTI, meningitis, - Hydrocephalus, brain tumors, Reye syndrome, - Intoxications, adrenal insufficiency, - RTA, inborn errors (PKU, maple syrup disease, galactosemia, fructose intolerance) Treatment: The underlying cause must be treated if possible. Supportive treatment includes bed rest, fluid and electrolyte replacement (oral glucose electrolyte solutions, or parenteral in severe cases or if indicated). Children become dehydrated more quickly and should be given an appropriate rehydration solution. Antiemetics include dimenhydrinate, chlorpromazine, or meperidine. 190 92) VARICELLA. HERPES ZOSTER a) Varicella (chickenpox) – highly contagious disease, occurring in children <10 yr. Usually mild, self-limited in otherwise healthy children, but may be severe in immunocompromised. A varicella

vaccine decreases the risk of disease b) Zoster (shingles) - a reactivation of varicella infection, occurring predominantly in adults who previously had varicella and who have circulating antibodies. Although zoster occurs in children, it is uncommon in children <10 yr. It’s an acute infection characterized by crops of vesicles confined to a dermatome and often accompanied by pain in this dermatome. Clinical features: Varicella - Incubation period (10-21 d) prodrome of mild fever, malaise, anorexia, and occasionally scarlatiniform or morbilliform rash. - Characteristic pruritic rash begins later, appearing first on the trunk & spreading peripherally. The rash begins as red papules develops rapidly into clear “teardrop” vesicles (1-2 mm) they become cloudy, breaking down into thin ulcerative lesions that crust before healing. Vesicles may occur on mucous membranes. - The severity of the illness ranges from a few lesions associated with a low-grade fever, to hundreds

of lesions associated with temperatures up to 40.6°C, to fatal disseminated disease in immunocompromised. In most children, it manifests as a generalized rash with mild fever and mild systemic symptoms. - Infectious period. Pt are infectious 24 h before the rash appears until all lesions are crusted (5-7 days after onset of rash). a) Zoster - Attacks may begin with pain along the affected sensory nerve that is accompanied by fever and malaise. These symptoms are more common in adults than in children - A vesicular eruption similar to the vesicular form of varicella then appears in a dermatome area and this eruption clears in 7-14 days. However, the rash may last as long as 4 weeks - The lesions are infectious (direct contact or inhalation of aerosolized infected epithelial cells). - Postzoster neuralgia is common in the elderly but rarely occurs in children. Diagnosis: usually obvious from clinical presentation. It unclear, VZV fluorescent Ab test should be performed

(scraping the base of vesicle). VZV, but also other herpes viruses may be demonstrated by the multinucleated giant cells with intranuclear inclusions. The definitive diagnosis is made by positive culture results from a vesicular scraping or 4x rise in Ab titre. 191 Therapy and prevention: Uncomplicated cases are treated with an antipruritic agent and daily bathing to reduce secondary infection. Immunocompromised children after exposure to infected persons should receive prophylaxis with VZIG and should receive IV acyclovir if they develop varicella or disseminated zoster. For varicella vaccine see topic 16B Complications: The most common include: - Encephalopathy, cerebellitis, Guillain-Barre sy, aseptic meningitis. - Pneumonia, hepatitis and arthritis. - Thrombocytopenic purpura, cellulitis, abscess. - Varicella is a risk factor for severe, invasive group A beta-hemolytic streptococcal disease. - Severe varicella occurs in immunocompromised children (20% mortality rate).

- Maternal varicella during the 1st trimester may be associated with congenital malformations. 192 93) BRONCHIAL ASTHMA. The most common chronic disease of childhood. Up to 80% of children with asthma develop symptoms before their fifth birthday. Atopy, the genetic predisposition for the development of an IgE mediated response to common allergens, is the strongest predisposing factor. Exposure to tobacco smoke is also a risk factor. Sensitization to inhalant allergens increases over time The principal antigens associated with asthma are aeroallergens such as dust, mites, animal dander, cockroaches, and moulds. Other triggers include exercise, cold air, cigarette smoke, pollutants, strong chemical odours, and rapid changes in barometric pressure. Psychologic factors may precipitate exacerbations. Pathologic features of asthma include shedding of airway epithelium, edema, mucus plug formation, mast cell activation, and collagen deposition beneath the basement membrane. The

inflammatory cell infiltrates include eosinophils, lymphocytes, and neutrophils. Airway inflammation contributes to airway hyperresponsiveness, airflow limitation, and chronicity. Persistent airway inflammation can lead to airway remodelling and irreversible changes. Clinical findings - Wheezing is the most characteristic sign of asthma. - Cough and shortness of breath. - Complaints “chest congestion”, exercise intolerance, dyspnea, and recurrent infections. - Chest auscultation during forced expiration reveals prolonged expiratory phase and wheezing. - Nostril flaring, intercostal and suprasternal retractions, and use of accessory mm of respiration. - Cyanosis of lips and nail beds may be seen with underlying hypoxia. - Tachycardia and pulsus paradoxus also occur. Airway hyperresponsiveness (decrease in FEV1) to nonspecific stimuli (histamine, metacholine, or exercise and cold air) is a hallmark of asthma. During acute asthma exacerbations, FEV1 is diminished and

the flow-volume curve shows typical characteristic. Hypoxemia is present early with a normal PCO2 level and respiratory alkalosis. Respiratory acidosis and increasing PCO2 suggest further airway obstruction and signal respiratory failure. A PaO2 less than 60 mmHg despite oxygen therapy and a PaCO2 over 60 mmHg are indications for mechanical ventilation in a child with status asthmaticus. Clumps of eosinophils on sputum smear and blood eosinophilia are frequent findings. Prognosis - Mortality and morbidity rates for asthma have increased. Children with mild symptoms generally outgrow the asthma, while pt. with more severe symptoms tend to have persistent disease 193 Treatment 1. Chronic asthma a) General measures: avoid exposure to smoke and allergens to which they are sensitized (identification is made by skin tests or in vitro testing), air pollution, and beta-blockers. b) Pharmacologic therapy: a stepwise approach is recommended. The preferred strategy is to initiate therapy at

a higher level at the onset to gain control and then step down. A rescue course of systemic steroids may be necessary at any step. Medications are now classified as long-term control medications and quick-relief medications. The former include anti-inflammatory agents (steroids or cromolyn and nedocromil). Long-acting bronchodilators (e.g salmeterol), and leukotriene modifiers (eg montelukast) Inhaled corticosteroids (e.g budesonid) are the most potent inhaled agents available Quick-relief medications include short-acting inhaled beta2-agonists such as albuterol, pirbuterol, or terbutaline by inhaler. It is better to use beta2-agonists as needed, not regularly Anticholinergic agents such as ipratropium by nebulizer may provide additive benefit. Systemic corticosteroids such as prednisone can be given for 3-10 days. c) Exercise induced bronchospasm - occurs during vigorous activity, reaches its peak 5-10 min after stopping, resolving in next 20 min. However, participation in normal

physical activity should be encouraged. Treatment before exercise is effective 2. Acute asthma a) General measures - involves early recognition of warning signs and early treatment. The child and parents must be able to assess asthma severity accurately. Prompt communication with the clinician is indicated with severe symptoms or a drop in peak flow or with decreased response to inhaled beta2-agonists. A short course of oral corticosteroids may be needed a) Hospital management - measurements of airflow limitation with PEFR or FEV1 and monitoring of the pt. response to treatment is necessary Other test include oxygen saturation and blood gases measurements. If the initial FEV1 or PEFR is over 50%, initial treatment can be with an inhaled beta2-agonist by inhaler. Oxygen should be given to maintain oxygen SAT at > 90% Oral corticosteroids should be given if the pt. responds poorly to therapy If the initial FEV1 or PEFR is under 50%, initial treatment should be with a high dose

inhaled beta2 agonist by nebulizer. Ipratropium may be added Systemic steroids should be instituted In pt. unresponsive to initial therapy, epinephrine may be administered subcutaneously For ongoing respiratory arrest, pt. should be intubated and ventilated with 100% oxygen and admitted to ICU. 194 94) COUGH – DD AND TREATMENT Cough is one of the most common respiratory symptoms of all ages. It results from stimulation of irritant receptors in the airway mucosa. It may have many different characteristics, depending on anatomic factors and the cause of irritation. a) Acute cough - Most cases are associated with respiratory infections (pneumonia, tracheobronchitis, bronchitis, sinusitis, pertussis), and the cough subsides with healing of infection. A history of sudden onset of choking and coughing is common after aspiration of a foreign body. Other causes include pulmonary edema, thermal or chemical irritation injury (smoke), and pulmonary embolism or hemorrhage. b) Chronic cough

- defined as daily cough for > 6 wks. It has a more diverse etiology, including: - allergy (asthma, hypersensitivity pneumonitis, rhinitis). - anatomic abnormalities (tracheoesophageal fistula, cysts, gastroesophageal reflux). - chronic infection (CF, sinusitis, aspiration pneumonia, abscess, TB, fungal pneumonia, histoplasmosis, AIDS-related infection). - environmental exposure to irritants (smoking, drug abuse). - foreign body aspiration, psychogenic (habit), and neurologic dysfunction. Children in the first several years of life often have frequent respiratory infections, esp if they are exposed to other children, as in day care centers. Cough that resolves promptly and is associated with a viral infection does not require further examination. However, cough that persists beyond 4-6 week and is not associated with classic viral URT infection requires investigation. The circumstances under which the cough occurs are important. Nocturnal cough suggests allergy (esp to

antigens in the child’s bedroom) or a drainage from sinuses. Cough associated with exercise suggests reactive airway disease or bronchitis/bronchiectasis. Cough arising in the morning is associated with excessive production of tracheobronchial secretions, such as occurs in asthma, bronchitis, bronchiectasis, or CF. Paroxysmal cough should suggest pertussis syndrome, foreign body aspiration, or CF. Chlamydial infections characteristically produce a repetitive, staccato cough. Children with a harsh cough often have an anatomic problem, such as subglottic edema (croup), tracheomalacia, or tracheal compression, but also may have habit cough. Habit cough disappears with sleep Cough waking up a child from sleep is usually the result of a pathological proce3ss. Tumors, lymphadenopathy, and pulmonary vasculitis are uncommon causes of chronic cough. 195 Complications: Severe coughing may result in: - rib fractures, chest wall pain, PTX. - rupture of conjunctival, nasal or anal veins,

syncope. - hernias, emesis, urinary incontinence, or rectal prolapse. - Severe repeated episodes of cough, as seen in pertussis, may interfere with oral intake and produce malnutrition, or may result in cyanosis. Diagnosis: X-ray, sputum examination. Children with allergies produce clear mucoid sputum, eosinophils are present. Purulent sputum suggests infection, and a specimen should be obtained for culture. Immunofluorescence, ELISA using serum are available for dg. of some infections Treatment: Cough suppressants, decongestants (sympathomimetic). Expectorants, antihistamines, antipyretics. ATBs – depending on the underlying disease. 196 95) OSTEOMYELITIS. In children occurs mostly in long bones of the lower extremities and less of the upper extremities. Etiology: S. aureus and S pyogenes account for more than 90% childhood osteomyelitis Salmonella sp. are common in pt with sickle cell anemia P aeruginosa is common in pt with puncture wounds of the foot. Pathogenesis: The

tortuous course of the nutrient vessel in the metaphyseal region of bone cause bacteria to be trapped there. If pus develops, it may elevate the periosteum Clinical features: - In young infants, fever may be the only manifestation. - Fever and localized bone tenderness are the most common symptoms in older children. - Local swelling, redness, warmth, and suppuration may occur later. - ½ of the pt. have a history of minor trauma Diagnosis: WBC & ESR are elevated. Blood cultures are positive in 50% Bone scan using Tc99m is positive 24 hours after symptoms begin. X-ray results become positive until 10 to 12 days after symptoms onset. MRI or CT scans are more specific, but used only in questionable cases Aspiration of the affected site shows the causative organism and determines whether an abscess is present ( surgical drainage). Therapy: ATB therapy should initially consist of parenteral oxacillin, unless a gram-negative organism or resistant staph is suspected. Appropriate

ATB is continued for 4 weeks Surgical drainage is necessary to treat an abscess. Complications: Chronic osteomyelitis may occur if treatment is inadequate. Joint infection may occur (esp in neonates and when the disease affects the shoulder or hip). 197 96) RENAL FAILURE. a) Acute renal failure A sudden inability to excrete urine of sufficient quantity or adequate composition to maintain body fluid homeostasis. Causes include acute renal disease, renal ischemia, renal vascular compromise, or obstructive uropathy. The hallmark of early renal failure is oliguria. The causes can be prerenal, renal, and postrenal Prerenal: most commonly due to compromised renal perfusion. Usually secondary to dehydration, although abnormalities of renal vasculature (renal vessel injury, renal arterial thrombosis) and poor cardiac performance (heart failure, cardiac surgery) may also be considered. Other causes include hemorrhage, trauma, nephrotic syndrome, or shock. Renal include acute ATN, GN, HUS,

acute interstitial nephritis, nephrotoxic injury, vascular nephropathy, intravascular coagulation (septic shock, hemorrhage), or severe infections. Postrenal: found in newborns with anatomic abnormalities. Obstruction of the bladder outlet should be relieved by insertion of a uretheral catheter followed by surgical correction. Early intervention may prevent irreversible renal injury and CRF. Delayed voiding, anuria, or poor urinary stream in the newborn period suggest obstruction. Ureteropelvic junctional obstruction usually presents as an abdominal mass. Complications include: - Fluid overload (HTN, CHF, pulmonary edema). - Electrolyte disturbances (hyperkalemia), metabolic acidosis, hyperphosphatemia, and uremia. Treatment Prerenal or postrenal factors should be treated first and the circulating volume maintained. The pt’s response is assessed by measurement of urinary output. If diuresis does not occur after these measures, furosemide may be used. If it does not help, dialysis

is indicated Immediate indications for dialysis are severe hyperkalemia, severe metabolic acidosis, fluid overload with hypertension or CHF, and symptoms of uremia, usually manifested in children by CNS depression. Peritoneal dialysis is generally preferred in children because it is easier, but it is also less effective than hemodialysis. Course and prognosis - oliguria usually lasts around 10 days. The diuretic phase begins with an increase in urinary output. During the recovery phase, signs and symptoms subside rapidly, although polyuria may persist for some time. If renal recovery does not occur, arrangements are made for chronic dialysis and eventual renal transplantation. 198 b) Chronic renal failure, CRF In children, it most commonly results from developmental abnormalities of the kidney (dysgenesis, agenesis, cystic changes) or urinary tract. Abnormal development of the urinary tract may not permit normal renal development. Obstructive uropathy or severe VUR nephropathy

continue to cause progressive renal insufficiency in children. In older children, the chronic GN and nephropathies, irreversible nephrotoxic injury, or HUS may also result in CRF. When CRF is the result of a low amount of normally functioning renal tissue, the inability to concentrate urine results in polyuria, polydipsia, and enuresis. Affected pt often fail to thrive An abnormally collecting system can lead to renal failure and infections. Children with long-standing CRF may present with complications such as: - Hyperphosphatemia, secondary parathyroidism, and rickets. - Anemia (from decreased erythropoetin synthesis). - In contrast, children who develop CRF due to chronic glomerulonephritides, will usually retain sodium and water so that presentations range from HTN to uremic symptoms. - Uremic symptoms occur late and include anorexia, nausea, malaise. - CNS features range from confusion, apathy, lethargy, and stupor to coma. - Uremic pericarditis and CHF may also occur.

- Growth failure depends on the age at presentation and the rapidity of functional loss. - Some of the chronic glomerulonephritides may progress unnoticed into renal failure. Treatment Before dialysis, preservation of remaining renal function and avoidance of complications is done. Controlling HTN and hyperphosphatemia and preventing UTI are important. Acidosis is controlled by Na-citrate solutions, as long as the added sodium won’t aggravate HTN. Hyperphosphatemia is controlled by dietary restrictions and dietary phosphate binders. Vitamin D should be given. When the blood urea level is high, dietary protein should be restricted As the GFR falls, potassium restriction will be necessary. Anemia may be treated with erythropoetin. The best tolerated treatment of ESRD is renal transplant. The prognosis is good Hemodialysis is a good method of treating older children with end-stage renal disease. Peritoneal dialysis is mainly for small children because it is easier and can be

performed at home. 199 97) DYSPNEA – DD AND TREATMENT It is an unpleasant sensation of difficulty in breathing. Dyspnea is a symptom, not a sign It is often described as breathlessness or shortness of breath on exertion. Other sensations include increased muscular effort required to expand the chest during inspiration or to expel air from the lungs, sensation of fatigue in the respiratory muscles. Only older children are capable of describing these feelings, infants aren’t. We think of dyspnea in an infant according to clinical symptomatology: signs of respiratory distress, tachypnea, use of accessory respiratory muscles, loud breathing (wheezing, stridor). Dyspnea may be acute or chronic Causes of acute dyspnea: 1. Bronchial asthma, foreign body aspiration, epiglottitis, laryngitis, acute bronchiolitis 2. Cardiac failure, pneumonia, PTX, chest trauma, 3. Neuromuscular disorders, CNS disorders (trauma, hemorrhage), hysteria Causes of chronic dyspnea: Dyspnea usually occurs

during physical activity in these cases. 1. Inadequately treated bronchial asthma, repeated aspiration, CF, bronchopulmonary dysplasia 2. Congenital heart diseases, extreme obesity, interstitial pulmonary diseases 3. Neuromuscular disorders, anemias, chronic upper airway obstruction, etc Treatment: directed against the underlying condition. It is important to know that a severe pulmonary dysfunction is not necessarily accompanied by dyspnea, and on the contrary, pt. with acute dyspnea may have only minimal changes in pulmonary function. 200 98) INFECTIONS OF THE SKIN a) Viral infections - Warts (verrucae) - benign epidermal proliferations induced by human papilloma viruses (HPV). It is very common in children and tends to recur. Commonly, several household pt are affected Clinical features: The incubation period is variable, but the average is 3 months. The usual sites are hands and feet. Certain types of HPV may induce warts in the perianal and genital regions (condyloma

accuminata) and in the larynx (laryngeal papillomatosis), esp in infants who have passed through a birth canal infected with such viruses. Some warts regress naturally Common warts usually appear in groups and can continue to arise at different sites. No prophylaxis can limit further infection. It is common for warts to recur at previously infected sites, even after successful eradication. Diagnosis: Lesions consists of flesh-colored papules that have a rough, papillated surface and may have capillaries on the surface, many of which are thrombosed and appear as black dots. Treatment: Home therapy consists of a keratolytic preparation containing salicylic acid. These agents cause maceration of the infected skin that can be then removed easily. Cantharidin, a blistering agent is a very effective and painless method. Liquid nitrogen is also effective but produces a burning sensation. Prevention: Warts are contagious and can spread by direct contact. Genital and perianal lesions spread by

sexual contact in adults, in children, sexual abuse should be suspected. - Molluscum contagiosum - a viral skin infection caused by the pox virus and characterized by discrete pearly 1-5 mm papules that often have a central depression. Although people of all ages can develop it, it is most common in childhood. The infection is contagious and is highly autoinocuable Mollusca are often seen in pt. with atopic dermatitis and HIV infection Clinical features: The pt. often has one lesion that is touched and spread to the adjacent skin All body areas can be involved, but most common are the face, flexures, chest wall, and buttocks. In adults, it is often transmitted sexually and involves the genital area. Without treatment, mollusca usually disappear spontaneously, but it may takes years. Diagnosis: clinical appearance, scrapings, or biopsy. Treatment: Cantharidin, a blistering agent, liquid nitrogen, extraction, or curettage. 201 - Herpes simplex - HSV 1 (orofacial) and HSV 2

(genital). Both viruses choose sensory ganglia as the site of residence after primary infection. The factors that awake them from this state are include alterations in immunity, physical injury, and various types of stress. Primary HSV 1 orofacial infections are common during childhood. Frequently asymptomatic but can cause painful stomatitis, pharyngitis, and fever with local adenopathy. It lasts for 1-2 weeks Primary HSV 2 infections are characterized by painful mucosa ulcerations and skin vesicles in the genital area, inguinal adenopathy, and fever. It lasts for 2-3 weeks Recurrent HSV 1 is characterized by painful labial vesicles and ulcerations, and recurrent HSV 2 by painful vulval, vaginal ulcerations in women and penile ulcers in the male. There is no fever or adenopathy. Treatment: Acyclovir (available in topical, oral, and IV form). b) Bacterial infections - Impetigo - a superficial pyoderma caused by group A beta-hemolytic strep (staph may be present but are thought to

be secondary invaders and are responsible for bullous impetigo). The organisms may invade a superficial abrasion, mosquito bite, poison ivy, or any wound. The lesions are characterized by clusters of erythema, shallow ulceration and a thin, purulent exsudate that forms a yellow crust. Regional lymphadenitis may occur, bacteremia is rare Deep skin infections with group A streptococci include cellulitis and erysipelas. In the latter, the skin is swollen and indurated, with a raised margin. Fever and vomiting are common Treatment: Mupirocin ointment. If it does not help, ATB (cloxacillin, penicillin) systemically c) Fungal infections - Dermatophytoses (tinea) - a dermatophyte is a fungus that infects skin, hair, and/or nails. Clinical diseases are called tineas and are caused by species of Trichophyton, Microsporum, and Epidermophyton. The site of infection determines the name: tinea capitis (scalp), tinea corporis (body), tinea faciei (face), tinea cruris (inguinal areas), tinea pedis

(athlete’s foot), tinea manum (hands), and tinea unguium (nails). Dermatophyte infections do not invade below epidermis Tinea capitis is more common in pediatric pt. Tinea corporis is common in all age groups Tinea pedis is the most common form, which is rare before puberty but very common later. Clinical features: Tinea capitis starts as a small group of scaly papules or pustules and spreads peripherally. Hair loss may occur Complications include secondary bacterial infection and scarring Tinea corporis occurs anywhere on nonhairy skin. The lesions are scaly plaques with erythematous papular borders, often with central clearing. Maceration and scaling of the interdigital spaces is the presentation of tinea pedis. Itching is common Moisture and warmth encourage growth of the 202 organisms, therefore, summertime and sweaty feet are worsening factors. Tinea unguium (onychomycosis) causes discoloration and thickening of the nail plate. Treatment: Tinea capitis and tinea unguium is

treated with systemic griseofulvin. In all cases, cultures should be obtained before griseofulvin therapy. Other tineas are cured by topical antifungal preparations, such as clotrimazole. - Tinea versicolor - a superficial fungal infection characterized by scaly macules (pink, tan, brown, white). It involves the chest, proximal arms, and neck It is caused by Pityrosporum ovale that is a skin saprophyte. Under certain conditions, it changes from the yeast phase to the filamentous form that causes the clinical lesions. It is not transmissible. It is less frequent in children than in adults Scabies - results from infestation of the spfc. layers of skin by the human mite Sarcoptes scabiei Clinical features: It is widespread and occurs in people of all age groups. It is most common in crowded areas and is highly contagious. Humans are the only source of the mite The lesion is a papule, papulovesicle, papulopustule, or a burrow. In older children lesions are distributed on the volar sides

of the wrists, ankles, finger web spaces, buttocks, and genitalia, groin, axillae, but always spare the head. In infants, distribution is less localized The lesions itch and may become secondarily infected. Diagnosis: Microscopic examination of the scrapings. Treatment: Permethrin cream, pyrethroid. It is applied to the entire skin surface from the neck down, and left on for 10 h, before being washed off. All household members should be treated at the same time. Linen and clothes should be laundered Diaper dermatitis - initiated by irritation and aggravated by infection. Erythema, maceration, pustules, and erosions may be seen in this condition. Clinical features: Nearly every infant has inflammation in the diaper region aggravated by prolonged contact with urine and feces. The course depends on whether infection (caused by bacteria or yeast) is present. Most common agent is Candida albicans, less common S aureus Treatment: Avoidance of occlusion and appropriate cleansing. Gentle

washing with warm water is usually all that is needed. The area should be dried and then covered with zinc oxide paste. If significant inflammation is present, hydrocortisone ointment will help decrease the erythema. If Candida is suspected, an antifungal cream is indicated 203 99) CONGENITAL ANOMALIES OF THE URINARY TRACT. a) Renal parenchymal anomalies About 10% of children have congenital GU tract anomalies, ranging from asymptomatic to lethal. Some asymptomatic anomalies have significant complications. Eg pt with horseshoe kidney have a higher incidence of renal calculi. Unilateral agenesis is usually accompanied by compensatory hypertrophy of the contralateral kidney. Ectopic kidneys are usually of no significance The most severe abnormality is complete agenesis. When the agenesis is bilateral, it causes early death Oligohydramnios is present and can result in pulmonary hypoplasia and peculiar (Potter’s) facies. 1. Renal dysgenesis - a spectrum of anomalies In simple

hypoplasia, which may be unilateral or bilateral, the affected organs are smaller than normal. In the various forms of dysplasia, immature, undifferentiated renal tissue persists. In some of the dysplasias, the number of normal nephrons is insufficient to sustain life once the child reaches a critical body size. A simple cyst within a kidney may be clinically unimportant. An entire kidney lost due to multicystic development with hypertrophy and normal function of the contralateral side may also be of no importance. Nonetheless, even a simple cyst could become a site for lithiasis, infection, or hematuria. 2. Polycystic kidney disease a. Autosomal recessive polycystic kidney disease is increasingly diagnosed on prenatal US Newborns can have Potter facies and other complications of oligohydramnios. This condition is incompatible with life. b. Autosomal dominant polycystic kidney disease is characterized by large cysts in both kidneys. Extrarenal cystic involvement of the liver, pancreas,

or lungs is not unusual Pt. are seen with combination of abdominal pain, flank mass, proteinuria, intermittent hematuria, hypertension, or UTI. Progressive renal failure develops later Management of end-stage renal failure is by dialysis or renal transplantation. 204 b) Distal urinary tract anomalies 1. Obstruction at the ureteropelvic junction may be the result of intrinsic muscle abnormalities, aberrant vessels, or fibrous bands. It can cause hydronephrosis and usually presents as an abdominal mass in the newborn. 2. Obstruction can occur in other parts of the ureter, esp as it enters the bladder, with resulting proximal hydroureter and hydronephrosis. 3. duplex ureter - the ureter is double An extra ureter may have an own pelvis Partial duplication of the ureter with division of the upper part of the ureter and the pelvis is called ureter fissus. 4. Ureterocele is a cystic dilation of the ureter inside the bladder wall – it may cause problems in emptying the urine into the

bladder. 5. Megaureter is characterized by congenital dilation and elongation with no obstruction in the urinary tract. These anomalies may be clinically asymptomatic or they may cause enuresis, poor voiding with following infection and hydronephrosis. Urinary tract obstructions are relieved surgically to minimize kidney damage. 6. Bladder exstrophy - clinically obvious and require surgery 7. Bladder outlet obstruction causes voiding problems; it may occur due to congenital hypertrophy of the sphincter, intravesical stenosis of the urethra, or abnormal innervation. 8. Aberrant posterior urethral valves – an obstruction of urine flow, which occurs almost exclusively in males. It usually presents as anuria or poor voiding in the newborn period Ascites may occur and the kidneys and bladder may be easily palpable. Surgical drainage of urine is urgently required to prevent irreversible damage. 9. Epispadias are caused by dorsal fusion defect of the urethra; can be partial or complete

(85%), Most severe form being associated with exstrophy of the bladder. More common in males 10. Hypospadias - displaced urethral opening caused by failure of fusion of the urethra In female hypospadias, the urethra opens into the vagina. In the male, the foreskin fails to become circumferential and appears as a dorsal hood. The urethral opening may be located on the underside of the penile shaft, at the penoscrotal junction, between the scrotal folds, or in the perineum. Prognosis for functional and cosmetic correction is good 11. Phimosis is congenital or acquired (inflammatory) constriction of the foreskin, which cannot be retracted. Paraphimosis is inability of the retracted constricting foreskin to be reduced distally over the glans. When either is present, surgical circumcision is indicated Excellent prognosis 205 Vesicouretheral reflux, VUR - reflux of urine from the bladder into the ureter may result in damage to the upper urinary tract by bacterial infection and

occasionally by increased hydrostatic pressure. It most often is due to congenital anomalous development of the ureterovesical junction Incomplete development of the intramural ureteral tunnel causes a failure of the flap valve action and permits backwash of bladder urine into the ureter and renal pelvis. Reflux can occur even in sufficient tunnel when there is bladder outlet obstruction with increased intravesical pressures or neurogenic vesical dysfunction. Bacteria can be easily transmitted by reflux to the upper tract, leading to parenchymal infection with potential loss of renal function. Chronically elevated bladder storage and pressure can result in hydrostatic damage of the kidney even without infection or reflux. Abdominal or flank pain, recurrent UTI, dysuria or flank pain with voiding, frequency and urgency, or signs of renal insufficiency may be secondary to VUR. Pyuria, hematuria, proteinuria, and bacteriuria may be present. Filling and voiding cystouretherograms

demonstrate reflux. IVU may show calyceal dilation and ureterectasis. Reflux may also be demonstrated by direct (catheter) radioisotope cystogram VUR is usually mild and resolves spontaneously over several years while antibacterial prophylaxis is maintained. When the reflux is accompanied by high pressure storage, the approach is to lower bladder pressure by pharmacotherapy and/or behavioral means. In severe cases, ureter may be reimplanted surgically. 206 100) LN ENLARGEMENT – DD AND TREATMENT Generalized lymphadenopathy is abnormal enlargement of more than 2 LN regions. Investigation: History: Ask about systemic symptoms such as: 1. Persistent or recurrent fever (infection, malignancy, collagen vascular disease) 2. Sore throat (infectious mononucleosis) 3. Cough (TB or fungal infection) 4. Epistaxis or easy bruising (leukemia) 5. Limp, or limb pain (juvenile rheumatoid arthritis, leukemia, neuroblastoma) Note duration of systemic symptoms and assess if they are progressing or

improving. Obtain a complete history of travel and animal exposure (brucellosis, tularemia, tropical diseases). Not recent immunizations and medications (serum sickness, drug reaction, phenytoin-induced lymphadenopathy). Assess possible exposure to TB and HIV Physical exam: Check the degree and extent. Discrete, mobile, nontender LN are palpable in most healthy children. An inflamed LN (lymphadenitis – see below) is soft, nontender, and sensitive LN enlargement caused by a lymphoma is more tender, painless, and nodes may fuse. Metastatic involvement of the LN feels typically hard, painless, and fixated to adjacent structures. Note thyromegaly (hyperparathyroidism), massive hepatosplenomegaly (malignancy, storage disease, infection), arthritis (collagen vascular disease, leukemia), or a characteristic rash or conjunctivitis (viral exanthema, JRA, SLE, Kawasaki disease, leptospirosis). Lab findings: Atypical lymphocytes are frequently associated with many viral illnesses. WBC with >

10% of atypical lymphocytes suggests mono, CMV, toxoplasmosis, viral hepatitis, or drug hypersensitivity. Tests for mono will differentiate this disease from the rest, however, it may be false-negative early in the course of EBV infection in very young children!!! Severe anemia, neutropenia, or thrombocytopenia accompanying generalized lymphadenopathy suggests malignancy, severe infection, or storage disease. Bone marrow examination, LN biopsy, chromosomes, fungal and viral serology and cultures, X-ray, antinuclear antibodies (rheumatologic diseases), VDRL tests (syphilis) should follow – according to suspected cause. Children with severe illness usually have either prolonged fever, unexplained weight loss, persistent cough, known exposure to TB, risk factors for HIV, or significant systemic toxicity. Treatment: See specific topics. Abscess requires surgical drainage 207 Regional LN enlargement It occurs as a reaction to local infection that is drained by the enlarged node. It

is prominent in strept disease (angina), TB or nontuberculous mycobacterial disease, tularemia, plaque, cat scratch disease, primary syphilis, lymphogranuloma venerum, chancroid, and genital herpes simplex. LN enlargement from edema and WBC cellular infiltration, the major sign of lymphadenitis, may be asymptomatic or may cause pain and tenderness. With some infections the overlying skin is inflamed, abscess formation may occur, and penetration to the skin will produce draining sinuses. If the cause of lymphadenitis is not clinically evident, LN aspiration and culture or biopsy is necessary. Treatment depends on the underlying cause. 208 101) TETANUS. Etiology: Clostridium tetani (anaerobic Gram + rod) with 2 forms: the spore, extremely resistant to external conditions and therefore can persist in the soil for many years, and the vegetative one, which produces neurotoxic exotoxin. Epidemiology: C tetani lives as a spore inside the bowel of many animals including humans. It gets

into the soil with the feces. When an injury occurs and the spores contaminate the wound (even minor and may be unnoticed), in anaerobic conditions the vegetative form develops from the spores and starts producing a toxin responsible for the clinical features. Although the spores are widely distribute in the environment (mainly soil), the disease is rare because of vaccination. Clinical features: The incubation period is 3 to 30 days. Short incubation is associated with a high infectious dose and has a bad prognosis of 100% mortality. 1. Slowly increasing muscle rigidity develops The chewing muscles are affected first trismus 2. Spinal muscles rigidity and tenderness of abdominal muscles are seen later 3. Painful spasms (generalized tonic convulsions) then become superimposed on the mm rigidity, resulting in typical facial grimacing (risus sardonicus) and arching of the back. 4. Spasms may be provoked by sudden sensory stimuli such as noises 5. ANS involvement may cause arrhythmia and

unstable BP This instability often causes death 6. Special forms of tetanus associated with a high mortality include gynaecological tetanus following criminal aborts and neonatal tetanus in which the cord serves as a wound. Management: In order to neutralize the circulating neurotoxin, human tetanus Ig should be given. Any evident wound should be surgically opened and debrided ASAP. Benzylpenicillin is the drug of choice to kill the bacteria. Mild spasms may be controlled by BZD, severe cases require intensive care unit management with assisted ventilation and muscle relaxants. Prognosis: Mortality is high (50%), rising to nearly 100% in gynaecological tetanus, in neonates, and in cases with short incubation period and rapid development of generalized spasms. After recovery (may take several weeks, no sequelae are observed. Prevention and prophylaxis: The most effective is active immunization with tetanus toxoid (see topic 16B). It starts at 3 months of age, boosting follows at the age

of 15 yr, subsequent boosting is recommended every 5-10 years. Children up to 15 yr are usually well vaccinated, and tetanus prophylaxis in wound management is not necessary. 209 102) DEHYDRATION. DISTURBANCE OF K+ AND NA+ HOMEOSTASIS Dehydration - Depletion of body fluids is one of the most common problems in clinical pediatrics. Children have a high incidence of GI diseases, and may demonstrate GI symptoms in nonGI cases. Infants and young children often decrease their oral intake when ill. Renal concentration mechanisms do not maximally conserve water in early life, and fever may significantly increase fluid needs. Dehydration decreases ECF volume, leading to decreased tissue perfusion, impaired renal function, compensatory tachycardia, and lactic acidosis. Caregivers must be particularly aware of dehydration occurring in breast-fed infants whose mothers fail to produce enough milk. Clinical evaluation of a dehydrated child should focus on: 1. Composition & volume of fluid

intake 2. Frequency and amount of vomiting, diarrhea, and urine output 3. Degree and duration of fever 4. Any administered medications and underlying medical conditions 5. Important clinical features a. Capillary refill time, postural BP & HR changes; lack of external jugular venous filling b. Dryness of lips and mucous membranes; lack of tears, oliguria c. Sunken fontanelle and altered mental status Children generally respond to a decrease in circulating volume with a compensatory increase in pulse rate and may maintain their blood pressure normal even in severe dehydration. A low or falling BP is, therefore, a sign of shock in children! Lab parameters include a high urine specific gravity, elevation of BUN (more than creatinine), a low urinary Na , and an elevated HTC or serum albumin level secondary to hemoconcentration. IV therapy rapidly expand plasma volume and prevent circulatory collapse (if > 10% of body weight is lost). A bolus of isotonic fluid should be given and

either colloid (5% albumin) or crystalloid (saline or Ringer’s lactate) may be used. When adequate tissue perfusion is demonstrated by capillary refill, decreased pulse & improved mental status, deficit replacement should be started. The amount of fluid that is needed for replacement is calculated from estimated deficits, ongoing losses, and maintenance needs. In case of metabolic acidosis, K+ acetate or HCO3 may be considered. If the pt is unable to eat for a prolonged period, nutritional needs must be met (hyperalimentation or enteral tube feedings). Oral rehydration for children with mild-moderate dehydration. Commercially available solutions contain Na, K, citrate or HCO3, and glucose. It should be given slowly over few hours (50ml/kg within 4 hours in mild dehydration, 100-150ml/kg over 6 hours in more severe dehydration). It is usually well tolerated Oral rehydration is contraindicated in low-consciousness 210 children or respiratory distress, in suspicion of acute

abdomen; in infants with > 10% volume depletion, hemodynamic instability; and in severe hypo- or hypernatremia. The basic principle for calculation of fluid and electrolyte requirements is estimation of caloric expenditure. Once known (from tables), the requirements for maintenance for each 100 kcal is 115 ml H2O, 3.2 mEq Na+, and 24 mEq K+ to replace normal losses Adjustments are required to replace ongoing losses, as in diarrhea, gastric drainage, excessive sweating, or body temperature (12% increase in basal rate for each 1 º C). The need of calories is based on daily activity A pt in bed can require up to 30% fewer calories. Deficit therapy will depend on the type, underlying cause, and duration of the deficit. Acute losses are less well tolerated than more chronic ones. The type of dehydration is characterized by the serum Na. If relatively more solute is lost than water, the Na falls, and hyponatremic dehydration ensues. This is important because hypotonicity of the plasma

contributes to further volume loss from the ECF into the intracellular space. Thus tissue perfusion is more significantly impaired. Hyponatremia 1. Hyponatremic dehydration – pt generally demonstrate signs and symptoms of dehydration The magnitude of the sodium deficit may be calculated by a formula. One half of the deficit is administered in the first 8 hours, and the rest over the following 16 hours. The danger of toorapid correction of hyponatremia include cerebral dehydration and injury due to fluid shifts from the ICF compartment central pontine myelinolysis. Hyponatremic dehydration also occurs in cerebral salt wasting associated with CNS insults, a condition characterized by a high urine output and elevated urinary Na due to an increase in atrial natriuretic factor. Treatment: replacement of urinary salt and water losses 2. Hyponatremic hyperhydration - may occur in edematous disorders such as nephrosis, CHF, or cirrhosis. Treatment involves restriction of Na and water and

correction of the underlying condition. Hyponatremic hyperhydration due to water intoxication is characterized by a dilute urine and is also treated with water restriction. Hypernatremia Although diarrhea is associated with iso- or hyponatremic dehydration, hypernatremia may develop in persistent fever or decreased fluid intake. If the serum Na falls quickly during treatment, the osmolality of the ECF drops more rapidly than that of the CNS. Water shifts from the ECF into the CNS and cerebral edema, seizures, and CNS injury may occur. Electrolyte concentration should be assessed every 2 hours in order to control the decline in serum 211 Na. Elevations of blood glucose and BUN may worsen the hyperosmolar state in hypernatremic dehydration and should be also monitored closely. Pt. with diabetes insipidus, whether the central or nephrogenic, are prone to develop profound hypernatremic dehydration as a result of high urinary water losses. Treatment involves restoration of fluid and

electrolyte deficits with subsequent water deprivation testing to distinguish responsiveness to ADH. Hypervolemic hypernatremia (salt poisoning), associated with excess total body salt and water, may occur as a consequence of improperly mixed formula, excessive NaCl or NaHCO3 administration, or in primary hyperaldosteronism. Treatment: diuretics and water replacement Potassium disorders The predominantly intracellular distribution of potassium is maintained by Na/K ATPase. Potassium is shifted into the ECF and plasma by acidemia and into the ICF by alkalosis, hypochloremia, or in conjunction with insulin-induced glucose uptake. The ratio of IC & EC K is the major determinant of the cellular resting membrane potential and contributes to the action potential in neural and muscular tissue. Abnormalities of K balance are potentially life-threatening. In the kidney, K is filtered at the glomerulus, reabsorbed in the proximal tubule and excreted in the distal tubule. Distal tubular K

excretion is regulated by aldosterone. Renal K excretion continues for some period even after the intake of K is decreased. Thus, by the time urinary K decreases, the systemic K pool has been depleted significantly. The causes of K loss are primarily renal. GI losses through nasogastric suction or vomiting reduce total body K. However, the volume depletion results in an increase in plasma aldosterone, promoting renal excretion of K in exchange for Na. Diuretics (esp thiazides), mineralocorticoids, and renal tubular diseases (such as Bartter syndrome) enhance the renal excretion of K. Clinically, hypokalemia is associated with: 1. neuromuscular excitability, decreased peristalsis or ileus, hyporeflexia, paralysis 2. rhabdomyolysis and arrhythmias The first priority in the treatment of hypokalemia is the restoration of an adequate serum K. In the presence of arrhythmias, extreme weakness, or respiratory compromise, give IV K. Hyperkalemia: due to: 1. decreased renal K excretion &

mineralocorticoid deficiency or unresponsiveness 2. K release from the ICF compartment 212 It is characterised by weakness, paresthesias, ascending paralysis, tetany and arrythmias. The severity of hyperkalemia depends on the ECG changes, the status of other electrolytes, and the underlying disorder. If ECG changes or arrythmias are present, start treatment promptly IV 10% calcium gluconate will rapidly ameliorate depolarization. Calcium should be given only with a cardiac monitor. The IV administration of a Henle loop diuretic, such as furosemide, can be helpful in lowering total body K. Administering Na and increasing systemic pH with HCO3 therapy will shift K from the ECF to the ICF, as will therapy with a β-agonist such as albuterol. In nondiabetic pt, glucose will enhance endogenous insulin secretion, lowering serum K. These therapies provide transient benefit. Ultimately, K must be reduced to normal levels by reestablishing renal excretion using diuretics or optimising

urinary flow, using ion exchange resins such as sodium polystyrene sulfonate or by dialysis. 213 103) BLEEDING DISORDERS - DD AND TREATMENT (see topic 3) 104) POLIOMYELITIS. An inflammation of the gray matter of the spinal cord. It is an acute viral diseases caused by polioviruses. There are 3 serotypes recognized Summer outbreaks are known since late 19th century (North Europe). Polioviruses circulate in nature every summer (they get into sewage with the feces) Incubation period is 7 to 12 days. After oral ingestion polioviruses can induce hidden, flu-like, or paretic illness. The manifestation depends on unknown host immune factors. After introduction of oral vaccination only very rare sporadic cases are seen in developed countries. Typical clinical manifestation was two-phased: Flu-like beginning fever, sore throat, headache, vomiting, neck stiffness, that was followed by a silent few days onset of paretic disease & loss of spfc and deep reflexes. The grey matter of

anterior spinal cord horns is usually affected, resulting in flaccid paresis of the extremities (one group of muscles or one or all extremities). However, the brain involvement was also frequent, just less obvious. When the gray matter of the medulla oblongata or the respiratory muscles are involved, death is likely to occur. In the majority of pt, the disease is mild, being limited to the respiratory and GI symptoms. Treatment symptomatic in the acute phase and long rehabilitation is necessary in the convalescent period to prevent muscle atrophy, spasms, and deformities. Strong physical activity during the disease increases the risk and severity of nerve involvement. Prophylaxis: Active immunization with either poliovirus vaccine live oral or inactivated in early childhood is useful as prevention (see topic 16B) 214 105) DIABETES INSIPIDUS. DISTURBANCES OF FLUID HOMEOSTASIS Diabetes insipidus It is a condition marked by the inability to concentrate urine appropriately despite a

normal osmotic gradient in the kidney. With loss of ADH secretion, 24-hour urine output may reach 10L/day Urine osmolality remains low. Etiology: 1. Isolated idiopathic defect (rare) 2. Part of panhypopituitarism 3. Head trauma 4. Surgical interruption of the pituitary stalk (eg for craniopharyngeoma) 5. CNS tumors and infections Clinical features: The pt. has polyuria and polydipsia even when water deprived The onset may be abrupt. The child prefers cold water Caloric intake diminishes, and growth and weight gain may fall off. Neurologic & visual complaints may be present if it’s secondary to a tumor Diagnosis: Water deprivation test is done: After a water load in the morning, the child is not given anything until the test is ended. Hours measurements of weight & urine output are recorded The test is terminated when 5% of body weight is lost, or the serum osmolality rises to 300mOsm/L, and the urine osmolality remains constant and dilute (< 250mOsm/L). At the end of the

test, a long-acting analogue of the ADH (dDAVP) is given intranasally, and the child may drink. A rise in urine osmolality indicates ADH-deficient diabetes insipidus This would not occur in nephrogenic diabetes insipidus and other renal diseases associated with decreased concentrating ability (e.g renal tubular acidosis) Therapy: desmopressin intranasally every 12 to 24 hours. Nephrogenic diabetes insipidus Rare X-linked recessive, with renal unresponsiveness to ADH. Male infants are severely affected Clinical features include polyuria, failure to thrive, and bouts of hyperpyrexia and vomiting, which lead to severe hypernatremic dehydration. Diagnosis: Failure to respond to ADH at the end of the water deprivation test in the absence of other renal disease suggests the diagnosis. Therapy: Providing a lot of fluids at all time 215 Psychogenic water drinking Compulsive water drinking is rare in children. The history of other neurotic behaviors, gradual onset, no getting up and

drinking at night, normal serum Na levels & osmolality may suggest it. On a water deprivation test, urine osmolality rises, whereas serum osmolality and Na remain normal. Syndrome of inappropriate antidiuretic hormone secretion (SIADH) Clinical features: there’s expansion of the vascular volume & hyponatremia lethargy, confusion, and seizures. In children, SIADH is occasionally associated with pulmonary and CNS disease (eg bacterial meningitis, pneumonia). It is also associated with some chemo (eg vincristine) Therapy: Fluid restriction. Symptomatic hyponatremia is treated with infusion of NaCl 216 106) GOITER. HYPO- AND HYPERTHYROIDISM – DD AND TREATMENT a) Hypothyroidism It may occur at birth (congenital hypothyroidism) or at any time during childhood or adolescence (juvenile hypothyroidism). Because of the importance of thyroid hormone for normal brain growth and development in the first 2 years of life, the clinical presentation is different. 1. Congenital

hypothyroidism: It affects males and females equally Etiology: - Developmental thyroid defect (thyroid agenesis or dysgenesis). - Defective biosynthesis of thyroid hormone (frequently resulting in goiter). - Transient congenital hypothyroidism may occur as a result of transplacental passage of maternally ingested goitrogens (antithyroid drugs) or maternal antibodies. Clinical features: Because thyroid hormones are not necessary for fetal growth, infants with congenital hypothyroidism are normal in size. The severity of symptoms correlates with the degree of hypothyroidism. These include: - The first symptom is prolonged neonatal jaundice. - Other symptoms develop in the first 1 to 2 mo feeding problems, lethargy, infrequent stools. - Physical findings coarse facies with large open fontanelles, large protruding tongue, hoarse cry, umbilical hernia, dry, cool skin, hypotonia, and delayed development. Diagnosis: Low total T4 or free T4, elevated serum concentrations of TSH.

Assessment of skeletal age by knee radiography may show retardation of skeletal maturation. Tc 99m thyroid scan may explain the cause: Absence of uptake in the thyroid indicates agenesis. Increased uptake implies enzymatic defect. Abnormal localization of uptake indicates ectopic gland Therapy: Thyroid hormone replacement (synthetic L-thyroxine). Prognosis: When the diagnosis is delayed beyond 3 months of age, a high proportion of children suffer permanent neurological impairment. Screening: Because it is a relatively common problem (1:4000) and early recognition may prevent neurologic sequelae, screening of the newborns has become widely applied. 2. Juvenile (acquired) hypothyroidism - When symptoms appear after the 1st year of life, it is an acquired hypothyroidism. This form is more common in girls than in boys Etiology: - Autoimmune destruction 2o to chronic lymphocytic thyroiditis (Hashimoto’s thyroiditis). - Ectopic thyroid dysgenesis. - Goitrogens (iodide cough syrup,

antithyroid drugs). - Surgical or radioactive ablation. 217 Clinical features: - Slow linear growth, delayed puberty. - Cold intolerance, small appetite. - Inactivity, constipation. School performance is not impaired - Physical findings coarse, puffy facies, flattened nasal bridge, immature body proportions, less speech and spontaneous movement; dull, dry, thin hair; rough, dry, pale, waxy skin. - Delayed tendon reflexes. Diagnosis: Low total T4 or free T4, elevated TSH. Delayed skeletal maturation, presence of circulating thyroid antibodies in case of autoimmune disease. Therapy: Thyroid hormone replacement. Prognosis: Catch-up growth is usually good, other symptoms and signs resolve completely. b) Hyperthyroidism Etiology: 1. Grave’s disease, or hyperthyroidism 2o to diffuse thyroid hyperplasia (diffuse toxic goiter), is the most common cause. It is an autoimmune disorder in which enlargement and hyperfunction of the thyroid is stimulated by circulating Ab

(binding to TSH receptors) leads to high free T4 levels that suppresses TSH. 2. Neonatal Grave’s disease is caused by transplacental passage of thyroid-stimulating Ab 3. Other causes are rare (autonomous hyperplasia, subacute thyroiditis) Epidemiology: Girls are more commonly affected (5:1), and there is often family history of Grave’s disease. The usual age of presentation is adolescence Clinical features: The onset of symptoms is insidious, including: - Increased appetite, weight loss, heat intolerance, frequent loose stools. - Emotional liability, deterioration of behavior, poor sleep. - Physical exam the child appears warm and flushed. - Thyroid storm (sudden exacerbation of symptoms) presents as tachycardia, fever, N/V. There may be proptosis and widened palpebral fissures. - The thyroid is diffusely enlarged, smooth, firm but not hard, and not tender. - The precordium is hyperactive, resting tachycardia, and widened pulse pressure are present. - The skin is

smooth, warm, flushed, and moist. - Fine tremor of outstretched fingers may be seen. - Grave’s ophthalmopathy is caused by lymphocytic infiltration of the conjunctiva, extraocular eye muscles, and retrobulbar soft tissue and may cause redness and edema of the conjunctiva, decreased mobility of the eye, and proptosis. It may be present despite treatment 218 Diagnosis: High total T4 or free T4 and total T3, and low suppressed levels of TSH. Therapy: 1. Antithyroid medication (propylthiouracil [PTU] or methymazole) Propranolol relieves symptoms until the thyroid hormone levels begin to fall (after 2 to 4 weeks). Surgery: Subtotoal thyroidectomy is indicated if medical treatment fails. Complications include postoperative hypoparathyirodism and recurrent laryngeal nerve damage. Most children require thyroid hormone replacement after surgery. Radioactive iodine ablation – although usually reserved for adults it may be good in some cases. Thyroid storm therapy: immediate

hospitalization, iodide, PTU, beta-blockers, supportive care. c) Thyroiditis - may be chronic, subacute, or acute 1. Chronic lymphocytic thyroiditis (Hashimoto’s thyroiditis) The most common thyroid disease in childhood and adolescence. Girls are more affected (2:1) Clinical features: Asymptomatic thyroid enlargement (goiter) is most common. The thyroid is diffusely enlarged, and the surface is tender. With long duration, the thyroid becomes hard and nodular Few children may be hypothyroid and very rarely, some may have thyreotoxicosis. Diagnosis: High titers of thyroid autoantibodies. Serum T4, TSH may be normal, or T4 normal with elevated TSH (compensated hypothyroidism) or T4 low + TSH high (hypothyroidism). Therapy: L-thyroxine in case of hypothyroidism. The disease may resolve completely (in 50%) 2. Subacute thyroiditis It is rare, nonsuppurative inflammatory disease of the thyroid, which has a viral cause. Clinical features: The child complains of sore throat and pain in the

area of the thyroid. Pain may be referred to the angle of the jaw or the ear and is worse on movement of the neck. Symptoms of systemic illness (fever, malaise) are frequent. The thyroid is tender and swollen Diagnosis: T4 may be moderately elevated and TSH suppressed in the early stages of the disease (discharge of preformed thyroid hormone from the gland). Thyroid radionuclide uptake is very low There is leukocytosis and elevated FW in the systemic phase of the illness. In the later stages of the disease (2 to 6 months), hypothyroidism is common but resolves in almost all pt. Therapy: Symptomatic (aspirin, NSAIDs). Steroids in most severe cases 3. Acute thyroiditis It is acute bacterial infection of the thyroid, which may be suppurative or nonsuppurative. The most common causative organisms are S. aureus, Strept hemolyticus, and pneumococci 219 Clinical features: Initially, there is an acute, toxic febrile illness with marked tenderness in the thyroid area that is exacerbated

by the neck extension. The thyroid is extremely tender, and there is increased warmth and erythema of the overlying skin. Diagnosis: Leukocytosis with a left shift and elevated FW. T4 and TSH are normal, as is uptake of radioactive iodine. US may show an abscess Needle aspiration may identify the bacteria Therapy: High dose parenteral ATBs. An abscess requires surgery Complete recovery occurs d) Thyroid nodules They are rare in children. There is a high incidence of thyroid nodules in young adults who received irradiation to the neck area during infancy or childhood. 220 107) SEPSIS. It is a life-threatening bacterial invasion of the intravascular compartment, which may or may not be associated with a focus of infection. Sepsis in infants and children Incidence and etiology: Sepsis is uncommon in immunologically normal children > 2 months. The most common agents are group B streptococcus and E. coli in infants < 2 months, and Neisseria meningitidis and S. pneumoniae in

infants and children > 2 months Clinical features: It should be suspected in any previously healthy infant or child who develops fever without an obvious focus of infection and appears ill. A petechial rash occurs commonly with sepsis due to N. meningitidis Petechial rashes can also occur with other infections. Diagnosis: WBC count (> 15.000, shift to the left), chest X-ray (if indicated), urinalysis, blood and urine cultures, and LP (if indicated). Therapy: Infant < 2 months: parenteral ampicillin and aminoglycoside (or cefotaxime). Infant+child > 2 months: third-generation cefalosporin. Sepsis in the immunocompromised pt. Defects in immune function may be congenital or acquired (see topics 17B and 13B). One of the most common cases of immunocompromise is a pt. with cancer whose neutrophil count is < 500 due to chemotherapy. Etiology: Common causative agents include Pseudomonas aeruginosa, gram-negative enteric rods, and S. aureus Staphylococcus epidermidis is common

cause of sepsis in children with central venous or intra-arterial catheters. Clinical features: Fever may be the only manifestation of life-threatening sepsis. These pt. must be examined carefully for a focus (including the oral and rectal areas) Diagnosis: WBC, chest X-ray, cultures of the blood and urine. Therapy: Empiric therapy consists of vancomycin and ceftazidime. Oxacillin, an aminoglycoside (e.g tobramycin), and an antipseudomonadal penicillin (eg ticarcillin) can be used alternatively 108) Congenital hypertrophic PYLORIC STENOSIS. See topic 20. 221 109) MYASTHENIA GRAVIS. PERIPHERAL NERVE PALSIES Myastenia gravis Etiology: usually sporadic. It is an autoimmune disease in which antibodies develop against the acetylcholine receptor protein at the motor end-plate. Clinical features: There are 3 different clinical forms: a) Neonatal myasthenia: Antibodies from the mother with MG are transmitted to the fetus transplacentally. The infant develops transient myasthenia in the

1st week of life, lasting < 2 mo b) Congenital myasthenia: Ptosis, usually the first symptom, is noted by 2 yr of age; swallowing difficulties and truncal weakness may follow. c) Juvenile myasthenia: similar to the adult form, but starting earlier (1st - 2nd decade). Associated diseases including rheumatoid arthritis, thyroiditis, thymoma, and diabetes mellitus may occasionally occur. Diagnosis: Pt. show normal muscle strength after receiving edrophonium chloride; the muscles weaken 1 – 5 min later. A repetitive nerve stimulation test causes rapid muscle fatigue Therapy: Pyridostigmine, an anticholinesterase agent, is helpful. Immunosuppression with corticosteroids may be necessary. Plasmapheresis, IVIG and thymectomy may help in some pt Prognosis: If the weakness remains limited to ocular muscles for 2 yr, the progression is limited. Peripheral nerve palsies Trauma, infections, postinfectious states, toxins (e.g lead), and genetic factors all may effect the axon, the myelin, or

both. a) Peripheral nerve injuries Clinical features include sensory loss as well as weakness and wasting of the muscles innervated by the affected nerves. The most common peripheral nerve injury in childhood is neonatal brachial plexus palsy. Varying degrees of upper extremity weakness and sensory loss are seen Therapy: Removal of the compressing force and reanastomosis of nerves, if possible, as well as minimizing complications, including contractures (physical program) and pain (with medication). Prognosis can be determined by EMG. Presence of re-innervation potentials is a recovery sign b) Hereditary sensory and motor neuropathy (HSMN) It is a group of inherited neuropathies . The most common of this group is Charcot-Marie-Tooth disease (peroneal muscular atrophy). Clinical features: Weakness begins in the foot muscles starting in the 1st decade of life; eventually the hand muscles are affected. Mild sensory loss may be present Diagnosis: EMG shows denervation, and nerve

conduction times are delayed. 222 Therapy: A rehabilitation therapy (physical and occupational) is indicated. Prognosis: Most disease in this group are mild, and life expectancy is normal. However, some pt become physically handicapped, becoming confined to a wheelchair by the 4th decade. c) Guillain-Barre syndrome It is a postinfectious demyelinating polyneuropathy. Lymphocytes sensitized to the basic protein of myelin have been identified, which supports the presumed autoimmune pathogenesis. Clinical features: 2 weeks after a viral infection or an immunization, weakness begins to develop in the distal muscles of the lower extremities, sometimes with paresthesias. The weakness progresses upward and centrally over 2 to 4 weeks, so that the diaphragm and cranial nerve musculature may eventually become involved. It lasts other 4 weeks, followed by recovery, which may take up to 1 year. At the height of the clinical manifestations, the CSF shows an elevated protein level without an

elevation of the WBC count. Therapy: It is supportive. Respiratory difficulties may require assisted respiration Steroids, plasmapheresis, or IV gammaglobulin is recommended for pt. with chronic progression Prognosis: 10% of pt. have residual deficits (weakness of foot muscles) Another 10% have a relapse, usually within the 1st year after recovery. Death is rare, but may occur Other presumably autoimmune, postinfectious neuropathies include Bell’s palsy and sixth nerve palsy. Brachial plexus neuropathies are associated with influenza vaccination d) Other peripheral neuropathies These are less common and include brachial and lumbar plexus neuropathies, hereditary sensory/autonomic neuropathies, Leber’s optic atrophy, or neuroaxonal dystrophy. 223 110) ACUTE HEPATITIS a) Acute viral hepatitis In childhood, acute hepatitis occurs most commonly as a result of viral infection. Diagnosis: Children with acute viral hepatitis have elevated serum levels of aminotransferases with or

without icterus. In severe cases, hepatic synthetic function may be affected, with resulting prolongation of the prothrombin time. a) Hepatitis A virus (HAV). The diagnosis is established by the finding of hepatitis A antibodies IgM is present for 1 to 3 months. IgG Ab represents past infection and immunity b) Hepatitis B virus (HBV). The standard marker is the presence of surface antigen (HBsAg) The presence of Ab directed against HBsAg (anti-HBs) indicates immunity. IgG directed against hepatitis B core antigen (anti-HBc) may indicate acute infection, chronic infection, or past infection). IgM against HBcAg (anti-HBc IgM) are more indicative of acute infection HBeAg is a marker of viral replication and infectivity and chronic infection if it is present for more than 2 months. It guarantees transmission of hepatitis B virus from mother to infant when prophylaxis is not performed. c) Hepatitis C virus (HCV). Anti-HCV IgG Ab is a marker for hepatitis C, not immunity Its appearance may

be delayed for up to 12 months after infection. Anti-HCV Ab persists in chronic hepatitis C but eventually disappears after the recovery from acute hepatitis C. PCR to detect HCV RNA in the serum is most sensitive marker of infection with HCV. d) Hepatitis D (delta hepatitis). Antibodies to delta virus (anti-HDV) become detectable within 3 to 8 weeks of infection. The highest titers of anti-HDV are found in chronic hepatitis D Therapy: No specific therapy. Strict bed rest is not necessary but high activity should be avoided Prognosis: The prognosis is excellent for full recovery from nonfulminant hepatitis A. Chronic active or chronic persistent hepatitis develops in 5% of pt. who have hepatitis B Without prophylaxis, vertical transmission at birth results in infection of more than 90% infants. Many of these infants become chronic carriers of HBV. Hepatitis C results in 50% in chronicity Prevention: - Hepatitis A: Family members, children, and staff exposed at day care centers, as

well as their sexual contacts, should receive immune globulin within 2 weeks of contact. A vaccine is available and is effective in preventing infection in endemic areas (e.g travelers) - Hepatitis B: People who have had sexual, percutaneous, or mucosal exposure should receive hepatitis B immune globuline (HBIG) plus vaccination. The vaccine is a recombinant vaccine that is effective in infants as well as older children, with few side effects. It is given as 3 doses, 1 and 6 months following the first dose. The vaccine is now recommended for all infants, family contacts of chronic carriers, and other high-risk population (health care workers, homosexual 224 men, IV drug abusers, dialyses pt, hemophiliacs). All infants should be immunized against hepatitis B. Infants of women who are serum HBsAg positive, esp if they are also HBeAgpositive, should also receive HBIG within 12 hours of birth b) Fulminant hepatitis It is severe acute hepatitis resulting in progressive liver failure

and hepatic encephalopathy. Etiology: - Viral infection: Fulminant hepatitis may follow any of the hepatitis viral infections described above. Hepatitis A is the most common cause of fulminant hepatitis in children from underdeveloped countries. Hepatitis B is a less common cause Delta hepatitis infection superimposed on chronic hepatitis B may convert a stable chronic persistent hepatitis B pt. to fulminant hepatitis. Hepatitis E is particularly virulent in pregnant women Other viral causes of fulminant hepatitis in infancy include herpes virus, echovirus, CMV, and EBV. - Metabolic causes: - Tyrosinemia, galactosemia, hereditary fructose intolerance. - Neonatal iron-storage liver disease, disorders of fatty acid oxygenation. - Bile acid synthesis defects. - Wilson’s disease, α1-antitrypsin deficiency, CF, Niemann-Pick disease, and glycogen storage disease may lead to this condition in the older child. - Hepatotoxic drugs: Fulminant hepatitis can be caused by overdose

(acetaminophen), through a genetic metabolic defect (INH), or through a hypersensitivity to a normal dose (Phenytoin) - Plant toxins (e.g Amanita phalloides mushroom) - Autoimmune chronic active hepatitis - Other causes: Infiltrative diseases (leukemia), ischemia, irradiation, cryptogenic Clinical features: Early symptoms include: - persistent anorexia and fever, progressive jaundice, and mental status changes. - physical exam shrinking liver size despite worsening clinical status, hyperventilation, ascites. - Lab tests show coagulopathy, hypoglycemia, hypokalemia, hypoalbuminemia, low BUN, low cholesterol, high blood ammonia. - CNS signs include agitation, stupor, and coma. Complications: GI bleeding, secondary bacterial/fungal infection, renal dysfunction, increased ICP. Therapy: - Supportive care: Fluid+electrolyte balance (hyponatremia, hypoglycemia, hypokalemia, and hypophosphatemia). H2-receptor antagonists-upper GI bleeding prevention Fresh frozen plasma

(correction of clotting abnormalities). 225 Endotracheal intubation and assisted ventilation in coma and ICP. Treatment of ICP Broad-spectrum ATBs and antifungals. - Minimalization of encephalopathy: Low serum ammonia levels – achieved by decreased protein intake and elimination of ammonia-producing bacteria in the bowel. - Liver transplantation: It has 70% survival rate (with conservative management only 30%). 226 111) MECKELS DIVERTICULUM. INTUSSUSCEPTION Meckel’s diverticulum It is a congenital sacculation of the distal ileum. It is found in about 2% of adult surgical pt, usually located within 3 to 6 ft of the ileocecal valve. Pathophysiology: In early fetal life, the vitelline duct runs from the terminal ileum to the umbilicus and yolk sac; this duct is normally obliterated by the 7th week. Failure to atrophy leads to several abnormalities; Meckel’s diverticulum being the most common. It is formed when all of the duct except the portion to the ileum is

obliterated. This true congenital diverticulum contains all coats of the normal bowel. 25% of diverticula removed surgically contain heterotopic tissue of the stomach (which contain parietal cells that secrete HCl), pancreas, or intestine. Symptoms, signs, and diagnosis: In children, repeated episodes of severe, bright-red bleeding occur because a peptic ulcer has formed in the adjacent ileum. Bleeding tends to be acute and profuse but usually is not severe enough to produce shock. In adolescents & adults, intestinal obstruction, manifested by cramps & vomiting is more common. Acute diverticulitis of Meckel’s may occur at any age. It is characterized by localized abdominal pain and tenderness below or to the left of the umbilicus; it is often accompanied by vomiting and it is similar to appendicitis except for location of the pain. Diagnosis is difficult. Occasionally, the diverticulum can be seen on barium X-ray Presence of acid-secreting cells permits the use of technetium

pertechnetate scanning. Complications and treatment: Intestinal obstruction is a dangerous complication (torsion and gangrene can be fatal if early operation is not done). A bleeding diverticulum requires resection 227 112) ATAXIAS IN CHILDHOOD Ataxia is a disorder of coordination of movement. It usually results from dysfunction of the cerebellum or its connections. This disorder is characterized by several findings: 1. Incoordination of limb motion resulting in dysmetria, intention tremor and dysdiadochokinesis 2. Loss of muscle tone 3. Inability to maintain posture & stability (wide-based gait, difficulty in walking a straight line) Etiology: The ataxias are classified as acute, intermittent, or progressive. - Acute ataxia: - Infection and postinfectious causes (poliovirus, influenza A and B, echovirus, coxsackie B, varicella – direct effect or postinfectious autoimmune attack – it is most common between 1 and 2 yr of age, the prognosis is good – 2/3 recovering

within 2 month and 1/3 will have persistent neurologic findings). - Intoxication (ethanol, phenytoin, some cytostatics). - Infarction and hemorrhage (it may proceed quickly to obtundation – this is an indication for rapid neurosurgical decompression). - - Seizure, migraine, and acute demyelinating polyneuropathy. - Occult neuroblastoma may present initially as ataxia – the etiology is paraneoplastic. Intermittent ataxia: - Multiple sclerosis, it may be the presenting sign. Other lesions such as optic neuritis and spasticity provide the diagnosis. - Certain metabolic states may present with ataxia (e.g a variant of maple syrup urine disease, Hartnup disease). - Progressive ataxia: - Subdural hematomas and tumors within the posterior fossa present with the gradual onset of ataxia, headache-often in occipital location, nausea, vomiting, and papilledema. The child may have neck stiffness. All children with ataxia should have a high-resolution CT or MRI performed!!! Many

degenerative diseases affect the cerebellum. - Ataxia-teleangiectasia is a phacomatosis that presents with ataxia, choreoathetosis, and teleangiectasias of the skin and conjunctiva. This is the second most common cause of progressive ataxia in children < 10 yr, after posterior fossa tumors. The diagnosis is usually clinical, but pt. have altered immunity - Friedreich ataxia is an autosomal recessive disease presenting before puberty with gait ataxia or scoliosis, followed in 2 yr by rapid progression of limb ataxia, loss of position sense, and areflexia. Cardiomyopathy is the most common cause of death - Mutiple sclerosis, Wilson disease, abetalipoproteinemia, or metachromatic leukodystrophy. 228 Diagnosis: Dysmetria is the inability to guide the limb to a target. This may be shown by asking the pt. to guide his finger to his nose In lower limbs, the analogous test is the heel-to-shin test, where the pt. is asked to place his heel on the contralateral knee and then run it

down his shin Both these tests will show dysmetria. Moreover, a tremor at the end of the movement may be seen Dysdiadochokinesis is tested by asking the pt. to slap the palm then the dorsum of the hand on the thigh quickly and repeatedly. Therapy: Treatment of the underlying cause if possible. 229 113) TUBERCULOSIS (TB) Caused by M tuberculosis. The tubercle bacillus is a rod, difficult to stain, and growing slowly Pathogenesis: Primary pulmonary tuberculosis in children results from inhalation and deposition of organisms in the peripheral lung. Some organisms are carried via the lymphatics to the regional LN that subsequently become enlarged. The peripheral lesion, which is often invisible until it calcifies, and the enlarged mediastinal nodes are called the primary complex. Hypersensitivity to tuberculin and tubercle (the peripheral lesion) formation requires 3-8 wks. The primary complex may dissolve then, or become necrotic or caseous. If the area of caseation is extensive, it

may rupture into a bronchus, be aspirated, and induce tuberculous pneumonia. If many bacilli are inhaled, and/or host defenses are impaired, becillemia follows. Hematogenous spread can deposit in many different sites, producing military lesions usually 3 – 6 months after the primary infection. Transplacental infection can present with disease at birth. Tubercle bacilli produce proteins that can produce a delayed-type hypersensitivity response (intradermal injection of these proteins induces a positive tuberculin test). However, immunity is not complete – reactivation and reinfection are well known. Newborn infants have some immunity from the mother (until maximum 3 mo of age) There is no evidence of transfer of TBC by human milk. Epidemiology: TB is most common in urban crowded slums. Epidemics occur in shelters for homeless and HIV-infected. Malnutrition depresses host defenses and increases risk of infection There is a high rate of TB among immigrants from developing countries.

Almost all cases of TB are acquired from close contact. Congenital TB can occur, with involvement of liver, spleen, and peritoneal surface from bacillemia via the umbilical vein. It is rare Clinical features: The disease is spread mostly by droplets from an infected individual who is coughing. Children get mostly infected from a parent or another close-contact adult Bovine TB is acquired by ingestion of milk from infected cows and presents as cervical adenitis or intestinal TB. The severity of infection is related to the size of the inoculum, the virulence of the organism, and sensitivity of the host. Age also plays a role Disseminated disease occurs more often in the 1st 2 yr of life and again at puberty. Severity is enhanced by underlying disease (measles, DM, malignancy) When host defenses are suppressed by cortisol agents, TB that has been asymptomatic and latent for years may become manifest. Presenting findings are low grade fever, anorexia, and cough Children may not have cough

& rarely produce sputum. Bacteria may be found in swallowed pulmonary mucus and can be recovered by gastric aspiration. Most children are not diagnosed by symptoms, but commonly by routine TB tests. Diagnosis: The tuberculin (Mantoux) test is the principal screening and diagnostic tool. The test is read after 48-72 hours. Induration < 5 mm is negative, 5-9 mm doubtful, and > 10 mm positive This test may produce severe reaction in some individuals. False-negative results may occur in 230 malnutrition or after measles or other viral disease or immunizations, it is necessary to test for anergy with another antigen, such as Candida albicans. Almost all persons have had contact with Candida and will give a positive response if T-cell mediated delayed hypersensitivity is normal. Previous immunization with BCG is not a contraindication to tuberculin testing – it rarely produces induration over 9 mm. Definitive diagnosis depends on culture of the bacilli Sputum or swallowed

organisms may be seen on smear or grown on suitable media. Unfortunately, growth is slow so results may be only after 3 – 6 weeks. Biopsy may be performed as well Treatment: isoniazide, pyrazinamid, rifampin, ethambutol and streptomycin. Hospitalization depends on the severity of illness. Prevention: Contacts of newly diagnosed case who have a tuberculin reaction of 5 mm or more should receive chemoprophylaxis for at least 3 months (isoniazide). Vaccination: BCG is the generic term for a number of live attenuated vaccines derived from M. bovis. Vaccines are recommended for all infants in Hungary In the US, tuberculin-negative infants from families with sputum-positive adults receive the vaccine. BCG is contraindicated in immunosuppression. 231 114) DISEASES OF THE ORAL CAVITY a) Infections Gingivitis and stomatitis refer to inflammatory disease of the gingivae (gums) and oral mucosa. Combined inflammation is called gingivostomatitis. 1. Necrotizing ulcerative gingivitis

(Vincent’s disease) This infection is more common in adults but may occur also in children. It results from a decreased resistance of the gingivae to infection by normal oral flora. Plaques are large and consist of fusiform bacilli and spirochetes. Clinical features include gingival pain, fever, malaise, and foul-smelling breath. Therapy consists of oral irrigation with oxidizing agents & ATB (penicillin, amoxicillin) 2. Aphthous stomatitis It is common and often recurrent oral mucosal lesion. It consists of circular shallow ulcers that are painful and occur anywhere on the oral mucosa. The lesions may be single or in clusters, and are covered by a gray membrane, & surrounded by a border of inflammation. Exact cause is unknown It is self-limited and it heals in 1 or 2 weeks without treatment. Saline mouthwash may be helpful 3. Herpetic gingivostomatitis (see topic 33B) This is the most common gingivostomatitis in children. The primary infection occurs 4. Herpangina It is one

of the manifestations of the infection with enteroviruses. It can occur in association with meningitis, exanthems, and other clinical presentations of the enteroviruses. Coxsackieviruses types A and B and echoviruses are the causative agents. It occurs almost exclusively in summer and fall, when enteroviruses are prevalent. Fever, sore throat, and pain on swallowing are the most common signs. The lesions are vesicles and ulcers surrounded by an erythematous ring. The lesions occur in the posterior pharynx The fever subsides in 2 to 4 days, but ulcers may persist for 1 week. No treatment is necessary 5. Hand-foot-and-mouth disease It is another infection caused by enteroviruses, which is characterized by vesicular lesions of the mouth, hands, and feet. 6. Candidal gingivostomatitis (“thrush”) It is common in newborns. It clears by 3 mo of age, except in severely diseased children Oral ATB therapy may predispose to it. When it occurs after the first year of life, a defect of

cell-mediated immunity should be considered. The lesions are grayish-white, on the buccal mucosa and dorsum of the tongue. If a scraping from the affected area is gram-stained, both yeast and pseudohyphae are seen. Culture on blood agar will yield Candida sp organisms Therapy consists of nystatin solutions. Clotrimazole, or fluconazole are alternatives 232 b) Teeth development Two sets of teeth include the 20 primary (deciduous) and the 32 secondary (permanent) teeth. Disorders of teeth development include enamel hypoplasia that may be caused by rickets, serious illness, amelogenesis imperfecta or the intake of tetracyclines. Because enamel formation is complete after the age of 10, tetracycline given after this age does not cause any damage to the enamel/ Abnormally shaped teeth are seen in congenital syphilis. Missing teeth may be genetic or associated with certain syndromes (e.g down syndrome) Delayed eruption of all teeth indicates impacted teeth or an endocrine, metabolic,

genetic, or nutritional disturbance, such as hypopituitarism, hypothyroidism, osteopetrosis, or rickets. Discoloration of teeth may occur as a consequence of excessive fluoride, prolonged neonatal jaundice, or tetracycline. Early tooth loss may be due to genetic causes (e.g Down sy), immunodeficiencies, enzyme deficiencies, or tumors (e.g leukemia) c) Dental caries It depends on the interaction of dietary carbohydrate and oral bacteria, specifically Streptococcus mutans, on the tooth surface, leading to pit formation that progresses to cavity (caries0 formation. If unchecked, the process erodes through the tooth, permitting bacterial invasion of the pulp that becomes painful (toothache). Further spread to the alveolar bone results in dental abscess Caries in primary teeth may disrupt the development of the permanent teeth. The best treatment is prevention and the most effective measure is fluoridation of the water supply. Reducing dietary carbohydrate intake is also important (esp

giving children continuously drinks containing sugar is very harmful). Brushing of teeth should begin by age 3 yr Regular dental visits should also begin at this age. Early dental treatment can save most carious teeth. Infection of the tooth itself is managed with filling or extraction. When the infection extends to the alveolar bone, oral ATBs are given d) Cleft lip and palate 1. Cleft lip Cleft lip, with or without cleft palate, occurs in 1:1000 births and is more common in males. It may be unilateral or bilateral. Multiple genetic and environmental factors play a role in etiology The recurrence risk in siblings is 4%. Most cleft lips are repaired shortly after birth In general, feeding is not a problem with isolated cleft lip deformities. 2. Cleft palate Development of the palate, which includes development of the hard palate, soft palate, uvula, and maxillary teeth, is completed by the ninth week of gestation. The incidence is 1:2500 233 Genetic factors are important in the

etiology. The recurrence risk is also 4% Cleft palates are common in pt. with chromosomal syndromes Surgical repair is done between 12 and 24 mo. In the immediate newborn period, respiratory and feeding problems may occur These pt. are fed by gavage or gastrostomy The main problems after cleft palate repair are speech and tooth disturbances and recurrent otitis media. 234 115) SEIZURES DISORDERS. FEBRILE SEIZURES EPILEPSIES a) Neonatal seizure disorders Defined as abnormal electrical discharges from the CNS occurring in newborns, usually manifested by stereotyped muscular activity or autonomic changes. Seizures are a frequent and sometimes serious neonatal problem. They may occur with any disorder that directly or indirectly affects the CNS and require an immediate evaluation. Etiology: Seizures can arise only from an abnormal CNS discharge, but this may be from a primary intracranial process (meningitis, CVA, encephalitis, intracranial hemorrhage, tumor) or secondary to systemic

or metabolic problem (hypoxia-ischemia, hypoglycemia, hypocalcemia, hyponatremia). Infections may cause seizures. Seizures are frequent with meningitis and also occur in sepsis Hypoxia-ischemia may occur before, during, or after labor and often occurs in premature infants with respiratory distress syndrome. Hypocalcemia is common in infants born to diabetic mothers, small-for-gestational age infants, and those with hypoxia-ischemia or other stress. It’s often associated with prematurity or difficult births Hyper- or hyponatremia may cause seizures. Hypernatremia may result from accidental oral or IV NaCl overloading. Hyponatremia may be dilutional when too much water is given orally or IV or may follow Na loss in stool or urine. Other causes of seizures are intraventricular hemorrhage, birth trauma, drug withdrawal, and CNS malformations. Maternal substance abuse (cocaine, heroin, BZD) is an increasingly common problem, & seizures can occur with acute withdrawal after birth.

Symptoms and signs: Neonatal seizures are usually focal and may be difficult to recognize. Migratory clonic jerks of extremities, alternating hemiseizures, or primitive subcortical seizures (respiratory arrest, chewing movements, persistent deviations of the eyes, episodic changes in muscle tone) are common. A grand mal seizure is infrequent Clonic muscle activity seen with hypertonicity must be distinguished from true seizure activity. Hypertonicity produces clonus only with stimulation, and holding the extremity still will stop it. Seizures occur spontaneously and their motor activity is still felt when the limb is held. Diagnosis and Prognosis: It includes laboratory findings, cultures, metabolic tests, skull X-rays (calcifications), EEG (focal abnormality during a seizure is a good prognostic sign, an EEG with diffuse abnormalities is a poor one), cranial US or CT (intracranial hemorrhage). The long-term prognosis after neonatal seizures is related to the cause. Early onset of

seizures is associated with the highest morbidity and mortality rates. The longer seizure activity continues, the more likely the infant is to have later neurologic impairment (e.g cerebral palsy, mental retardation) 235 Treatment: Therapy should be directed primarily to the underlying pathology and secondarily to the seizures. It is usually unnecessary to stop seizures in progress, since they generally are self-limited If needed, phenobarbital is the drug of choice. b) Seizure disorders There are two kinds of seizure disorders: an isolated, nonrecurrent attack, such as may occur during a febrile illness or after head trauma, and epilepsy – a recurrent, paroxysmal disorder of cerebral function characterized by sudden, brief attacks of altered consciousness, motor activity, sensory phenomena, or inappropriate behaviour caused by excessive discharge of cerebral neurons. Seizures may be precipitated by exogenous factors (sound, light, and touch). Repeated seizures characterize

epilepsy. Epileptic seizures occur most commonly at the extremes of life. The incidence is highest in the newborn period and higher in childhood than later in life Epilepsy in childhood often remits (30% chance of a second seizure). Factors promoting recurrence: 1. Difficulty in getting the seizures under control 2. Neurologic dysfunction or mental retardation 3. Age at onset under 2 years 4. Abnormal EEG at the time of discontinuing medication The type of seizures also determines prognosis. Seizures are caused by any factor that disturbs brain function. Seizures and epilepsy are often classified as symptomatic (the cause is identified or presumed) or idiopathic (the cause is unknown or presumed genetic). The younger the child the more likely the cause can be identified Idiopathic or genetic epilepsy most often appears between ages 4 and 16 years. Symptoms and signs: The key to diagnosis is the history. An aura (a sensory or psychic manifestation) sometimes precedes the seizure itself.

The pt. describes a feeling of fear, numbness or tingling in fingers, or bright lights in visual fields Sometimes there are no warnings before a seizure comes. The parent may report that the pt eye deviated to one side or that pallor, trismus, or body stiffening occurred first. All the events prior to, during, and after the seizure can help to classify the seizure and may help to determine if the event was an epileptic seizure or a pseudoseizure. Classifying the seizure may aid diagnosis or prognosis and may suggest lab tests and medications. Diagnosis: Idiopathic epilepsy must be distinguished from symptomatic epilepsy. Focal seizures imply a focal structural lesion in the brain; generalized seizures are more likely to have a metabolic cause. A thorough history is very important EEG and serum glucose, sodium, magnesium, and calcium follow. A lumbar puncture is indicated if infection is suspected 236 Prognosis: Drug therapy completely eliminates seizures in 1/3 of pt. and reduces

frequency in another 1/3. Usually, these pt can eventually discontinue drugs Most pt. become neurologically normal between seizures Types of seizures a) Simple partial seizures consist of motor, sensory, or psychomotor phenomena (according to the affected brain area) without loss of consciousness. It can happen at any age In jacksonian seizures, focal motor symptoms begin in one hand and “march” up the extremity. Other focal attacks can first affect the face area, then spread down the body to involve the arm and sometimes a leg. Some seizures proceed to generalized convulsions b) Complex partial seizures, the pt. loses contact with the surrounding for 1-2 minutes First, the pt. may stare, perform automatic purposeless movements, say sounds without understanding what is said, and resist aid. Confusion continues few minutes after motor attack. These seizures may develop at any age Aura (e.g child running to adult from “vague fear”, epigastric discomfort, odd taste or smell,

hallucinations) is common before the onset of the seizure. Unlike absences, complex partial seizures occur singly and last longer. c) Generalized seizures cause loss of consciousness, and motor function from the onset. Such attacks often have a genetic or metabolic cause. They may be primarily generalized (bilateral cerebral cortical involvement at onset) or secondarily generalized (local cortical onset with subsequent bilateral spread). Types: • Infantile spasms (West sy) are characterized by sudden flexion of the arms, forward flexion of the trunk and legs extension. Seizures last few seconds and repeated many times a day They occur only in the first 3 years of life and then replaced by other types of seizures. • Absence seizures (“petit mal”) consist of brief attacks manifested by a 10-to 30-sec. Loss of consciousness and eyelid flutterings. Affected pt don’t fall or convulse; they stop activity suddenly and resume it just as suddenly after the seizure, with no post

symptoms or even knowledge that an attack has occurred. Absence seizures are genetic and occur predominantly in children. They usually occur when the pt is sitting quietly • Generalized tonic-clonic seizures typically begin with aura or cry; they continue with loss of consciousness and falling, followed by tonic, then clonic contractions of the muscles of the extremities, trunk, and head. Urinary and fecal incontinence may occur Seizures usually lest 1 to 2 minutes. They may appear at any age 237 • Atonic seizures are brief seizures in children. They are characterized by complete loss of muscle tone and consciousness. The child falls to the ground, so that seizures pose a serious risk of trauma, particularly a head injury. • Myoclonic seizures are brief, lightning-like jerks of a limb, several limbs, or the trunk. They may be repetitive, leading to a tonic-clonic seizure. There is no loss of consciousness These seizures may appear at any time in childhood (usually 2-7

years). d) Febrile seizures are associated with fever (> 38.8°C) They affect 4% of children aged 3 mo-5 years. Benign FS are brief, solitary, and generalized tonic-clonic; complicated FS are focal, last > 15 minutes, or recur >2x in 24 h. Acute respiratory illnesses are most commonly associated with FS. Recurrent FS occur in 50% but do not worsen the long-term outlook e) In status epilepticus, seizures follow one another with no intervening periods of normal neurologic function. Generalized convulsive SE may be fatal It may result from too-rapid withdrawal of anticonvulsants. Confusion may be the only manifestation of the nonconvulsive type of SE! Infection or metabolic disorders are the most common cause of SE in children. 25-75% of children experience SE with their initial seizure Tumor, stroke, or head trauma, which are common causes of SE in adults, are uncommon causes in children. SE occurs most commonly in children <5 yr Treatment: The ideal treatment is the

correction of specific causes. However, even when a biochemical disorder, a tumor, meningitis, or another specific cause is being treated, anticonvulsant drugs are often required. a) General management - pt. and parents must understand the problem of seizures and their management. Children should engage in physical activities for their age and social groups Loss of sleep should be avoided. Alcohol intake should be avoided since may cause seizures Prompt attention should be given to infections. Driver’s license will be issued to an epileptic if he or she is under a physician’s care and free of seizures for at least 1 year. In the pregnant teenager with epilepsy, the possibility of teratogenic effects of anticonvulsants must be weighed against the risk from seizures. During a seizure, injury should be prevented Protecting the tongue should not be attempted because the teeth may be damaged. Clothing around the neck should be loosened, and a pillow placed under the neck. The pt. should

be rolled onto his side to prevent aspiration b) Anticonvulsant therapy - No single drug controls all types of seizures, and different drugs are required for different pt. Pt rarely require several drugs The drug of choice is started at low dose and increased to the standard dose over 1 week. Blood levels are measured to determine whether the therapeutic dose has been reached. If toxic blood levels or toxic symptoms occur before seizures are controlled, a second anticonvulsant is added. The initial anticonvulsant is 238 then withdrawn gradually. The effective drug should be continued without interruption until 1 seizure-free year. The most effective anticonvulsants for long-term use include Acetazolamide, Carbamazepine, Clonazepam, Ethosuximide, Gabapentin, Lamotrigine, Phenobarbital, Phenytoin, Primidone, Topiramate, and Valproate. Status epilepticus can be terminated by diazepam IV or lorazepam IV. Phenytoin IV may prevent recurrence. Anesthetic IV doses of phenobarbital or

lorazepam may be necessary in refractory cases; in such cases, intubation and O2 therapy are required to prevent hypoxemia. Benign febrile convulsions do not require anticonvulsant treatment. c) Surgical therapy - Some pt. have seizures that are refractory to medical treatment Most pt. whose seizures originate from a local area of abnormal brain function improve markedly when the epileptic focus is resected. Some are completely cured d) ACTH and corticosteroids - These drugs are indicated for infantile spasms which cannot be controlled by anticonvulsant drugs. Duration of therapy is guided by cessation of clinical seizures and normalization of EEG (usually few weeks to few months). e) Ketogenic diet in treatment of epilepsy - A ketogenic diet should be recommended in myoclonic seizures and absence seizures not responsive to therapy. Ketosis is induced by a diet high in fats and very limited in carbohydrates with sufficient protein for body maintenance and growth. The mechanism for the

anticonvulsant action is not understood. The diet is most effective in children under age 8 years. 239 116) MANAGEMENT OF ALLERGIC DISORDERS (see topic 98) 117) CONGENITAL DISORDERS OF LUNG- PLEURA, MEDIASTINUM & DIAPHRAGM. 1. Pulmonary hypoplasia Histologically - there is decreased number of alveoli and capillaries. The severity depends on the time during gestation when the insult occurs. Etiology: 1. Oligohydramnios (due to ROM or lack of fetal renal defect 2. Intrathoracic space-occupying lesion (diaphragmatic hernia, pulmonary tumor) Clinical features: severe respiratory distress & cyanosis. CXR shows small lungs, PTX, pleural effusion. When oligohydramnios present, the characteristic facies of Potter’s syndrome are present Kidneys may be normal, large, or absent, if there is a renal cause. Therapy: Ventilatory support + treatment of pulmonary hypertension 2. Diaphragmatic hernia Displacement of abdominal contents into the thorax through a defect in the diaphragm.

Types: Hernias through the foramen of Bochdalek - most commonly seen. The defect is on the left, in the posterolateral portion of the diaphragm. It results from failure of the pleuroperitoneal canal to close. Pulmonary hypoplasia results from compression of the affected lung by the GI organs This is the most urgent of all neonatal thoracoabdominal emergencies. Hernias through the foramen of Morgagni are rare. The hernia is on the right, in the anterior portion of the diaphragm, through defects that are secondary to a failure of the retrosternal segment of the septum transversum. Frequently, the hernia contains only omentum Clinical features: Severe respiratory distress with cyanosis and dyspnea is seen shortly after birth. Breath sounds are diminished on the affected side and heart sounds are shifted to the right. X-ray shows air-filled bowel in the left hemithorax. Therapy: Intubation, oxygenation and mechanical ventilation, decompression of the intestinal tract with a nasogastric

tube, correction of metabolic acidosis, and surgical removal of the abdominal contents from the thorax with repair of the hernia. Prognosis depends on the degree of lung hypoplasia and presence of other anomalies about 50%. 3. Congenital lobar emphysema It is caused by developmental abnormalities in either the conducting airways or the alveoli that result in the air trapping in the affected lobe of the lung. 240 Clinical features: Mild-to-severe respiratory distress, including tachypnea and cyanosis. X-ray shows overinflated area of the lung. Therapy: Most pt. require removal of the involved lobe 4. Hyaline membrane disease (see topic 9B) 5. Persistence of the fetal circulation, or persistent pulmonary hypertension of the newborn It affects term infants who have experienced acute or chronic in utero hypoxia. It is seen frequently in infants with meconium aspiration syndrome (topic 41A). Pathophysiology: Failure of the pulmonary vascular resistance to fall with postnatal lung

expansion and oxygenation. With persistence of the fetal circulation, the pulmonary vascular resistance continues to be high, and may be higher than the systemic vascular resistance. This results in shunting of the deoxygenated blood. The RL shunt can occur at both the atrial level (foramen ovale) and through the ductus arteriosus. Because the lungs are bypassed, the blood is not oxygenated and hypoxemia occurs. Clinical features: Rapidly progressive cyanosis, mild-to-severe respiratory distress. Diagnosis: History of perinatal hypoxia, cyanosis, negative cardiovascular examination and X-ray. Echo shows no heart disease, increased pulmonary vascular resistance, and the shunt. Therapy and prognosis: Supplemental oxygen, mechanical ventilation, hyperventilation, support of systemic blood pressure, and sodium HCO3 and pulmonary vasodilators. Inhaled nitric oxide May lower the pulmonary vascular resistance. The mortality rate is high 6. Bronchogenic cyst This congenital cyst lined with

bronchial epithelium is usually found in the mediastinum. Symptoms result if the cyst compresses the airway or if the cyst becomes infected and opens through tracheobronchial communication. Therapy: Surgical removal of the cyst. 7. Pulmonary sequestration It is a cystlike mass o nonfunctioning lung tissue, which lacks normal communication with the tracheobronchial tree. This sequestration is nourished by systemic arteries Clinical features: Infection can result if a fistula develops between the sequestration and the airway or digestive tract. These children have a history of recurrent, progressive pulmonary sepsis X-ray shows a density, contrast bronchography shows an area that fails to fill. Aortography shows the anomalous arterial supply. Therapy: Surgical removal. 241 118) MENINGISMUS SYNDROME – DD AND TREATMENT ( see topic 128) 119) CARE OF THE NEWBORN AFTER DELIVERY. PEDIATRIC EMERGENCIES IN THE DELIVERY ROOM – RESUSCITATION. The Apgar score The Apgar score should be

done at 1 and 5 minutes of age. In case of severely depressed infants, a 10-minute score should also be recorded. Although the 1 and 5 minute Apgar scores have almost no predictive value for long-term outcome, serial scores do provide a useful description of the severity of perinatal depression and the quality of resuscitation. The Apgar score is assessing the oxygenation, ventilation, and degree of asphyxia. Five signs HR, respiratory effort, muscle tone, reflex irritability, and skin color-are examined and assigned a score of 0, 1, or 2. Score of 8-10 reflects good oxygenation and ventilation and indicates no need for resuscitation. Score of 5-7 indicates a need for stimulation and supplemental oxygen. Score < 5 indicates a need for assisted ventilation and cardiac support. The Apgar score is a useful method of communicating the well-being of the newborn. However, urgently needed resuscitation should not be delayed while a full examination is performed. Bradycardia or poor

respiratory effort alone indicates a need for immediate resuscitation. Pediatric emergencies in the delivery room - Resuscitation Maintenance of body heat: The infant should be dried and provided with radiant heat. It is important to avoid hypothermia, which will increase the newborn’s oxygen consumption. 1. Establishing the airway: Immediately after delivery, the infant’s head should be placed in a neutral position & an airway established by clearing the mouth, nose, and pharynx of thick secretions or meconium. 2. Ventilation: In most cases, drying off, suctioning, and tactile stimulation are adequate to induce spontaneous ventilation. If ventilation is adequate, supplemental oxygen may be given to improve HR or skin color. If supplemental oxygen is not enough, mechanical ventilation should be initiated, using mask and bag ventilation. If spontaneous ventilation improves, mechanical ventilation should be stopped and supplemental oxygen given. If the response is poor or if

airway obstruction occurs, an endotracheal intubation & mechanical ventilation should be done. 3. Circulation: If HR is less that 60 (or 60-80) beats per minute, cardiac compression is initiated; if HR does not improve, epinephrine is administered via an umbilical venous catheter or endotracheal tube. If HR is 80 or greater but there is poor perfusion or weak pulse, a plasma volume-expanding agent is given. 242 4. Drug support: Sodium HCO3 should be reserved until it is clear that a metabolic acidosis exists. Naloxone is helpful for poor respiratory effort secondary to maternal narcotic use during labor. Naloxone is contraindicated in an infant born to narcotic-addicted mother Special problems requiring resuscitation: a) Meconium aspiration syndrome (topic 41A): Meconium-stained amniotic fluid may be a sign of perinatal asphyxia. Although the infant may be depressed because of asphyxia, it is important to remove the meconium from the airway before any attempt is made to

ventilate the infant. b) Choanal atresia is a membranous or bony obstruction of the posterior nasal passages. It is a life-threatening anomaly, and failure to recognize it may result in respiratory arrest. c) Progressive respiratory distress or cyanosis: if it occurs in an infant despite appropriate resuscitation, it suggests an underlying disorder of the cardiopulmonary system, which requires immediate investigation. Such disorders include cyanotic heart disease (see topic 43A), congenital or acquired disorders of lung (see topic 39C, 41A), and sepsis (topic 36B). Newborn physical examination Goals of newborn physical examination are to identify major and minor congenital anomalies, sequelae of birth trauma, and neonatal medical problems, as well as to determine gestational age and appropriateness of size for gestational age. - Skin (see topic 11B) - Head and neck: - Head and face frequently have sequelae of the birth process (bruises and asymmetries). Most of these sequelae

resolve spontaneously. Palpation of the skull determines contour, sutures, and the size of fontanelles. - Eyes: Conjunctival or scleral hemorrhages resolve spontaneously. The presence of red reflex excludes lens opacities (e.g cataract, retinoblastoma) - Ears: Patency of the ear canal should be determined. Malformed or low-set ear may be associated with auditory or renal abnormalities. - Nose: Obstruction of the nasal passages results in respiratory distress. - Mouth: Palpation may reveal a submucosal bony cleft of the palate. - Neck must be hyperextended to inspect adequately for masses. Congenital masses include goiter, cystic hygroma, branchial cleft cysts, and thyreoglossal duct cysts. A webbing of the neck is typical for Turner syndrome. 243 - Chest: - Clavicles are palpated for fracture (difficult delivery). - Respiratory rate, pattern, and chest asymmetry, retractions, grunting, and nasal flaring must be determined. - Cardiac location is screened. Soft

systolic heart murmurs are commonly heard in the 1st 24 hours of life (disappear spontaneously). - Abdomen: It is convex and moves with respiration. Liver edge and spleen should be palpated Because the most common abdominal masses involve genitourinary tract, palpation of the kidneys is important. - Inguinal region and genitalia: - Femoral pulses must be palpated (to check for coarctation of the aorta). - Male genitalia – note urethral meatus, testes, and search for a bulge in the groin and scrotum (hernia or hydrocele). - Female genitalia - check vaginal and urethral openings, normal-sized clitoris to exclude ambiguous genitalia and vaginal atresia. In normal infants, vaginal discharge that is mucoid or bloody results from the influence of maternal hormones. - Anus is inspected for patency and placement. Extremities: Developmental dysplasia of the hip (1:1000) is much more common in girls and breech deliveries. Trauma to the cervical nerves during delivery may result in

asymmetric arm movements, which indicates Erb palsy (C5, C6) or Klumpke palsy (C8-T2). Metatarsus adductus (fore part of the foot is adducted & supinated) may result from intrauterine positional compression. - Back: The spine is palpated for sinus tracts or overlying lesions such as lipomas, hairy tufts, or hemangiomas (signs of a covered neural tube defect). - Neurologic examination: State of consciousness, transitions from waking to sleeping, and strength of cry should be noted. Cranial nerves, primitive reflexes, and tone should be noted Gestational age assessment It is important to know an infant’s gestational age because its behavior and further problems can be predicted on this basis. Accurate recall of the date of the last menstrual period is the best indicator of gestational age. Other obstetric signs include: 1. last menses 2. fundal height 3. 1st heart auscultation 4. early US 5. foot length 6. ant vascular lens capsule (supp to be gone at 34W) 7. Bellard

score assessing neuromuscular strength & tone; variety of physical characteristics eg skin, lanugi, plantar surface, eyes, ears, genitals. Adding the score will give GA 244 A postnatal examination can be also used because fetal physical characteristics and neurologic development progress in a predictable manner. The birth weight and gestational age must be evaluated to determine if the infant’s weight is appropriate for gestational age (AGA), small for gestational age (SGA) or intrauterine growth restricted (IUGR), or large for gestational age (LGA). Factors such as the size of the parents, the birth weight-gestational age distribution of infants previously born to the parents, and variations from one population to another must be taken into consideration. An important distinction, particularly in SGA infants, is whether the growth disorder is: - Symmetric (weight, length, and occipitofrontal circumference < 10th percentile). Symmetric growth restriction implies an

event of early pregnancy: chromosomal abnormality, drug or alcohol use, congenital viral infections. - Asymmetric (only weight < 10 %). Asymmetric growth retardation implies a problem late in the pregnancy, such as pregnancy-induced hypertension or placentar insufficiency, or maternal age > 35 years, and poor weight gain during pregnancy. In general, the outlook for normal growth and development is better in the asymmetrically growthrestricted infant whose intrauterine brain growth has been spared. SGA infants have increased morbidity and mortality rates. They are at risk for fetal distress during labour and delivery, polycythemia, hypoglycemia, and hypocalcemia. LGA babies (often born to diabetic mothers) are at risk for birth trauma, congenital anomalies, hypoglycemia, hypocalcemia, polycythemia, cardiomyopathy, & hyperbilirubinemia. Procedures 1. Prophylaxis of gonococcal ophthalmia is recommended for all newborns (1% silver nitrate solution or a sterile ophthalmic

ointment containing 1% tetracycline or 0.5% erythromycin) 2. Vitamin K - every newborn should receive it to prevent hemorrhagic disease and coagulation disorder. 3. The state-sponsored newborn genetic screening tests (for inborn errors of metabolism such as phenylketonuria (PKU), galactosemia, sickle cell disease, hypothyroidism, and CF) is performed just prior to discharge, after 24-48 hours in hospital if possible. 4. All newborns should be vaccinated with the 1st dose of hepatitis B recombinant vaccine Infants at high risk for perinatally acquired hepatitis B infection should also receive a dose of hepatitis B immunoglobulin within 12 of birth. 245 120) CYANOSIS IN ACQUIRED DISEASE Cyanosis refers to a bluish discoloration of the skin and mucous membranes. Central cyanosis affects the skin, mucous membranes, lips, and conjunctiva. It occurs secondary to significant arterial desaturation. A minimum of 40-50 g/L of desaturated hemoglobin must be present in order for cyanosis to

be clinically evident. Cyanosis is due to pulmonary, cardiac, CNS (hypoventilation), neuromuscular (hypoventilation), or hematologic etiologies. Acrocyanosis (peripheral cyanosis) is a benign bluish discoloration of the hands and feet that occurs due to peripheral vasoconstriction, usually in young infants. The etiology of cyanosis varies according to age of the pt. For infants, the birth history and age of onset are important. For older children, a history of trauma, possible ingestion, or choking may be helpful. Physical signs of certain genetic syndromes may also be helpful in evaluation Older children should be assessed for signs of chronic or progressive illness, including growth parameters, clubbing, and stigmata of neuromuscular or hepatic disease. Arterial blood gases help to distinguish pulmonary from cardiac etiologies. - Congenital heart disease (see topic 43A). - Cyanosis of apnea – cyanosis commonly accompanies apnea in children of any age. Causes: - - infection

(RSV, pertussis, sepsis, meningitis), and GERD. - seizures, and neuromuscular disorders causing hypoventilation. Cyanotic infant syncope are common paroxysmal episodes characterized by sudden lack of inspiratory effort during crying. Cyanosis, opisthotonus, tonic, tonic posturing or clonic movements, and a brief loss of consciousness typically follow. The child is fully recovered within 1 minute. These spells typically occur between 6-18 months They are usually precipitated by an event that is frustrating, frightening, or painful to the child. No diagnostic evaluation is indicated. - - Cyanosis can occur with ingestion of agents that cause: - respiratory depression (e.g narcotics, or sedatives) - airway edema and obstruction (acid and alkali products). - neuromuscular disease, such as botulism. Methemoglobinemia may occur because of abnormal Hb or deficiency of enzymes involved in heme reduction. Certain drugs or toxins (eg oxidizing agents in drugs or anesthesia, nitrates

in well water) can also be responsible, esp in young infants who have low levels of methemoglobin reductase activity. Mild forms of congenital methemoglobinemia may appear later in infancy or childhood due to infections or exposure to precipitating agents. Check metHb level to confirm diagnosis. A low pulse oximetry SAT, dark brown arterial blood & normal aPO2 are suggestive 246 - Other causes that can induce acute cyanosis include: - cardiac tumors, cor pulmonale. - foreign body aspiration, primary pulmonary HTN. - hypoglycemia, hypocalcemia. Chronic cyanosis may be also caused by CF, pulmonary A-V malformation, cor pulmonale, and pulmonary hypertension. 247 121) RESPIRATORY DISEASES OF THE NEW-BORN INFANT - DD + TREATMENT Typical features Tachypnea, respiratory rate > 60 breaths/min., retractions (intercostal, sternal), expiratory grunting, and cyanosis on room air. Transient tachypnea (Retained fetal lung fluid) Respiratory distress is typically present from

birth, usually associated with moderate oxygen requirement. It is usually following a short labour or cesarean section It probably results from decreased lymphatic absorption of the fetal lung fluid. Resolution occurs within 12-24 h Aspiration syndromes The infant is typically term or near-term, with some fetal distress prior to delivery. The fetal hypoxia triggers the passage of thick meconium into the amniotic fluid. The contaminated amniotic fluid is swallowed and aspirated at birth with the initiation of breathing. With severe fetal hypoxia, the meconium may be aspirated prenatally because of fetal gasping. Diagnosis is made by the presence of meconium in the amniotic fluid, combined with respiratory distress (present from birth and manifested by a barrel chest appearance and coarse breath sounds), and X-ray (diffuse infiltrates + hyperinflation). When the amniotic fluid contains blood or meconium, suctioning of the infant’s mouth and nose as the head is delivered is recommended

to prevent aspiration of these secretions with the onset of breathing. Only after the trachea is cleared of any meconium should spontaneous or artificial ventilation be initiated!!! Babies with aspiration are at risk of pneumothorax (because of uneven ventilation with segmental overdistention) and pulmonary hypertension. Treatment: ventilatory support, ATBs, systemic blood pressure support Congenital pneumonia Onset of respiratory distress may be at birth or delayed for several hours. The lungs are the most common site of infection in the neonate. Most commonly, infections ascend from the genital tract during labor, by the vaginal or rectal flora (group B strep, E coli, Klebsiella). Shock, poor perfusion, and neutropenia are evidence of pneumonia. Pneumonia should be suspected in older neonates with recent onset of tachypnea, retractions & cyanosis. Except from bacteria, also viruses and Chlamydia can cause the diseases Because no signs or laboratory findings can confirm the

presence or absence of pneumonia with certainty, all infants with respiratory distress should receive broad-spectrum antibiotic therapy until the diagnosis of bacterial infection can be ruled out. 248 Pneumothorax, PTX It is the presence of free air in the pleural space. The air may be under tension (tension PTX) Asymptomatic, spontaneous PTX occurs in 1% of healthy newborns. Symptomatic PTX occurs in the infant who is receiving mechanical ventilation or who has underlying lung disease (hyaline membrane disease, emphysema, meconium aspiration) Symptoms + signs: cyanosis, tachypnea, and elevation of the affected hemithorax. Auscultation: diminished breath sounds on the affected side. X-ray shows collapsed lung surrounded by area of air within the hemithorax. Mediastinal structures are shifted to the opposite side. Therapy: In mild cases - supplemental 100% oxygen, in severe cases - air evacuation by aspiration. Constant suction should be applied until the air leak resolves. Apnea

It is the stopping of breathing for > 20s. It often occurs in preterm infants (apnea of prematurity) and reflects immaturity of the respiratory control mechanisms in the brain stem. Clinical features: Bradycardia (< 80), periodic breathing, intermittent hypoxia. Diagnosis of apnea of prematurity is made after excluding other reasons for apnea (infection, intracranial hemorrhage, airway obstruction, GERD, seizures, hypoxia, pulmonary edema, etc). Therapy of apnea of prematurity: tactile stimulation, maintenance of core body temperature, supplemental oxygen, respiratory stimulants (theophylline, caffeine). For other causes of apnea treat the underlying condition. Whatever the cause, any infant with respiratory distress should be given adequate oxygen to maintain a PaO2 of 60-70 mmHg and a saturation by pulse oximetry of 92-96 %. Other supportive treatment includes IV glucose, antibiotics, and volume expanders, if needed. Intubation and ventilation should be undertaken for signs of

respiratory failure. Most of the respiratory conditions affecting the term infant are acute and resolve in the first several days. Meconium aspiration and congenital pneumonia are associated with long-term pulmonary morbidity (chronic lung disease) and mortality (10-20 %). 249 122) AUTOIMMUNE DISEASES Autoimmune diseases are a consequence of complex pathological processes. They are characterized by abnormal reactions to body antigens (autoantigens) combined with exogenous factors. They can be divided into systemic (not organ specific) and localized (organ specific). Etiology: It is still not clear. It is not known what makes a normal reaction which is good for the body to turn into an immunopathological reaction which is harmful for the body. However, there are two main factors: genetical predisposition and exogenous agents. Probably a combination of several genetic and exogenous factors play a role. Some autoimmune diseases are associated with certain HLA haplotype. Sex hormones

may also play a role, because some autoimmune diseases are predominant in girls (SLE, some forms of JCA, Sjögren sy) or activated during puberty. Exogenous agents usually act as trigger factors. The most suspected are infections. Infectious agents may be the source of antigen, they may mimic own antigens, or they may change the antigens of the organism in such way that they start an autoimmmune reaction. Other factors include UV radiation, drugs, or some food particles. Immunization may probably play a role as well. Pathogenesis: The organ damage is caused by all types (except type I.) of immunopathological reactions in the classification of Coombs ad Gel. Clinical picture: It is very variable (depends on the source of antigen, on the affected organ, immunopathological reaction and the age of onset). Laboratory features: It is necessary to confirm an autoimmune disease. It includes: 1. Increased ESR, CRP and other APP 2. Presence of specific autoantibodies, hyperimmunoglobulinemia

(mainly IgG), sometimes decreased complement activity and low C3 and C4. 3. The examination of cellular immunity from peripheral blood is routinely done but has a limited value. It is valuable if it is performed directly in the affected organ (biopsy) to determine the type of immunopathologic reaction and activity of the disease, which can be useful in planning the therapy. HLA typization is performed in some cases 4. Serological tests are useful in differential diagnosis In most autoimmune diseases, there are increased antibodies against many organisms (bc of B-cells activation) – e.g EBV, HSV, CMV, toxoplasmosis. This is important to remember – such pt are often treated as if they had the infection!!! 5. Histological and immunofluorescence tests in tissues taken by biopsy are very important 250 6. Hematologic tests show a complete blood count with differential, and coagulation parameters. Systemic autoimmune diseases and autoimmune cytopenias are accompanied by a decrease in

blood elements. Thrombocytopenia is typical for SLE and antiphospholipid syndrome. On the contrary, in active stages of systemic diseases, there may be thrombocytosis. Leukopenia is typical for SLE & Sjögren sy Leukocytosis is present in systemic vasculitis. 7. Biochemical examinations may also show several changes that are usually not specific Other examinations: - Echo may show pericardial effusion. - EMG is important in the diagnosis of myasthenia gravis and myositis. - LP helps in diagnosis of vasculitis affecting the CNS. The may be even autoantibodies present in the CSF. It is also important in the diagnosis of multiple sclerosis Therapy: In childhood, it must be aggressive enough and prompt to stop the process in the beginning, but not too harmful – not damaging the pt. by side effects The therapy is usually complex – by specialists from different specialization. Nonspecific immunosuppressive therapy (corticosteroids, cyclosporin A, in severe cases also cytotoxic

drugs) is the most important. Plasmapheresis is indicated in diseases with high concentrations of autoantibodies or immunocomplexes (SLE, ANCA-positive vasculitis, antiphospholipid sy, myasthenia gravis). High doses of normal human gammaglobulines are indicated in Kawasaki disease and thrombocytopenic purpura. NSAIDs are mainly used in JCA. Sulfasalazines are used in nonspecific IBDs It is also important to monitor the side effects of such drugs and treat them effectively (growth retardation, electrolyte dysbalances, and osteoporosis from corticosteroids), G-CSF in case of bone marrow suppression due to the use of cytotoxic drugs. In organ specific diseases, it is usually difficult to find the inflammatory process in the beginning – immunosuppression has no effect in such cases – it is too late. Therefore, the consequences are treated only (e.g hormonal substitution in case of endocrine system dysfunction) Other therapeutic means include rehabilitation (JCA and dermatomyositis) or

psychotherapy. a) Systemic autoimmune diseases It includes JCA, SLE, dermatomyositis, sclerodermia, mixed connective tissue disease, overlap syndrome, vasculitis, Sjögren syndrome, rheumatic fever, poststreptococcal GN, and antiphospholipid syndrome. 251 b) Autoimmune diseases that are affecting a specific organ system and accompanied by systemic symptoms It includes ulcerative colitis, Crohn’s disease, Celiac, and autoimmune hepatitis. c) Localized (organ-specific) autoimmune diseases Autoimmune reaction is directed against structures that are typical for a specific organ. Usually, we can find specific autoantibodies in the serum in such cases. They usually cause destruction of the tissue. However, they may also stimulate (eg Grawes-Basedov disease) or blocate (e.g myasthenia gravis) the target tissue Some diseases are not accompanied by the presence of autoantibodies (e.g multiple sclerosis) 252 123) SINUSITIS. ETHMOIDITIS Sinusitis is an inflammation of the mucous

membrane lining the paranasal sinuses. a) Acute sinusitis may involve one or more sinuses. Inflammation of the ethmoid sinuses (ethmoiditis) is most common in children, because these are the only sinuses that are fully developed at birth. The maxillary sinuses may also be involved but not until the age of 18 months. Frontal and sphenoidal sinusitis are rare before age 10 yr, because these sinuses develop only after age 6 yr. b) Chronic sinusitis occurs after prolonged episodes of inadequately treated acute sinusitis that result in permanent changes of the mucosa. Sterility is no longer in the sinus Pt. may have acute infectious exacerbations Predisposing factors: The most common are viral upper respiratory tract infection, allergy, and asthma. Other contributing factors include periodontal disease, rapid changes in altitude, swimming, trauma, exposure to tobacco smoke, and immunologic defects. Etiology: The predominant microorganisms recovered from both children and adults with acute

sin. are the same that cause acute otitis media (S. pneumoniae, H influenzae, and M catarrhalis) S. aureus and anaerobic organisms are associated with chronic sinusitis Viral infections or allergies may cause sinusitis, but after there may be a bacterial superinfection. Clinical features: - Common symptoms in children > 5 yr are fever, facial pain, headache, and sore throat. - In younger children nasal discharge, cough persisting > 10 days, eye swelling, and fever. - Suggestive signs include periorbital swelling, localized tenderness, and malodorous breath. Diagnosis: - X-ray: The presence of air-fluid levels or complete opacification is strong evidence of acute sinusitis. Sinus CT gives the best views of the sinuses - Transillumination of the frontal and maxillary sinuses may provide good information in older children. Opacity suggests sinusitis, normal light transmission suggests no infection - Sinus aspiration and culture can identify the specific microbial cause.

- Biopsy is rarely performed but can be useful as well. Therapy: a) Antimicrobial therapy: Initial treatment is directed against the most common bacterial causes. The drug of choice is amoxicillin. An alternative ATB is given if treatment fails b) Sinus irrigation or surgical drainage is indicated in those who do not respond to antimicrobial therapy and in those with intraorbital or intracranial complications. c) Supportive care: Antihistamines in pt. with associated allergic rhinitis Nasal decongestants, saline nose drops, or nasal corticosteroids are sometimes prescribed. 253 124) PNEUMONIA. COMPLICATIONS IN INFANCY AND CHILDHOOD It is an inflammation of the lung parenchyma. There are numerous infectious causes, including viruses, bacteria, fungi, parasites, and rickettsiae. Although most cases are caused by viruses, ATBs are prescribed because the precise cause is initially not determined. 1. Bacterial pneumonia Etiology: - neonates group B strep are common pathogens -

children >3 months pneumococci and group A strep. Other S aureus, gram (-) enterics, Mycoplasma pneumoniae, Chlamydia trachomatis, anaerobes (esp associated with aspiration), H. influenzae type b, and M tuberculosis Clinical features: In older children (>6yr) it’s classic (like adults). In children <6 it’s variable Older children are first seen with mild upper respiratory tract symptoms (cough, rhinitis), followed by onset of fever, tachypnea, chest pain, and shaking chills. Physical examination reveals decreased breath sounds and rales on the affected side. Younger children may initially be seen with nonspecific manifestations (fever, malaise, GI complaints, restlessness, and chills). Respiratory signs may be minimal and include tachypnea, cough, grunting, and flaring of the alae nasi. Signs of pneumonia may be weak, with no rales Diagnosis: Lab findings include leukocytosis (neutrophils) and dense, focal infiltration on CXR. Specific diagnosis may be made from culture

or rapid antigen testing of blood, alveolar or pleural fluid, or urine. Gram stain & culture of the sputum is helpful, but difficult to obtain from children Therapy: a) Neonates should be hospitalized and treated with IV ampicillin and an aminoglycoside or with ampicillin and cefotaxime or ceftazidime. In case of resistant staphylococcal infection, a penicillinase-resistant penicillin should be used. Infants should also receive IV fluids, supplemental oxygen, ventilatory support, and chest physical therapy. b) Children can be observed at home and receive oral amoxicillin. In severe cases, hospitalization is necessary and IV cefuroxime or ceftriaxon are used. For the most severe cases, initial therapy consists of very broad-spectrum ATB (oxacillin or vanco). Complications: Pleural effusions, empyema, abscess, sepsis, RDS, septic shock. 254 2. Viral pneumonia Etiology: RSV is the most common viral cause. Other causatives include parainfluenza, adenovirus, and enterovirus. Less

common are rhinovirus, influenza virus, CMV, VZV and HSV Clinical features: It begins with several days of rhinitis and cough, followed by fever and more pronounced respiratory tract symptoms, such as dyspnea and intercostal retractions. The symptoms of viral pneumonia are less fulminant than in bacterial pneumonia. Diagnosis: Lab findings include lymphocytes observed on CBC and diffuse, bilateral infiltrates on CXR. Specific diagnosis can be made by rapid tests for viral antigen and culturing nasopharyngeal and rectal specimens for viruses. Therapy: a) Antiviral therapy: Ribavirin by aerosol for 5 to 7 days (respiratory syncytial and influenza viruses). Oral amantadine shortens the course of influenza A infection Acyclovir IV for 7 to 10 days against herpes simplex or varicella zoster viruses. b) Supportive care includes administration of IV fluids, and supplemental oxygen, ventilatory support, and chest physical therapy. Complications: Bacterial superinfection complications of

bacterial pneumonia. 3. Other causes of pneumonia a) M. pneumoniae is the most common nonviral cause of pneumonia in children > 6 years Clinical features: It is less severe than bacterial pneumonia. Hospitalization is rarely necessary The onset of illness is gradual; fever, headache, and malaise are seen for 2 to 4 days before respiratory symptoms develop (a nonproductive cough). Pharyngitis is also common Diagnosis: Lab findings: CBC shows mild leukocytosis. CXR shows interstitial infiltrate Cold agglutination levels are elevated. Specific diagnosis: Elevation in Ab titre in convalescent serum or isolation of M. pneumoniae from sputum or throat. Therapy: Erythromycin. b) Pneumocystitis carinii produces a progressive pneumonia in immunocompromised hosts (AIDS, congenital immunodeficiency, cancer, cancer chemotherapy). Clinical features: In infants, it begins as a mild illness with low-grade fever and cough but progresses to severe respiratory distress with cyanosis. 255

Diagnosis: Clinical course + bilateral generalized granular pattern on chest X-ray. Definitive diagnosis is made by demonstration of P. carinii on specially stained smears from tracheal or bronchial washings, lung aspirates, or lung biopsy. Therapy: Trimethoprim-sulfamethoxazole is the drug of choice. Pentamidine is an alternative 125) DEVELOPMENT OF THE CHILD. DEVELOPMENT MILESTONES ACHIEVEMENT. Development applies to somatic, psychological, and cognitive growth and to behavior. 1. The first year Normal children follow a trajectory of increasing physical size and increasing complexity of function. Growth charts provide an overview of the normal growth trajectory of children, thus alerting the clinician to what is atypical or disturbed. The child triples his birth weight within the first year and achieves 2/3 of his brain size by age 3 years. The child progresses from a totally dependent infant at birth to a mobile, verbal person who is able to express his needs by age 2-3 years. It

is critical to identify developmental disturbances during these early years because there are sensitive periods when developmental problems can be treated effectively. From a motor perspective, children develop in a cephalocaudal direction. They can lift their heads at 3 months, sit at 6 months, crawl at 9 months, walk at 1 year, and run by 18 months. The child learning to walk first has a wide-based gait. Next, he walks with legs closer together, the arms move medially, a heel-toe gait develops, and then a symmetrical swing of arms by 18-24 mo. Fine motor development: the grasp begins as a raking motion involving the ulnar aspect of the hand at age 3-4 months. The thumb is added to this motion at about age 5 months as the movement shifts to the radial side of the hand. The thumb opposes the fingers for picking up objects at age 7 months, and the neat pincer grasp emerges at age 9 months. Most young children have symmetrical movements. Asymmetries of movement noted in the first 2-3

years of life, should be investigated Finally, children have 5-10 comprehensible words by age 1 year; by age 2 years they are putting 2-3 words into phrases; 50% of which their caregivers can understand. The 1st 2 years of life are described as the sensorimotor period, during which infants learn how to link sensory input from the environment with a motor response. Infants build on primitive reflex patterns of behavior (termed schemata; sucking is an example) and constantly incorporate new experiences. The schemata evolve over time as infants accommodate new experiences Enhancement of neural networks through dendritic branching occurs in spurts throughout the sensorimotor period. In the first year of life, the infant’s perception of reality revolves around itself and what it can see or touch. However, before age 6 months the object ceases to exist once it leaves 256 the infant’s field of vision. At age 9-12 months, the infant realizes that objects exist even when not seen. It

attaches first to the image of the mother or primary caregiver because of his or her emotional importance and is a critical part of attachment behavior. In the second year, children manipulate objects by using instruments, first by imitation and later by trial and error. Freud describes the 1st year of life as the oral stage because many of the infant’s needs are fulfilled by oral means. Nutrition is obtained by sucking, and self-soothing occurs through sucking on fingers or a pacifier. The baby’s needs are totally met by the mother and the mother is manifesting “narcissistic possessiveness” of the infant. This is a very positive interaction A parent who is depressed or cannot respond to the baby’s expressions and cues can have a profoundly adverse effect on the child’s future development. Age 8-9 months is also a critical time in the attachment process because this is when separation anxiety and stranger anxiety appear. The infant experiences new events that do not match

previously known schemata. This causes uncertainty and fear In stranger anxiety, the infant analyzes the face of a stranger, detects the mismatch with previous schemata, and responds with fear or anxiety, leading to crying. In separation anxiety, the child perceives the difference between the primary caregiver’s presence and absence. The child becomes uncertain and then anxious and fearful. This begins at age 8 mo, reaches a peak at 15 months, and disappears by the end of 2 yrs 2. The second year Once the child can walk independently, it can move away from the parent and explore the environment. Although the child uses the mother as “home base”, returning to her frequently for reassurance, it takes a major step towards independence now. This is an emerging sense of self The child frequently uses self-asserting “no”. Ego development during this time should be fostered but with appropriate limits. Children also begin to understand the feelings of others and develop empathy.

They hug another child who is in trouble or become concerned when one is hurt. The failure to develop empathy is one of the cardinal deficits in autistic behavior. Children also begin to understand right and wrong and parenteral expectations. They also take pleasure in their accomplishments and become more aware of their bodies. After age 18 months, toddlers have the sensory capacity for awareness of a full rectum or bladder and are physically able to control bowel and urinary tract sphincters. Children must be given some control over when elimination occurs. If parents impose severe restrictions, the achievement of this developmental milestone can become a battle between parent and child, and long-term struggles of control predisposing to encopresis may develop later. 257 Freud termed this period the anal stage because the developmental issue of bowel control is the major requirement. Although Freud described the by-products of anal regularity on personality development,

including punctuality, reliability, cleanliness, and conscientiousness, these terms simply represent abilities emerging at the time toilet training is also being mastered. 3. Language development: ages 1-4 years Communication is important from birth, particularly between infant and caregiver. By age 2 months, these interactions include vocalizations. Babbling begins by age 6-10 months and reaches a peak at age 12 mo. Then the child uses individual words to represent objects or actions The number of words acquired by 18 mo varies significantly, with an average of 20-50 words. Recurrent otitis media, or any other condition associated with hearing loss, has a negative effect on the achievement of early language milestones. Receptive language develops more rapidly than expressive language. Word comprehension increases at age 9 months, and by age 13 months the child’s receptive vocabulary may be 20-100 words. After age 18 months, expressive and receptive vocabularies increase dramatically

The child begins to put together words and phrases and begins to use language to represent symbolic world. The child begins to manipulate the symbolic world; sorts out reality from fantasy imperfectly; and may be terrified of dreams, and foolish threats. Most of the child’s perception of the world is egocentric or interpreted in reference to wants, needs, or influence. For example, children may focus their understanding of divorce on themselves (“My father left because I was bad”). The child may make a mental connection between a sibling’s illness and a recent argument, or a wish for the sibling to be ill. The child may feel guilty Magical thinking blossoms between ages 3 and 5 years. Fantasy facilitates development of role playing, sexual identity, and emotional growth. In their play, children often create magical stories that reflect issues with which they are dealing, such as aggression, relationships, fears, and control. At this stage, other children become important, such

as in a pre-school group. Play becomes more co-operative. Freud described the oedipal phase between ages 3-6 years, when there is a strong attachment to the parent of the opposite sex. 4. Early school years: ages 5 – 7 years The child is ready relate to peers in a more interactive manner than through parallel play. The brain has reached 90% of its adult weight. Sensorimotor coordination abilities facilitate penciland-paper tasks and sports, both part of the school experience By first grade, there is more pressure on the child to master academic tasks – recognizing numbers, letters, and words and learning to write. The child is able to perform mental operations concerning concrete objects that involve 258 manipulation of more than one variable. The child is able to order, number, and classify Magical thinking diminishes greatly, and the reality of cause-effect is better understood. 5. Middle childhood: ages 7 – 11 years Freud characterized ages 7-11 years as the latency

years, during which children are not bothered by significant aggressive or sexual drives but instead devote their energies to school and peer group interactions. In reality, throughout this period there is a gradual increase in sex drive, manifested by increasingly aggressive play and interactions with the opposite sex. Fantasy still has an active role in dealing with sexuality before adolescence, and fantasies often focus on movie and music stars. For the 7-year old child, the major developmental tasks are achievement in school and acceptance by peers. Academic expectations require the child to concentrate on, attend to, and process increasingly complex auditory and visual information. Children with learning disabilities may have difficulty in these tasks and may receive negative reinforcement from teachers or even parents. Such children may develop a poor self-image manifested as behavioral difficulties. The pediatrician must evaluate potential learning disabilities in any child who

is not developing adequately or who presents with emotional or behavioral problems. 259 126) IRON DEFICIENCY ANEMIA. The most common cause of anemia in children. Most cases result from inadequate intake of iron; however, loss of iron through hemorrhage must be considered in the DD. Nutritional iron deficiency develops with rapid growth, when iron storage is not enough for the growing body. This occurs in: - Infancy (low birth weight, d/t low iron storage or when the diet is composed only of milk or cereals). - Adolescence (rapid growth). - Blood loss can occur: - Prenatally (fetal blood goes into maternal circulation (fetomaternal transfusion), or into the circulation of a twin (twin-to-twin transfusion). - Perinatally (obstetric complication). - Postnatally (trauma, surgery, parasites, IBD). Clinical features: It is most commonly seen at 6-24 months of age. Mild cases are asymptomatic, more severe manifests with irritability, anorexia, lethargy, and easy fatigability.

On examination, the milk-fed infant is fat and pale, has tachycardia, and a systolic murmur. In most severe cases, signs of CHF may develop. Diagnosis: Small, pale RBCs on the peripheral smear; the reduction in MCV, MCH and MCHC are proportional to the severity of anemia. The serum iron level is low, whereas the iron-binding capacity (transferrin level) is high, and the percentage of saturation is low. The serum ferritin level is decreased (reflects low iron stores in the bone marrow). Therapy: Mild cases are managed with oral iron administration. Dietary counselling must be provided to caregivers to give the pt. enough of dietary iron Severe cases are managed with slow transfusion with packed RBCs. ADD B12 & FOLIC 260 127) HEART MURMUR AND CYANOSIS IN THE NEWBORN - DD AND TREATMENT. 1. Tetralogy of Fallot An anatomic abnormality with severe or total RV outflow tract obstruction and a ventricular septal defect, allowing RV unoxygenated blood to bypass the pulmonary artery and

enter the LV and aorta (the aorta is enlarged and overriding the septal defect). This is the most common cyanotic lesion, accounting for 10% of congenital heart disease. An upper left sternal border ejection murmur of RV outflow tract obstruction is heard shortly after birth, followed by gradual development of cyanosis. However, newborns with complete pulmonary obstruction (pulmonary valve atresia) and ductus arteriosus-dependent pulmonary blood flow present with severe cyanosis and a continuous murmur of ductal flow. Older infants have an ejection murmur, right axis deviation, and RV hypertrophy on ECG. CXR shows a small heart and diminished pulmonary blood flow. Pt. with tetralogy are usually small and thin The degree of cyanosis is variable The fingers show varying degrees of clubbing depending on the severity of the cyanosis. In some newborns and older infants, hypercyanotic spells (sudden onset of cyanosis) with anxiety, air hunger, respiratory distress, and altered consciousness

may occur, usually precipitated by activity. In infants with cyanosis due to ductal closure, prostaglandin E1 infusion maintains ductal flow until surgical palliation by a systemic-pulmonary anastomosis can be done. Treatment of hypercyanotic spells consists of supplementation with O2, placing the infant in a knee-chest position, and administering morphine. Cathetrization, angiography, and surgical palliation or total repair are needed urgently. 2. Pulmonary atresia with intact ventricular septum In this uncommon condition, the pulmonary valve is atretic. The ventricular septum is intact The right ventricle is reduced in size. Following birth, the pulmonary circulation is maintained by the PDA. Accordingly, a continuous infusion of PGE1 must be started as soon as possible after birth to maintain ductal patency. Pt. are cyanotic at birth Cyanosis progresses as the ductus arteriosus starts to close A blowing systolic murmur associated with the PDA is heard at the pulmonary area. In

addition, a pansystolic murmur is heard at the lower left sternal border, because most of the children develop tricuspid insufficiency. Surgery should be undertaken as soon as possible 261 3. Tricuspid atresia - relatively rare condition ( < 1%) is characterized by complete atresia of the tricuspid valve. As a result, no direct communication exists between the right atrium and the right ventricle. Tricuspid atresia may be divided into two types, depending on the relationship of the great vessels (with or without transposition of the great arteries). Because there is no communication between the right atrium and the right ventricle, the entire systemic venous return must flow through the atrial septum (either an ASD or patent foramen ovale) into the left atrium. Accordingly, the left atrium receives both the systemic venous return and the pulmonary venous return. Complete mixing occurs in the left atrium arterial desaturation The development of the right ventricle depends on

the presence of a LR shunt at the ventricular level. Therefore, severe hypoplasia of the right ventricle occurs in those forms in which there is no VSD or the VSD is small. Symptoms, signs, and diagnosis Symptoms develop early in infancy. Except in pt whose pulmonary blood flow is high, cyanosis is present at birth. Growth and development are poor, and the infant shows fatigability on feeding, tachypnea, dyspnea, anoxic spells, and right heart failure. Clubbing is present if the child is old enough. The present murmur is harsh, blowing, heard best at the lower left sternal border X-ray shows small pulmonary artery segment, enlarged right atrium, and decreased pulmonary vascular markings (except for cases with high pulmonary blood flow). ECG shows left axis deviation, right atrial hypertrophy, and low right ventricular forces. Echocardiography is diagnostic (absent tricuspid valve). Treatment In infants with high pulmonary artery flow, a Fontan procedure (connection of the systemic

venous return to the pulmonary artery) is performed at age 2-4 years (when the infant begins to outgrow the VSD and cyanosis increases). 4.Hypoplastic left heart syndrome It includes a number of conditions in which lesions of the left heart result in hypoplasia of the left ventricle. It occurs in about 3% of infants with congenital heart disease The most common lesions are mitral atresia, aortic atresia, or both. Following birth, survival depends on a PDA as there is inadequate antegrade flow into the systemic circulation and the ductus arteriosus provides flow into the aorta. Children are stable at birth, but deteriorate rapidly as the ductus starts to close with findings of CHF, followed by clinical shock with acidosis secondary to inadequate systemic perfusion. Untreated, age at death is 5-7 days 262 X-ray shows cardiac enlargement with pulmonary venous congestion. The ECG shows right axis deviation, right atrial and ventricular hypertrophy. Echocardiography is diagnostic Color

flow Doppler shows retrograde flow in the ascending aorta. Treatment and prognosis PGE1 is essential initial management. Further management depends on balancing pulmonary and systemic blood flow, because the right ventricle provides both. Therapy is directed to increase pulmonary vascular tone using hypoxia or hypercapnic ventilation. This must be done with extreme care. Surgical options are to utilize the right ventricle as the systemic ventricle while creating an aortopulmonary anastomosis for pulmonic blood flow, or listing the child for cardiac transplantation. A decision not to intervene and allow the infant to die is also an option A 5-year survival after the operation is 50% at the best centres. 5.Complete transposition of great arteries It is the second most common variety of cyanotic congenital heart disease, accounting for 5%. The male-to-female ration is 3:1. The disorder is caused by an embryonic abnormality in the spiral division of the truncus arteriosus, in which the

aorta arises from the right ventricle and the pulmonary artery from the left ventricle. In 60% of cases there is a patent foramen ovale or PDA Left unrepaired, transposition is associated with early pulmonary vascular obstructive disease. Transposition of great arteries is classified in two types (with or without VSD). Because the aorta arises directly from the right ventricle, life would not be possible unless mixing occurred between the systemic and pulmonary circulations. Oxygenated blood from the pulmonary veins must reach the systemic circulation. In pt with intact ventricular septum, mixing occurs at the atrial and also at the ductal level. However, these communications are small, and the ductus closes soon. These pt are therefore severely cyanotic VSD pt will have better mixing & less cyanosis Many of the neonates are large and severely cyanotic at birth. They have no murmur and lack respiratory distress. Only infants with a large VSD are less cyanotic and they have a

prominent murmur. CXR shows a narrow base because the great vessels are superimposed The neonatal ECG is normal. The diagnosis should be confirmed immediately by echocardiography; palliation by balloon atrial septostomy to improve atrial mixing may be necessary. In infants with severe hypoxemia, PGE1 produces ductal opening, improving pulmonary blood flow. Surgical repair by arterial switch technique with coronary artery reimplantation is performed within the first 7 days of life. 263 Total anomalous pulmonary venous return This accounts for 2% of all congenital heart lesions. The pulmonary venous blood drains into a confluence behind the left atrium but the confluence is not connected to the left atrium. The pulmonary blood therefore finds another route to rejoin the circulation via the systemic veins. This leads to complete mixing at the level of the right atrium. The clinical presentation depends on the method of return and whether that return to the systemic veins is

unrestricted or restricted. The most common form of return is unobstructed via the left superior vena cava. Because venous drainage from the body comes to the right atrium, a RL shunt is always present in the atria. This is either ASD or patent foramen ovale Because of the mixing, the left atrial and the systemic arterial saturation levels equal that of the right atrial saturation. Children with unobstructed venous return have a high pulmonary blood flow, and they present with cardiomegaly and early CHF. The children are small and have respiratory infections They do relatively well after birth. An ejection systolic murmur is heard at the pulmonary area X-ray shows right heart and pulmonary artery enlargement. ECG shows right axis deviation and right atrial and ventricular hypertrophy. Echocardiography is diagnostic Children with obstructed venous return include all pt. with subdiaphragmatic return These infants present shortly after birth with severe cyanosis. They require early

corrective surgery Echocardiography shows a small left atrium and ventricle and a vessel parallel to the descending aorta and to the left of the inferior vena cava. Color flow Doppler is diagnostic Cardiac cathetrization and angiography are diagnostic for any type of anomalous return. Treatment Surgery is always required. If there is obstructed venous return, surgery must be performed during the newborn period. In infants with unobstructed return, surgery may be delayed for weeks to mo The results of surgery are excellent except for a complication of pulmonary vein stenosis, occurring in 5% of operated children. 6.Truncus arteriosus It accounts for less than 1% of congenital heart disease. A single great artery arises from the heart giving rise to the systemic, pulmonary, and coronary circulations. A VSD is always present The number of valve leaflets varies from 2-6. The valve may be sufficient, insufficient, or stenotic In this condition, blood leaves the heart through a single common

exit. Therefore, the saturation of the blood in the pulmonary artery is the same as that in the systemic arteries. If pulmonary vascular resistance is normal, the pulmonary blood flow is greater than the systemic 264 blood flow and the saturation is relatively high. If pulmonary vascular resistance is elevated, pulmonary blood flow is reduced and oxygen saturation is low. High pulmonary blood flow is the most common presentation. Pt are acyanotic Early CHF occurs A hyperdynamic precordium, increased pulse pressure, a normal S1 with an ejection click, and a loud single S2 are characteristic. A pansystolic murmur is audible at the lower left sternal border Low pulmonary blood flow pt. have cyanosis early and do poorly The manifestations include fatigability, growth retardation, dyspnea on exertion, and CHF. S1 and S2 are single and loud A systolic murmur is heard at the lower left sternal border. ECG shows combined ventricular hypertrophy. Chest X-ray shows cardiomegaly and

increased pulmonary blood flow. Diagnosis is confirmed by echocardiography and usually requires cathetrization and angiography before surgical repair. Repair must be performed early to avoid pulmonary vascular disease 265 128) MENINGITIS. This term refers to an inflammation of the leptomeninges, which include the arachnoid membrane, the subarachnoid space (with CSF), and the pia mater covering the brain. Classification and etiology: There are two major classifications: bacterial and aseptic. These are distinguished on the basis of CSF: a) In bacterial meningitis, CSF abnormalities include increased CSF pressure, high WBC count (predominant neutrophils), an increased protein level, a lowered glucose level, and Gram’s stain, cultures, and latex agglutination positive for bacteria. The cause varies with age of the pt: 1. Neonates, the most common causes are group B strep, E coli, and other coliforms, and Listeria monocytogenes. 2. In infants and children, the most common causes are

pneumococci, meningococci and H influenzae. Rare causes include Borrelia burgdorferi (Lyme disease) and M tuberculosis b) In aseptic meningitis, the CSF does not contain bacteria. The CSF pressure is increased, it has a mildly elevated WBC count (predominant mononuclear cells), mildly elevated protein level, and a normal glucose level. The cause is usually viral The most common causes are enteroviruses (coxsackieviruses and echoviruses). Clinical features: it is (esp bacterial) a severe infection. In very young, symptoms are less specific - CNS involvement manifests as severe headache, lethargy, confusion, irritability, seizures, vomiting, and a bulging fontanelle. - Meningeal involvement manifests as neck or back pain and Brudzinski’s sign (neck flexion causes flexion of the legs) and Kernig’s sign (inability to extend the leg after the thigh is flexed to a right angle). Nuchal rigidity is a sensitive sign in children older than 12 months - Nonspecific features: fever,

irritability, poor feeding, and petechial lesions (N. meningitidis) Diagnosis: Bacterial meningitis is a medical emergency. Diagnosis must be done immediately and therapy must be prompt. The diagnosis is made from LP Bacterial cultures of the CSF are very accurate. Viral culture results are positive in less than half of pt Cultures of tissue or fluid other than CSF may help find the specific cause (e.g blood culture for bacteria or throat and rectal cultures for virus). CSF findings in bacterial and viral meningitis Therapy: a) Antimicrobial: Initial therapy (before identification of the causative agent) is based on age: - Neonates and infants < 2 months are given ampicillin (against group B strep and Listeria monocytogenes) and an aminoglycoside (against coliform bacteria) or ampicillin and cefotaxime (against group B streptococci and coliform bacteria). - Infants and children > 2 months are given ceftriaxone or cefotaxime (against S. pneumoniae and N. meningitidis) plus

vancomycin (against resistant S pneumoniae) 266 The duration of therapy is usually 2 to 3 weeks in neonates and 1 to 2 weeks in older children. b) Supportive care: - Fluid restriction to minimize cerebral edema and treatment of SIADH. - Maintenance of intravascular volume by IV fluids if shock is present. - Anticonvulsants if seizures occur. - Assisted ventilation if respiratory failure occurs. - Subdural taps for evacuation of extensive subdural effusions. c) Corticosteroid therapy for H. influenzae type b meningitis improves CSF findings and decreases the incidence of hearing loss and neurologic sequelae. d) Follow-up care: Children after meningitis should have a complete neurologic examination at the time of discharge, including a vision test, hearing test, and developmental assessment. Periodic monitoring should be carried out for at least 2 years. Meningismus This term refers to irritation of the brain and spinal cord with symptoms simulating meningitis, but

without actual inflammation (lumbar puncture findings are normal). It may be caused by: - Lesions in the head area (severe tonsillitis, lymphadenitis colli, otitis media, sinusitis acuta, retropharyngeal abscess). - SAH (see topic 30A). - It can accompany systemic illnesses (rheumatic fever, shigellosis, pneumonia, dehydration, viral infections). - Acute cerebellar hemorrhage or infarction can produce tonsillar herniation and consequent nuchal rigidity, followed by obstructive hydrocephalus. This may be confused with meningitis if fever is present. LP in such case may cause death from herniation !!! Treatment: Specific treatment according to the cause. 267 129) RENAL TUMORS. The most common malignant renal tumor of childhood is the Wilms‘ tumor. Neoplastic embryonal renal cells of the metanephros give rise to this tumor, which is composed of a mixture of cells. Epidemiology: It accounts for 6% of all childhood cancers. It occurs during the 1st 5 years of life Occurrence

is equal in males and females. Clinical: Most children with newly diagnosed Wilms tumor have localized disease and appear well. - Abdominal mass - the most common presentation. It is usually asymptomatic and detected by the mother. The mass is within the kidney and displaces & distorts the renal collecting system - Hypertension occurs in ¼ of all pt. and may be related to renin production by the tumor cells or to compression of the renal vasculature. - Hematuria is not common, if it is present, it is microscopic. - Abdominal pain occurs with hemorrhage into the tumor. - Fever and anemia may be present. Associated abnormalities that occur in a few pt. include: a) Aniridia - Children with sporadic aniridia are at increased risk for development of Wilms tumor. In many of them, deletion of chromosome 11 may be found Children with sporadic aniridia, chromosome deletion, and Wilms tumor may also have mental retardation and genitourinary tract abnormalities. b) Hemihypertrophy

Staging: Stage I: tumor localized to kidney, total resection is possible. Stage II, III: more advanced localized, surgical resection dose not result in complete tumor removal. Involvement of regional LN Stage IV: disseminated disease with hematogenous mets (lungs, liver, bone, brain) or distant nodes. Stage V: both kidneys are involved. Only 15% have stage IV at diagnosis Therapy: - Surgery: Removal of the primary tumor, LN, and selected metastases. - Radiotherapy: Treats residual local disease and selected metastases. - Chemotherapy: Varies in duration and intensity, depending on the stage and histology. Prognosis: Pt. with localized disease and favorable histology (90% of cases) have a 90% survival chance. Poor prognosis (unfavorable histology, metastases) pt have < 70% chance 268 130) HEART MURMUR IN NONCYANOTIC NEWBORN - DD AND TREATMENT. 1. Patent ductus arteriosus Definition: Failure of the duct between pulmonary artery and aorta to close. It closes spontaneously in

normal term infants at 3-5 days of age. It is a common finding in premature infants. Persistent patency of the ductus in term infants occurs in 1:2000 live births. It is more common in females In premature infants, PDA frequently occurs with increased pulmonary blood flow and further compromise of gas exchange, particularly in infants with respiratory distress syndrome. The infant has bounding pulses, a hyperdynamic precordium, and increased pulmonary closure sound. Typically, a pulmonic area murmur occurs; it may be continuous, or systolic alone. ECG is usually normal for the degree of prematurity, but LV volume load may be increased. X-ray shows cardiomegaly and an increase in pulmonary artery flow. Echocardiography images the length of the ductus and confirmation of the direction and degree of shunting. In full-term infants, PDA is usually diagnosed after age 6-8 weeks on the basis of a continuous murmur at the upper left sternal border. The peripheral pulses are full, with a

widened pulse pressure, and the ECG may reflect LV volume load. X-ray shows prominence of the left atrium, LV, and ascending aorta & an increase in the pulm blood flow if the ductus has significant flow. Treatment In premature infants with compromised respiratory status, closure should be attempted using fluid restriction, diuresis, maintenance of good oxygenation, drugs (e.g indomethacin), or surgical ligation at age 1.5 to 25 years In full-term infants, surgical ligation or transection is indicated if heart failure occurs. It should be performed at age 6 months to 3 years to remove the risk of infective endocarditis. 2. Atrial septal defect An opening in the interatrial septum. It constitutes about 10% of congenital heart disease, more common in females. There are 3 major types The most common is the ostium secundum It is in the middle of the septum in the region of the foramen ovale. The ostium primum type is low in position and is a form of atrioventricular septal defect.

Children with ASD most often have no cardiovascular symptoms. Cyanosis does not occur unless pulmonary hypertension develops (and LR shunting decreases leading to RL shunting). Some pt. develop easy fatigability as adults The typical grade 3/6 flow murmur is audible at the upper sternal border with splitting of the second heart sound in respiration. The murmur is caused by increased blood flow across the pulm valve The murmur presents at age > 1 yr when pulmonary blood flow has increased significantly. 269 No murmur is heard from the flow across the ASD. A diastolic murmur can often be heard at the left sternal border. This murmur is caused by increased flow across the tricuspid valve X-ray shows cardiomegaly with dilation of the right atrium, right ventricle, and pulmonary artery as well as increased pulmonary blood flow. ECG shows moderate right axis deviation, increased RV volume, and normal LV forces. The defect is detected by echocardiography Treatment: Surgical repair.

Small shunts are left alone, but must be observed (danger of developing pulmonary hypertension and RL shunting with systemic embolization). 3. Atrioventricular septal defects An atrioventricular septal defect is a congenital cardiac abnormality that results from incomplete fusion of the embryonic endocardial cushions. These defects account for 4% of CHD Atrioventricular septal defects are divided into incomplete and complete forms. The complete form consists of a posterior VSD, a low ASD that is continuous with the ventricular defect, and a cleft in both the septal leaflet of the tricuspid valve and the anterior leaflet of the mitral valve. In the incomplete form, any of these components may be present. The complete form results in large LR shunts, tricuspid and mitral regurgitation, and marked pulmonary HTN. The hemodynamics in the incomplete form are dependent on the lesion present Symptoms and signs: In the incomplete form, children are clinically like those with ASD. They are often

asymptomatic. Pt with the complete form are usually severely affected CHF develops in infancy and pneumonias are common. In the complete form, at age 4-6 weeks, a systolic murmur develops. The heart is enlarged (both the right and left sides). S2 is split, with an accentuated pulmonary component When severe pulmonary vascular obstructive disease is present, right ventricular enlargement is dominant. Cyanosis develops with predominant RL shunts. Diagnosis: X-ray shows enlargement of all four chambers, and increased pulmonary vascular markings. On ECG, left axis deviation is present in all forms First-degree heart block is present in 50% of pt. Ventricular hypertrophy is present Echo is the diagnostic method of choice Cardiac cathetrization and angiography are appropriate in some cases. Treatment: Surgery is required in the 1st yr of life prior to pulmonary hypertension development. 4. Ventricular septal defects One or more openings in the interventricular septum that may undergo

spontaneous closure in infancy, may lead to heart failure, may require surgical closure, and may be accompanied by pulmonary vascular disease. The incidence is 3/1000 live births 270 Symptoms and signs Small ventricular septal defects are frequently heard as loud, harsh, pansystolic murmurs at the lower left sternal border during the early months of life and are not accompanied by hemodynamic abnormality. More significant VSD are heard at age 2 to 3 weeks as pulmonary vascular resistance decreases and LR shunting increases. A loud, harsh, pansystolic murmur grade 3/6 is heard at the lower left sternal border. Infants with large shunts or unstabilized heart failure are at higher risk for severe viral or bacterial pneumonias, often needing ventilatory support. Cardiomegaly, heart failure, growth failure, or infective endocarditis may occur. Diagnosis: X-ray shows cardiomegaly, LA and LV enlargement, and increased pulmonary arterial flow. ECG findings are LV volume load, increasing

RV hypertrophy as pulmonary artery pressure rises. Cardiac cathetrization and angiocardiography determines VSD location, and pulmonary vascular resistance. However, echocardiography can give similar information Treatment: Digitalis, diuretics, salt restriction, and afterload reduction and treatment of respiratory infections may control heart failure. Heart failure usually disappears by age 1 or 2 years, as the defect becomes less significant and may not require surgery. In children with large shunts, surgical repair is done in the first few months of life. All children with VSD should receive prophylaxis against infective endocarditis. 5. Pulmonic valve stenosis Narrowing of the pulmonic valve orifice due to congenital pulmonic valve abnormalities, with restricted opening. It constitutes about 10% of congenital heart disease In newborns, pulmonic valve stenosis with severe obstruction to RV outflow may present as an emergency with RL atrial shunting. It requires immediate diagnosis: by

ECG showing diminished RV forces (when the RV is hypoplastic) or RV hypertrophy; by X-ray showing decreased pulmonary blood flow; by echocardiography, showing a severely narrowed valve with restricted movement; and by cathetrization and angiography to evaluate RV function. In older children, cyanosis is not present, although poor peripheral color may result from prolonged circulation. An ejection murmur, usually with a prominent ejection click, is found at the upper left sternal border. The pulmonic component of S2 is progressively delayed and diminished ECG shows increasingly severe RV hypertrophy. X-ray shows a prominent main pulmonary artery segment with increasingly severe obstruction. The pulmonary vascular markings are narrow Treatment In newborns, immediate creation of a systemic-pulmonary anastomosis is necessary for adequate pulmonary blood flow. Until this can be done, temporary palliation by maintaining ductal patency with prostaglandin E may be lifesaving. Pulmonary

valvotomy may also be indicated These 271 procedures are palliative; further surgery may be necessary in later childhood to improve RV outflow. The preferred approach is balloon valvuloplasty with cathetrization 6. Aortic stenosis Narrowing of the aortic valve orifice due to congenital aortic valve abnormalities, usually a bicuspid valve. It accounts for 5% of clinically apparent cardiac defects, but the true prevalence is much higher. Obstruction may be valvular, subvalvular, (subaortic), or supravalvular Supravalvular obstruction is uncommon. Severe obstruction may be accompanied by chest pain and syncope Symptoms, signs, and diagnosis Uncommonly, symptomatic valvular stenosis may occur in infancy and may produce severe LV outflow tract obstruction with severe LV dysfunction, heart failure, ischemic changes, and poor systemic output. More commonly, it presents later in childhood as a bicuspid aortic valve with stenosis and sometimes with insufficiency. There are usually no

symptoms, and an ejection systolic murmur is heard loudest at the upper right sternal border, an accentuated aortic closure sound is also heard. Infective endocarditis or endarteritis is a complication of bicuspid aortic valve Subvalvular obstruction is due to a fibrous ridge or muscular hypertrophy and it has similar findings. It is usually asymptomatic ECG shows LV hypertrophy and ischemia as LV dysfunction worsens. X-ray reflects poststenotic dilation of the ascending aorta. Evaluation by echo, cardiac cathetrization, and angiography may be necessary. Treatment Life-threatening heart failure in infancy requires immediate aortic valvotomy, either by balloon valvuloplasty or surgical valvulotomy, to prevent death. Valve replacement is required when the child has grown. Repair of subvalvular obstruction requires removal of the abnormal tissue Antibiotic prophylaxis against infective endocarditis is indicated in any pt. 272 131) ENDEMIC TYPHUS. TULARAEMIA BRUCELLOSIS 1) Endemic

typhus (Rat-flea typhus) An acute febrile disease caused by Rickettsia typhi and transmitted to humans by rat fleas. Etiology and epidemiology: The causative agent resembles other rickettsiae in morphology and in intracellular parasitism. The animal reservoir includes wild rats, mice, and other rodents Rat fleas transmit the agent to humans. The distribution is sporadic and worldwide Symptoms, signs, and prognosis: After an incubation period of 6 to 18 days - Shaking chills develops with headache & fever. Fever lasts about 12 d & gradually reduced - The early rash is sparse and discrete. Later, the rash (small pink macules in the axillae and upper trunk, excluding the face, soles, and palm, thereafter becoming dark and maculopapular) and other manifestations (headache, splenomegaly, hypotension, petechiae) are similar to those of epidemic typhus but are much less severe. Mortality is low Diagnosis: Serologic tests, isolation and identification from blood or tissues, and

identification of the agent in skin or other tissues by immunofluorescence. PCR may help in fast diagnosis Treatment: It should begin early, as the rash appears, with tetracyclines or chloramphenicol. Prophylaxis: Incidence has been decreased by reducing rat and rat-flea populations, excluding rats from food depots and residencies, trapping and rodenticides. No effective vaccine exists. 2) Tularemia Identification: A zoonotic bacterial disease presenting with a variety of clinical manifestations related to the route of introduction of the organism. Caused by Francisella tularensis, gram (-) rod - Ulceroglandular type presents as an ulcer at the site of introduction of the organism, and swelling of the regional LN. - Glandular type shows no apparent ulcer, only enlarged and painful LN that may suppurate. - Oropharyngeal type – due to ingestion of organisms in contaminated food or water may produce painful pharyngitis, abdominal pain, diarrhea and vomiting. - typhoid type –

by inhalation may cause pneumonic involvement. - Oculoglandular type – with conjunctivitis and enlarged local LN. Reservoir: Wild animals (rabbits, deer, rodents) Mode of transmission: Through the bite of arthropods, including wood ticks, dog ticks, deer flies, and mosquitoes; by inoculation of skin or oropharyngeal mucosa with contaminated water, through aerosols, contact, or ingestion of insufficiently cooked meat of infected animal hosts Incubation period: 3-5 days. Not directly transmitted from person to person Pathogenesis: Organisms localize in the reticuloendothelial cell. Endotoxin (?) 273 Treatment: Streptomycin Prevention: Live attenuated vaccine for persons in high risk occupation, protection against tick bites, wear gloves when handling animals 3) Brucellosis Identification: A systemic bacterial disease => fever, headache, weakness, sweating, chills, depression, weight loss, aching, suppurative infection of liver and spleen. Infectious agent: Brucella

abortus, melitentis, suis, canis; Gram (-) Occurrence: Worldwide, esp Mediterranean countries (Africa - north, east) Reservoir: Cattle, swine, goats, and sheep Mode of transmission: By contact with tissues, blood, urine, vaginal discharge. Aborted fetus and placentas, ingestion of raw milk of infected animals Incubation period: 5 - 60 days Pathogenesis: Organisms localizes in reticuloendothelial cells, virulence is associated with intracellular survival. Endotoxin Treatment: Tetracycline Prevention: Pasteurization of milk, and vaccination of cattle. Infection usually requires slaughter of the herd. Human vaccine is available, but it efficacy is not completely proven Persons inoculated with strain 19 should be given doxycycline as well. 274 132) FOREIGN BODY IN THE RESPIRATORY TRACT. Death from mechanical airway obstruction is most common in the first year of life. Foods and coins are the most common causes of nonfatal choking. Balls and deflated balloons, which tend to occlude the

airway, are common causes of death. Clinical findings: Symptoms may be minimal, esp if the foreign body is small. Many of the episodes are non-witnessed. In general, the younger the child, the higher the involved anatomic site - The larynx is the most common site in children <1 year. - The trachea and bronchi are most common in children 1 to 4 years. - Upper airway obstruction may cause asphyxiation and be a threat to life. It is manifested by gagging, choking, wheezing, cyanosis, and dysphonia. - Lower tract obstruction may be tolerated for longer periods, esp the more distal the obstruction. It is manifested by wheezing or asymmetric or absent breath sounds. - Generally, foreign body aspiration should be suspected in children who have sudden onset of respiratory distress associated with gagging, coughing, and stridor. Management: It consists of nonintervention if the child is can cough, breathe, or speak! A natural cough is more effective. Fingers should not be put

blindly into the mouth to search for a foreign body. This might lodge the object in the airway a) Infant: Alternating sequences of four back blows and four chest thrusts are administered until spontaneous breathing is re-established. Back blows: Straddle the infant over the rescuer’s arm, with the head below the trunk, while supporting the head. Give the blow with the heel of the hand b/w the infant’s shoulder blades Chest thrusts: Turn the infant over onto the rescuer’s thigh, with the head supported. The thrusts are performed like chest compressions. b) Child: Conscious child: Encourage the child to cough spontaneously. If the cough becomes ineffective, subdiaphragmatic thrusts (Heimlich manoeuvre) are performed until the object gets out. The rescuer is behind the victim, holding his fists against the subxiphoid region The thrusts is given as a quick upward movement. Unconscious child: Abdominal thrusts are performed with the child on the ground. c) Contraindications: Airway

clearing maneuvers should not be made if there is gradual onset of respiratory symptoms over hours, esp if these signs are associated with fever or other signs of infection. The Heimlich manoeuvre should not be performed on children < 1 yr because injury to abdominal organs is more likely in this age group. d) If necessary, rigid bronchoscopy should be performed under general anesthesia. Occasionally, thoracotomy and bronchotomy may be required. 275 133) 134) HIV INFECTION IN INFANCY AND CHILDHOOD Infection with HIV causes AIDS. It is a disorder associated with deficiency in T-cell immunity, and, in children, T-cells and B-cells abnormalities. Worldwide, over 1 million children have died as a result of AIDS since the first case was described in 1983. The number of perinatally acquired HIV has declined because of the use of zidovudine (ZDV) therapy for HIV-infected pregnant women and their newborns. However, the total number of cases in children and adolescents continues to

increase because of continued perinatal and non-perinatal transmission. Cause: HIV-1, a retrovirus, is the case of AIDS. 90% of childhood HIV occur because of perinatal transmission from the mother The risk of perinatal transmission is 15-35%. The risk is reduced by two thirds if zidovudine is given to the infected mother during the last two trimesters and to the infant during the 1st 6 weeks of life. Breast feeding can also transmit HIV to the infant Transmission via blood, blood products, or organ transplantation has diminished because of efficient screening. Adolescents require infection via sexual contact or IV drug abuse Classification (according to Centre for Disease Control): a) Category E (perinatally exposed infant) describes HIV-exposed infants in whom infection has not been confirmed or excluded. b) Category N (infected infant) describes HIV-infected infants who are not symptomatic. c) Category A (mildly symptomatic infection). Manifestations include lymphadenopathy, HSM,

parotitis, recurrent upper respiratory infections, and otitis media. d) Category B (moderately symptomatic infection). Manifestations include chronic oral candidiasis, diarrhea, failure to thrive, anemia, thrombocytopenia, lymphocytic interstitial pneumonitis, cardiomyopathy, and a variety of infectious diseases that reflect some degree of immunodeficiency, but which are not AIDS-defining conditions. e) Category C (severely symptomatic infection) include AIDS-defining conditions: 1. Recurrent, serious bacterial infections 2. Opportunistic infections include P carinii pneumonia, Mycobacterium avium complex, candidal esophagitis, disseminated cryptococcosis, toxoplasmic encephalitis, cryptosporidiosis, and disseminated TBC 3. Wasting syndrome 4. Progressive neurologic disease can be manifested by developmental delay, encephalopathy, paresis, dystonia, and peripheral neuropathy. Cerebral atrophy and intracranial calcifications may be seen on the CT scan. 276 Diagnosis: 1) Serologic

testing (ELISA and Western blot). Maternal antibodies may persist for up to 18 months in exposed infants, hence, with few exceptions, positive HIV serologic findings in such infants are not diagnostic until 18 months. 2) HIV p24 antigen or HIV RNA detection (by PCR). Not all infected infants have positive results; a negative test does not exclude HIV, but a positive test generally means infection. 3) HIV culture of blood lymphocytes. When available, this is the most reliable test Prognosis: The majority of children with HIV will eventually die from complications of AIDS. Infants with significant clinical disease in the 1st year tend to progress most rapidly. Infants remaining asymptomatic in the second year are likely to remain disease free for several yrs. Therapy: General management includes antiretroviral therapy (combinations of nucleoside and nonnucleoside reverse transcriptase inhibitors, and protease inhibitors), treatment of secondary infections, prophylaxis against PCP

(TMP-SMX), and nutritional support. Prevention: Change of behavior (education, counseling). ZDV for infected pregnant women and infants after birth. 277 135) DISORDERS OF LIPID METABOLISM (Hypercholesterolemia, hyperlipidemia) a) Primary hyperlipidemia These disorders are a heterogenous group of genetically determined diseases. The expression of the diseases may be affected by diet, obesity, diabetes, or other environmental factors. Certain types of primary hyperlipidemias predispose affected people to premature atherosclerosis and others to pancreatitis or neurologic sequelae, usually in adulthood. Identification of individuals with hyperlipidemia: It depends on the measurement of fasting total cholesterol, triglyceride, and HDL cholesterol levels. Elevated LDL cholesterol and low HDL cholesterol are associated with increased CV risk. LDL cholesterol may be calculated as: LDL = total cholesterol – (HDL cholesterol + triglyceride/5). Although triglyceride levels rise slowly with

age, cholesterol levels are relatively constant throughout childhood and adolescence. 1. Familial hypercholesterolemia (FH) - characterized by marked elevation of total and LDL cholesterol levels with normal triglyceride levels. It is inherited as an autosomal dominant trait Heterozygous FH occurs in 1:500 people. Heterozygous FH is associated with a high risk of coronary heart disease by the age of 30 to 40. There is only 50% of LDL receptor activity Homozygous FH is rare, Myocardial infarction in 1st decade of life and death by the age of 20 years are common. It is caused by near-total deficiency of LDL receptors, which prevents uptake and catabolism of LDL cholesterol. Therapy: Heterozygous form: Diet low in cholesterol and total fat and an increased polyunsaturated-to-saturated fat ratio. If it is not sufficient, cholestyramine, a bile acid-binding resin that removes cholesterol from the enterohepatic circulation, is given. Homozygous form: It is very resistant to therapy. Some

lowering of cholesterol may be achieved by portocaval shunting or liver transplantation. 2. Familial combined hyperlipidemia (elevated cholesterol or triglycerides, or both) This is the most common inherited hyperlipidemia in children and is dominantly inherited and variably expressed in family members. Adults have premature coronary artery disease and peripheral vascular disease. Treatment is the same as for FH 3. Mild-to-moderate hypertriglyceridemia This may be an early manifestation of familial combined hyperlipidemia. In adults, it is associated with obesity, glucose intolerance, and hyperuricemia, and it may be aggravated by alcohol and some drugs. 278 4. Severe hypertriglyceridemia It may be caused by familial deficiency of lipoprotein lipase, which results in hyperchylomicronemia. This may result in recurrent pancreatitis, HSM, and eruptive xanthomas. Treatment consists of severe restriction of dietary fat Severe hypertriglyceridemia 2o to elevation of both chylomicron

& VLDL levels is rare in children. b) Secondary hyperlipidemia Hypercholesterolemia may be secondary to hypothyroidism, nephrotic sy, diabetes, and liver disease, or it may be induced by a diet high in saturated fat and cholesterol. Elevated triglyceride levels may be present in poorly controlled diabetes, obesity, glycogen storage disease, and renal failure, or it may be secondary to use of certain drugs (oral contraceptives, thiazides diuretics). Treatment is directed against the primary cause or removal of involved drugs 279 136) SUDDEN INFANT DEATH SYNDROME (SIDS) – DD AND MANAGEMENT Definition: The onset of unexpected death of any infant or young child in which a thorough postmortem examination fails to show an adequate cause. SIDS is the most common cause of death (30%) between 2 weeks and 1 year of age. Distribution is worldwide, occurring in 1.5/1000 live birth in the USA Incidence is increased in the cold months, in lower socioeconomic groups, in premature infants,

in infants who experienced episodes of severe apnea during resuscitation, in siblings of SIDS victims, and in infants born to mothers who smoke during pregnancy. Almost all SIDS deaths occur when the infant is thought to be sleeping Etiology, Diagnosis, Prevention The cause is unknown, although it is most likely due to dysfunction of neural cardiorespiratory control mechanisms. The dysfunction may be only intermittent or transient The connection between SIDS and recurrent prolonged apnea is very small. Many studies link a prone sleeping position with increased risk of SIDS. Other factors include soft bedding (lamb’s wool), waterbed mattresses, smoking in the home, and an overheated environment. The diagnosis cannot be made without adequate autopsy to rule out other causes. It is recommended that infants be placed supine for sleep. Every effort should me made to avoid an overheated environment, to avoid overwrapping the infant, to remove soft beddings, and to avoid smoking during and

after pregnancy. Management Parents who have lost a child to SIDS are unprepared for the tragedy; because no definitive cause can be found, they usually have excessive guilt feelings. Family members require support, it is necessary to help them with their grief and to inform and counsel the parents concerning the cause of death. Autopsy should be performed quickly to exclude other causes sudden unexpected death. 280 137) VASCULITIS (E.G HENOCH-SCHÖNLEIN PURPURA) The vasculitis syndromes are a group of disorders related to inflammation of blood vessels. Clinical pictures depend on the size and site of involved vessels. 1. Henoch-Schönlein purpura A disease of unknown origin characterized by colicky abdominal pain, purpuric rash, and arthritis. Renal involvement characterized by hematuria, proteinuria & HTN is commonly associated. It affects any age group, but is most common between 6 mo and 7 yr. It is more common in males More cases occur in the spring and fall, following an

URT infection, which suggests that an infection may play a role and the disease may be immune-mediated. Clinical features: - Rash - initially urticarial and later papular and purpuric. These lesions occur on the extensor surfaces of the lower extremities and on the buttocks and hands. - Colicky abdominal pain may be associated with GI hemorrhage. - Some pt. (usually those with gross hematuria) may develop permanent renal damage Diagnosis: The purpuric rash is essential for the diagnosis, with joint, renal, and GI symptoms being manifest either before, during or after the appearance of the skin rash. Treatment, prognosis: Supportive. The overall prognosis is good 2. Kawasaki disease (mucocutaneous LN syndrome) It is a clinical syndrome of unknown etiology characterized by panvasculitis. There’s high fever, conjunctivitis, dermatitis, arthritis, and later, aneurysms of coronary vessels. Etiology: The syndrome is a reaction to something, perhaps the toxin of an unknown organism.

There are abnormal immunologic findings during the course of the disease. Clinical findings: It is most common in children < 5 yr from upper socioeconomic groups. - There is sudden onset of high fever and a generalized skin rash that may resemble scarlet fever or a drug reaction, and sometimes involves palms and soles. - Desquamation of skin of digits is common during recovery. - Edema of the extremities may be marked. - Lips and mucous membranes are inflamed, and the tongue has a strawberry appearance. - Cervical lymphadenopathy may be prominent. Fever may be present for few weeks - Several weeks after, coronary artery aneurysms appear in 20%. Carditis occurs in 40% - Death occurs in 1% (from cardiac complic.) Diagnosis: From clinical picture. No single test is available Platelet count may be high, acute phase reactants are high, immunologic response is abnormal. Treatment: Aspirin (high dose), gama-globulin. There is no method of prevention 281 3. Rare

vasculitis syndromes in childhood Polyarteritis nodosa, PAN A serious illness. It affects medium sized vessels throughout the body Manifestations include: - Fever, skin rashes, arthralgias or arthritis. - Nodular lesions, abdominal pain, renal disease, hypertension, - CNS, PNS, cardiac, pulmonary, and ocular disease. Diagnosis depends on biopsy. It probably follows some infection, mediated by an immune complex Wegener granulomatosis There’s systemic necrotizing vasculitis is with necrotizing granulomas of the upper and lower respiratory tracts chronic rhinitis & sinusitis, nasal perforation, or chronic pulmonary disease may follow. Renal involvement is present Diagnosis: biopsy. Untreated, both diseases are progressive and fatal Treatment: cyclophosphamide or corticosteroids. Behcet syndrome Vasculitis involving oral & genital mucosa, associated with iridocyclitis, joint (knee) involvement, fever and erythema nodosum. GI symptoms may also occur Cause is unknown It is most

common in the Middle East and Greece, esp at age 20, but can occur in younger pt. More common in males. Treatment: corticosteroids. 282 138) METABOLIC DERANGEMENTS (ACIDOSIS, ALKALOSIS) 1. Acidosis a) Respiratory acidosis Causes: due to accumulation of CO2 as the result of pulmonary hypoventilation. May occur with any cause of respiratory failure, including pulmonary disease, neuromuscular & CNS depression. Compensation: by renal conservation of HCO3 and increased excretion of H+. Therapy: directed at restoring adequate ventilation. Alkalinizing agents should not be used b) Metabolic acidosis Causes: due to accumulation of acid or extensive loss of HCO3. 1. Accumulation of acid occurs with ingestion of acid (eg salicylate intoxication, which also causes respiratory alkalosis by stimulating central ventilation). 2. Excess production of acid (eg lactic acidosis, diabetic ketoacidosis) 3. Decreased excretion of acid (eg renal failure) These forms of acidosis usually have a wide

anion gap. 4. Excess loss of HCO3 commonly occurs with diarrhea It may also occur in renal disease Metabolic acidosis resulting from HCO3 loss usually has a normal anion gap and is called hyperchloremic metabolic acidosis. Compensation: by increased respiratory minute ventilation (hyperventilation), leading to reduction in PCO2 and returning pH to normal. Therapy: In mild-to-moderate cases, in which respiratory compensation has occurred and renal function is normal, therapy is directed at the underlying course. Although controversial, in severe acidemia (pH < 7.2), alkali therapy may be given to increase the serum HCO3 concentration and decrease the energy expended by compensatory respiratory effort. The amount of HCO3 given should correct the pH to 7.2; it is essential to avoid fully correcting the base deficit, which places the pt. at risk for overshoot alkalosis Alkali should never be given unless adequate ventilation can be ensured. Renal tubular acidosis A heterogenous group of

disorders, all of which are characterized by hyperchloremic metabolic acidosis and tubular dysfunction, but usually not renal insufficiency. Children with RTA may have growth failure and episodes of vomiting and dehydration. 3 types: 1) Distal RTA (type I) is characterized by failure of the distal nephron to secrete dietary acid necessary to maintain acid-base homeostasis. The urine cannot be maximally acidified and new HCO3 generated. Chronic positive hydrogen ion imbalance results in buffering by bone This leads to increased skeletal Ca resorption, hypercalciuria & increased risk of nephrocalcinosis and stones. 283 2) Proximal renal tubular acidosis (type II) characterized by decreased proximal tubular reabsorption of HCO3. It results in marked HCO3 leak, which disappears when the serum HCO3 falls below the threshold level (15mmol/L). The defect may be isolated or may occur with other proximal tubular abnormalities, such as glycosuria, aminoaciduria, or depressed phosphate

reabsorption. Diffuse proximal tubular dysfunction is termed Fanconi’s sy. 3) Type IV RTA is a group of disorders, all of which are characterized by defects in distal tubular H+ and potassium secretion, leading to hyperchloremic metabolic acidosis and hyperkalemia. Therapy: RTA is treated by alkalinizing agents. Adequate therapy restores normal growth Doses of either HCO3 or citrate must be sufficient to correct the acidosis completely. 2. Alkalosis Due to any process that increases the body’s pH via either reduction in PCO2 or increase in HCO3. a) Respiratory alkalosis Causes: caused by the excessive loss of CO2 as the result of hyperventilation, which is usually centrally mediated (e.g salicylate intoxication, head injury, hysteria) Compensation: The attempt at renal compensation is through increased HCO3 excretion. Therapy: It is directed at the cause of the hyperventilation. b) Metabolic alkalosis Causes and compensation: caused by loss of H+ or increase in base. Compensation

is by a small and unpredictable decrease in minute volume to allow the PaCO2 to rise slightly. The most common cause in children is the use of diuretics, which leads to volume contraction and potassium and chloride depletion. These changes, in turn, lead to increased HCO3 reabsorption and aldosteronism, with increased hydrogen ion secretion. Volume contraction with significant chloride and potassium loss as a result of recurrent vomiting is another common cause. A gain of base can be the result of excessive alkali administration Less common causes include Bartter’s sy, chronic steroid administration, dietary chloride deficiency, chronic potassium depletion, and posthypercapneic states. Therapy: It is treated by restoring intravascular volume and replacing potassium and chloride deficits. Correction of the underlying cause (eg surgery for pyloric stenosis, discontinuing diuretic therapy) is essential. The use of acid to correct alkalosis through infusion of ammonium chloride or dilute

HCl is rarely indicated and should be reserved for severe cases. 284 139) DISEASES WITH HEMOLYSIS Hemolytic anemias are caused either by intrinsic defects of the red blood cells (intracorpuscular) or factors extrinsic to red blood cell (extracorpuscular). In general, intracorpuscular defects are hereditary and extracorpuscular defects are acquired. Hemolytic anemias associated with extracorpuscular defects: a. AutoAb or isoantibodies Ab-mediated hemolytic anemias The 2 major types include: autoimmune hemolytic anemias that are the result of Ab generated by an individual’s immune system against his own RBCs, and isoimmune hemolytic anemias that result from Ab produced by one individual against RBC of another of the same species. Typical antibodies involved include: IgG class that are usually warm-reactive (they have ,maximal activity at 37 ºC). Hemolysis occurs extravascularly due to trapping of opsonized RBC by macrophages in the spleen and other RES organs. IgG are associated

with autoimmune diseases, lymphomas, viral infections. Sometimes, there is no underlying cause These antibodies are identified by the Coombs’ test. IgM are usually cold reactive (maximal activity at cold temperatures). They agglutinate RBC and activate the complement, causing lysis of RBC Hemolysis occurs intravascularly. IgM are associated with mycoplasmal pneumonia, EBV, and transfusion reactions. Autoimmune hemolytic anemias: It may be idiopathic or the result of infectious agents, drugs, lymphoid neoplasm, or disorders of immune regulation (SLE, agammaglobulinemia). Therapy depend on the cause and clinical conditions. Because most cases of childhood autoimmune hemolytic anemia are idiopathic or postinfectious and self-limited, supportive care (bed rest and oxygen) or transfusions and corticosteroids are the most common therapies. Splenectomy or immunosuppressives are indicated in severe cases Isoimmune hemolytic anemias are seen in hemolytic disease of the newborn. Isoimmune

hemolytic anemias are often associated with hemolytic transfusion reactions (e.g transfusion of group A blood into an individual with type B blood). b. Microangiopathy Microangiopathic hemolytic anemias In these conditions, RBC suffer mechanical damage due to irregularities in the vascular endothelium (severe HTN, CRF, artificial heart valves, HUS, giant hemangioma, or DIC. The resulting hemolytic anemia occurs because of RBC fragmentation in diseased small vessels. c. Toxic chemicals d. Infectious agents 285 Hemolytic anemias associated with intracorpuscular defects These defects are in the membrane, hemoglobin, or enzymes. These disorders are hereditary with the exception of paroxysmal nocturnal hemoglobinuria. 1. Membrane defects hereditary spherocytosis, hereditary eliptocytosis (abnormally shaped RBC with variable RBC destruction & hemolytic anemia) & NPH. - Hereditary spherocytosis - an autosomal dominant disorder with a defect in the major protein of the RBC

membrane. The defect leads to membrane fragmentation and formation of small spherical RBC (microspherocytes). Microspherocytes cannot be deformed and therefore, they have difficulty in traversing small blood vessels. The spleen has the smallest vessels in the body, thus, the rigid microspherocytes are trapped in its microvasculature. Clinical features: It manifests in the newborn as jaundice, which is sometimes so severe that it requires exchange transfusion. Infants and children have pallor and splenomegaly After a viral infection, there may be aplastic crisis. Gallstones may develop in teenagers Physical examination usually reveals pallor, icterus, and mild splenomegaly. Mild anemia & reticulocytosis. Diagnosis: confirmed by increased osmotic fragility Therapy: Folic acid supplementation (to meet the needs from increased RBC turnover) and RBC transfusion (during aplastic crises). Definitive therapy is splenectomy, which makes the symptoms milder. However, the defective cells

persists after splenectomy Because the asplenic pt. is susceptible to sepsis caused by encapsulated gram-positive cocci, special precautions should be taken (e.g splenectomy not < 6 yr, immunization against pneumococci, meningococci, and H. influenzae before splenectomy, daily penicillin prophylaxis after splenectomy. All febrile illnesses should be treated as potential sepsis) - Paroxysmal nocturnal hemoglobinuria - an uncommon, acquired membrane disorder. RBCs are unusually sensitive to complement. Hemolysis is maximal during sleep There are attacks of hemoglobinuria, which usually occur at night. Therapy is symptomatic When the anemia is severe, transfusion of packed RBCs (which should be washed to remove complement) is indicated. 2. Enzymopathies They involve either the glycolytic pathway (pyruvate kinase deficiency) or the hexose monophosphate shunt (glucose-6-phosphate dehydrogenase (G6PD) deficiency. - Pyruvate kinase deficiency It is autosomal recessive and clinically

heterogenous. There is inadequate production of ATP in the RBCs = not enough energy that results in loss of potassium and gain in sodium. 286 Reticulocytes with their increased metabolic demands are particularly vulnerable. Reticulocyte count is low in comparison to other hemolytic anemias. - Glucose-6-phosphate dehydrogenase (G6PD) deficiency The most common RBC metabolic disorder. It is X-linked The cells do not generate enough of reduced gluthathione that protects the RBC from oxidant agents. Hemoglobin is oxidized hemoglobin damage follows Damaged cells are removed by the RES system or lyse intravascularly. Most common is an episodic hemolytic anemia that is usually drug-induced (sulfonamides, salicylates – oxidizing agents). Pt should avoid these drugs 3. Hemoglobinopathies They result from a qualitative change in the structure of one of the globin chains. - HbS disorders It is the most common cause of hemolytic anemia in the African-American population. The molecular

defect results from an abnormal autosomal gene that substitutes one amino acid with another in the beta-globin chain. This results in an unusual solubility in the deoxygenated state. Under hypoxia, the hemoglobin aggregates into long polymers, which distort the RBC into a sickle shape. The consequences are shortened RBC survival and microvascular obstruction, which leads to tissue ischemia and infarction. Heterozygots: 10% are heterozygots – they have both the normal Hb A and the abnormal Hb S. It is usually asymptomatic, unless there is hypoxemic stress. There are usually no sickle cells on peripheral smear. It may be diagnosed by hemoglobin electrophoresis or solubility tests It is important to detect it for purposes of genetic counseling. No specific treatment is needed However, deep hypothermia, hypoxia associated with severe pneumonia, exercise at high altitudes, or tourniquet surgery should be avoided. Homozygots: The high levels of HbF during fetal life and the first few

months of postnatal life protect the pt. The earliest manifestations occur at 4-6 months (symmetric, painful swelling of the hands and feet – this is caused by avascular necrosis of the bones). Progressive anemia with jaundice and splenomegaly begins to develop. There are 2 major life-threatening problems: - Splenic sequestration crisis (sudden strong trapping of the RBCs by the spleen that may lead to hypovolemic shock and death. - Overwhelming infection: The spleen is not sufficient in filtering the blood-born microorganisms and severe infections may develop (sepsis, osteomyelitis). - Aplastic crises can occur at any age after erythropoiesis suppression following a viral infection such as parvovirus B19. 287 - Vaso-occlusive episodes can involve any tissue. It can produce abdominal pain, bone pain, cerebrovascular accident, pulmonary infarction, hepathopathy, or hematuria. The episodes are precipitated by infection, dehydration, chilling, stasis, or acidosis. - Late

manifestations: In the 20s the pt. has already long-term consequences of chronic anemia, tissue hemosiderosis, and tissue infarction (CHF, renal damage, leg ulcers, etc.) Therapy: These pt. have functional asplenia, therefore, the same precautions as in spherocytosis are needed. Pre3vention of vaso-occlusive episodes includes avoidance of de3hydration, hypoxia, chilling, and acidosis. Treatment: analgesics for pain, transfusions, and use of agents elevating the HbF level (hydroxyurea). - Thalassemias These hereditary hemolytic anemias are characterized by decreased synthesis of one or more globin subunits of the hemoglobin molecule. Alfa-thalassemia results from reduced synthesis of alfa-globin chains, and beta-thalassemia from beta-globin chains. It is common in Mediterranean, southeast Asians, and Africans. Only some types produce hemolytic anemia, others produce hypochromic, microcytic anemias. 288 140) FUNGAL INFECTIONS There are 3 major categories of fungal infections: -

superficial (caused by Candida, Dermatophyte, Malassezia – see topic 33B). - systemic occurring in a normal host (caused by Coccidioides, Histoplasma, and Blastomyces). - systemic that are opportunistic infections (caused by Candida, Pneumocystitis, Aspergillus, Malassezia, and Cryptococcus). Immunosuppression, foreign bodies (e.g central catheter), ulcerations of GI and respiratory mucosae, broad-spectrum antimicrobial therapy & HIV are major risk for opportunistic infections. Lab dg. may be difficult because of small number of fungi present in the lesions, slow growth, and difficulty in distinguishing colonization from infection. Tissue biopsy with fungal stains and culture is the best method. Blood cultures may be negative. Serology and antigen detection are useful in some cases only a) Blastomycosis - transmission by inhalation of spores. Severe disease is more common in adults Primary infection is unrecognized or produces pneumonia. Infection is usually self-limited in

normal pt. Chronic pulmonary disease may develop Cutaneous manifestations usually represent disseminated disease. Slowly progressing ulcerative nodules are typical There may also be bone, CNS, and genitourinary involvement. Diagnosis: isolation of the fungus from sputum or skin lesions. Ab test is available Life-threatening infections are treated with amphotericin B Itraconazole is suggested for less severe infections. b) Coccidiodomycosis – infection results from inhalation or inoculation. Even travel to an endemic area (some parts of USA) may allow infection. Human-to-human transmission does not occur More than 1/2 of infections are asymptomatic. Dissemination is rare Clinical features: Symptoms vary from mild fever and arthralgia to severe influenza-like illness with high fever, nonproductive cough, pleurisy, myalgias, headache, and anorexia. Signs vary from none to rash, rales, hilar adenopathy and pulmonary consolidation. Chronic pulmonary disease may develop. Skin disease

usually manifests as maculopapular eruption Lab findings: Direct examination from respiratory secretions, pus, spinal fluid, or tissue + culture. Routine laboratory tests are nonspecific. Ab tests are available Treatment: Mild pulmonary infection requires no therapy. Amphotericin is used for severe cases Itraconazole or fluconazole is preferred for less severe disease. Excision of pulmonary cavities or abscess may be needed. Most pt recover Disseminated disease may be fatal 289 c) Histoplasmosis - Endemic infections are common at all ages and are usually asymptomatic. Over 2/3 of children are infected in endemic areas (some parts of USA). Reactivation is very rare in children; it may occur years later usually due to immunosuppression. Infection is acquired by inhalation of spores. Clinical features: 50% of infections are asymptomatic and are diagnosed by the presence of calcifications in lungs and spleen and a positive skin test. Pneumonia (45% of presentation) – it is usually

mild or moderate. 5% of pt have disseminated disease Lab findings: Diagnosis is made by demonstrating the organism by histology or culture. Detection of histoplasmal antigen is the most sensitive test. Antibodies may be also detected Treatment: Mild infections do not require therapy. Disseminated disease responds to amphotericin B. Chronically immunosuppressed pt require lifelong azole therapy d) Candidiasis - Disease due to Candida is usually caused by Candida albicans. It is normally present in small numbers on skin and mucous membranes. Normal bacterial flora and normal immunologic barrier are factors preventing Candida invasions. Disseminated infection is almost always due to prolonged broad-spectrum ATBs therapy, instrumentation, or immunosuppression. Pt. with DM are esp prone to superficial Candida infection; thrush and vaginitis are most common Clinical features: - Oral candidiasis presents as adherent white plaques with mucosal ulceration. Lesions may be few and asymptomatic

or extensive, extending into the esophagus. Thrush is very common in normal infants in the 1st weeks of life; it may last weeks despite topical therapy. Spontaneous thrush in older children is unusual. Steroid inhalation for asthma predisposes to thrush Infection with HIV should be suspected if there is no reason for oral thrush. - Skin infection: - Diaper dermatitis – see topic 33B. - Vulvovaginitis is seen in sexually active girls or diabetics, and occurs at any age in girls receiving ATBs. Thick, cheesy discharge with intense pruritus is typical The vagina and labia are erythematous. - Cutaneous dissemination with scattered red papules or nodules. - Paronychia and onychomycosis occur in normal children, but are often associated with immunosuppression or hypoparathyroidism. - Enteric infections – Esophageal involvement in immunocompromised is most common. It is manifested by substernal pain, dysphagia, painful swallowing, and A/N/V. 290 - Pulmonary infection:

Candida is normally present in respiratory tract secretions, therefore, tissue invasion demonstration is needed to diagnose tracheitis or pneumonia. It is rare except for immunocompromised and intubated for long time. - Renal infection: Candida is associated with instrumentation or anatomic abnormalities. - Endocarditis, myocarditis, meningitis, and osteomyelitis are seen in immunocompromised. - Disseminated candidiasis: It usually mimics bacterial sepsis but it does not respond to ATBs. It is common in neonates, esp premature. Lab findings: Yeast cells are seen in scrapings or other samples. Pseudohyphae are suggestive of invasion. Culture is definitive Blood cultures may take few days until results Treatment: Oral candidiasis is treated by oral nystatin suspension. Oral ketoconazole or fluconazole are effective in older children with Candida not responding to nystatin. Discontinuation of ATBs and corticosteroids is advised when possible. Skin infection: It responds t5o a

lotion or cream containing nystatin, amphotericin B, or imidazole. Another approach is to keep the area dry; a heat lamp and nystatin powder may be used. Vaginal infection is treated with clotrimazole, miconazole, or nystatin suppositories or creams, usually applied once for 7-14 days. Candida balanitis in sexual partners should be treated Oral azole therapy is also effective. Renal infection: Oral flucytosine or fluconazole. Removal of an indwelling catheter is necessary Renal abscess requires amphotericin and surgical debridement. Systemic infection: It is dangerous and resistant to therapy. Removal of all infected tissues (such as heart valve, central catheter) are necessary. Optimal treatment consists of amphotericin B infusion for at least 4nweeks. Correcting predisposing factors is important (eg discontinuing ATBs and immunosuppressives, and improving control of DM). Addition of flucytosine may help If amphotericin B is not tolerated (it is quite toxic), fluconazole is an

alternative. e) Cryptococcosis - Cryptococcus neoformans is a soil yeast. Inhalation cause the infection Infections are quite rare in children, even in heavily immunocompromised such as with HIV. Clinical findings: Meningitis is the most common clinical form following dissemination from a pulmonary focus. Symptoms of pulmonary disease are subacute and nonspecific Skin, bones, or other organs involvement may follow dissemination. Lab findings: The spinal fluid has lymphocytic pleocytosis. Direct microscopy reveals the organism in sputum or spinal fluid. The capsular antigen may be detected Treatment: Amphotericin B and flucytosine for systemic infections. Therapy is for 6 weeks Fluconazole is an alternative. Relapses are common in immunosuppressed 291 f) Pneumocystis carinii infection - Pneumonia caused by this organism is associated with immunosuppression. It is found in 60% of AIDS pt After total body irradiation, it is so common that prophylaxis with

trimethoprim-sulfamethoxazole is often prescribed. Immunocompetent infants are also at risk in the first weeks of life. Onset in preterm infants occurs between 2 and 12 weeks, with the major findings cough and tachypnea. The course is 15-50 days when treated. Coinfection with pathogenic viruses is common Clinical features: Pt. are usually afebrile The chest X-ray sometimes looks worse than the pt Lab findings include elevated IgM, with mild leukocytosis and sometimes eosinophilia. 2 to 5 mo after onset, IgG antibodies are present. Lung lavage or lung aspirate may reveal the organism It can be seen with suitable stains. In some pt, biopsy is preferred Treatment: Pt. after transplantation, with HIV, or who have had one prior episode of PCP are candidates for prophylaxis (trimethoprim-sulfamethoxazole, pentamidine, dapson). TMP-SMX for 12-14 days is effective to treat the infection. If the response is poor, pentamidine IV recommended for 20 days. Mortality is 100% if untreated With

treatment, it is 25-50% as long as the host is immunocompromised. Complete recovery is possible 292 141) PARENTERAL NUTRITION. FLUID REPLACEMENT a) Parenteral nutrition Indications: It should be considered when there is uncertainty about the adequacy of GI motility, there is evidence of severe malabsorption, or a period of at least 1 week without regular nutrition. Perioperative parenteral nutrition is used in adults to prevent the catabolic state induced by surgery. The parenteral nutrition provides protein in the form if free amino acid solutions, dextrose as the carbohydrate source, and fat emulsion to increase the calorie load + provide essential fatty acids. - Amino acids: These are provided as solutions of essential and nonessential aminoacids. - Glucose: This is the basic component. It is readily available and well tolerated It also improves nitrogen balance, by sparing amino acids used in gluconeogenesis. However, premature neonates and critically ill pt. may have

decreased sensitivity to insulin, and thus, less ability to utilize large amounts of infused glucose. Insulin is not recommended for small infants, since it may cause hypoglycemia. Osmolarity of the solutions and volume restrictions are limiting factors for glucose supply to small neonates. - Fat: Fat solutions are a good way to provide calories. It can be given via peripheral vein It is a good source of essential fatty acids. However, there are some problems: - Lipoprotein lipase activity may not be sufficient and fat emulsions are slowly metabolized, building up in the circulation and taken up by macrophages in the liver, lung & other organs. - The high oxidative rate caused by fat may reduce respiratory function of premature infants. - Lipids also inhibit glucose oxidation resulting in hyperglycemia. To minimize these risks, fat should be given in low doses, increased gradually, and monitored. - Micronutrients: Vitamins and minerals are added to PN shortly before use

(some vitamins are stable for short time only). There are commercially available multivitamin preparations Complications: They are related to catheter placement (mechanical) or contamination (septic) and to the metabolic effects of the components. Care of the central line is one of the most important tasks Many catheter infections can be treated without removing the line, but no clinical response to antimicrobial therapy or isolation of Candida are indications for immediate line removal. PN cholestasis results from poorly understood factors, but its incidence is higher in low birthweight, asphyxiated, or stressed neonates receiving PN for long time. Intermittent PN and administration of even small amounts of enteral formula to stimulate bile flow are preventive. Bone demineralization occurs in small neonates fed with PN for long time. It is not known whether calcium, phosphorus, or vitamin D is the main etiologic factor. Drugs interacting with nutrients, affecting the bioavailability,

metabolism, or utilization, also complicate PN. b) Fluid replacement (see topic 34C) 293 142) ATOPIC DISEASES (RHINOCONJUNCTIVITIS ALLERGICA, ATOPIC DERMATITIS). Allergic disorders are among the most common problems seen by pediatricians, affecting over 25% of the population in developed countries. The prevalence has increased recently The allergic cascade - allergic responses begin with sensitization to an allergen in a genetically susceptible host. The CD4 cells of allergic pt respond to allergens by producing IL-3, 4, 5, and 13 These cytokines are involved in IgE synthesis and in the differentiation and activation of eosinophils. Allergen-specific IgE binds to receptors on mast cells, basophils, and Langerhan’s cells distributed throughout the skin and respiratory tract and in the circulation. Cross-linking of receptor-bound IgE by allergen activates effector cells to release allergic mediators. This immediate allergic reaction can cause sneezing, hives, vomiting, or

anaphylaxis As part of the allergic cascade, mononuclear cells and eosinophils are recruited to the site of the acute reaction and can cause late phase responses. It is likely that a number of infiltrating cells such as monocytes, neutrophils, and basophils and resident cells such as epithelial cells and fibroblasts participate in this inflammatory process even in the absence of allergens. Atopic dermatitis It is a chronically relapsing inflammatory skin disease typically associated with respiratory allergy (asthma and allergic rhinitis). Atopic dermatitis typically presents in early childhood, with onset rarely before age 2 months but prior to age 5 years. Exotoxins such as enterotoxins and TSST-1 secreted by S aureus, can act as superantigens, contributing to atopic dermatitis. These pt. make specific IgE antibodies directed against the toxins found on their skin Symptoms and signs - Acute atopic dermatitis is characterized by intensely pruritic, erythematous papules associated

with excoriations, vesiculations, and serous exsudate. - Subacute atopic dermatitis is characterized by erythematous, excoriated, scaling papules; - chronic atopic dermatitis by thickened skin with accentuated markings (lichenification) and fibrotic papules. Pt may have all three types together - During infancy, atopic dermatitis involves primarily the face, scalp, and extensor surfaces of the extremities. In older pt, the flexural folds of extremities are the predominant location of lesions Lab findings Identification of allergens involves careful history taking, skin prick tests, or in vitro tests. IgE levels are usually elevated. Many pt have positive tests with food and inhalant allergens Some pt. have elevated specific IgE to the yeast Pityrosporum ovale and to the dermatophyte Trichophyton rubrum. 294 Complications atopic keratoconjunctivitis & increased susceptibility to infections. Treatment 1. General measures Avoidance of irritants such as detergents,

chemicals, and certain foods, as well as extremes of temperature and humidity is important in managing this disease. 2. Hydration Hydration combined with moisturizers will help skin healing and reduce the need for topical corticosteroids. 3. Corticosteroids – only topical and the least toxic (weaker) 4. Tar preparations 5. Wet dressings 6. Anti-infective therapy – for suppurative infections S aureus (antibiotics), viruses (oral acyclovir), or P. ovale (topical antifungal agents) 7. Antihistamines – to reduce pruritus Prognosis - most children outgrow it by adolescence, but relapses are common. Allergic conjunctivitis Usually caused by inhalant allergens. Pt often have other allergies (rhinitis, eczema, asthma) Clinical features include tearing, itching, edema, and redness of the conjunctiva. The cornea is not involved; thus, no scarring occurs. Therapy: drugs (ocular decongestant-antihistamines, oral antihistamines, topical mast cell stabilizers, or topical NSAIDs. Vernal

conjunctivitis A severe bilateral inflammatory disorder occurring mainly in the spring and summer in preadolescent boys. The disease tends to resolve after puberty Clinical features include intense itching, tearing, photophobia, and an ocular mucous discharge containing many eosinophils. Unlike allergic conjunctivitis, vernal conjunctivitis can result in corneal damage with ulceration and scarring. Therapy is with topical lodoxamide (mast cell stabilizer) and corticosteroids. 295 Allergic rhinitis Pathophysiology: Itching, sneezing, and hypersecretions are related to the effects of histamine. Mast cells in the nasal mucosa also regulate the local blood flow in the mucosa by release of vasoactive mediators. Parasympathetic pathways also play a role in hypersecretion and congestion Similar to asthma, both early and late-phase inflammatory processes occur. Classification and clinical features: a) Seasonal allergic rhinitis: nasal symptoms include congestion; pruritus; and increased

thin, watery, clear mucous secretions (rhinorrhea). Other symptoms include loss of the senses of smell and taste, chronic cough and clearing of the throat due to postnasal discharge, and chronic malaise and fatigue. Epistaxis, nasal or sinus polyps, and persistent serous otitis media and sinusitis may also be problems. The nasal turbinates are edematous and pale, they are covered with a thin, clear secretion. Children have a typical facial feature such as dark shadows under the eyes and dental malocclusion. Other findings include conjunctivitis, nasal polyps, and sinusitis. Nasal mucosal scrapings show large amounts of eosinophils b) Chronic or perennial rhinitis may have an allergic or nonallergic basis. Vasomotor rhinitis results from local autonomic imbalance. Inhalant or airborne allergens: Pollens, molds and fungi, household dust, cockroach, animal allergens (dander, hairs, dried saliva, and feathers). Cats are the most highly allergenic of the common household pets Contributing

factors: Although odors and fumes (tobacco smoke, perfumes) are not allergens, they are primary irritants of the mucosal epithelium. Other contributing factors include weather conditions, temperature changes, infections, air pollution, endocrine hormone imbalance, stress. Therapy: a) Avoidance: This is the most direct and safest mode. Even after removal of a pet, it may take 20 to 24 weeks before the allergen levels are reduced, even with extensive cleaning. b) Drugs: Antihistamines control rhinorrhea and pruritus. First-generation antihistamines are sedative Second-generation antihistamines are not sedative. Decongestants reduce congestion. Cromolyn (topical) blocks mediator release from mast cells, for both early & late-phases. Corticosteroids (topical sprays) affect late-phase allergic reaction. Atropine-like drugs (topical sprays) treat vasomotor rhinitis. c) Desensitization (immunotherapy) - useful in seasonal allergic rhinitis caused by inhalant pollen. 296 143)

OBSTRUCTIVE UROPATHY This term refers to structural or functional changes in the urinary tract that block normal urine flow, sometimes leading to renal dysfunction (obstructive nephropathy). It is common at all ages Etiology & pathogenesis: may be acute or chronic, partial or complete, and unilateral or bilateral. May occur at any level from the renal tubules (casts, crystals) to the ext. uretheral meatus and may result in increased intraluminal pressure, urinary stasis, UTI, and calculi. In children congenital anomalies of the urinary tract are usually the cause. Other causes include stenosis, valves, calculi, tumors, colliculus seminalis hypertrophy, or neurogenic bladder. Obstructive nephropathy (renal insufficiency, renal failure, or tubulointerstitial damage) may be due to increased intratubular pressure, local ischemia, or associated UTI. Pathologic findings include dilation of the collecting ducts and distal tubules and chronic tubular atrophy with little glomerular damage.

Symptoms and signs: They vary with the site, degree, and rapidity of onset. - Pain is common due to distention of the bladder, collecting system, or renal capsule. - Upper ureteral or renal pelvic lesions lead to flank pain or tenderness, whereas lower ureteral obstruction causes pain that may radiate to the ipsilateral testicle or labia. - Pain may be severe with acute complete ureteral obstruction (e.g calculus) - Acute dilation from a downward displaced kidney or after fluid load (e.g osmotic diuresis) produces intermittent pain. Pain is typically minimal or absent with partial or slowly developing obstructive uropathy (e.g ureteropelvic junction obstruction, pelvic tumor) - Ureteropelvic obstruction may cause hydronephrosis flank mass that may be palpable. - Anuria is highly suggestive of obstructive uropathy. However, normal or even polyuric urine output does not exclude partial obstructive uropathy. - Other findings may include hypertension, hyperkalemia, and

metabolic acidosis. - Lab studies may show elevations of BUN and creatinine. Urinalysis is most often normal or with small number of cells. Diagnosis: US may assess the renal parenchyma and the degree of hydronephrosis. CT or IVU localizes the level of obstruction. Kidney size may indicate the duration of obstruction Cystourethroscopy with cystourethrography is indicated if urethral obstruction is suspected. Treatment: Most cases can be corrected; a delay in therapy can lead to irreversible renal damage. Treatment consists of elimination of obstruction by instrumentation (endoscopy, lithotripsy) or surgery. Prompt drainage for hydronephrosis may be needed (percutaneous technique) Lower uropathies may require catheter drainage. ATB given if there is pyelonephritis or urosepsis 144) RUBELLA, SCARLATINA (SEE TOPIC 86). 297